Wikipedia:Reference desk/Archives/Humanities/February 2006

From Wikipedia, the free encyclopedia

February 1[edit]

Garage Again[edit]

I have another garage question. Before there were cars, were there a such thing as putting your horse carriage or wagon in something that is called a garage today, but wasn't called garage in those days? I mean a special room connected to the house for horse carriages or wagons, were they around?

Carriage house? —WAvegetarianCONTRIBUTIONSTALKEMAIL00:43, 1 February 2006 (UTC)[reply]
  • This has already been asked several times recently, and been answered several times. See here. --BluePlatypus 01:19, 1 February 2006 (UTC)[reply]

State of the Union Designated survivor[edit]

Concerning President Bush's State of the Union address this evening, has anyone seen details concerning who is tonight's designated survivor? Or, is this information covered until the address is over and everyone's home tucked safe in their beds? :-) — Scm83x talk 01:20, 1 February 2006 (UTC)[reply]

Ah, I just saw the report on C-SPAN that the DS isn't known until everyone gets into the chamber and reporters count heads to see who is missing. Smart money says Condoleezza Rice, though, as she's in London. — Scm83x talk 01:38, 1 February 2006 (UTC)[reply]
It was James Nicholson, the Secretary of Veterans Affairs. Also interestingly, there were only four Supreme Court justices there (that I saw): Roberts, Alito, Thomas, Breyer. --zenohockey 04:08, 1 February 2006 (UTC)[reply]

ancient rituals[edit]

I have a question I am having a difficult time finding and answer to hopefully you can help. The ancient inhabitants of Asia filled a suspect's mouth with this item. If the suspect had trouble spitting this item out, it was beleived he was lying and was judged guilty. What was the item? This is for a research project I have to do for a CJ class. Thank you so much for taking the time to help out.

Jody

Rice. http://en.wikipedia.org/wiki/Lie_detector#History -LambaJan 05:12, 1 February 2006 (UTC)[reply]

O. Henry recursive story[edit]

I am searching for the name of a short story by O. Henry that I once read. It involved a person telling a story whose main "plot" was something to the effect of "[When I was little, my father told me, 'Years ago, when I was but a boy, my father took me out by the lake and said, "Let me tell you an important story ... "']". Besides its recursive structure, the story also used nested quotation marks, at least ten (?) levels deep. Thanks, Ardric47 02:17, 1 February 2006 (UTC)[reply]

I don't know it exactly but you may want to check out Wikisource:Author:O. Henry and see if any of the titles bring it to mind. 68.39.174.238 21:21, 2 February 2006 (UTC)[reply]

The Truth,The Theory of Intelligent Design & The Separation of Church & State (cont.)[edit]

You said that teaching intelligent design in public schools violates the separation of church and state because it is advancing religious viewpoints.

But what if we don't teach the beliefs and teachings of creation of one or a particular religion?I mean, what if we don't mention anything about the God of a particular religion and say that the intelligent designer could be the God of any religion?

Please continue the discussion on the same thread and don't start a new one (see the top of the page). And please sign your questions with four tildes (like this: ~~~~), then we can keep track of who is asking what question.
To attempt an answer: No matter how you slice it, Intelligent Design still has no place in science class because it is not, by any stretch of the definition, science. Every advance the field of biology and ecology in the last two centuries is based on the theory of evolution - if it was so far wrong, so you think we would have GM crops now? Deciphered DNA? If microorganisms didn't evolve, then the drugs we used against them fifty years ago would still be effective - but they do evolve (into more resistant forms), so we have to develop new ways to fight them. Check out a whale skeleton - see those little bones halfway down its spine, those that look like the remains of... legs?
The scientific consensus on evolution is about as strong as you can get with fundamental theories, arguably stronger than, for instance, the consensus on gravity - where we still have a number of unanswered questions - but you're not going to start saying that gravity is an illusion and that all things fall because a diety wants them to, right? (see intelligent falling). To put it bluntly, Intelligent Design is scientific nonsense - sorry, but it's true. Even the judge asked to arbitrate the case in Dover, PA, said clearly what the vast majority of scientists and non-US laypeople were thinking, that ID is an attempt to introduce Christian religious teachings into the science classroom, where I am sure you agree it doesn't belong. The purpose of ID is not to "teach the controversy" (because there is none), it is not to "open students' minds to alternatives" (because, in that case, we should be teaching students other 'alternative' theories, such as a flat Earth, or the four humours). The purpose of ID is to disguise religion as science, and therefore should not be taught in the science classroom.
However, if you do want to teach ID in schools, it can well be discussed in Ethics or Philosophy class, along with the (in that context) other equally valid explanations for Creation of the the Navaho, the Sumerians and the Australian Aborigines. — QuantumEleven | (talk) 10:12, 1 February 2006 (UTC)[reply]
I'm not going to feed the troll, so I'll only pass on a cute story from the news. A physician said that he will ask a patient if he believes in evolution or not before prescribing antibiotics. If yes, he will give only sulfa drugs. If no, he will give Cipro. After all, antibiotic resistant bacteria are evolved. If there is no evolution, then sulfa ought to work on all of them. Geogre 14:12, 1 February 2006 (UTC)[reply]
I like to argue that any parent who tries to deny medical treatment to their kids because "God will save them if he wants them to live" should be shot in the head, because "God will save them if he wants them to live". StuRat 22:58, 4 February 2006 (UTC)[reply]

History[edit]

Can anyone name a key turning point in the British Home Front wars. This can cover any date or event between 1914 and 1945. Thanks

I'm sure the articles on World War I and World War II, and those linked off of them, can help you with your homework. Dismas|(talk) 13:58, 1 February 2006 (UTC)[reply]

Tenzing Norgay's nationality[edit]

Which country did Tenzing Norgay belong to ? Was he Nepalese, Chinese (Tibetan) or Indian ? Did he ever change his citizenship officially ? What was his nationality during the time when he scaled the Everest ? Jay 11:48, 1 February 2006 (UTC)[reply]

The article you link says it all, really - he was Nepalese, although there were occasional rumours that he may have actually been Chinese. He lives some of his life in India. To the best of my knowledge, he was a Nepalese citizen at all times, and certainly he was during the 1953 ascent. Grutness...wha? 12:06, 1 February 2006 (UTC)[reply]
I guess so too, but several General Knowledge and text books say he was the first 'Indian' to scale Everest. Jay 03:58, 2 February 2006 (UTC)[reply]

Oposition Leaders[edit]

Does anyone know where to find a list of worldwide oposition leaders (goverments) and links to them?

Thanks,

I suppose that depends on your perspective. When groups oppose, they oppose. Some see Osama Bin Laden as an opposition leader, while others see George W. Bush as one. Whatever your definition, your best resource is the news. Opposers love to publicly oppose, so consquently, all of the major names are repetedly mentioned. -LambaJan 20:51, 1 February 2006 (UTC)[reply]
Presumably he's talking about Opposition (parliamentary) leaders. That would be a reasonably well-defined set. I have not looked for a list, though. --Anonymous, 2006-02-02 02:02 UTC!!


I meant what anonymous capted, parlamentary oposition leaders, Osama Bin Laden? he is not an oposition leader he is a terrorist and/or and assassin

the olympic games of 1936[edit]

Which ones, the 1936 Summer Olympics or the 1936 Winter Olympics? Also, please read the top - this is not a search engine! — QuantumEleven | (talk) 14:03, 1 February 2006 (UTC)[reply]

What is your question? -LambaJan 16:01, 1 February 2006 (UTC)[reply]
Whatever it is, the answer is either Jesse Owens or Leni Riefenstahl. :) --BluePlatypus 18:57, 1 February 2006 (UTC)[reply]

Meissonier 1876[edit]

I purchased what looks like a print from a yard sale. I have researched all over the books and internet about this and have come up empty handed. I have been trying on and off for the past four or five years. So with falling upon zippo I was wondering if you could help? This etching, print, is signed in the lower right hand corner MEISSONIER 1876, and in the lower left corner (which is torn off) is a small picture of a man. The picture itself is two musketeers on a horse outside a house or tavern drinking out of glasses, and three people, one a lady petting the horse and two men. I am curious about the small picture in the lower left corner. Can you give me some insight to this as I have not been able to find anyhting on it at all. Thank You, Sincerely Donna

Is www.wallacecollection.org/c/w_a/p_w_d/f/p/p289.htm this your print? LarryMac 16:25, 1 February 2006 (UTC)[reply]

The print that you have is of a painting called "The Halt" by J.L.E Meissonnier. Most likely, you purchased an etching that was done for a collection of art prints that were shown at the 1893 Columbian Exposition, which was located in Chicago, Illinois. I am not sure what the small picture of the man is in the lower left corner. I hope this helps some. Shelley ........................................................... Hello,

I am from Canada and it seems that I am in the same position as as the owner of the scetch. I have the same scetch in my home also. From what I read it is exactly the same, would like to get your email address so I can send pictures of it to confirm? also I was curious as to what is the origin as well as some value associated with the picture. I did not notice any numbering on the picture to indicate that there were several duplicates made. Dale - canada. [email protected]

Cartoon love music[edit]

What is the name of this classical piece often played in cartoons when one character falls in love with another character? It goes (probably wrong key): f#' g', b c' d' a g c', a a b g#... —Keenan Pepper 15:35, 1 February 2006 (UTC)[reply]

Have you tried this link? www.thomasbending.co.uk/torus/ Maid Marion 16:01, 1 February 2006 (UTC)[reply]
Wow, that's such a bad system. Why can't I tell it the intervals if I know them? I ended up with stuff like "Camptown Races", which sounds nothing like the piece I'm looking for. —Keenan Pepper 16:14, 1 February 2006 (UTC)[reply]
How about this one? www.musipedia.org/ I just Googled 'identifying tunes', which threw up lots of possibilities that you might want to explore. Maid Marion 16:22, 1 February 2006 (UTC)[reply]
This one's much better. Turns out it's from Tchaikovsky's Romeo and Juliet. Thanks! —Keenan Pepper 16:50, 1 February 2006 (UTC)[reply]
AHH!! I knew it was by Tchaikovski! I couldn't answer it because I couldn't find it. I thought it mighta been from Swan Lake. -LambaJan 20:33, 1 February 2006 (UTC)[reply]

Bruce Dickinson of Iron Maiden[edit]

Bruce said something about Metallica in Metal Hammer. I read it in an interview. Heres the question -

E - I read in Metal Hammer that you made a very bold statement about Metallica. Bruce - Yea, it's true. E - I agree with it but this is something that you haven't walked that fine line in a long time. B - No, but it's true. I thought should I go ahead and say this and I thought f**k it.


Anyone knows what it was? I couldnt find it on Metal Hammer archive.

Name for pre-modern warfare[edit]

Hi, I'm trying to translate an article from Chinese and I'm stuck on a word. You shouldn't have to know Chinese to know the answer, though. The author describes U.S. tactics in Afghanistan as "fei xian shi" or "non-line-style", in the sense (I believe) of not having two armies lined up against each other. Is there a better word for this in English?

Thanks. 150.174.234.101 19:30, 1 February 2006 (UTC)[reply]

It's possible that the author is being more precise than that. One type of attack in classic warfare is the "attack in echelon." That tactic was best used by Napoleon, but it certainly existed before. (You have your troops lined up and they charge one after the other, moving across the field. When your opponent shifts to meet the first attack, he's weaker for the second rank, then weaker still on the third, and the result is that he is flanked.) It may be simply "non-echelon" that you're looking for. You have to go to non-professional armies altogether before you find them attacking and defending in a mob. Every army that we know of as an army had formation (lines). Geogre 19:39, 1 February 2006 (UTC)[reply]
What is the time period? If this is after the Sino-Japanese War, then you should not translate Feixian. It is referring to the county of Feixian, the headquarters of the Eighth Route Army. It is therefore comparing the American's tactics to those of general Zhu De. --Kainaw (talk) 01:49, 2 February 2006 (UTC)[reply]
In terms of just looking for a generic term, I think fastfission is right with asymmetric warfare. You might also look at guerilla warfare. Marskell 08:13, 2 February 2006 (UTC)[reply]
But, if this is referring to the Feixian County style of combat, it is not guerilla or asymmetric warfare. The questioner translated the name of the county as "non-line". That would be like saying South Carolina style combat is attacking from the south with some ladies named Caroline.
Feixian County became famous because of the Eighth Route Army. They would infiltrate attacking forces. By the time an attacking force was at the border of Feixian County, it would be facing opponents in the county, opponents who snuck through their lines and were now behind them, as well as many who infiltrated and became part of their own ranks. So, claiming the Americans used Feixian tactics means that they used people who could infiltrate the opponents ranks, spy on them, and surround them before they could attack. It has little to do with how troops line up. --Kainaw (talk) 13:58, 2 February 2006 (UTC)[reply]

One term that somewhat applies is a proxy war. In such a war, the main enemies do not confront one another directly, but each back some minor power in their war. In Afghanistan, for example, the US supported the Northern Alliance against the Taliban. The Taliban was originally backed by Pakistan and al-Quada, but that support had somewhat evaporated by the start of the war. StuRat 22:16, 4 February 2006 (UTC)[reply]

Successful crimes[edit]

Are there any lists on wikipedia, showing the most successful crimes throughout history? (Henningklevjer 20:36, 1 February 2006 (UTC))[reply]

I was unable to find such an article on the Wikipedia. Perhaps you can write it. -LambaJan 21:23, 1 February 2006 (UTC)[reply]
Not that I know, but there's this list which is pretty close: list of famous bank robbers and robberiesҠieff 21:37, 1 February 2006 (UTC)[reply]
By definition, the most successful crimes are the ones that nobody knows about. What are you actually asking about? What do you mean by "successful"? JackofOz 09:32, 2 February 2006 (UTC)[reply]
Of those who got away with their crime, Jack the Ripper is probably the most notorious. I believe nobody has yet been convicted for the Northern Bank robbery when so much cash (£26.5 million) was stolen that the Northern Irish currency had to be reprinted. --Shantavira 13:44, 2 February 2006 (UTC)[reply]
Murders in the millions, and embezzlement in the billions, have been attributed to heads of state (typically after they are deposed, and most often conveniently dead). Of course, they may have had the power to make their acts legal, but they typically didn't bother, perhaps because it would be bad PR. Anyway, they take some beating. Notinasnaid 15:21, 2 February 2006 (UTC)[reply]


Those crimes you know about but dont have the slightest clue how it was done can be considered most successful. It doesnt have to necessarily be something undiscovered. I may find out my brothers passowrd somehow and read his emails. He'd never know that. Its not something brilliant. So its a valid question.

If D.B. Cooper survived, I think he'd get my vote....at least in my top 10. Jwrosenzweig 06:34, 3 February 2006 (UTC)[reply]

Hatha Yoga[edit]

Hello. I looked under Hatha Yoga which was very infromative but my main question is, how would one describe the difference between Hatha Yoga and other yogic paths such as, raja or bhakti yoga for example?

Thank you so much for your time!

Leigh

Have you seen the section called "diversity of yoga" in our article on yoga? I think it describes the differences well: "Thus, yoga may include love and devotion (as in Bhakti Yoga), selfless work (as in Karma Yoga), knowledge and discernment (as in Jnana Yoga), or an eight-limbed system of disciplines emphasizing meditation (as in Raja Yoga)."
This differs from Hatha Yoga, which is the "holistic yogic path, including moral disciplines, physical exercises (e.g., asanas (postures) and Pranayama (breath control), and meditation. The Hatha yoga predominantly practiced in the West consists of mostly asanas (postures) and exercise."
Also, looking at our articles specifically on Raja Yoga or Bhakti Yoga may help. Hope this was useful, --Lox (t,c) 21:24, 3 February 2006 (UTC)[reply]

Religious beliefs[edit]

I need to know the names of all and i mean all the religions recognized by the department of corrections, in the United States. Not all the little ones, but the top 30 most recognized. I know everyone is going to consider theirs a big deal but I need this to be serious.

There is a Department of Corrections for each US state. It is not a federal department. —Keenan Pepper 23:22, 1 February 2006 (UTC)[reply]
Do not forget about the federal Bureau of Prisons. The www.bop.gov/inmate_programs/religious.jsp BOP website doesn't mention any official recognition of particular religions. It just says, "Inmates can observe religious holy days and wear and use religious items consistent with their faith as long as this is consistent with policy and with the security, safety, and good order of the institution." -- Mwalcoff 00:28, 2 February 2006 (UTC)[reply]

Beside a Sikh's knife, what religious items would be inconsistent with security? Rmhermen 16:33, 2 February 2006 (UTC)[reply]

Articles of clothing related to Hijab (Burqas, niqabs and dupattas) can cause problems at security checkpoints as well, since they prevent identification. Other obscuring clothing (turbans, Nun's habits and loose robes may have to be removed if there's a problem at the metal detector. The www.sikhcoalition.org/FAAGuidelines.pdf FAA Guidelines detail how theyre dealt with. GeeJo (t) (c)  17:18, 2 February 2006 (UTC)[reply]
Also, a couple of years ago there was a set of lawsuits by prison practitioners of minority religions like Wicca and Asatru. IIRC, some of the (printed) materials of the particular branch of Asatru were considered to exacerbate racial tensions in the prison facility. Open flames are another potentially problematic part of worship. FreplySpang (talk) 21:14, 2 February 2006 (UTC)[reply]

Asperger Syndrome, Dating, and Marriage[edit]

I'm the only son (2 sisters won't help any, in terms of passing the lineage), and not only that, but I'm the only male 1st AND 2nd cousin with my last name. Therefore, I must pass on my lineage. It's no less than mandatory.

However, if I ever successfully pull off having a relationship with a possible girlfriend, when would be the right time to tell her about my disorder?

Not only do I have that, but I don't have a job either. The only way I'm not homeless is by having Supplemental Security Income i.e. a government paycheck. What the heck do I tell her when she asks,

"So, where do you work?"

"I'm a student; I don't work anywhere."

She may understand that, as it's more ideal to study w/o a job, so I can have more time to do my studies. However, she'll still be curious:

"So then, what do you do to have a roof over your head?" (i.e. to have an apartment)

I do NOT want to answer this: "Due to my Asperger disorder, it's a whole lot harder for me to land a job, even jobs below a living wage, so I told this to the government, and they've been sending me disability checks ever since."

She'd leave me if I did. What am I supposed to say or do instead? How else should I answer her?

For those of you who are about to suggest that I pursue girls with Asperger's, I don't think that's a good idea because I want to raise normal, mentally healthy children. Both parents w/ Asperger's will bring the probability of traiting offspring with it to 100%! (not unless Gene editing gains momentum to the point that it effectively deals with autistic genes, which I think won't happen for quite a while.)

--Shultz 23:25, 1 February 2006 (UTC)[reply]

There are a number of editors on Wikipedia with AS. I suggest you check out this link and ask others how they are dealing with it. —WAvegetarianCONTRIBUTIONSTALKEMAIL00:09, 2 February 2006 (UTC)[reply]
Eh? You talk of Asperger as if it was a horrible, disabilitating mental disease that must be controlled. It is not like that. Also, it is not yet fully understood what causes autism, so your fears are unfounded on our current scientific knowledge. I have a bad case of Asperger and I don't see it as a direct obstacle in my professional life. In fact, I think it gives me a lot more insight and grasp on reality than most, "normal" people. A good deal of the greatest humans that ever lived had, or are suspected to have had, Asperger's syndrome, and these people changed the world we live on forever. I think what you really need is to change the way you deal with your condition. (And, on a personal note, I don't find your reasons to raise a family valid at all. Have children if you really want to, not because you think you must, or you're likely to fail as a parent.) ☢ Ҡieff 00:12, 2 February 2006 (UTC)[reply]
Well, I find your attitude of passing on lineage as slightly weird. Are you nobility or something? (and if you are, it still strikes me as rather pre-20th-century). But enough of that. I'd say: Don't lie. There's no point in lying. If the girl isn't going to want you because of these things, then it's not going to make much difference if you tell her up front or later. The fact that you lied would be worse. A person who will want a serious relationship with you isn't going to care too much about these things anyway. Having an impressive job or money or whatever may make someone interested in you, but it doesn't make them fall in love. That's up to your personality, and your personality is not your disorder. However, some people might need help to better understand the disorder as to better be able to see you for you, and not the aspberger part. So there's everything to gain to be upfront about it. Although it wouldn't be the first thing I'd tell people, "Hi! I've got aspberger!", but certainly once you get to the level of talking about personal things. --BluePlatypus 00:43, 2 February 2006 (UTC)[reply]

You know, there is no test for Asperger syndrome, and the diagnosis is often simply the opinion of one person judging another (or himself). I often remind families to accept a diagnosis like that only if it is clearly advantageous (like qualifying for extra resources at school), because such a diagnosis has its downside, by limiting others' expectation or your own expectation of what you can do and not do. When I first read your question, "should I tell her", my first impulse was to say, "if you really have it you won't have to tell her, she'll figure it out". However, on reflection, only you can decide how much of your social behavior you are willing to own and control and how much you get to blame on AS. If you don't like this answer you can be relieved that I am not the kind of doctor who makes the final diagnosis about this, just the kind of doctor who gets asked what it means or whether it is right, and the kind of doctor who realizes when 50 editors here claim the diagnosis that the diagnostic criteria and social significance of the disease have changed enormously in twenty-five years. Labels have consequences and it sounds like you are examining that. Good luck. alteripse 01:12, 2 February 2006 (UTC)[reply]

Shultz: If you think the fireworks would be bad if you tell her now, what do you think they'll be like if you delay, and tell her later? If you actually think she would leave you if you tell her about your condition, then look at it for a moment from her point of view: It is cruelty to not tell her. You're just wasting her time. If, however, you're incorrect and she doesn't care, then you've got it out of the way and can enjoy life again. Just tell her and get it over with; one way or another, you can move on.
By the way, this question indicates you need a lot more relationship experience before you decide to get married. I recommend at least two serious girlfriends in exclusive relationships for a minimum of 1 year each. This will give you the needed experience in communication and trust. Then you have my permission to marry. Junkboat 04:08, 2 February 2006 (UTC)[reply]
ONE YEAR? Wow, I've never lasted that long. Ever. How would I ever pull it off? (I'm still in college so I'm sure something could be done.)
I wouldn't fret so much - after all, these are just one person's idea of a guideline :) Also, don't forget that as you're still in college, you have many, many years ahead of you in which to meet people, so take things at your own pace. Whilst personally I would agree with Junkboat, that several long-term relationships (I would even put the figure at two years) are very important before making even more long-lasting committments such as marriage, I know quite a few people who have gone a different path and seem to be doing fine. In the end, it's your life and your decisions - but a point to think about, if you can't manage to maintain a relationship for a year, how do you think you will maintain a marriage for (possibly) several decades? This is not meant to put you off, far from it, I would more see it as an indication that you still need some more time to find out what you want in a relationship and how to make it work over long time spans. Practice makes perfect :) — QuantumEleven | (talk) 13:33, 2 February 2006 (UTC)[reply]

I couldn't not respond to this, even if the thread is so long. I wholly agree with the honesty policy. Honesty to her about all of the things she should know about you, including this; and honesty to yourself about how your condition actually affects your life. I also agree with what was said about practicing a higher level of emotional maturity. But going out with someone for a year so that you can learn about relationships is just as silly as marrying so that you can pass along your surname, in that both could easily be construed as using the person. A relationship will not flourish if it's planted in the soil of this mindset. You seem to have a good bit of emotional maturity when it comes to your relationship with your family, or else you wouldn't be so eager to please them. I'm not a doctor, but my advice is to furthur develop your relationships with your family, your plants and pets, your neighbors, friends, Higher Power, room, school, money and self. That's where you learn about relationships. Till that soil and plant a marriage in it. But really, don't be rude, don't have a prolonged relationship with someone unless there's a chance it will last forever. -LambaJan 23:55, 3 February 2006 (UTC)[reply]

Asperger was conviced that many people with that peculiar attitude (a disease only in occidental terms) "would use their special talents in adulthood." Please be confident, there are girls who won't fly when they know you more. --DLL 20:31, 4 February 2006 (UTC)[reply]

February 2[edit]

Hubert H. Humphrey speech[edit]

I would love to read the entire transcript of Hubert Horatio Humphrey's 1948 address to the Democratic National Convention. Better yet, I would love to view or purchase a video of his actual address to the convention. Does anyone know where this could be obtained? --71.145.191.15

Your wish is our command: www.americanrhetoric.com/speeches/huberthumphey.html for the text. I could find recordings of later speeches available inexpensively by googling, but not this one. alteripse 02:46, 2 February 2006 (UTC)[reply]

Thanks a million for your wonderful help. --71.145.191.15 15:01, 2 February 2006 (UTC)[reply]

I'm glad this page was helpful to you, but for the future, you should note the suggestion at the top about checking the content we already have. One of the "External links" to the article on Hubert H. Humphrey is www2.austincc.edu/lpatrick/his1302/WhenAllTheExperts1.html an account of the 1948 campaign, which, as noted in our article, includes the complete text of the speech (as Appendix #1, near the bottom). JamesMLane t c 17:29, 3 February 2006 (UTC)[reply]

Poem[edit]

While browsing through poetry, I came upon this piece by Gertrude Stein.

"Chain boats are merry, are merry blew, blew west, carpet."

I have a few questions about this poem.

1. What on earth is a chain-boat? 2. What on earth does a carpet have to do with a chain boat? 3. What's significant about a chain-boat to write a poem about it? 4. Am I missing something?

--User:Shadarian 03:11, 2 February 2006 (UTC)[reply]

You might want to read the whole poem. It's all rather strange: www.bartleby.com/140/2.html User:Zoe|(talk) 03:26, 2 February 2006 (UTC)[reply]
A chain boat is a like a ferry that uses a chain (as opposed to a rope) attached to both destinations to drag itself back and forth across the water. They are used where it is too far to have a rope extended above the water. A chain can safely be submerged. It goes up and over a wheel on the boat. When the wheel is turned, the boat moves along the chain. --Kainaw (talk) 03:26, 2 February 2006 (UTC)[reply]
After reading the poem, Gertrude is apparently thinking of "gravy boats" and using the phrase "chain boats". Is there a type of gravy boat normally called a chain boat? --Kainaw (talk) 03:39, 2 February 2006 (UTC)[reply]

"Chain boats/chin boats are merry/very are very blue, bluest, carpet": As is often the case with Stein, it's elaborate wordplay with a suggestion of vertical signification. (Imagine that words have two meanings. Now, instead of one moving forward, semantically, while the other is rejected as an ornament or a supplement, both meanings move forward semantically to the next double word/double signification, and when one line of signification is cut off, it may become dominant again in a few words.) As for what you may or may not be missing, that's a question of whether or not you want to understand her the way her fans do. Personally, I don't. I don't like, enjoy, or appreciate her poetry. I'm fussy, though. Her most credible fan made the case to me that the feminine language is fundamentally non-linear and that, at least as Stein would have understood things, the female language is non-Oedipal and defined by Freud's "polymorphus perversity," where the whole body is involved in ego and language definition. She, therefore, speaks a feminine language by speaking a language that refuses to eliminate one meaning as it opens up another. <shrug> I'm pretty phallocentric in my poetry preference, I guess. Geogre 03:51, 2 February 2006 (UTC)[reply]

Still, where does carpet come in from? Rmhermen 16:25, 2 February 2006 (UTC)[reply]
I seriously hesitate to answer. All I'll say is that I think I know, but I don't know I know, so I'll leave it as carpet, and not car-pet, and say that perhaps, as in "Lifting Belly," the subject of domesticity and the subject of sexuality and sensuality are all tied together in a way that a "masculine" poet would not do. Geogre 19:27, 2 February 2006 (UTC)[reply]

Wow - I didn't know that that was all considered one poem. Then again, I know nothing about Gertrude Stein, or poetry. Her work sure is...interesting. I suppose that you have to be into the whole poetry thing to grasp her work, something I am not by any means. --Shadarian 00:32, 3 February 2006 (UTC)[reply]

Keep on browsing. You never know what you might find in poetry. Some of us have found a lot, and that's not even counting what folks read into it, which may or may not have value. It does for them, I guess. It's a free world. But it's more fun to find the there that's there. --Halcatalyst 05:09, 4 February 2006 (UTC)[reply]

What was the first US Supreme Court ruling to overturn a previous USSC ruling?[edit]

Inspired by an anon's question over at Talk:Brown v. Board of Education#another random question: What was the first US Supreme Court ruling to overturn a previous ruling of the US Supreme Court? Tempshill 03:57, 2 February 2006 (UTC)[reply]

Certainly, you have no need to be told there are many articles in Wikipedia on the US Supreme Court. I poked around a little and found List of United States Supreme Court cases, which might be a good jumping-off point. In the end, though, if you really want to know, you may need to go to a law library and ask the reference librarian. --Halcatalyst 19:54, 2 February 2006 (UTC)[reply]
You know, I never saw that list article before. Thanks. I figure you're probably right on the reference librarian. Tempshill 21:06, 2 February 2006 (UTC)[reply]

BRIC SHARES[edit]

is there any value to the 1979 BRIC shares or any news on the court case outcome?

Doesn't look good. See www.askchriscarter.com/files/faqs.html under the question about BRIC shares. Tempshill 07:26, 2 February 2006 (UTC)[reply]

Music technology use in the transition of musical theatre to film[edit]

Hi I'm looking for information on how music and the technology of music is used differently in theatre and film productions of the same show. I want to know what benefits making a musical theatre production into a film has through use of technology and how films have been adapted through their music from their theatre production. Thanks

I can't think of any general works offhand, though they have been written. I suspect you might do better looking at the ways individual works have been altered for film. (Chicago, Little Shop of Horrors are two good examples). Take a look at www.musicals101.com/lecturetopics.htm. Also make sure you understand the difference between www.filmsound.org/terminology/diegetic.htm diegetic and nondiegetic songs. - Nunh-huh 02:30, 4 February 2006 (UTC)[reply]

Thanks! Yeah I do know the difference between those. I'm already taking West Side Story and analysing it, along with phantom of the opera as the latter is more modern and i want to look at how the change in available technology has affected the way the film has been produced. Just finding a hard time getting resources about it.

What is the GDP per capita for Zimbabwe over the past 10 years?[edit]

I am having trouble finding this data over the internet, can anyone help me please?

Try an almanac. Hmm, I guess there's no such thing as a "wikimanac", is there? We have just about every other reference book... =P —Keenan Pepper 16:37, 2 February 2006 (UTC)[reply]
Here's the GDP for the latest 6 years www.indexmundi.com/g/g.aspx?c=zi&v=65 www.indexmundi.com/g/g.aspx?c=zi&v=65 —This user has left wikipedia 17:42 2006-02-02

Your tax dollars at work (if you're an American): the CIA World Factbook. There you can find tons of information on any country you're interested in, including www.cia.gov/cia/publications/factbook/geos/zi.html Zimbabwe. There it says the GDP per capita (purchasing power parity) in Zimbabwe is $1,900 (2005 est.) You'll need to do a little more research to get the 10-yr average. --Halcatalyst 19:41, 2 February 2006 (UTC)[reply]

If the reports on the recent policies of Mugabe, the GDP has probably taken a hit in the last few years. And given the increasing amount of hostility between Zimbabwe and the rest of the world, there's fewer outsiders there to report about it. So the numbers are probably increasingly uncertain. That said, the CIA's guess is probably as good as any. --BluePlatypus 20:38, 2 February 2006 (UTC)[reply]
It's a guess based on a lot of research and resources and money, and it's likely to be quite accurate unless the result is foreordained. That can happen. --Halcatalyst 05:02, 4 February 2006 (UTC)[reply]
In addition to Mugabe's attacks on foreigners, such as the white farmers there from colonial Rhodesia days, which damage the economy, he also attacks the poor. Their black or grey market stalls, where they sell goods to make a living, are destroyed along with their homes. The economic damage from this destruction is substantial. StuRat 21:47, 4 February 2006 (UTC)[reply]

Iran / Security Council[edit]

What are the legal ramifications of Iran being referred to the Security Council? Why is it being so hotly contested? Is it possible that China and Russia will oppose any sanctions on Iran? Is there a list of Securty Council Resolutions? Thanks, —This user has left wikipedia 17:36 2006-02-02

  • The Security Council has a wide variety of diplomatic approaches that it can utilize, one after another, from talking about the issue, to a a mild public rebuke, to sanctions of various sorts, to, in the most extreme and unlikely case, a request that UN member nations use military action to invade the country; see Korean War. Any permanent member of the Security Council can block any of this with the veto power, including China and Russia. For the list, see List of UN Security Council Resolutions. It's divided into subpages to contain the hundreds of resolutions. Iran is hotly contesting this because under the terms of the Nuclear Non-Proliferation Treaty, which it has signed, most of the developing nations of the world agreed with the superpowers not to pursue research on nuclear weapons, and in exchange the superpowers promised to help the developing nations build nuclear power reactors and infrastructure. Iran says that it is merely pursuing nuclear power per the terms of the NPT, but the US has been saying that the actual reason for Iran's nuclear research and procurement has been in order to build a bomb. Tempshill 19:12, 2 February 2006 (UTC)[reply]

3 categories of money- M1, M2, M3[edit]

What is included in each of the 3 categories of money? Why is one item categorized in M1 for example and not in M3? How are these classifications helpful to the Federal Reserve Board?

Did the money supply article answer your questions? Tempshill 19:02, 2 February 2006 (UTC)[reply]
The main goal for categorization of money is liquidity vs solidity. Liquidity means flowing : banknotes (M0) can be obtained and exchanged quickly. M3 exists only in scriptures and will stay longer. --DLL 20:38, 4 February 2006 (UTC)[reply]

Mellon Arena in Pittsburgh, PA[edit]

How many times has the dome been opened in the history of the arena?

I don't know, but I looked around and found some interesting facts.
  • In initial construction of the roof cost $2 million. It consisted of eight sections cut at 45-degree angles, six of which are able to rotate on 3,000 feet of rails. The girders, 34 feet above the floor, slope at a 13-degree angle. With each section of the roof powered by five motors, the arena can open completely in about two and a half minutes.
Other dimensions of the dome were impressive for their time:
Weight: 3,653 tons
Diameter: 417 feet
Area: 166,000 feet of stainless steel sheeting
Height: 155 feet above the ice surface
  • In 1995, a bigger scoreboard was added, but produced one major drawback: the dome could no longer be opened completely due to the extra weight.
  • The roof isn't opened that often and never for hockey games, but it works well.
hockey.ballparks.com/NHL/PittsburghPenguins/ -LambaJan 03:51, 4 February 2006 (UTC)[reply]

Means for breaking "the fourth wall" in plays[edit]

Hi! I am writing an essay on "The Skin Of Our Teeth" and "The Glass Menagerie." I have chosen the topic of breaking the fourth wall in each play. I have said that the authors have broken the wall by directly addressing the audience in their plays, by changing the scenary, by projection of screen images, by reminding the viewers that they are seeing a play.

My question is if I can say that symbolism is also a mean that the authors use to break the fourth wall, because sometimes symbolism helps the playwright better convey the central aims of his play.

I would appreciate it if you can help me.

Thank you!

P.S.: From reading the rules I did not fully understand how you would respond to my question, that is why I am giving you my e-mail.

mariayten -at- yahoo.com

Once again, thank you!

username: Mariayten

  • Elena *
Symbolism has been used as such many times. It can be something as simple as the line, "All the world is a stage and all the men and women are merely players." This is funny only because it is said by a player on a stage - which reminds the audience that it is a play. --Kainaw (talk) 23:37, 2 February 2006 (UTC)[reply]
Typically, breaking the fourth wall requires something quite overt. I would caution you to be precise in your terminology (I say this as a person who gets such essays from time to time) and to be sure to say that these are methods of decreasing the illusion or the illusionistic identification of the audience with the characters and not exactly breaking the fourth wall. Plays that break the fourth wall are quite old. In the English theater, Knight of the Burning Pestle breaks it by having an "audience member" demand to be in the play. Ancient Greek theater probably had no fourth wall to begin with, as the chorus and such moved among the audience, and Roman theater probably had a good deal of direct leering, gesturing, and costuming to allude to the audience. Bertolt Brecht wished to make characters who could not foster illusionistic identification, so the "director" speaks to the audience frequently. My point is that breaking the fourth wall is a fairly specific and overt act, and these other things you mention are disillusioning, distancing, or alienating effects. In those terms, symbolism can, of course, be part of it. (I would also warn you to be careful and not to mistake a single instance of staging for what is intrinsic to a play. Our Town has no choice but to break the fourth wall, but a given production of True West can do things to break the wall or not. (I once saw a production of Everyman that broke the fourth wall by having the whole audience get up and move from one theater to another to another for the three phases of the play.)) Geogre 02:40, 3 February 2006 (UTC)[reply]

Question regarding Henry VIII[edit]

Karl Marx, in The German Ideology, states: "These vagabonds, who were so numerous that, for instance, Henry VIII of England had 72,000 of them hanged..." Is this true? Where can I find more information? --Tothebarricades 19:42, 2 February 2006 (UTC)[reply]

homepages.force9.net/tradeck/cjd/cjd9304.htm This has some information. JackofOz 23:59, 2 February 2006 (UTC)[reply]

PATRIOT expiration[edit]

According to the articel here on the PATRIOT Act, the sunsetting parts expire tomorrow. Is that true, or does an update need to be made? I'm wondering because I've heard nothing about this act expiring or efforts to renew it past it's current extention from other people, media sources, etc. 68.39.174.238 20:37, 2 February 2006 (UTC)[reply]

It was true, however the US House of Representatives voted today to extend it until March 10th to allow further time for negotiating the terms of a renewed act. As of right now, the extension still has to be approved by the Senate. LarryMac 21:08, 2 February 2006 (UTC)[reply]
Yearrgh! SOO close! It looks like it goes by a month and some @ a time... 68.39.174.238 22:41, 2 February 2006 (UTC) PS. I wonder what the final version'll be ;)[reply]
The final version will not be much different than it is now. The primary problem with the Patriot Act is that nearly everything ever reported or told about it is untrue. For example, CNN ran a story this morning that the Patriot Act was going to expire. It made no mention of the Sunset Provisions. So, the story was untrue. There are also many stories about the "Terrorism Law" (aka the USA PATRIOT Act) being used for non-terrorism investigations. It is untrue that it is a terrorism law. It frustrates me because there is so much to dislike about the Patriot Act, but what most people dislike about it are things that are not true - such as the common claim that it allows ANYONE in the government to snoop your library records at ANY time for ANY reason without any court oversight of any kind. There isn't a bit of truth to that, but I hear it over and over and over. --Kainaw (talk) 23:44, 2 February 2006 (UTC)[reply]
It is "untrue" that the "Uniting and Strengthening America by Providing Appropriate Tools Required to Intercept and Obstruct Terrorism Act" is a terrorism law? You could've fooled me. --BluePlatypus 00:50, 3 February 2006 (UTC)[reply]
As I said, it is frustrating that people want to read the silly acronym and ignore the text of the act. If you read beyond the title, you would see that it covers a hell of a lot more than terrorism. It is about federal investigations of all kinds. Why is this frustrating? When you get hit with a Patriot Act investigation, you go "Hey! I'm not terrorist!" and the judge says, "You should have read the law. It isn't a terrorist law. It is a Federal investigation law." Then, you wish you listened to more than just the press so you could protect yourself. --Kainaw (talk) 02:20, 3 February 2006 (UTC)[reply]
It still is an anti-terrorist law. The name of the law, the justification for it and the entire debate around it centered on the fact that the law was intended to fight terrorism. The entire argument was about terrorism. Look at the transcripts of the Congressional debates if you like, and the more recent ones on renewing it. The fact that the law itself does not target people involved in terror crimes specifically is a point of criticism. Because fighting terrorism was the argument used to sell it to Congress. What you are doing, is turning that on its head and saying that since it doesn't target terrorists specifically, that means it's not an anti-terrorist law, and that critics therefore have no place to complain about it being used against non-terrorists. That completely ignores the arguments used to justify it. If you like the Patriot Act the way it is, then that's fine. But don't pretend that its proponents didn't present it as an anti-terrorist measure. They did. That is simply undisputable. And given that they did, people have every right to criticize the law if they feel that it does not accomplish that goal. If someone promises me a car and gives me a tractor, pointing at the vehicle and saying 'Oh, but it's quite obvious it's a tractor, so why do you keep complaining it's not a car?' does not mean I don't have a right to complain. --BluePlatypus 02:47, 3 February 2006 (UTC)[reply]
You are missing my point as much as possible. It was sold as an anti-terrorism law. It is titles as an anti-terrorism law. Inside the actual act, there are some anti-terrorism topics. However, most of it is not anti-terrorism. It is about Federal investigations of many things besides terrorism. So, we have a situation where everyone is arguing about terrorism and letting all those little bad parts of the Patriot Act get a free ride to avoid crtitical analysis. Using your analogy, it is like the government told us we were getting a car but we got a tractor. Now, everyone keeps complaining that it is a bad car and I am saying that it isn't even much of a car. It is a tractor. Every time I say that, someone says, "Oh, you love this bad car. You are anti-American." I just can't get them to see that while everyone is calling it a car, it is actually a tractor - and a bad tractor at that. But, we can't fix the tractor when everyone is worried about fixing the car we don't have. --Kainaw (talk) 03:23, 3 February 2006 (UTC)[reply]

Apropos en.wikiquote.org/wiki/Patriotism patriotism. DirkvdM 08:37, 3 February 2006 (UTC)[reply]

It seems to me, that whenever someone calls something the "Patriot Act", or something even worse like the "Elimination of Evil Act", you have to be deeply skeptical, just because of the name. It's name implies that anyone who opposes it is not patriotic, or is even treasonous. Any act that needs to use such methods to protect itself from the light of day must include some truly frightening items, which the authors want to keep hidden. StuRat 21:30, 4 February 2006 (UTC)[reply]

Medieval Hierarchy[edit]

Hello, Wikipedia People!

Sorry if I'm not doing this right, but I just want to ask; What was the medieval hierarchy of lords and counts and everything (Is a duke more powerful than a count, or vice versa)? Was the queen even on the hierarchy? Really, really sorry if I have stuffed this up somehow, and I'm pretty sure I have. P.S. Great website. --210.86.106.100 20:58, 2 February 2006 (UTC)[reply]

Check British_peerage#Ranks, which seems to have the list (Starting with the duke, who'se the highest). The Queen's the "Font of honor", but that may or may not answer your question. 68.39.174.238 21:07, 2 February 2006 (UTC)[reply]
It's actually fount of honour. JackofOz 21:51, 2 February 2006 (UTC)[reply]
It's not quite pertinent for the Medieval time period, as it concerns only British titles in a peerage system that has been considerably changed and codified since then. As always, power came from having a big army and productive territories. Sometimes titles corresponded to that reality, and sometimes not. - Nunh-huh 21:25, 2 February 2006 (UTC)[reply]
Crap, missed that one. 68.39.174.238 22:40, 2 February 2006 (UTC)[reply]
  • Seems Nunh-huh was faster at conveying the gist of what I was going to say, but I'll go ahead anyway. See Feudalism. 'lord' is any noble. In Britain it can also be someone who is the owner of a stately home. In general you can't really say "This is what the feudal heirarchy was" because it varied quite a lot from country to country. Most European monarchies had systems of peerage similar to England's after the middle ages, but they were not always rooted in a feodal system like England's. The same title could also mean quite different things, see for instance the shifting meanings of Earl, Voivod and Ban. --BluePlatypus 21:45, 2 February 2006 (UTC)[reply]
  • Indeed, and "big army" was also "a great deal of profitable land or a small amount of extremely profitable land." Thus, a small barony that contained a pilgrimage site might generate a large enough income to support a large army and/or castles (and castles were the equivalent of half an army by themselves), and a large dukedom of swamps and mountains might give one a great deal of money but also an enormous border that needed to be defended against a dozen enemies -- thus reducing the effective strength of the title. Geogre 02:43, 3 February 2006 (UTC)[reply]

Apollo's hooker girlfriend in Battlestar Galatica[edit]

I am watching season 2 episode 14 right now and I am wondering if I missed an episode because Apollo's hooker girlfriend seem to have came out of no where with a long back story attached. Was this just poor writing?

Poor writing? Who knows? She did come out of nowhere, and you're supposed to figure out the back story as it goes. The primary back story was Apollo's dead wife. They've done this before, after all. Quite a few characters appear with an apparent history with the principal characters (the crime lord, e.g.). Geogre 02:45, 3 February 2006 (UTC)[reply]

Was she really a "hooker" (prostitute) on the show ? About the only thing I recall from that show was the cool robot dog "boxy", with pistons for legs. I really wanted one of those when I was a kid ! StuRat 21:13, 4 February 2006 (UTC)[reply]

on the original show, she was certainly an ex-hooker, but they called it something like a "companionator" or something (that's definitely not it, though). More geisha than whore. I can't think of the actual word. - Nunh-huh 01:36, 5 February 2006 (UTC)[reply]
Do you mean Cassiopeia ? She was a "socialator". StuRat 02:29, 8 February 2006 (UTC)[reply]
I think they mean the new show, not the old one--205.188.117.12 14:48, 5 February 2006 (UTC)[reply]
apollo is a women in the new series, so i doubt she has a lesbian relation with a former protitute, tv shows usually don't go that far :) Boneyard 10:46, 7 February 2006 (UTC)[reply]
Apollo is not a woman in either series. You are thinking of Starbuck or Boomer. Argyrios 22:16, 9 February 2006 (UTC)[reply]
Perhaps a gigolo ? StuRat 02:29, 8 February 2006 (UTC)[reply]

Are there any good news websites which can tell me indepth about something[edit]

I'm looking for a good news website which will tell me about something, and describe it in detail. Like tell me what is happeneing in Iraq right now, and tell me what the political parties in iraq are, and who is doing what, and will give me a primer, or an overview. If it's brief, it would be even better. Also, are there any websites, which give brief news, like really, really brief? Thanks Wikipedians!

If you want background context to the news, Wikipedia itself is a pretty good place to start. --Robert Merkel 23:46, 2 February 2006 (UTC)[reply]
Or Wikinews of course. GeeJo (t) (c)  23:48, 2 February 2006 (UTC)[reply]
well, I don't like WikiNews...Honestly, it's articles are really low in volume. Wouldn't rely on it...Any other resources?
You can use www.google.com/ to search for news media by nation, language etc. and thus get at news media written in nations like Iran, Pakistan, Egypt, and other places where the spin will give you many different perspectives.
You can also visit news.google.com/ and note the advanced search function, to get at stories recently in the news in a great diversity of news media.
Other search engines have similar features.

User:AlMac|(talk) 01:41, 3 February 2006 (UTC)[reply]

I'm sure you'll appreciate this. A timeline with clickable really-breifs! news.bbc.co.uk/2/hi/middle_east/4192189.stm -LambaJan 03:28, 3 February 2006 (UTC)[reply]

February 3[edit]

Ford Focus Commercial[edit]

There's been a recent Ford Focus Commercial with some uplifting instrumental music. It includes a "sparkle trail" and a Focus exploding together with sparkles in the middle of a city intersection. Does anyone know the song? --Doctorcherokee 02:09, 3 February 2006 (UTC)[reply]

Search here: adtunes.com/, www.commercialbreaksandbeats.co.uk/index.asp, www.whatsthatcalled.com/, www.songtitle.info/. -LambaJan 03:17, 3 February 2006 (UTC)[reply]

Cunard Cruise Line Specials[edit]

I have inheirited a #2A Brownie Camera Model B with a sticker of some sort on it's side that reads "Cunard Vacation Specials" I am trying to find out the history behind this sticker. The last production date on camera was 1902, and I believe it belonged to my grandfather who immigrated from Europe.Would appreciate any info on the history behind this sticker. Thank you

Ok, now I inherited the Cunard line's most beautiful boat and there was a camera in the inventories. Please send it back 'cause it's missing now. --DLL 20:40, 4 February 2006 (UTC)[reply]

You can see www.pbase.com/shaner/camera_lot_1 a slew of these cameras online. This was just the first site I located when I googled "Brownie Camera Model B". As for the sticker: it might have been a giveaway by the cruise line, but to be sure you'd have to ask a specialist dealer. I found one Googling "cunard vacation specials", but there must be others. --Halcatalyst 18:22, 5 February 2006 (UTC)[reply]

Don't Go Breaking My Heart[edit]

There is a song with the lyrics "don't go breaking my heart, I won't go breaking your heart," and it includes the line "nobody knows it." My Google search tells me that it's by Elton John, and that may very well be the version I'm looking for, but to be sure, does anyone know of any other versions of this? It includes a man and a woman singing. Thanks. Captain Jackson 05:36, 3 February 2006 (UTC)[reply]

  • "Don't Go Breaking My Heart" by Elton John and Kiki Dee. Charted around 1975 or so. phobos.apple.com/WebObjects/MZStore.woa/wa/viewAlbum?id=521523&s=143441 Here's the iTunes link. --Aaron 05:46, 3 February 2006 (UTC)[reply]
  • It was "Don't Go Breaking My Heart" by Elton John and Kiki Dee. It topped the UK charts, the US charts www.allmusic.com/cg/amg.dll?p=amg&token=ADFEAEE4781ED947A57520EFB71670E1FA5CDE2DFE62D9BA5007656392940454880C71ED59E695CFAEF96AB679AFF962A7500DD5C0EB55ECAD1B&searchlink=ELTON|JOHN&uid=CAW050602030326&samples=1&sql=11:tgfe4j570wat~T51 and Australia www.onmc.iinet.net.au/top/1976.htm in 1976. Unfortunately, for Kiki Dee, her career stalled after that. Capitalistroadster 08:39, 3 February 2006 (UTC)[reply]
    • Unfortunately for the rest of us, Elton John's didn't. Grutness...wha? 00:17, 4 February 2006 (UTC)[reply]
      • Surely there must be some Elton John songs you like. Dmn Դմն 18:02, 5 February 2006 (UTC)[reply]
    • Yeah, OK...one or two - but mostly the older ones. Grutness...wha? 00:28, 6 February 2006 (UTC)[reply]
  • If you're looking for other versions, The Mr. T Experience once did a quite funny cover version of the song -- Ferkelparade π 09:02, 3 February 2006 (UTC)[reply]
  • In 1994, Elton released "Don't Go Breaking My Heart" again in the UK, this time with American drag queen RuPaul. It reached #7. Dmn Դմն 18:02, 5 February 2006 (UTC)[reply]

Gun control in countries besides U.S.[edit]

How is gun control treated in countries such as Switzerland and Israel, countries where a large portion of the population serve in the armed forces, or otherwise have easy access to weapons?

  • There are no neutral point of view answers to this question! This www.cato.org/dailys/05-13-00.html article from the conservative Cato institute has some unsourced statements about gun control and crime rates in those two countries. The obviously biased National Rifle Association also has www.nraila.org/Issues/Articles/Read.aspx?ID=72 some statements on the issue. On the other side of the issue, the Coalition for Gun Control www.guncontrol.ca/Content/international.html also discusses Switzerland, and has some informative international charts. Deborah-jl Talk 15:40, 3 February 2006 (UTC)[reply]
That was informative, but it wasn't really what the poster was asking — political opinion from countries such as Switzerland/Israel, rather than about them. GeeJo (t) (c)  16:33, 3 February 2006 (UTC)[reply]
  • I'd say the Swiss do not consider themselves to have 'easy access' to weapons. The Swiss are under strict regulations on how those (military-issued) guns are to be stored, and every bullet must be accounted for. They don't view themselves as 'gun owners', because they don't own the gun, and they can't do what they want with it. In fact, there's very little they can legally do with it. The idea popular in the US, of using those weapons to protect their private property is a rather alien one to most Swiss I know. --BluePlatypus 18:11, 3 February 2006 (UTC)[reply]
  • Here's a brief overview for Switzerland (Disclaimer: I'm a Swiss lawyer, this is not legal advice, it may be completely inaccurate):
  • Gun purchases at stores require a permit that must be issued by police to all adult persons that fulfil certain obvious criteria, e.g. no felony convictions. However, gun ownership itself, and gun purchases among private persons, require no permit at all, though sellers can be prosecuted if they sell guns to obviously unsuitable persons.
  • Various restrictions apply e.g. to weapon type (automatic weapons are banned), and the sale of guns to citizens of certain nations, e.g. from Ex-Yugoslavia is also illegal.
  • The wearing of guns in public requires a special permit which is granted very restrictively; it requires that a special need for personal protection be shown.
  • Military assault rifles are issued to all army members, i.e. (in theory) all male citizens, together with 50 live rounds which must be used only in case of invasion. They are kept at home and can be purchased after completion of the military service for a nominal fee, though modified for semiautomatic operation. It is estimated that between 500'000 and 1 million SIG 550 and SIG 510 assault rifles are stored in private homes.
  • A tightening of gun laws is currently being discussed in the wake of a 2001 gun massacre in the cantonal Parliament of Zug; however, no severe tightening is considered politically feasible.
--Sandstein 22:13, 3 February 2006 (UTC)[reply]

Howard Cosell quote about John Lennon[edit]

Our article on John Lennon has the following quote from Howard Cosell announcing Lennon's death.

"Cosell: This, we have to say it, remember this is just a football game, no matter who wins or loses. An unspeakable tragedy confirmed to us by ABC News in New York City. John Lennon, outside of his apartment building on the West Side of New York City, the most famous perhaps of all the Beatles, shot 5 times in the back, rushed to Roosevelt Hospital, dead…on…arrival."

A reader has contacted the help desk stating that Cosell stated that Lennon has been shot twice. The quote is unsourced. While I have left a message on the John Lennon talk page seeking a source, the last comments were a month ago so I don't know how long it will take to get a response.

I would be grateful if someone had a book or could access a reliable source for Cosell's announcement so that we can be sure about this issue. Thanks for any help you can give. Capitalistroadster 08:04, 3 February 2006 (UTC)[reply]

I found this: www.995themountain.com/guides/aahtml/wav/00jl806.wav. It is the quote and it says that he was shot twice. Lapinmies 08:10, 3 February 2006 (UTC)[reply]

David Letterman Ratings[edit]

Hey someone please help me. I'm getting so frustrated. I want to know how many people in america. watched David letterman with me tonight. I know that a rating system by the name NIELSEN or somewhat close to that is responsible for recording this data. Where can I get this info? Please help me.

Can't help you with the number of people who watched the show tonight, but check out Nielsen ratings. JackofOz 09:48, 3 February 2006 (UTC)[reply]

How to keep illegal immigrants out of the U.S.[edit]

My teacher had the idea of putting bounty's out on all illegal immigrants after giving them one year to get their shit together. If in that one year they can't hold down a good job and filer ther papers and pay their taxes, then they are out of here. Does anyone else have any other good ideas? I really think that immigration is a very large problem. We need to handle this soon. As we sit here hundreds are coming in everyday. When will we get some balls and tell our delegates to tighten down the border. Is it because we live in an elitist society and they are letting them in on purpose.

To you question about any ideas: If you feel strongly on this matter, my suggestion is that you vote for a representative who shares similar values, or run for election yourself. - Akamad 11:25, 3 February 2006 (UTC)[reply]
Your teacher is an idiot and should be fired. What does elitism have to do with this? --Nelson Ricardo 12:01, 3 February 2006 (UTC)[reply]
I hope that you feel better after you get that get that off your chest, but I can't find a question in there except a question of whether anyone else has "good ideas." Since you regard bounty hunting immigrants as a good idea, I'm not sure what you would consider a good idea. Geogre 12:07, 3 February 2006 (UTC)[reply]
I had the idea of putting a bounty on people who misuse apostrophes, but I haven't yet found a candidate for office who shares my feelings. LarryMac 15:51, 3 February 2006 (UTC)[reply]
There aren't enough prison cells or bullets, and the bounty hunters would inevitably have to commit suicide. (It is the #1 punctuation error I see.) I'd rather see bounties on anyone who uses "where's" or "there's" with a plural. Invest in gunpowder stocks now. Geogre 16:06, 3 February 2006 (UTC)[reply]
I found that www.angryflower.com/aposter.html these posters make excellent gifts. GeeJo (t) (c)  16:29, 3 February 2006 (UTC)[reply]
  • Ah, but my defense lawyer think he can get me off, see we've got a medical doctor who's going to testify that I've got Tourettes and can't control my right pinkie-finger, leading to uncontrollable bursts of apostrophe's. See? --BluePlatypus 18:30, 3 February 2006 (UTC)[reply]
Yes. That's a great idea. Now, if only the Native Americans had thought of that first. --BluePlatypus 18:32, 3 February 2006 (UTC)[reply]
I assume that by "bounty" you mean not shooting immigrants but rather (Dog the Bounty Hunter-style) tracking them down and kicking them out. It's certainly an idea, but there are obvious problems: 1) tracking illegal immigrants long enough to put a bounty on them is exceedingly difficult, 2) the cost of bounties would be excessive, and 3) it might raise moral objections among the sort of people who enjoy having such things. I suggest you read the websites of the numerous respectable anti-immigration groups out there. Googling "illegal immigration" will get you started. --George 21:04, 3 February 2006 (UTC)[reply]

One thing to consider is that the US economy is dependant on the cheap labor of illegal immigrants. Without them, food prices would increase substantially, for example. Any method to stop illegal immigrants, such as a wall on the Mexican border, would therefore need to increase legal immigration quotas for unskilled laborers, to compensate. StuRat 04:34, 4 February 2006 (UTC)[reply]

The problem is : you can keep out a would-be immigrant. You can only oust an immigrant since he already made his move (to help you with the best of his capacities). --DLL 20:44, 4 February 2006 (UTC)[reply]
Do you mean we can't keep out a would-be immigrant ? Sure we can, we just need a government that actually wants to stop illegal immigration, not just arrest a token few to get the public off their back. Building a wall with barbed wire, motion detectors, and cameras and sufficient guards to go to the location of any attempted crossing and stop it would not be beyond our capabilities. If the Israelis can do it, we could, too. StuRat 02:59, 5 February 2006 (UTC)[reply]
I guess this huge wall would have to cover the Canadian border and all of the ocean front too. I can't see people who paid big for an ocean view voting for that one... Notinasnaid 10:15, 5 February 2006 (UTC)[reply]
The majority of the illegal immigrants come across the Mexican border, and would have no way to get to Canada to try to cross from there. Since Canada is not contiguous with any country other than the US, it is difficult for illegal immigrants to arrive in Canada from anywhere other than the US. Boatloads of illegal immigrants, such as those which have come from Cuba, are quite easy to detect with radar and thus no wall is needed. Therefore, the wall would be highly effective at stopping illegal immigration. StuRat 15:50, 5 February 2006 (UTC)[reply]
(I meant you can. Is keep out the same as put out ?) --DLL 20:24, 10 February 2006 (UTC)[reply]

Does EHarmony work as successfully as advertised? Is it worth my money?[edit]

I'd like to know how well EHarmony really works, in terms of successfully blossoming relationships with partners found on that site. What are all the positive & negative reports (truthfully), and must I go ahead with it? (I wish I'd make a more specific-sounding question, but I hope it's specific enough.) --Shultz 12:48, 3 February 2006 (UTC)[reply]

It's kind of off topic, you know. You might look at computer dating, if we have an article on it. Since the 1960s, there have been matchmakers that have attempted personality inventories to find matches. The problem is the theory of attraction and the degree of social skill. Do opposites attract? Do complements attract? Is unison better than counterpoint, or is harmony possible with personality inventories? Supposing that harmony (finding someone different enough but not too different) can be determined by inventory and questionnaire, to what degree will past experiences color reactions? To what degree with physical attraction override personality? To what degree will personal psychological dilemmae mess things up? Supposing all of those are no bar, how sociable is a person? Are both going to "feel like it" when the date arrives? Will both talk? Finally, will people lie on their inventories in such a way that the inventory will catch it? Thus, it is as you would expect: there are successes for people without significant baggage who are not socially maladroit, and then there are many more failures. Ultimately, the answer is not in our stars, but our selves, and such things can get you to meet people, but they can't get you to open up nor the other person to take an interest. Geogre 14:04, 3 February 2006 (UTC)[reply]
Well our article on eHarmony talks a bit about this. As it's my major academic interest, I usually keep up with the latest research in intimate relations psychology and I haven't seen anything about it in any of the journals. Neither is any of their research staff shockingly famous in the field, though it's hardly surprising that there aren't a lot of major academics eager to get into the online dating world. I suspect they also have a rather conservative view of relationships, which may not be suitable for everyone: They are explicitly committed to the increasingly less popular notion of lifelong marriage, and they don't match same-sex couples, oddly claiming that they just don't know enough about matching them. (I say this is odd as there's little evidence that same-sex couples are looking for anything different from opposite-sex couples.) I'm no psychometrician, but their matching profile didn't strike me as particularly spectacular: It had some of the expected controls but it's certainly no MMPI.
All that said, I took their personality profile and it was quite consonant with what I already know about myself and my tastes - a good sign. Given the likelihood of increased sincereity among eHarmony members - remember, they're all looking for a scientific, guaranteed lifelong marriage - I think it's likely to be one of the better dating sites out there if that's what you're looking for. I just don't necessarily buy it's grand claims. --George 20:58, 3 February 2006 (UTC)[reply]

Vietnam[edit]

Why Did The US get involved in Vietname —Preceding unsigned comment added by Blair yates (talkcontribs)

Because the United States was allied with France and they were asked to help France in their recolonization of Indochina. France at the time was still trying to rebuild their own country and people were afraid at the time that Soviet Union wanted to rapidly expand their influence into the area. The article First Indochina War deals more in depth with the subject. Dr Debug (Talk) 14:15, 3 February 2006 (UTC)[reply]
See also Domino theory for some of the US's foreign policy thinking that got it to continue after France withdrew. Geogre 16:03, 3 February 2006 (UTC)[reply]
Ho Chi Minh first contacted the US to help him get the French out (he had even drawn up a constitution based on that of the US). When he got no reaction (at all - maybe someone forgot?) he turned elsewhere. And a logical choice was the big neighbour, China. Which happened to be a socialist country. And that did draw the attention of the US, so they found themselves fighting on the 'wrong side' (from an anti-colonialist point of view). I learned this from a BBC documentary. But there's nothing about this in the Vietnam war article. Actually, Ho Chi Minh is only mentioned once, way down, under the column 'Other noteworthy figures'. Considering the size of the article that is a very strange omission.
The US was allied with France through WWII (I suppose that's what Dr Debug means), but so were the Netherlands. So why would the US help France against Vietnam but not the Netherlands against the newly declared Indonesia? Actually, the US stance was quite the opposite; they favoured a very rapid independence of Indonesia. So I don't think it has anything to do with the alliance between the US and France. DirkvdM 09:46, 4 February 2006 (UTC)[reply]
The Indonesian Independance war is a lot earlier, because that was fought between 1945-1949 and the US involvement with the Indochina war didn't start until 1950 - even though it could well have started before that on an even smaller scale. The Indonesian war meant that a considerable amount of the Marshall plan were being spend fighting a war.
The US was partially financing the First Indochina War. In the beginning the fighting was mainly done by French soldiers, but the US was one of the main backers. Ultimately 80% of the war costs as mentioned in the famous speech by Martin Luther King was being handled by the Americans. The amount of actual advisors was limited in the beginning. It wasn't until de Gualle decided to retreat that the US started to increase their increase their advisor to about 16,000. The actual Veitnam war didn't start until the Gulf of Tonkin in 1964 and the financing was the period 1950-1954, so that was 10 years earlier, but after the Indonesian war. Dr Debug (Talk) 18:03, 4 February 2006 (UTC)[reply]

Note that the US was, generally, anti-colonial after WW2, as demonstrated by their opposition to British and French attempts to retain control of the Suez Canal and the US granting of independence to it's colony in the Philippines. However, they considered colonialism better than communism, which was the choice in Viet Nam. StuRat 19:38, 4 February 2006 (UTC)[reply]

There was also a general lack of understanding or memory in high levels of government of some distinctions between people seeking independence, and how the communists were exploiting civil wars. The reality was that the Vietnamese wanted independence, after a long history of colonialism. They accepted whatever help they could get. Had the USA helped them become independent from colonialism, then today Vietnam might be a major nation in the West. But instead, the US chose to help Vietnam continue to be subject to outside rule. The rest is history. User:AlMac|(talk) 23:36, 4 February 2006 (UTC)[reply]
You could also just try Background to the Vietnam War. - BanyanTree 18:26, 7 February 2006 (UTC)[reply]

Can the torture of prisoners be considered a hate crime[edit]

Can the torture of prisoners eb considered a hate crime?

Hate crime is a rather silly phrase meaning 'a crime motivated by dislike of a group to which the victim belongs', so it all depends on the motivation for torturing them. If you are torturing the prisoners because they are black, then yes. Conceivably if you are torturing them because you don't like prisoners, then yes (although it would be an unusual case). If you are torturing them because you dislike their personalities, then no. Markyour words 15:10, 3 February 2006 (UTC)[reply]
Yup, what Mark said. In any case torturing a prisoner is wrong unless you're the CIA where everything they do is illegal.
It depends on whether you're asking a moral question or a legal one. If it's merely a matter of morality, just about any activity that negatively affects another human being or group of human beings can can be spun as a "hate crime". But if you're asking whether or not there's an actual law on the books, that depends entirely on where you live and the circumstances of the so-called torture. --Aaron 15:28, 3 February 2006 (UTC)[reply]
  • If you're torturing someone, I don't think the issue of whether it can be labeled a "hate crime" or not is particularily relevant. --BluePlatypus 18:04, 3 February 2006 (UTC)[reply]


Technically torture against prisoners would not be considered a hate crime in international standards, however it does violate the Geneva Convention of 1948 and its two Protocols. Depending on who was tortured, it can be also considered a crime against humanity and/or ethnic clensing, which is based on the idea of a hate crime. The people actually committing the torture, if against orders, can be tried in their own country, or if with approval by their home government, in the International Criminal Court. If the torturers are acting under the orders of their government, usually it is the government leaders held responsible. For instance, the Nuremburg trials condemned the leaders of the Nazi party responsible for ordering the torture and deaths of Jewish citizens of Eastern Europe, not specifically the soldiers who performed the actions. The Geneva convention does recognize the chain of command within militaries and the severe actions that might be faced if a soldier does not follow orders. --Sam H.

If you're thinking of Guantanamo Bay and the like (it's difficult to answer such a question when it isn't clear what specifically you mean) then there will be some hate involved in as far as a link between the prisoners and the attacks on the pentagon and NY WTC is assumed (and that will often be the case, if only in 'feeling' I presume). But to legally pronounce it a hate crime the torturers would have to admit that. Which would have been the case for the nazi concentration camps (they started off with that purpose, so there would be little point in denying it). But in the case of Guantanamo Bay, they're more likely to come up with anti-terrorism reasons, because that has always been the official version. DirkvdM 09:57, 4 February 2006 (UTC)[reply]

shadow cabinets[edit]

Hi I am a research assistant and i am doing a project on Canadian parliament behaviour. I am looking for a list of members of all Shadow Cabinet from today to 1975. I am looking for the sources you guys used on wikipedia because you're the only who seem to have access to that information on the web. Please let me know how I could get that information at [[[E-mail address removed]]] Thank you!

Please don't list your e-mail address. --Impaciente 18:43, 3 February 2006 (UTC)[reply]

But if anyone has a clue where to find....please respond!

Well, first, the roles of members of shadow cabinets shift around just like those of the official cabinet. So you're looking for a moving target. At any point in time, the lineup of the shadow cabinet would be what it was, but the next day somebody might have died or been sacked.
I'd suggest you first scour all our articles on Canada, following up on the most likely-looking links, which may take you to some government or other records which would have the particulars you need. But frankly, it doesn't sound like an Internet project to me. Hye thee to the library! --Halcatalyst 22:45, 3 February 2006 (UTC)[reply]

The term for a list of an author's works[edit]

This word is on the tip of my tongue, but I'm looking for the specific term of the listing of an author's works. Its similar to a bibliography, except the listing isn't to specify where information was used from. Its not quite an anthology, because it is the work of a single author, nor is it an omnibus, because its only a listing of the works, not all the works themselves.

For instance, the "(insert term here)" for Jane Austen would be: Emma Lady Susan Mansfield Park Northanger Abbey Persuasion Pride and Prejudice Sense and Sensibility

I'd still call it a bibliography. Notinasnaid 18:54, 3 February 2006 (UTC)[reply]
There are other terms, but I too think bibliography to be best. Notably, Merriam-Webster gives as the second definition for the word: "a list of the works of a [singular] author". Joe 19:06, 3 February 2006 (UTC)[reply]
Oeuvre? Oddly, it redirects to Opus number, but it shouldn't. The bibliography is the list, and the oeuvre is what is listed. Markyour words 19:04, 3 February 2006 (UTC)[reply]
Corpus? Although I think that's more 'body of work'. --BluePlatypus 20:07, 3 February 2006 (UTC)[reply]

Bibliography is a field of study. An enumerative list of books and other publications is often called a bibliography. But your example of a list of Jane Austen's novels is simply a handlist, if you want a term commonly used by bibliographers. If you add publication information to the list, it becomes a bibliography (maybe a "working bibliography"). As you remarked, Anthology and Omnibus are terms far removed from what you're looking for. --Halcatalyst 22:36, 3 February 2006 (UTC)[reply]

Wikipedia itself uses "bibliography" as a section title in articles on writers, and listed below you'll usually just find the names of the works but no publication details. See Charles Dickens, for example. This is an accepted meaning of "bibliography". --- Well, it contains years of publication. But I assumed that by "publication information" you meant more than just dates. Maybe I assume too much. JackofOz 23:19, 3 February 2006 (UTC)[reply]
Well, I wouldn't argue with that. The point is, bibliography (in its etymology) means writing about books. So I guess you could call a (written) list of books a bibliography. Hey, it's the weekend! Let's let our hair down. --Halcatalyst 02:41, 4 February 2006 (UTC)[reply]
Truth knows no weekend. (Hey, that's good, I think I'll add it to my Favourite Quotes). JackofOz 02:58, 4 February 2006 (UTC)[reply]
Truth? --Halcatalyst 04:53, 4 February 2006 (UTC)[reply]
Seems truthiness really is stranger than fiction. And more all-pervasive. JackofOz 11:12, 4 February 2006 (UTC)[reply]
I'd call it a "list of publications". – b_jonas 12:11, 4 February 2006 (UTC)[reply]
Complete works (only after the author's death). --DLL 20:52, 4 February 2006 (UTC)[reply]
So, if you compile a list of a living author's works, and want to be able to call it "complete", it is necessary to kill the author. StuRat 23:54, 4 February 2006 (UTC)[reply]
We noticed you had a short Wiki-break recently, StuRat. Perhaps you came back from your holiday a little too soon, and your brain has not been fully rested. JackofOz 00:02, 5 February 2006 (UTC)[reply]
I was searching for a site that would appreciate my sense of humor. Finding none, I was forced to return here. StuRat 02:51, 5 February 2006 (UTC)[reply]
Yes, that would be quite some challenge. But don't worry, we practise Wiki-tolerance here. JackofOz 04:59, 5 February 2006 (UTC)[reply]

Ancient Larson Cartoon[edit]

Around a decade ago (give or take a few years; I was, like, eight, so I don't remember), I watched an animated cartoon on TV (I don't remember what channel, but it was long before cable) based around Gary Larson's Far Side comics. It was an hour or two long, and drawn in the same style, possibly even by Larson himself. I don't remember much in the way of details, but if you saw it, you know you did. We didn't know anything about it (name, who made it, etc.), and didn't know how to get hold of a copy, but I figured I'd eventually see it again or find some reference to it. Well, I remembered it today (a small miracle in itself), and finally have the ability to look for it. There's no reference to it on Google, but I know the Reference Desk won't fail me. Black Carrot 20:03, 3 February 2006 (UTC)[reply]

  • www.imdb.com/title/tt0109873/ Tales from the Far Side and www.imdb.com/title/tt0120270/ Tales from the Far Side II. --BluePlatypus 20:06, 3 February 2006 (UTC)[reply]
I love you. Black Carrot 22:30, 3 February 2006 (UTC)[reply]

Gilbert Stewart[edit]

I have a Gilbert Stewart portrait, verified as a G.S. I have been told the portrait is of Sir Edmund Sexton. I would like to find out who this genleman was and, more importantly, verify the name of the individual. How can I do this on the net? —Preceding unsigned comment added by 69.19.14.26 (talkcontribs)

I think you probably mean Gilbert Stuart, né Stewart. As the article states, he spent a dozen or so years in England and could have painted Sir Edmund Sexton. Google and Yahoo searches on "Sir Edmund Sexton" turned up nothing. Your best bet might be the Oxford Dictionary of National Biography or another such reference in a good library. --Halcatalyst 22:15, 3 February 2006 (UTC)[reply]

Is this "Edmund Sexton Pery", later "Viscount Pery"? Try Googling those names. --Shantavira 18:06, 4 February 2006 (UTC)[reply]
From www.thepeerage.com/p3673.htm thePeerage.com: "Edmond Sexton Pery (M) #55944 Last Edited=10 May 2003. Edmond Sexton Pery was also known as Viscount Pery." So he would have been addressed as "Lord" rather than "Sir." --Halcatalyst 21:09, 4 February 2006 (UTC)[reply]
Well, there was more than one "Edmond Sext[o/e]n Pery". Edmond Sexten Pery (1719-1805) was Speaker of the Irish House of Commons 1771-85; afterwards (1785-1806) Viscount Pery. So whether he was addressed as Lord depends on when he was being addressed. There was another Edmond Sexten Pery (1797-1860), third son of Edmond Henry Pery, 2nd Baron Blentworth of Mallow, created Viscount Limerick on 29 Dec 1800 and created Earl of Limerick 22 Jan 1803. I suspect the Speaker was the more likely, both by prominence and chronology, to have had his portrait done by Stuart. But if the name is simply "Edmond Sexten", he was father of Susannah Sexten, herself heiress of her nephew, another Edmond Sexten, and wife of Edmund Pery, they being parents of Edmund Pery who married Dymphna Stackpole, the latter two being parents of Rev. Stackpole Pery, father of Edmond Sexten Pery (1719-1805), the Speaker. So it may have been a Sexten and not a Pery portrait.- Nunh-huh 00:21, 5 February 2006 (UTC)[reply]

February 4[edit]

The message/ meaning of the song "Hitler in my Heart"[edit]

I would like to have an explanation regarding the meanings of the song "Hitler in my Heart" by Antony and the Johnsons - It is not clear to me: He (Antony) loves him (Hitler) ? He hates him ?He is being cynical about him ?? Also - Is there a site in which there's a discussion/review regarding songs/lyrics - i.e. meanings/symbols etc.? Thanks RONNIE

  • Well, I looked at the lyrics here www.alwaysontherun.net/antony.htm, and I'm under the impression that the song really isn't so much about Hitler. I think that the song is about coming to terms with your regrets and inadequecies; accepting them as learning opportunities. Hitler is a metaphor for all of the unkind things he found in his heart when he was looking for kindness. He resigned to be honest and accept it for what it is and therefore called it Hitler. Then he looked deeper at the consequences of unkindness and became very upset at all of the pain. This is represented by the extended metaphor of the Holocaust. He then looked even deeper and found the silver-lining, so to speak, which is the corpses feeding the flowers. It's really quite a moving song. I think I'd like to hear it sometime. -LambaJan 02:55, 4 February 2006 (UTC)[reply]
  • I've never heard of this group, but I found www.wnyu.nyu.edu/static/?a=antony this interview with "Antony" where he sort of talks about the meaning of the song. In any case, a simple www.google.com/search?q=%22Hitler%20in%20my%20Heart%22&sourceid=mozilla2&ie=utf-8&oe=utf-8 Google search on "Hitler in My Heart" pulls up a number of hits that contain reviews of the song and/or the album it comes from. --Aaron 03:00, 4 February 2006 (UTC)[reply]

Can you give any examples of any Hate Crimes[edit]

If they are famous ones, that's good, but even otherwise, it would be good.

Can you please be a little more specific. A huge number of crimes have been motivated by hate in one way or another. JackofOz 03:16, 4 February 2006 (UTC)[reply]
The killings of Emmett Till and Matthew Shepard. Zafiroblue05 03:31, 4 February 2006 (UTC)[reply]
Well, both those would qualify under most definitions, but neither resulted in a prosecution under a hate crime law. - Nunh-huh 04:40, 4 February 2006 (UTC)[reply]
Too much love for great men leads to committing crimes. See John Lennon. --DLL 13:04, 4 February 2006 (UTC)[reply]
The Holocaust. Markyour words 14:25, 4 February 2006 (UTC)[reply]
See Category:Hate_crimes. David Sneek 17:06, 4 February 2006 (UTC)[reply]

How about the current burning of Southern churches in the US ? www.registerguard.com/news/2006/02/04/a4.nat.churchfires.0204.p1.php?section=nation_world StuRat 23:11, 4 February 2006 (UTC)[reply]

Reading the article, it is not suspected that the burnings are hate crimes.--192.160.130.12 14:37, 7 February 2006 (UTC)[reply]
  • Is that a real newspaper? It looks like it completly ripped off the formatting from the NYT online--205.188.117.12 14:45, 5 February 2006 (UTC)[reply]
Sure, it's a real paper in Eugene, Oregon, USA. It may very well use the same font/typeface as another paper. Those can't be copyrighted, as far as I know. StuRat 23:56, 5 February 2006 (UTC)[reply]

Crime, Law and Justice[edit]

Hello i was justing wondering if anyone can help me with a current criminal issue in Australia concerning Riots and the effectiveness of the current law in achieving justice with the issue, and the change which could be made to achieve greater justice with arguments for and against the change. Also the effectiveness of the legal mechanisms by which the change could be bought.

Or please give me some references

Thank You Your help will be appreciated

I think you might want to check google news, or other search engine news, for background on events leading up to the riots. In my opinion, the problem cannot easily be resolved by legislation or justice system, since it is rooted in attitudes of the people:
  • The original aboriginal people of the country.
  • The settlers from Britain who were sent there as to a penal colony from a system in Britain that at the time was far from contemporary standards of justice.
  • The settlers from Britain who were unwanted orphans, easy to get rid of that way, such as in the WW II era.
  • The settlers from other ethnic backgrounds who have cultural standards that clash, a problem with many nations, more so in France for example.
  • A failure of many nations to help with the assimilarion of their people.

User:AlMac|(talk) 10:00, 6 February 2006 (UTC)[reply]

Who is the Author of "Turned"[edit]

I am supposed to do an essay on Turned but i forgot my copy at school and wanted to find a copy on the web. But i forgot the author! would you know who it is? The plot is about the Marroners that hire a young girl to work for them and then the husband seduces the girl, gets her pregnant, the wife finds out and takes the young servant girl and herself away from the man. Thank you, --86.142.205.246 09:42, 4 February 2006 (UTC)[reply]

A quick Google search of the words "Marroner" and "Turned" found that it's a short story by Charlotte Perkins Gillman. Grutness...wha? 09:52, 4 February 2006 (UTC)[reply]

a copy of "Turned"[edit]

I have found the author she is Charlotte Perkins Gliman, But i have desperately tried finding a copy of the story and i cannot. Please would you be so kind as to know obtain it or find a website where it is. Thank you so much --86.142.205.246 10:01, 4 February 2006 (UTC)[reply]

Charlotte Perkins Gilman (note the spelling). Here's the bibliographical information:

  • "Turned." Forerunner 2:9 (1911): 227-32. "The Yellow Wall-Paper" and Other Stories. Ed. Robert Shulman. Oxford: Oxford UP, 1995. 182-191.

Here's a www.georgetown.edu/faculty/bassr/heath/syllabuild/iguide/gilman.html short teacher's guide which includes "Turned." Sorry, I couldn't find the text online. Why do you need the text online? --Halcatalyst 19:45, 4 February 2006 (UTC)[reply]

Film discussion[edit]

does anyone know of any discussion groups or mailing lists that i can subscribe and be a part of, on the topic of film and cinema.

i seem to only be able to find groups on movie reviews or on something within film more specific, but i'd like a group which is focused on the general subject as in film criticism, filmmaking, film theory etc..

thanks in advance to everyone who answers! gelo 11:21, 4 February 2006 (UTC)[reply]

  • Perhaps the forums of www.studentfilms.com are a good place to start. - Mgm|(talk) 17:10, 4 February 2006 (UTC)[reply]
  • There would be listservs devoted to film discussion. Perhaps Google academic would lead you to some of those. As for signing up, you'd have to query them individually. You need to be prepared to decide whether you're interested in film theory, film comment/criticism, or technical film subjects. Neither you nor the list members will be overly happy if you choose incorrectly. For a web-based approach, look at some of the film journals that are to your liking and investigate their web presence. Are you a Film Threat person or a Cinaeste person? Look at their web sites and see what goes on, if they have discussion areas, etc. These are all going to be far, far more demotic than a list server, but they will be somewhat limited to people who read that kind of magazine, so that might be some discrimination and might lead away from discussions of the Brangelina Baby Bump. Geogre 23:56, 4 February 2006 (UTC)[reply]

Authors name?[edit]

There is a famous author who's mother was buried alive .I believe he was British. She was intombed not actually buried.I can't remember his name. This is interesting of course because it happened before his birth.

Thank You ,Obscureious


From a nice Google excursion :
  • "Death/buried alive

Whether true or not, the fear of it happening was real. In the nineteenth century numerous patents were given in England and the US for devices by which the elect could decline the nomination, either by signalling to people on the outside (bells, flags, electric lights activated from inside the coffin) or escaping on their own. One very common device was a bell mounted on the coffin lid, with a string running down inside the coffin which the corpse could pull to call for room service. A bell like this figures at one point in Nicholas Meyer's book "The Great Train Robbery" (a light-fiction work with great period details, BTW)." tafkac.org/death/buried_alive.html --DLL 13:02, 4 February 2006 (UTC)[reply]

Otherwise, I guess your question is something like a test. --Halcatalyst 20:55, 4 February 2006 (UTC)[reply]
This seems like a legitimate question to me (even if it turns out that no such person exists). Ardric47 04:31, 8 February 2006 (UTC)[reply]
No, no, it's ringing a bell (not the coffin kind). I don't think it was an author, I'm thinking about a famous roman or something... Or maybe you're thinking of that story by Poe. Or maybe it's Robert E. Lee, who www.thewindjammer.com/nefarious/html/diedonce1.html apparently was born after his mother's 'death'. But... oh.. goshdarnit, it's on the tip of my tongue! If I remember it, I'll tell you... СПУТНИКССС Р 04:50, 5 February 2006 (UTC)[reply]

18th century public art exhibitions in England[edit]

Hello, I was interested to know how the visual arts were effected due to the advent of public exhibitions in England? Everything I read says that it was a time that changed visual arts, but I cannot find a specific example. Thank you--Gus M.

Public exhibits were not very common, so far as I know. Art was generally in private homes, but these had exhibits, and the king/queen would also open to the public. What changed everything in England, at least up to mid-century, was the salon culture. Wealthy patrons began to collect art as art, instead of collecting pictures of grandpa or the estate. Not only did public interest in portraiture increase (Joshua Reynolds, e.g.), but a public taste began to emerge as salons began to be more accessible. Another huge effect was the increase in circulating libraries. These were in operation fairly early (everyone has a different idea of when, exactly), but by mid-century they were all over the place, and this takes me to the other huge thing: printing. It's true that the printing press was centuries earlier, but printing presses didn't proliferate in that protestant nation (and that's important in terms of censorship and the pro-printing ethic) until the late 17th c. Once printing presses are all over the place, wood cut, lithography, and plates hit a high point, and books on art as well as books doing lithography at a high quality were showing up both in the public and the library sphere. I don't know when or how many public exhibits the nobles did, and I hope someone else will know that. Geogre 13:45, 4 February 2006 (UTC)[reply]

Cashiers Checks vs. Money Orders[edit]

I am trying to determine if a bank can place a stop payment on either of these. Your article indicates that a bank may place a stop payment on a Money Order but states nothing about whether a bank can place a stop payment on a cashier's check. I appreciate your help.

Derrick

In the US, a cashier's check is already currency, the bank's currency, against the bank's credit, so it cannot be stopped to my knowledge. It is an old "bearer note," meaning that the banks themselves treat it as better than cash. Theoretically, they could fail to honor it, but no receiving bank has an incentive to do that, and the issuing bank considers it already spent money. Geogre 17:28, 4 February 2006 (UTC)[reply]
On the other hand, a money order may be issued by a non-bank. The bank may not be able to check on the credit worthiness of the issuer, so it cashes the money order at its own risk. It may choose not to take that risk. --Halcatalyst 18:40, 4 February 2006 (UTC)[reply]

Mahamrityunjaya Mantra[edit]

Doesn't seem to be anything here.

Here is the google search for it.

maps.google.ca/search?q=Mahamrityunjaya+Mantra&btnG=Search+the+Web

Request it at Wikipedia:WikiProject Hinduism or write it yourself. deeptrivia (talk) 15:10, 4 February 2006 (UTC)[reply]

fraternal jacket patch[edit]

Do you have any idea what fraternal organization used this patch? I'd like to get it to the right place. Thanks - J. Mangels, museum curator--204.212.230.7 15:24, 4 February 2006 (UTC) [email protected][reply]

I'm afraid you forgot to show us the patch. --Halcatalyst 20:44, 4 February 2006 (UTC)[reply]

Dutch politics and Afghanistan[edit]

Since there is no Dutch reference desk (because all the Dutch hang out here?), I'll ask this here. I just read in the book 'Politieke Basisinformatie' that the Dutch government makes laws together with the 'Staten Generaal' (parliament), but that the government (the assorted ministers) does several things on its own, which parliament can only criticise (unless they decide to cause the government to fall), namely international treaties, defense and the appointment of mayors, (provincial) governors and judges. But there was a fierce debate over the sending of troops to Afghanistan, which hung on the approval of parliament. But that was a defense thing (well, not if you take it literally, but that's a different matter), so according to the book, parliament could not have stopped it. Is that true, and if so, what was the fuss about? DirkvdM 16:15, 4 February 2006 (UTC)[reply]

Parliament could have stopped it by dismissing the cabinet. But the fuss was about something else: the cabinet members wanted parliament to take the decision, and vice versa, because nobody wants to be responsible when things go wrong. It was quite funny, I thought. David Sneek 17:03, 4 February 2006 (UTC)[reply]
If that is so, then the presentation in the media (and by the politicians) has been lousy. There was a lot of fuss about D66 making the cabinet fall (which was nonsense because the mission is outside the 'government accord' (regeerakkoord) and thus had nothing to do with the cabinet). But, apparently, the real issue was if PvdA (together with 'small left') would make the cabinet fall. Because that was the only way they could have stopped it. There's a lot of complaining about alienation between civilians and the government, but that's not surprising if elementary stuff like this is not explained properly. DirkvdM 08:40, 5 February 2006 (UTC)[reply]
I'm sorry; my first sentence above should have read: If the government had simply decided to send troops to Afghanistan, then the only way parliament could have stopped it was by dismissing the cabinet. But the cabinet members never made that decision, they moved the problem to parliament instead. David Sneek 09:22, 5 February 2006 (UTC)[reply]
Ok, the original issue seems to be out of the way now. But I thought the cabinet had made a decision (the fuss over the word 'besluit'). As I understand it, all ministers had agreed (including the D66 ones), only parliament disagreed. But they have no say in defence matters, so why was it moved to parliament? DirkvdM 09:50, 6 February 2006 (UTC)[reply]
This is where you're mistaken, I think. Even though all cabinet members supported the mission, they did not take the official decision to go through with it until after www.volkskrant.nl/den_haag/article208190.ece it was given the green light by parliament. The problem was moved to parliament because of the bizar position of D66; their ministers supported the mission, but their parliamentary fraction was opposed and threatened to stop backing the cabinet if it decided to send troops. David Sneek 11:09, 6 February 2006 (UTC)[reply]
But if all ministers agree with a decision is a decision then not automatically taken by the cabinet? Or do you mean that 'Balkenende' decided that a mission like this needs approval of parliament? Where 'Balkenende' here means just the cabinet, because I don't think the guy himself has that much power (his vote in the cabinet is equal to that of the other ministers). But why would he seek such approval for this specific mission if the law says he doesn't need it? And why wasn't any of this important stuff covered in the media in stead of regurgitating the same stupid things over and over again? And is this a question or a complaint? :) DirkvdM 08:09, 7 February 2006 (UTC)[reply]
im not sure if the tweede kamer needs to give it's ok for such a mission, but very soon the cabinet already said they wanted to have it because the mission should have a broad political backing. and if they could do decide on it themselves it wouldn't be wise because the tweede kamer wouldn't accept that. i didn't find the news reporting on it that bad, it was clear enough for me. Boneyard 10:56, 7 February 2006 (UTC)[reply]

jazz music[edit]

I'm having trouble finding anything on this question. The question is : Does our society appreciate Jazz?

Sounds a bit like a homework question.. But to get you going: Whose society? Define 'appreciate'? As in "popular", as in "held in high esteem", or perhaps as in "sensitive to"? Depending on what you mean, think of some way to measure that, e.g. "How many jazz recordings are sold?", "What is the public perception of jazz musicians? Are they regarded more as artists or as simple entertainers, or both?", "What is the perception of jazz audiences? Are they viewed as common folk, an elite, or anybody? Young or old? What demographic do people think of?". Then decide if you want to compare it to other societies, or if you want to compare time periods? --BluePlatypus 22:14, 4 February 2006 (UTC)[reply]
Well, that's a tough question. Assuming you're asking about the US, jazz records make up about 3% of total record sales, far less than the big genres of rock and hip hop and so on, but about the same as classical. (cource: RIAA website) So our society votes with its money for Coltrane as often as for Chopin. Smooth jazz radio stations are also quite popular in some demographics, and college radio plays more than its share of jazz. Now whether that constitutes a proper appreciation is a rather subjective question. I leave that up to you. --George 22:19, 4 February 2006 (UTC)[reply]
What reason is there to assume (s)he is talking about the USA? Anyway, the question says "our society", not "our country". DirkvdM 08:45, 5 February 2006 (UTC)[reply]

Muhammed's Arm Bone in Cordoba Mosque[edit]

I have just returned from a trip to the wonderful former Mosque of Cordoba. A travel guide website (www.sacred-destinations.com/spain/cordoba-mezquita.htm) states that:

"Under Abd ar-Rahman II (822-52), the Mezquita held an original copy of the Koran {Qur'an} and an arm bone of the prophet Mohammed. It became a Muslim pilgrimage site second only to Mecca in importance."

This is the only reference to such a phenomenon that I have found despite searches on Google.

Can anyone shed any light on this for me? __Carlyt 22:22, 4 February 2006 (UTC)[reply]

  • Well, supposedly Muhammed is buried in the Masjid al-Nabawi in Medina. By any rate Medina is certainly the #2 pilgrimage site after Mecca. It sounds a lot like an Islamic version of the old medieval Christian tradition of having relics (of dubious authenticity) to attract pilgrims. And of course, there's no 'original' copy of the Koran, since it was first communicated orally. Some muslims believe the original book is in heaven. Although in this case maybe they mean one of Uthman's copies. I'd be doutful of the authenticity of the claims, although it is quite possible that it was believed at one time, or at that time.--BluePlatypus 23:34, 4 February 2006 (UTC)[reply]
    • Medina is now the second major pilgrimage site, but remember that much of Spain was moslem for some of its history, and it was indeed a major region for pilgrimage at that time. Not sure whether Cordoba was its holiest site, though. Grutness...wha? 23:58, 4 February 2006 (UTC)[reply]

US Department of State domestic responsibilities[edit]

When Richard Nixon resigned the US presidency, the letter of resignation was addressed to Henry Kissinger, the Secretary of State. Why? Does the Department of State have, in addition to responsibility for foreign policy, some kind of official role in maintaining the government or the executive branch?

HD67.42.183.19 22:52, 4 February 2006 (UTC)[reply]

  • It's because the law says that that's who the president resigns to. www.law.cornell.edu/uscode/html/uscode03/usc_sec_03_00000020----000-.html US Code Title 3, chapter 1, §20. As for why it's the Secretary of State and not say, the Speaker of the House (which would be my first guess) is a good question I don't have the answer for. --BluePlatypus 23:23, 4 February 2006 (UTC)[reply]
The secretary of state handles some government house-keeping functions, like receiving Electoral College votes and, I think, confirming the ratification of constitutional amendments. That's why she's called the secretary of state and not the secretary of foreign affairs. -- Mwalcoff 00:35, 5 February 2006 (UTC)[reply]

The reason for this is historical. In England during the 1700's and prior,the the chief generallist minister was called the secretary of state. He ran everything not under another department, (such as the treasury or the admiralty). This happened to include foriegn affairs, as time past the their became more than one secretary of state, gradually the also became specialized. When the U.S came into existance, the state department and its secretary was appointed to handle all affairs not under another department, such as war and Treasury. Because of federalism, most of this agency of defaults actions where in the realm of foreign affairs. As the federal government increased the scope of its powers and responsibilites, instead of farming these powers and responsiblities out they where given to new agencies. As a result the state department became seen as a agency of foriegn affairs.


It is the Presidential Succession Act of 1792 that decided the Secretary of State would be the one to take any presidential resignations. www.senate.gov/artandhistory/history/minute/Presidential_Succession.htm This page is a good description of the political infighting that led to that Act being passed. --Aaron 01:56, 5 February 2006 (UTC)[reply]
I don't think the above account is quite accurate. I believe Congress originally named the position "secretary of foreign affairs," then renamed it after adding the domestic duties shortly thereafter. -- Mwalcoff 02:03, 5 February 2006 (UTC)[reply]

Thanks-this is all helpful. HD

Consider the job responsibilities of the various state Secretaries of State. They're responsible for things like voter registration, public records, etc. It seems to be a generic function of a Secretary of State. User:Zoe|(talk) 00:17, 6 February 2006 (UTC)[reply]

Replying to Halcatalyst's Question[edit]

I really need that text, either online or written for my essay. i need quotes to sustain my answers, but it cannot be done without the text! thank you though --86.142.47.93 23:02, 4 February 2006 (UTC)[reply]

You are not going to have much luck getting what you need, unless you post your question in context of whatever thread you are referring to. User:AlMac|(talk) 23:39, 4 February 2006 (UTC)[reply]
I believe this to be in reference to the Charlotte Perkins Gilman text "Turned", about which questions posed above. 64.233.179.104/search?q=cache:rbCW28IJe-EJ:ee.1asphost.com/shortstoryclassics/gilmanturned.html+Charlotte+Perkins+Gilman+Turned&hl=en&gl=us&ct=clnk&cd=2 This link appears to provide a cached copy of the short story (the site hosting the page appears to be down); I'm not certain that the story is complete, inasmuch as I haven't a hard copy with which to compare it and don't feel in any case like reading it to essay a determination about its completeness. Nevertheless, I imagine that it's better than nothing (the teacher might not notice that all of your quotes came from the short story's first page...). Joe 23:46, 4 February 2006 (UTC)[reply]
By Geogre, I think you've got it! I spent over an hour yesterday afternoon without success, trying to locate the text. I even used the same key words. How you do dat? --Halcatalyst 17:34, 5 February 2006 (UTC)[reply]

February 5[edit]

What on Earth is going on?[edit]

What do you think on Earth is going on?There has been alot of suffering,problems and terrible events in the world after 1914, the beginning of World War I(and not just during that time). Great wars,the threat of nuclear weapons,famines,natural disasters(especially earthquakes), disease,crime,immorality,terrorism,pollution,unemployment, and fear/insecurity,etc are all the terrible things in this world during the past century.Our free world is becoming more and more terrible and dangerous!Nobody, not even the rich,powerful, and famous, have security and immunity from those bad events. What can we do about all those bad events going on in the world?How can we protect ourselves from those bad events?How can we stop all those people from making our world more and more terrible and dangerous?

If you don't know much about this, you should do some research about it!One way to find out more about this is by reading Matthew 24,Mark 13, and Luke 17:20-37 of the Bible. Bowei 00:21, 5 February 2006 (UTC)[reply]

That is not a question. Please take your apocalyptic rants elsewhere. In the meantime, perhaps I can direct you to Timeline of unfulfilled Christian Prophecy? It's incomplete. --BluePlatypus 00:40, 5 February 2006 (UTC)[reply]
That link goes to a blank page for me, anyone else having the same prob ? StuRat 04:30, 5 February 2006 (UTC)[reply]
Very well put, Blue. Joe 00:55, 5 February 2006 (UTC)[reply]

Actually, I am asking questions.I'm asking about the present state of the world.Bowei 01:12, 5 February 2006 (UTC)[reply]

While it is true that you posed questions, it is equally true that you answered them in the sentences suivantes. You advise us, even, that we ought to read certain chapters and verses of the Bible in order that we should better be able to answer the question. It stands to reason that, if, indeed, you were posing questions to which you were genuinely looking to answers (to be provided by one who reads this page), you wouldn't be capable of suggesting the very resources one ought to regard in order to provide those answers (in the fashion you would deem appropriate). Finally, though I surely think myself to be capable of properly adjudging the state of the world, I suspect that the answers I might provide you would be very different from those you seek, and I expect that the same would hold true for the answers of many others here. Joe 01:18, 5 February 2006 (UTC)[reply]
  1. What can we do about all those bad events going on in the world - Volunteer, or if you can't, make charitable donations to humanitarian organisations.
  2. How can we protect ourselves from those bad events? - Take an interest in politics, and support candidates whom you believe to be most likely to keep a high standard of living for people in your area.
  3. How can we stop all those people from making our world more and more terrible and dangerous? - Get involved, write to your local Member of Parliament/Congressman/etc and give them your opinions. If you strongly disagree with a policy, look to activism, or any of the above suggestions. Ultimately, if you want something to happen, you may have to do something about it. While the Bible gives fairly good moral guidelines, reading alone won't make a difference in the outside world. GeeJo (t) (c)  01:22, 5 February 2006 (UTC)[reply]
  • He needs to study his church history a bit. Folks have been sure that this is the End since, well, 50 AD. Things are certainly bad, and they seem worse, but they've always been bad and seemed worse. Jesus said that when you hear someone say that the end is coming and that Christ has returned, don't bother: live as if it were the end by living piously. Worry about yourself. Walking around with a sandwich board isn't really going to be very pro(fi/phe)table. Geogre 02:48, 5 February 2006 (UTC)[reply]

How badly things are going will depend on where you are, but let me use the US as an example. The Iraq and Afghan wars are tiny compared with Viet Nam, Korea, WW2, or WW1. The worst war in US history, measured as a percentage of the population killed, percentage of homes destroyed, etc., by far would be the US Civil War. The death toll from the recent hurricanes is far less, than, say, the 1906 San Francisco earthquake or the Galveston Hurricane of 1900. As for diseases, there are new diseases, like AIDS, but many old deadly diseases, like polio, whooping cough, scarlet fever, malaria, tuberculosis, leprosy, etc., have been virtually eliminated. On the whole, I would say things are getting better, not worse, at least in the US. StuRat 04:38, 5 February 2006 (UTC)[reply]

I really mean it.If you haven't read those parts of the Bible, then I think you should.Decide for yourself whether it is talking about the present world.Bowei 05:52, 5 February 2006 (UTC)[reply]

Well I really read it, and not for the first time - I'm not half bad at Koine Greek, you know. As to whether I think it is talking about the present world, certainly not. In Mar 13:29-30 Jesus says "So ye in like manner, when ye shall see these things come to pass, know that it is nigh, even at the doors. Verily I say unto you, that this generation shall not pass, till all these things be done." He is plainly saying that the horrors he has just described will occur within the generation of those he is addressing. The Greek γενεά could mean "age" more generally, but almost certainly doesn't in this case. It's uniformly translated "generation" in English Bibles; the Vulgate renders it "generatio"; the 1912 Luther Bible renders it "Geschlecht"; the 1960 Reina-Valera renders it "generación." (Some might say the latter is as ambiguous as the English, but I don't find it or the English ambiguous.) Thus Jesus cannot be talking about our present world. And let me hasten to add that in this case his prediction didn't come true.
Let me also, if I may, add a personal note. Those of us who volunteer at the Ref Desk do so because we fancy ourselves knowledgable people who can, together, usually answer even the most obscure of questions. The words of Jesus, however, are in no way obscure - they're among the most important words in Western history, however you measure that - and I'd wager that several of us here know quite a bit more about the Bible and Christianity than you do. That makes this a very poor place to proselytize, ignoring the fact that it's against the rules. So please save us both the trouble and don't do it. --George 06:40, 5 February 2006 (UTC)[reply]
Was 1914 an important year for your god that would make you specify the time after that year? If I'm not mistaken, and I doubt very highly that I am, there was a lot of suffering, famine, natural disasters, crime, & etc. (dogs and cats living together) before 1914 as well. Dismas|(talk) 06:57, 5 February 2006 (UTC)[reply]
1914 is a very important year for framing the question suitably to reach the presupposed conclusion. You can't just pick any old year if it doesn't support your position! Imagine what'd happened if they chose 1950. They'd have to explain away the end of the cold war, fall of communism, the dissolution of colonial empires, the spread of democracy, the global increase in wealth, health and welfare and a half-century of peace between European states, something which has not happened since they came into being (or before that). Not to mention the vast steps in literacy, education and science. No, 1914 is a great starting point, conveniently located at the dawn of modern warfare. The only bad thing is that it's too obvious. I'd be smarter and pick 1900. In truth though, it wouldn't matter what year I pick, because I still have the weapon of Selection bias. I get to decide what gets included and what's not, and I don't have to compare to a larger timeframe if I don't want to. --BluePlatypus 00:36, 6 February 2006 (UTC)[reply]

Was the world ever really safe in the first place? I don't think so, actually I think that it's a lot safer now... a LOT safer, than it was before '1914' (Idunno why u put that specific year as a division). and u say that the rich aren't safe either....well, where they ever really 'safe'? I think regular people of this time live far better than say a king in the middle ages...you sound like my father, you are probably a neo-luddite (or however it is spelled) and a Christian fundamentalist trying to drive people to accept something you say using fear and not reasonable judgement...

Now I feel kind of sorry for saying things this way to toy since I looked at your user page and you are just a kid, but I want to give you a little piece of advice, since I'm from a x-tian (catholic-obsessive) family myself, USE YOUR REASON please, or else you will be in serious trouble...don't let anyone make you believe something just because they threathen you with things like hell if you don't, that's evil, and I know your parents raised you that way, and it was a struggle for me too, but even parents can be wrong because they are human beings too.

and maybe you are right when you say the world is crazy... I think it is but we shouldn't give up our reason because of that...it's like people with a christian mentality called crazy people who asked themselves questions (and made the world a better place even for x-tians, and that drives me mad! because they aren't even thankful!)and...i think this is off topic but this just made me mad...I think creativity will always be called 'insane' at the beggining, and then even chistians ( or musslims or whatever, meaning all those censoring life hating biggots who criticize technology but own every gadget they want)will reap the bennefits! it's this way all over the world! I'd give u examples but thinking about them would just make me mad...--Cosmic girl 17:10, 7 February 2006 (UTC)[reply]

Christian doctrine of Eternal Security / Once Saved always Saved / Dennis Nigrelli[edit]

It seems like the Doctrine of Eternal Eternal is believed by some Christian sects and not by others. I know that the Bible can be used as justification for both sides of the discussion. My question is This ; Does Not the gift of Free Will make it a no Brainer that we are given a choice and that at any time we can choose not to Believe or to believe. How do the Follwers of The Eternal Security Doctrine deal with the gift of free will as it applies to them and the Once saved always saved point of view.Dennis Nigrelli

The way to salvation has been a point of doctrinal contention among Christian denominations since the Reformation. Classical Protestant doctrine teaches that a person does not earn salvation but must trust (sola fide) in God's grace that he has elected you for salvation, which saves you the anxiety over whether you have performed enough good works or shown strong enough faith to deserve salvation. Post-Tridentine Roman doctrine is that a combination of works and faith is necessary, and many more recent fundamentalist denominations have made salvation contingent upon a person's act of displaying sufficient faith by choosing to be "born again". There are many other nuances and variations as this is a central question for a Christian. The idea that God will eventually choose all human beings for salvation was rejected in the early Middle Ages as the Pelagian heresy but has been gaining appeal again in some churches. alteripse 01:16, 5 February 2006 (UTC)[reply]
Specifically, what you're referring to is a Calvinist and Baptist version of the perseverance of the saints: one view of Christian divine grace. Here is the question in a nutshell: If God grants grace to those with faith, then does that grace cover sins, or is salvation revoked with each sin? This is related to whether an act of genuine faith carries with it such totality as to make the person new in essence as well as new in state? For some (and they're a small minority), "once saved, always saved." These people believe, if you wish to be precise about it, that true faith means the inability to revoke salvation because overt sins of will are proof that the faith was not there in the first place. However, the doctrine is quite often perverted, and this has been increasingly common lately among some stripes of evangelical. When one such was presented with Christ's admonition, "Let him who is without sin cast the first stone," he responded that he could cast the first stone, because he was without sin since he was born again. This view is absolutely unsupported by any non-congregational church (i.e. any church with an official theology), but the congregational churches are such that, inevitably, one will teach the doctrine only half way and open the door to such abuses. (This is in addition to the questions of total depravity involved in the Pelagian debate. (It's also worth noting that, although Semi-Pelagianism is officially heterodox, it is the operative theology in many Christian churches.)) Geogre 02:44, 5 February 2006 (UTC)[reply]
What is the teaching called, that though through their free will many may reject Him, yet God is so patient and his Love so strong, his Grace so generous, that in the end all will accept him and be saved, even the worst of sinners, the Judases and the Hitlers? --Halcatalyst 17:03, 5 February 2006 (UTC)[reply]
The word you are looking for is Universalism. (It's the sixth definition in the article) DJ Clayworth 20:09, 6 February 2006 (UTC)[reply]

When I was taught this doctrine as a child, it was explained to me that by accepting Jesus into my heart, God changed me and I would find myself wanting to do the will of God and I would be a child of God and just like no matter what I did I would always be a child of my Earthly parents, so too, once God accepted me as his child I would always no matter what I did continue to be his child and when I died I would go to heaven; but just like parents punish their own children more than stranger's children, so too would God disciple me, just like he disciplines the Jews who are also his children under another covenant. (I am an atheist now.) WAS 4.250 05:28, 5 February 2006 (UTC)[reply]

Battlefield: 1942[edit]

Yesterday I bought Battlefield: 1942: The Complete Collection. Why doesn't anything happen when I call in for artillery support? Can I do anything with the engineering wrench other than defusing mines? And besides machine guns, what can I destroy with the TNT? Thanks. Captain Jackson 03:47, 5 February 2006 (UTC)[reply]

Apparantly it's a common problem with the AI of the game that a call for artillery support has no effect (see sharok.net/english/gaming/bf1942/gameintro.html here). The wrench should be able to fix pretty much anything from damaged tanks, to jeeps, etc. Likewise, you should be able to destroy (or severely damage) pretty much any soldier or vehicle with the TNT if it's placed or thrown in the right place. --Canley 14:22, 5 February 2006 (UTC)[reply]
This is WP:RD/Humanities. Still, war(games) are too much human to let them out. --DLL 21:59, 6 February 2006 (UTC)[reply]

Japanese pop culture question[edit]

The following links show a particular genre of Japanese performing arts. What is it called?

  1. www.ebaumsworld.com/matrixpong.html
  2. www.ebaumsworld.com/videos/karatemaster.html

Thanks in advance.

--Robert Harrisontalk contrib 07:03, 5 February 2006 (UTC)[reply]

Are you referring to the men dressed in black that act like special effects/stuntmen? I think you may be thinking of bunraku - I remember seeing a performace in Japan that was very good - similar to what you are thinking of. СПУТНИКССС Р 23:53, 5 February 2006 (UTC)[reply]

The Ayaan Hirsi Ali biograpfy translation[edit]

Dear Sir/Madam

I have unsuccessfully tried to find an answer to the following question(s):

Has the Ayaan Hirsi Ali biography been translated into English?? If so, could you please tell me the English title of the book and possibly the name of the publisher?

Thank you very much for your time.

Kind regards, Jens Kollenberg Australia

Could you tell us what book you mean? As far as I know, no biography of Ayaan Hirsi Ali has been published in Dutch. She wrote two books herself, De zoontjesfabriek and De maagdenkooi (to be published in English as The Caged Virgin www.amazon.com/gp/product/0743288335/sr=1-1/qid=1139129112/ref=pd_bbs_1/104-1932379-1295965?%5Fencoding=UTF8), but those are not strictly autobiographical works; her own experiences are described only occasionally, as part of a general critique of islam. David Sneek 08:49, 5 February 2006 (UTC)[reply]


Hi and Thank you very much for your quick reply. I read in a Norwegian newspaper (Aftenposten) on Saturday that Ms Ayaan Hirsi Ali's book, written in Dutch, had been translated into Norwegain. The title of the book in Norwegian is: "Krev Din Rett", which in English translates into something like: Demand your rights. Your mentioning of one book to be translated and published under the name of "The Caged Virgin" would probably be the one I'm thinking of. Would you also know the name of her Dutch/English publisher? Once again, many thanks for your good work and time. Kind regards, Jens Kollenberg, Australia

This is what I found - clicking the ISBN numbers will take you to a page that helps you locate the book at libraries, booksellers etc.:
  • The Virgin Cage, Free Press ISBN 0743288335 (available April 25, 2006)
  • De maagdenkooi, Augustus, ISBN 9045702126
  • Krev din rett, Cappelen, ISBN 8202256461
David Sneek 20:26, 5 February 2006 (UTC)[reply]


David, Thank you very much! Servus, Jens

Dennis Kyte information[edit]

I am trying to find some bio information on the author/illustrator Dennis Kyte with little [no!] success. Can anyone point to a link or provide any information on dennis.

many thanks

TH 84.69.110.126 12:56, 5 February 2006 (UTC)[reply]

  • If this is a living and working person, check with the publishing house of one of his works. Most publishers provide a small biography of their illustrators and authors. From that you will be able to figure out other sources to employ. Geogre 16:46, 5 February 2006 (UTC)[reply]
  • His www.getcited.org/mbrz/10116939 getCITED page lists 11 books for which he has been an illustrator (including one for which he was also the author); most are published by Little Simon, which I gather is a division of Simon & Schuster. The www.simonsays.com/content/destination.cfm?sid=33&pid=427751 homepage for the imprint, however, doesn't list Dennis Kyte as an author from whom the label has any books available. In sum, I don't know what to tell you... Joe 18:50, 5 February 2006 (UTC)[reply]

thanks for your help Geogre and Joe I'll see where it gets me.

TH 81.77.78.116 19:11, 6 February 2006 (UTC)[reply]

Spartan History[edit]

I am trying to do a research paper on the history of the Sparty, also trying to find information on how the name Sparty came about and how Michigan State got that name. I need more of the facts and information on the Spartan and not the history of the sports at MSU.

Thank you. Robin

See Sparta. In ancient Greece, Spartan culture was known for its obsession with physical exercise and military discipline, which is why the name is often used by sport teams. Spartak Moscow, AC Sparta Praha, Sparta Rotterdam, etc. David Sneek 20:38, 5 February 2006 (UTC)[reply]
Spartans were also known for being cruel and having little use for anything non-military. Their houses were devoid of art and much of anything else, hence the use of the word "spartan" meaning sparse furnishings. StuRat 21:23, 5 February 2006 (UTC)[reply]
Also of note is the fact that much of what is known of Spartan history and society was written by the Athenians, Sparta's main rival, and so what has come down to us may be a distorted view. The claim that the Spartans left disabled children out to die is often seen as propaganda. MeltBanana 23:42, 5 February 2006 (UTC)[reply]
"... disabled children out to die" : this was true in Europe, read Paris : "The herdsman, unable to use a weapon against the infant, left him exposed on Mount Ida, hoping he would perish there", and not only in Sparta. It is still common in Asia or Africa, google for it and see by yourself. --DLL 21:55, 6 February 2006 (UTC)[reply]

European heads of state[edit]

I am trying to find out which european head of state wears a crown but is not a monarch. Can anybody help, it would be much appreciated. JI Possibly the Pope.

Sounds like some kind of trick question. One answer could be the heads of state of Monaco, Luxembourg and Liechtenstein, if you define monarch as "king or queen". Another idea would be Simeon II, who was formerly the Prime Minister and former Tsar of Bulgaria. Although he was PM and not President and therefore not head of state. As a pun, it could be the presidents of the Czech Republic or Slovakia, since their currencies are named 'koruna' (crown). --BluePlatypus 22:26, 5 February 2006 (UTC)[reply]
You may be using the word "monarch" to mean an hereditary ruler. Our article explains that it can also include non-hereditary heads of state such as the pope. The pope has a crown called the "Papal tiara", but the practice of actually wearing it seems to be dying out. The last pope to wear it at his coronation was Pope Paul VI, in 1963. JackofOz 00:53, 6 February 2006 (UTC)[reply]
Ah! That's probably it. Best guess so far, anyway, since he clearly has a crown (even if he doesn't use it, but how many sane people go around with crowns on their heads nowadays?) and is head of state, and since he's not elected, he isn't a 'monarch' in the more common usages of the term. --BluePlatypus 01:16, 6 February 2006 (UTC)[reply]
Note that the pope is elected by the College of cardinals. Crypticfirefly 01:34, 6 February 2006 (UTC)[reply]
Ah but we know better, don't we, BluePlaytpus. See also elective monarchy, particularly the section titled "Current". JackofOz 02:13, 6 February 2006 (UTC)[reply]
Whoops, I meant to write "since he is elected" and it apparently came out as the opposite. But there are of course elected monarchies, but it's just not what people usually think of, which is why I wrote 'the more common usages of the term' (at least that part came out right). Most monarchies are elective in one way or another, if not the king, then at least the dynasty, since they tend to have a vote among the aristocracy or parliament if the line dies out. I think Norway was the most recent European country to elect a line of kings, in 1905. Finland, independent only 13 years later chose to be a republic, which says a lot about what WWI did for the popularity of monarchy. --BluePlatypus 02:26, 6 February 2006 (UTC)[reply]
Whilst the Pope doesn't use the crown any more, the term "triple crown" used to be an only-slightly-obscure term used to describe the Pope or the Papacy. It's obsolete now, but might still crop up... especially in riddles like this. Shimgray | talk | 21:15, 6 February 2006 (UTC)[reply]
The trick bit of the question might refer to something like in Dutch politics, where 'the Crown' (de Kroon) refers to the assorted ministers plus the king. In a country which had a king, this might be a term that is still in use despite the fact that there is no more monarch. So the elected head of state might be said to 'be' (or represent) the Crown, although that is not quite the same as physically wearing it. DirkvdM 09:57, 6 February 2006 (UTC)[reply]
Joseph II, Holy Roman Emperor didn't wear a crown. – b_jonas 20:05, 7 February 2006 (UTC)[reply]
How about Queen Elizabeth? England isn't really a monarchy anymore. Her role is more symbolic than anything. I guess that would mean she's not the head of state then... So nevermind. The Pope is probably still the best answer. -LambaJan 22:03, 7 February 2006 (UTC)[reply]
The English and Dutch queens are titular heads of state. So they've got the title, but then that's it. DirkvdM 09:26, 8 February 2006 (UTC)[reply]
Oh, so the question was "wears a crown but isn't a monarch". I misunderstood it to "is a monarch but doesn't wear a crown". Oopsie. – b_jonas 11:52, 8 February 2006 (UTC)[reply]
Titular means the person doesn't perform any of the functions of a monarch, but is a monarch in name only. That is very different from what Queens Elizabeth and Beatrix do in reality. Both of them perform all the functions required of a monarch in a modern constitutional monarchy. What has changed is the concept that a monarch rules, ie. goes out and fights battles, and makes laws by decree. They certainly no longer rule like that, but they still reign. I'd say that the UK and the Netherlands are monarchies in a very real sense. And QE2 is the head of state, not just of the UK but of 15 other Commonwealth Realms such as New Zealand, Canada, Australia etc. JackofOz 12:36, 8 February 2006 (UTC)[reply]
I can't speak for the monarch of England (etc), but the Dutch monarch has no real power at all. He is supposed to sign all laws. But what if he doesn't? Former prime minister Kok said that that would constitute a crisis. So it doens't happen. During a discussion about this, one exception came up, namely that the former queen Juliana said she wouldn't sign any death warrants. But no source was given for this, and if it is true then it was probably something that would not have been done anyway (it was about sentencing war criminals, by the way). A nice example of the difficult situation was in Belgium, where the king didn't want to sign a law concerning abortion. So he decided to abdicate for a few days to let the law pass. A clear demonstration of the farce the whole royal circus is. There, here, and in England (etc) as well, I thought. DirkvdM 20:54, 8 February 2006 (UTC)[reply]

I think the Baudouin abdication was a good example of a monarch having power. He refused to sign the law, and said the only way it would be passed was without him as monarch. So they passed a special law enabling him to abdicate for a short period. If he were truly a rubber stamp, he would just have acquiesced, however unwillingly. Another case is the Australian constitutional crisis of 1975. Prime Minister Gough Whitlam had a majority in the parliament but Governor-General Sir John Kerr dismissed his government anyway, despite the convention that the G-G always acts solely on the advice of his Prime Minister. It is often said that if the same crisis had occurred in Britain, the Queen would never have done what Kerr did but would have found a different solution. However, the fact is he did do what he did. The G-G is the queen's representative. He signs bills into law by giving Royal Assent on her behalf. He acts as if he were a monarch. Whether Kerr acted appropriately or not is still a matter of ongoing debate after 30 years, but nobody has ever said he did not have the technical constitutional power to do what he did. Powers that are rarely exercised are still powers. JackofOz 00:44, 12 February 2006 (UTC)[reply]

Huh? Boudewijn (so that's 'Baudouin' in English?) couldn't stop the law. He was powerless. He could not even acquiesce, the law forced him to sign, so the law had to be changed (didn't know that last bit). So he's a powerless slave to the system. Hardly an example of him having power. About the G-G, I don't know how similar your system is to that here in the Netherlands, but it sounds similar. And here, the 'governor' of a province (the 'commissioner of the king') is assigned by the crown, which is basically the cabinet (the government). So there's lots of royal naming, but the guy is really (rather indirectly) democratically chosen, by the ministers, who in turn have been chosen by the directly chosen parliament. No royal power here either.
I think I've discovered a weak spot of yours. You fancy kings and queens. Which do you fancy more? :) DirkvdM 08:02, 12 February 2006 (UTC)[reply]

Ancient quotes on the Olympics[edit]

I've been trying to find a quote from an ancient figure (preferably a philosopher) on the ancient Olympics for a paper I'm writing. I've only been able to find one, a fairly vague example from Epictetus that references the Olympics. Are there any such references?

Thank you very much.

In the play Lysistrata by Aristophanes, a reproach is made to the Spartans and the Athenians who are at war with each other:
In no uncertain terms I must reproach you,
both sides, and rightly. Don't you share a cup
at common altars, for common gods, like brothers,
at the Olympic games, Thermophylai and Delphi?
I needn't list the many, many others.
The world is full of foreigners you could fight,
but it's Greek men and cities you destroy!
Aristophanes, Lysistrata, 1131
David Sneek 21:27, 5 February 2006 (UTC)[reply]
Seargled "Olympia games" ... first hit :
Did politics ever affect the ancient Games?
"While the Olympic Games were being celebrated, Alexander had it proclaimed in Olympia that all exiles should return to their cities, except those who had been charged with sacrilege or murder. He selected the oldest of his soldiers who were Macedonians and released them from service; there were ten thousand of these. He learned that many of them were in debt, and in a single day he paid their obligations... Diodorus Siculus, Library ,"

www.perseus.tufts.edu/Olympics/pol.html try this site. --DLL 21:42, 6 February 2006 (UTC)[reply]

social values in Middlemarch by George Eliot[edit]

Hi all, in Middlemarch by George Eliot (female, English, 19th C novelist) there is a character called Tertius Lydgate, and the book discusses his nature, specifically his "spots of commonness". This is in chapter 15, page 140-1 of the Oxford World Classics edition. There is one part of it that I don't get at all. It says that one of these "spots of commonness" was that his impressive intellectual distinction did not carry over into his judgements about "the desirability of its being known (without his telling) that he was better born than other country surgeons."

What I want to know is: is it desirable (for a cultured person) that others should know this, and why? It seems as though both answers could make sense - yes it is desirable that others should know his birth by inferring from his demeanour, or no that they should not know because he should not try to display his birth too ostentatiously. The quote only refers to "without his telling" not "without his getting the message across in some other blatant way, so that he doesn't have to bother telling anyone." The Mad Echidna 22:33, 5 February 2006 (UTC)[reply]

That's irony. George Eliot's sentences are terribly arch, and she was an absolute master -- one of the greatest -- at subtle digs at fashionable society hidden in asides. It's "common" to not be arrogant. I.e. she's accusing the nobility of pigheaded arrogance. Geogre 00:46, 6 February 2006 (UTC)[reply]
(whoops, edit conflict) Yes. I draw the same interpretation, that he probably was telling people explicitly about his birth, which is wrong in terms of etiquette. So this is rather 'common' behaviour. At the same time I feel Eliot is criticizing the apparent hypocrisy of this code: Nobles are show-offs who want people to know that they're noble, while their own code of etiquette prohibits them from telling people. --BluePlatypus 01:11, 6 February 2006 (UTC)[reply]

Hi, Mad, great book, great character, easily the best fictional physician of the 19th century. One of the things you have to like about George is her dry irony. My favorite line in the chapter, not very subtle but describing such a realistic character, is, "...he was an emotional creature, with a flesh and blood sense of fellowship which withstood all the abstractions of special study. He cared not only for "cases," but for John and Elizabeth. Especially Elizabeth."

His spots of "commonness" were his unexamined social prejudices, represented as the very ordinary types of social values that he would have scorned had they been the object of his intellectual passions. One of them was the value of being thought "better born" (i.e., gentry) than some of the other doctors. You can infer that George was both mocking it and acknowledging that it would have been of some value to a new practititioner, but that Lydgate would probably have denied feeling that way. alteripse 01:02, 6 February 2006 (UTC)[reply]

Camigliano, Caserta, Italy[edit]

Hi,

My family is from Camigliano, Caserta, Italy. I hve been told that its name came from two war gods. Is there anyway that I could find this? Also my grandfather had a cousin named Monsignor Carmine Rocco would you know of a site to find anything on him? Thank-you so much for your time.

Toni

It's not quite clear what you mean. What was named after two war gods? The name of your family (which is?) or the name of the town? If you mean Carmine Rocco the Archbishop, there is a site about him www.capuaonline.it/blocknotes/luglio2002/index5.html here if it's the same person (it's in Italian). --Canley 00:21, 6 February 2006 (UTC)[reply]

February 6[edit]

How does Hugh Jintao look only 40, although he's 9?[edit]

His article/biography says he was born in 1997, but he looks nowhere near 9.

He's 9, but looks over 20 years older. How does he do that???

Contrast this to John Evander Couey, who is 22, but you'd think the numbers were switched on his age.

So, what did Hugh Jintao do, and John Couey not do?

Those things are mostly genetic. Although I would assume Couey probably hasn't taken as good care of himself. The conditions of the pictures play a part too. And perhaps Jintao dyes his hair, his eyebrows appear to be a shade lighter. --BluePlatypus 00:40, 6 February 2006 (UTC)[reply]
Jintao's photo is an official government photo, which means there is probably make-up or photo editing involved. In Couey's prison photo, that won't be the case. --Chl 03:25, 6 February 2006 (UTC)[reply]
And if you'd just been arrested, had your clothing replaced with a bright orange jumpsuit, and been told that you could be going to prison for a very long time, I doubt that you'd look all bright eyed and bushy tailed either. Dismas|(talk) 08:18, 6 February 2006 (UTC)[reply]
What really amazes me is that so many people apparently believe his surname is spelled "Hugh" that we need a redirect. He's the President of China forgodsake, not Welsh. JackofOz 08:34, 6 February 2006 (UTC)[reply]
Also, a lot of people don't know the naming systems are switched among the Chinese. Westerners automatically assume that the first name is the first name. Orientals take English names too. For example, in my high school, I knew of a foreign exchange student named "Eun-Jung Kim", although her English name became "Chloe Kim". However, many Asians take similar-sounding English names too. "Taa-mee", a Chinese name, would have an obvious choice- "Tommy." --Shultz 20:25, 6 February 2006 (UTC)[reply]
Also there currently seems to be no standard for writing Chinese names in English. Reading reviews of Memoirs of a Geisha, I have seen the star's name as both Zhang Ziyi and Ziyi Zhang. Irritating and confusing. I've never seen Jintao Hu, though. pfctdayelise 08:09, 13 February 2006 (UTC)[reply]
And Vietnamese names are even more confusing. After years of bafflement, I've now discovered Vietnamese have a family name, an intercalary or middle name, and a given name, in that order. But they are addressed and referred to not by their family name but by their given name (eg. Mr John, not Mr Smith). So when the original name is reversed to conform to Western order, it becomes virtually impossible to tell which name is which any more. I still remember the assassination of Ngo Dinh Diem, and always wondered he was called "President Diem" when his surname was Ngo. Some things are worth a long wait to find out. JackofOz 08:45, 13 February 2006 (UTC)[reply]
A whole mix of factors: as mentioned, airbrushing (very in in China) and hairdye (likewise); the photo is probably several years old; and East Asians do tend to age well (until they reach about 80, when they really start to shrivel). Perhaps also the inner satisfaction of being such an enlightened and judicious ruler? Markyour words 12:47, 6 February 2006 (UTC)[reply]
Stay away from the sun? --Vsion (talk) 14:33, 9 February 2006 (UTC)[reply]

termination[edit]

I have looked everywhere I can think of and can't find and answer so here goes. Can someone who you are not employed by terminate you if your employer asks them too? Even though they do not work for that company. Elizabeth

Unfortunately, that particular bit of nastiness is old, at least in the US. Companies have hired human resources firms to "administer" terminations. However, this is only a termination by the company, and they have merely out-sourced the actual axing to a "consultant." There are firms that make their generous yearly profits this way -- by coming along and doing the terminations for squeamish executives. N.b. these companies have to have been hired to do it, they have to have the full authority of the company, so they are, in a sense, the replacement Human Resources Department of your company for this one particular function, but a consultant cannot fire you because you have annoyed or hindered the consultant's work. Geogre 02:33, 6 February 2006 (UTC)[reply]

Camigliano,Caserta,Italy[edit]

Hi once again,

Thank-you for the information on Arch Bishop Carmine Rocco this is him. I was told that the village of Camigliano was named after two war gods and this is how it got its name. Also my families last name is DiBernardo. Thank-you so much for the information that you have already passed on to me. I appreciate this so very much.

Toni

Please continue the thread where you started it, here-above. DirkvdM 09:59, 6 February 2006 (UTC)[reply]
The only name I see Camigliano could be related to is Camillus who was not ever a demi god, only a roman statesman. Please correct if necessary. --DLL 21:32, 6 February 2006 (UTC)[reply]

Thank-you once again. But I was given a book that was written for a celebration for its twin city in France the name escapes me right now. But it mentions another name too which is Marte. A friend hads the book now and I'm hoping that she can translate it better than I did, have you ever heard of Marte? Thanks again Toni

End of metaphysics[edit]

why do some philosophers say it's the end of metaphysics? and I don't mean it in the sense that Wolfgang S. put it or postmodernism. I mean it in general, why is metaphysics supposed to be over? is it because of a 'metaphysical skepticism'?--Cosmic girl 04:33, 6 February 2006 (UTC)[reply]

They say there can be no Metaphysics because (they claim) nothing can be known beyond the physical. "Meta" is Greek for "beyond." This is the materialist, empirical, rationalist view. Yes, it can be called metaphysical skepticism. --Halcatalyst 05:07, 6 February 2006 (UTC)[reply]

I see, thank you :)--Cosmic girl 05:19, 6 February 2006 (UTC)[reply]

But that wouldn't be something new. That statement can be made at any point in time. It has not changed with the development of physics (nor can it ever). So "the end of metaphysics" is a strange way of putting it. DirkvdM 10:03, 6 February 2006 (UTC)[reply]

Is there any good source or book about 'metaphysical scenarios' or 'metaphysical hypothesis' ? because reading Kant is so boring that I don't even want to read his prolegomena or however it's spelled...and I think I basicly know all the metaphysics of other philosopers based on their philosophy...and the one I like the most is the one of the skeptics, and Buddhist metaphysics (but real buddhism, not new agey buddhism)--Cosmic girl 14:44, 6 February 2006 (UTC)[reply]

Keep in mind the difference between philosophy (metaphysics) and spirituality, though they're not incompatible. Buddhists don't have a "metaphysics" but rather the opposite, (making a word up here) an "intraphysics." Have you looked at Ten spiritual realms? --Halcatalyst 19:00, 6 February 2006 (UTC)[reply]

thanx :) --Cosmic girl 22:37, 6 February 2006 (UTC)[reply]

I'm hardly a professional philosopher, so take this for what it's worth, but I can't help but remember just a few decades when political philosophy was also a fairly dead subject among American philosophers. Then A Theory of Justice came along and suddenly everyone had an opinion. I think the relative dearth of serious metaphysicians is similar. Certainly there are perfectly explicable historical reasons, but I can't escape the feeling that we just need a good new perspective to reexcite interest in these age old questions.
As for a good book about metaphysics on a not-quite-intro level, I like www.amazon.com/gp/product/0415261074/qid=1139269793/sr=2-1/ref=pd_bbs_b_2_1/103-4486384-6819837?s=books&v=glance&n=283155 Metaphysics : A Contemporary Introduction --George 23:56, 6 February 2006 (UTC)[reply]

thank you :) --Cosmic girl 23:12, 7 February 2006 (UTC)[reply]

Word Origin[edit]

Please help me find the origin of the word, "magazine" I am refering to magazine in terms of a weapon magazine. Like an M16 magazine.

Thank You,

Harley

First, note we have a language ref desk which might be a better spot: Wikipedia:Reference desk/Language. M-W lists Middle French for "magazine" though someone with access to the OED could give a fuller answer. The first thing I notice is that word did not undergo the Great vowel shift given that that i is pronounced "ee". This means it was likely borrowed sometime after the 15th century. Marskell 12:07, 6 February 2006 (UTC)[reply]
My etymological dictionary says it originated in Arabic (makhazin, plural of makhzan, storehouses), via Italian (magazzino), then via French (magasin). In the sense of a place for storing things, it also appears in modern French and Russian with the meaning "shop". JackofOz 12:26, 6 February 2006 (UTC)[reply]
Makhsan, i believe, is the origin of the word magazine in all of its meanings; a magazine (for weapons) is where you store the powder. In French, a magasin is a store. And a magazine, like the book, is a place where you store lots of information. СПУТНИКССС Р 14:44, 6 February 2006 (UTC)[reply]

World Economy[edit]

I heard in the discovery chanel that the bible code predicted a downfall of the world's economy in 2002...has this any truth to it? and what do economists think about this?--Cosmic girl 14:05, 6 February 2006 (UTC)[reply]

Yes, and using the same methodology, Moby Dick predicted the death of cs.anu.edu.au/~bdm/dilugim/diana.html Princess Diana. I wonder how the whale feels about it. --BluePlatypus 14:19, 6 February 2006 (UTC)[reply]

haha yes I know, and I don't believe in the BC at all, but I was asking if there was indeed such a 'collapse' and how is it going now, I mean, did the world ever recover from it? and ... what whale are you talking about? :S --Cosmic girl 14:31, 6 February 2006 (UTC)[reply]

Well, the NASDAQ lost about 2/3 (!) of it's value between March 1 2000 and 2001, due to the bursting of the Dot-com bubble. It continued down in 2001 and 2002, although by not as much. Then there was of course the events of Sept 11 in 2001. So 2002 was not a good year by any standards (and you could've predicted that at least several years before), but I wouldn't say it was a 'collapse' (2001 would've been a better candidate). It was certainly not comparable to the 1929 crash. Mostly, we're just in a recession, which many if not most economists consider a normal thing to happen from time to time within an economic cycle. The world economy has been recovering from that. Some economists are worried about the US economy in the long term because of its increasing budget and trade deficits. But that kind of worry isn't new either. --BluePlatypus 15:13, 6 February 2006 (UTC)[reply]

thank you :) --Cosmic girl 15:19, 6 February 2006 (UTC)[reply]

Just remember, prominent economists have succressfully predicted seventeen of the last five recessions! Shimgray | talk | 21:09, 6 February 2006 (UTC)[reply]
BTW, where does that whale name come from (I mean, where did Melville take it ?). --DLL 21:16, 6 February 2006 (UTC)[reply]
From Mocha Dick, apparently, a real whale c.1820ish. Shimgray | talk | 23:08, 6 February 2006 (UTC)[reply]

What whale? --Cosmic girl 22:34, 6 February 2006 (UTC)[reply]

The whale they're talking about is Moby Dick, the subject of a rather long and frequently not enjoyed book. ᓛᖁ♀ 22:51, 6 February 2006 (UTC)[reply]
Well, call me Ishmael if you like, but I enjoyed it. JackofOz 22:56, 6 February 2006 (UTC)[reply]

Moby Dick is one of the best books in the world if you like reading detailed descriptions of a by-gone era. I loved parts of it. If you don't care to know minute details of the whaling trade about a century ago, then it's not for you. WAS 4.250 01:27, 7 February 2006 (UTC)[reply]

awesome, I'll try to read it sometime :P --Cosmic girl 03:22, 7 February 2006 (UTC)[reply]

I did not know that Moby had that enormous number of barnacles. I read about Mucha : how do you derive from this to Moby - maybe a question for RD:Language ?--DLL 20:38, 10 February 2006 (UTC)[reply]

Joan of Arc Interview[edit]

Hello, My name is Ariel Hernandez, I am a high school student at Eisenhower High school in Houston, Texas. I am currently a history fair performance on Joan of Arc, and I was wondering if i could interview you via email on this subject. Thank you for your time. Here are the interview questions:

  1. How did Joan of Arc change history?
  2. How did Joan of Arc take a stand in history?
  3. Why is Joan of Arc important?
  4. Explain Joan of Arc's impact on the world?

(I've reformatted your question to make it more legible. — QuantumEleven | (talk))

Hello, welcome to Wikipedia Ariel. I wonder who you are directing this question to? This is a group of people that answer questions here, on this page. I think the first thing you should do is read Joan of Arc right here in the encyclopedia, then if there are any unanswered questions, please let us know. Right now, though, you may struggle to convince us that this isn't just an inventive way to get someone else to do your homework – I hope you can see why. Notinasnaid 15:47, 6 February 2006 (UTC)[reply]
Supplementary question: What or where is "Arc"? --Shantavira 16:00, 6 February 2006 (UTC)[reply]
There is controversy over the name. perso.wanadoo.fr/musee.jeannedarc/nom.htm sums up the possibilities. It could've been "Darc", which later was interpreted as "d'Arc" and thus "of Arc". Strangely the article doesn't make any mention to it. It's probably one of those questions that will never be answered. --BluePlatypus 16:29, 6 February 2006 (UTC)[reply]
  • Why are you learning about religous-pseudo-historical-mythology in history class?--152.163.100.72 01:55, 7 February 2006 (UTC)[reply]

nevermind, you said you're from texas--152.163.100.72 01:56, 7 February 2006 (UTC)[reply]

    • Joan of Arc is a significant figure in the history of Medieval France and the Hundred Years' War, and to try to pretend that religion doesn't exist is to deny large chunks of history. User:Zoe|(talk) 03:24, 7 February 2006 (UTC)[reply]
  • Don't they sell Joan of Arc brand hot dogs and marshmallows for roasting over open fires ? StuRat 06:40, 7 February 2006 (UTC)[reply]
Not really??? How about prefabricated fireplace logs? - Nunh-huh 07:03, 7 February 2006 (UTC)[reply]
That's as tacky as the "Harold Holt Memorial Swimming Centre" in Melbourne. Holt was an Australian Prime Minister who drowned in the surf. JackofOz 11:21, 7 February 2006 (UTC)[reply]
The University of Colorado has an Alferd Packer Memorial Grill, named for a man convicted of cannibalism. User:Zoe|(talk) 16:32, 7 February 2006 (UTC)[reply]
OK, that's number one now on my list of "places not to eat sausage". - Nunh-huh 23:01, 7 February 2006 (UTC)[reply]
Also read the French page for a different perspective because there was a lot of English and French propaganda both ways about her. Jameswilson 00:15, 8 February 2006 (UTC)[reply]

Refuge[edit]

How much is known about the Indian practice of taking refuge, before it was adopted by Buddhism? ᓛᖁ♀ 16:17, 6 February 2006 (UTC)[reply]

There is a little information at Refuge (Buddhism) under "History", but I'm afraid I don't know more than that. The concept of taking refuge in principles, rather than gods, seems to be unique to Buddhism, so it would have meant something rather different in a Hindu context. --Shantavira 19:33, 6 February 2006 (UTC)[reply]

Refuge (Buddhism) seems like Faith in Buddhism to me. Is there a difference?? WAS 4.250 01:38, 7 February 2006 (UTC)[reply]

Refuge does involve faith to a certain extent. The practice is an expression of faith in the ability of one's teacher to provide guidance, faith that the essential teachings are correct, and faith in the support of the Buddhist community — taking refuge is an affirmation of one's faith in Buddhism. ᓛᖁ♀ 02:02, 7 February 2006 (UTC)[reply]

Kepi caps[edit]

I wondering about the design of the civil war kepi cap. Why is it designed like it is? Is there a function for the flat top? Thank You, CP --206.145.168.130 17:27, 6 February 2006 (UTC)[reply]

Usage of the kepi cap has been much broader than you might have realized. --Halcatalyst 23:56, 6 February 2006 (UTC)[reply]

Small tangent... Is "kepi" pronounced like the word "kept" with a long E in place of the T, i.e. "Kep-ee"? Dismas|(talk) 07:51, 7 February 2006 (UTC)[reply]
The French (where the cap originated) call it képi (KAY-PEE). My American dictionary says kep'i. --Halcatalyst 13:23, 7 February 2006 (UTC)[reply]

Homosexuality and Wrestling[edit]

There has long been an uneasy relationship between professional wrestling and homosexuality. Many people who are not fans of the sport claim that it is very homo-erotic and that the fans derive some type of latent sexual enticement by looking at them in the ring. Many wrestling fans deny that their sport is any more 'homosexual' than boxing, soccer, basketball, or any fighting sports. However, in the WWE, many promoters like Paul Heyman and Pat Paterson are actually questionably or openly gay. Is it a 'gay sport'? Why exactly would a sport like professional wrestling seem to be homosexual in nature?

There are gay players, officials, promoters, and spectators in every sport you could name. Their personal sexual orientation has no bearing on anything. (By the way, there are quite a lot of straight players, officials, promoters, and spectators in every sport you could name. Their personal sexual orientation has no bearing on anything either). I've never heard of wrestling, or any other sport for that matter, being particularly "gay". Maybe the amount of close body contact between the wrestlers causes some people to add 2 and 2 and make 69. No doubt some individual spectators might derive some homoerotic thrill from watching the 2 guys. They might also get a similar thrill from watching swimmers or lots of other sportspeople. To balance the equation, a lot of straight men, for example, get a heterosexual thrill from watching sportswomen in action. What spectators derive from what they see - in sport as in any other field of public performance - is a matter for them. Any "uneasy relationship" between wrestling and homosexuality would seem to a subjective opinion that cannot be supported by any facts. JackofOz
Hmm, perhaps the tights, the compromising positions, the "ultra-masculinity" also associated with muscular men wearing leather jackets over hairy, bare chests full of tattos, along with a spiked collars in a typical San Francisco gay bar ? Or perhaps the fact that the Greeks used to wrestle naked ? StuRat 19:21, 6 February 2006 (UTC)[reply]
Hairy chests? Most professional wrestlers shave. User:Zoe|(talk) 03:30, 7 February 2006 (UTC)[reply]
"in a typical San Francisco gay bar" ?? Stereotype much? LarryMac 17:05, 7 February 2006 (UTC)[reply]
Really. Typical SF gay bars are like bars anywhere else (except usually a bit cleaner and better smelling.) Now, if you want the correct stereotype, you need to say "typical San Francisco South-of-Market leather bar". Got it? --jpgordon∇∆∇∆ 04:17, 9 February 2006 (UTC)[reply]
Since some heterosexual men enjoy watching women wrestle for reasons beyond the simple competitivity of it, I suppose it's not hard to make the assumption (rightly or wrongly) that men who find other men attractive might get more out of watching male wrestling than was originally intended. As a heterosexual male I have to confess it's not just the chance of witnessing record winning performances I sometimes tune into women's athletics for. I am but human... --bodnotbod 18:34, 11 February 2006 (UTC)[reply]

The "boundary between subject and object".[edit]

What exactly is "the boundary between subject and object"? I believe this is a mind-brain problem, but nowhere can I find an explanation on what it is. Thanks.206.117.237.56 19:06, 6 February 2006 (UTC)[reply]

No one can answer that, really. It is the difference between the self and the not-self, between the subjective state (in your head) and that which is not subjective. It vexes us. Geogre 19:45, 6 February 2006 (UTC)[reply]
Thank you. 206.117.237.56 21:11, 6 February 2006 (UTC)[reply]
"Subject" and "object," or subjectivity and objectivity, and how they may be related, have been philosophical problems for millennia, psychological problems for centuries, and a scientific problem for maybe ten years now. Some people may think they have the answer, or that their discipline will allow them to root it out, but that remains doubtful. Geogre has it right. But I can confidently predict there will still be millions of words spoken and written about about the subject, because it's an important one. --Halcatalyst 23:50, 6 February 2006 (UTC)[reply]
There are two answers. One is philosophical and very difficult to answer (see above comments). The other is cognitive -- how humans distinguish between self and non-self -- which is probably not as hard to answer, though I don't know the answer to it. The two types of answer are not likely related to one another at all, because the brain takes lots of shortcuts on things like this. --Fastfission 02:28, 7 February 2006 (UTC)[reply]
Have a look at the article on Cognition. Psychologists look at this subject in a variety of ways. One approach is even called "cognitive ontology," where ontology refers back to the very old philosophical question of being: what does it mean to be, or exist? --Halcatalyst 18:35, 7 February 2006 (UTC)[reply]
The article on Consciousness is also highly relevant. Consciousness -- awareness -- is an old reality, of course, but one that scientists used to consider un-studyable because of its subjective nature. It was thought that subjective and objective could never meet. (I would call this a materialistic bias.) Now, with sophisticated tools to study the brain, that attitude has changed, and scientists from many disciplines are participating in what some have called the science of consciousness. --Halcatalyst 18:43, 7 February 2006 (UTC)[reply]
Thank you for your help, Halcatalyst and Fastfission. 206.117.237.56 19:00, 7 February 2006 (UTC)[reply]

House of lords mistake[edit]

In the "House of Lords" page somebody put an inscription between lines as follows

"The House you of are Lords going is to an strugggle unelected to body remove, consisting this of two"

I didnt remove it because I did not know how.

Regards

Thank you. David Sneek 19:43, 6 February 2006 (UTC)[reply]
The text is correct now, it goes : "of two archbishops and 26 bishops ...". There's one question left : do we have also to remove them ? --DLL 21:06, 6 February 2006 (UTC)[reply]
You managed to edit here, so what was the problem there? DirkvdM 08:23, 7 February 2006 (UTC)[reply]

I never edited a page I didnt want to make mistakes, cheers

indian ritual[edit]

dear reader,

i'm looking for information about indians that climb in high poles with a sort of wheel on top , attach themselves with rope to that wheel and turn, and turn....

'Los Voladores de Papantla' is what you're after: try www.kivu.com/multimedia/mexindians.html here to start off. Markyour words 19:59, 6 February 2006 (UTC)[reply]
Dang it, you beat me to it. Anyway, I was going to suggest es.wikipedia.org/wiki/Voladores_de_Papantla The Voladores of Papantla for a picture and a short summary (if you speak Spanish), but I see that site is much better. СПУТНИКССС Р 20:04, 6 February 2006 (UTC)[reply]
BTW, why don't we have an article about it? СПУТНИКССС Р 20:06, 6 February 2006 (UTC)[reply]

When I meet Koreans online, why are they ALWAYS from the South?[edit]

(Foreword: I already know why, but your answer would benefit the readers that don't yet.)

Other than overseas Koreans, whenever I meet a Korean online, why are they ALWAYS from the South? Have I been looking for North Koreans in all the wrong places? Where can I find online North Koreans?? --Shultz 20:21, 6 February 2006 (UTC)[reply]

In South Korea, nearly everyone has an internet connection, in the North virtually no one, I suspect. And given the nature of the North Korean regime, North Koreans might feel somewhat apprehensive about meeting foreigners online. David Sneek 20:32, 6 February 2006 (UTC)[reply]
North Koreans have a poor sense of self-worth, and so pretend to be from the south. Markyour words 20:38, 6 February 2006 (UTC)[reply]
It's like most "Indian Restaurants" are run by Pakistanis. OTOH, I wonder how many North Koreans even know about the internet! deeptrivia (talk) 21:59, 6 February 2006 (UTC)[reply]
I assume that most X-restaurants-run-by-Y-groups phenomena is a factor of economics, not self-worth. Where I grew up, most Chinese restaurants were run by Koreans, not because they weren't proud to be Korean, but because people knew what Chinese food was (and there was a big market for it), whereas Korean food was still somewhat unknown (a few "Korean BBQ" places popped up eventually, but they were never as popular as the Chinese places). Similarly, "Indian food" means something to more Americans than "Pakistani food" does (I have no idea what this latter is, in comparison to the former, though I don't claim to know anything about food). --Fastfission 02:24, 7 February 2006 (UTC)[reply]
But they want a separate article on Cuisine of Pakistan on wikipedia. There's just one Indian cuisine. The article was earlier named "Cuisine of India" and was renamed to recognize the fact that the cuisine in the whole of the Indian subcontinent is the same, since it was one country earlier. It's like having separate articles on "Cuisine of North Korea" and "Cuisine of South Korea", and then North Koreans opening "South Korean restaurants" all over the world. Do you get my point?
According to www.opennetinitiative.net/bulletins/009/ this page, "North Korea also controls its citizens' access to the Internet. Access points are few in number and extraordinarily expensive; the country's first cybercafe opened in May 2002, but the cost for 30 minutes of Internet time was equal to one month's wages for the average North Korean citizen." Which, if true, would explain it pretty well. --Fastfission 02:21, 7 February 2006 (UTC)[reply]
  • North Korea has an almost total embargo on outside communications. People in N.K. are in general very unaware of the outside world, except as it is portrayed in the official government propaganda. As said above, internet access is extremely rare and certainly monitored. Visitors to the country are not usually permitted to speak freely with people on the street. Radios sold in North Korea are normally modified to only recieve a single, government channel. The USA and South Korea broadcast Korean-language transimissions into the country (e.g. RFA, VOA). A significant number of North Korean defectors have listened to these broadcasts, so it's understandable why the North Korean regime wants to stop that. Freedom of speech has always been an enemy of dictatorships. And in general, the harsher the dictatorship, the tighter the controls on free speech. --BluePlatypus 06:06, 7 February 2006 (UTC)[reply]
  • The North Korean English news service is a humorous combination of lies and incompetence. I read one story where they repeatedly accused the US of "hooliganism", (is that parachute-dropping soccer thugs into their stadium ?), then called themselves "North Koran". I sent them an e-mail offering to correct their future articles in case they again forget the name of their country. I suppose they will need to go kidnap a dozen Americans as they did Japanese when they needed Japanese translators. Apparently it doesn't occur to them to put an add in the paper. StuRat 06:27, 7 February 2006 (UTC)[reply]

Sassoons WW1 poetry[edit]

Many years ago I read a book on sassoons poetry and on stuck in my mind ending with the line she had no need of them for she was death. it was if I recollect correctly about the end of everything. I might even be wrong about the author but it was definetly one of the war poets.

After re-reading this three times I'm still not sure - do you have a particular question you want to ask? (please see the rules at the top) — QuantumEleven | (talk) 08:15, 7 February 2006 (UTC)[reply]
Are you asking which Sigfried Sassoon poem ends with a line suggesting that "she has no need of them, for she was death?" If it's Sassoon, I think it may be one of his later poems, but it sounds more like him than, say, Owen. Other than that, the Chadwyck-Healy database, if you have access to it, would catch it in a moment. Geogre 11:09, 7 February 2006 (UTC)[reply]
www.poemhunter.com/siegfried-sassoon/poems/poet-6672/page-2/ This site claims to hold all of Sassoon's poems. Happy browsing! --Halcatalyst 03:56, 8 February 2006 (UTC)[reply]

Puritans and Amerindians[edit]

Topics on the question: Why did Puritan-Amerindian relations fail. --12.226.43.77 22:03, 6 February 2006 (UTC)[reply]

To begin with, Wikipedia doesn't editorialize (i.e., present analysis, however thoughtful, as "true"). "Why did these relations fail?" is not a question the encylcopedia can answer as such, and it is probably better taken up in good secondary and tertiary sources (see Jared Diamond, for example, for an examination of "contact moments"). From a certain perspective, Puritans did not fail: the Native American tribes they encountered often don't exist today while Puritanism (however altered) remains an important part of the American discourse.
As for Wiki pages, I'd point you to Pilgrims and you leave there; sorry, I don't have anything more specific. You might also look at (the admittedly very underweight) First contact (anthropology) for general info.

Puritans in the strict original sense refers specifically to Massachusetts Bay Colony in the first half of the 17th century, when most of the settlers were Puritans and the government was arguably somewhat of a theocracy. I think neither was still true by the end of the century. They were initially somewhat successful in maintaining peaceful relations with the Indians, but this was fairly easy because the settlers were few in number and area and the Indian tribes in Massachusetts had been more than decimated by infectious diseases which presumably originated from first European contact. The real "failure" of relations led to King Philip's War at a time when non-Puritan English were beginning to settle the Connecticut River valley at a steadily increasing rate. The war was New England-wide with atrocities by both sides but it led to the near extermination of New England Indians and set the stage for continued hostilities over the next century. Is that more helpful? alteripse 23:46, 6 February 2006 (UTC)[reply]

It is also worth noting that, where there was a crown colony, as opposed to a charter, official relations between England and the native tribes were different from those between France and native tribes. However, as indicated above, that wasn't "Puritan." That was later by a hundred years. Geogre 11:12, 7 February 2006 (UTC)[reply]

February 7[edit]

poem[edit]

I have memory of a (famous?) poem I heard or read decades ago that I can't get a grip on. Irish or Scottish author maybe. 1800's maybe. Someone loses a lover and claims in his sorrow that nothing can ever be ok again, even the stars are not needed anymore, they might as well go out. It was very moving, but I can't seem to recall enough of it to track it down with google. Can anyone help? WAS 4.250 01:53, 7 February 2006 (UTC)[reply]

www.egr.unlv.edu/~rho/interests/other/poems/w.h.auden/funeral.blues.html Funeral Blues by W.H. Auden, known nowadays from the movie Four Weddings and a Funeral. - BanyanTree 03:21, 7 February 2006 (UTC)[reply]
I thought that love would last for ever; I was wrong.
The stars are not wanted now: put out every one;
Pack up the moon and dismantle the sun;
Pour away the ocean and sweep up the wood,
For nothing now can ever come to any good.
Thanks, that was it. I copied above the last 5 lines that blew me away (I visualized a god disassembling the universe because the god's love has died.) WAS 4.250 14:34, 7 February 2006 (UTC)[reply]
If you'll forgive the plug, I can hardly help but be reminded of "I Don't Believe In the Sun" by the Magnetic Fields:
The only stars there really are
Were shining in your eyes
There is no sun except the one
That never shone on other guys
The moon to whom the poets croon
Has given up and died
Astronomy will have to be revised

--George 17:34, 7 February 2006 (UTC)[reply]

Mythology/Religion/Philosophy[edit]

What are some similarities and differences of mythology, religion, and philosophy? I need just a little bit of insight or ideas if anyone has any.

Thank you so much!

Megan

Religion is a kind of mythology. Philosophy is different from the two in that it tries to be based on critical thinking instead of tradition. All three have in common that they try to explain the world and give moral guidelines. --Chl 04:07, 7 February 2006 (UTC)[reply]
I'd say a religion is a set of beliefs and practices guided by a particular morality. A mythology is a collection of stories that provide a cultural context for a religion. Philosophy is a sort of meta-religion that tells us what morality is and what it means to believe. ᓛᖁ♀ 04:28, 7 February 2006 (UTC)[reply]
As a self-proclaimed philosopher, may I object to being called meta-religious. One of the big arguments between philosophers is what philosophy really is. But one definition I think all can agree with is that it is an attitude, taking a step back to get a different view on reality. Whichever reality that may be. The notion that philosophy is no more than metaphysics is a common misconception I constantly have to fight. Philosophy can be about anything, as long as it hasn't been caught in rules yet, because at that point it becomes a science. Since before that, everyone who deals with it is a philosopher, the ones who come up with the original rules that are the first step towards a science are philosophers. This is why philosophy is also called the 'Mother of all Sciences'. So Newton and Galilei were philosophers. Those are my heroes in philosophy, not Plato and the like. DirkvdM 08:33, 7 February 2006 (UTC)[reply]
Maybe the absence of rules is why Iris Murdoch said: "In philosophy if you aren't moving at a snail's pace you aren't moving at all." JackofOz 11:10, 7 February 2006 (UTC)[reply]

Objectivism[edit]

I don't understand Rand's argument which says that to state that a conscious being created the universe, would be a contradiction to the 'primacy of existence'...--Cosmic girl 16:35, 7 February 2006 (UTC)[reply]

Does www.geocities.com/gallantcrow/ar_argument.html this help? WAS 4.250 16:49, 7 February 2006 (UTC)[reply]

Thank you so much, I'll read it.--Cosmic girl 17:28, 7 February 2006 (UTC)[reply]

I think it's pretty self evident: she rejects the existence of a supreme being as an infringement on her supreme selfishness. If you are almost grown up, read that stuff fast because it fascinates college freshman and strikes most intelligent adults as the most puerile self-centeredness imaginable. alteripse 21:57, 8 February 2006 (UTC)[reply]

why self centeredness? I think she just said what she thought, I don't see why speaking your mind can be called self centeredness...and also, yeah I think Ayn Rand oversimplified some stuff and ignored some other stuff, and pressumed she knew almos everything,but I don't criticize her self centeredness since it's the way she thought and also I think it's not so far from what human nature is...

I'm only a practical atheist, but I wouldn't go as far as saying I'm a philosiphical atheist since that would be assuming stuff of which I have no clue if they exist or not or have any truth to them.--Cosmic girl 03:21, 9 February 2006 (UTC)[reply]

It sounds like you're claiming that "speaking your mind" shouldn't be called "self-centeredness"? If anything, speaking one's mind to voice a selfish opinion is more self-centered than abstaining from speaking an opinion out a genuine or hypocritical concern for others' opinions. Honesty and self-centeredness certainly dont exclude each other.
Second, if you don't criticize people for expressing "the way they think", on what grounds do you criticize a bad idea? Don't you think a horrible opinion or bad idea can be held and expressed with sincerity? Or does it "not matter what you think as long as you express it honestly"? alteripse 12:15, 9 February 2006 (UTC)[reply]

I think it doesn't matter what you think as long as you express it honestly, like you said...but that doesn't mean it can't be criticized! it can, and it should, so the best ideas can 'win'...we would lose many good ideas if we didn't speak them out for concern of what others may think... ideas should be criticized on logical grounds always.--Cosmic girl 16:52, 9 February 2006 (UTC)[reply]

Why does the Swiffer logo look so much like the Sprite logo?[edit]

File:Sprite.JPG

See how they look so strikingly similar? It appears that Swiffer purposefully based their logo off of Sprite. Did they really? If so, isn't that a type of trademark infringement?

--Shultz 18:59, 7 February 2006 (UTC)[reply]

You are right, I see the resemblance! haha, but I don't know what Swiffer is...--Cosmic girl 19:25, 7 February 2006 (UTC)[reply]

Cosmic girl, Read the article. --Shultz 20:27, 7 February 2006 (UTC)[reply]

Swiffer is a brand of dust mop sold in the US. As for any resemblance, there are five reasons why one company might copy another logo:

  1. To fool consumers into thinking their product is the other product.
  2. To convince consumers their product is virtually identical to the other product.
  3. To take on some of the "attitude" of the product copied, without the advertising expense.
  4. They are just lazy and find it easier to steal a logo than come up with their own.
  5. Sheer coincidence.

The first two don't seem to work for such different products. The third item is possible, but I would bet on the fourth myself. I think I see the Nike "swoosh" in the Swiffer logo, too. StuRat 21:00, 7 February 2006 (UTC)[reply]

I favor the fifth. They aren't really that similar, and what is similar about them is pretty basic. The 'swoosh' shape is easy to come up with an has been used in lots of things, not just Nike. Also, the swish shape almost perfectly imitates the path the mop takes through dust in their ad, so I think that's probably what their inspiration was. The colors are a basic range from yellow-green to forest green to blue, which is pleasant and is used all over the place, and I've been seeing white italic block letters in logos my entire life. There could be a connection, and I see how you could say one resembles the other, but I'd bet it's just coincidence. Black Carrot 20:01, 8 February 2006 (UTC)[reply]
If there's any connection it's probably like the reason why red features in most fast-food decor: it gets people out of the dining room faster. Someone somewhere might've found some particular effect of the white letters on green and blue that both Sprite and Swiffer hope to exploit. — Laura Scudder 23:14, 8 February 2006 (UTC)[reply]

Why Tuesdays for new music?[edit]

Why is Tuesday the day for new releases of music and movies (on DVD etc.)? I'm in my 40s and Tuesday has been new-music day for as long as I can remember. Did it originally have to do with distribution, or not competing with new movies in the theater on Friday, or was there some other rationale?

Also, is Tuesday the day in other countries besides the USA?

Thanks for your collective wisdom.

I don't know why certain days are selected, but in Australia, movies are usually released on Thursdays. - Akamad 19:44, 7 February 2006 (UTC)[reply]
In the UK, new music is released on Mondays. As an interesting example, this year David Gilmour will be releasing his new album On an Island on March 6 in the UK which is a Monday, while it won't be released until March 7 in the U.S. March 6 is also coincidentally, Gilmour's 60th birthday. Dismas|(talk) 21:38, 7 February 2006 (UTC)[reply]
Tuesday was selected so every store could have the new release. Given the size of the United States it was not possible at the time to ship out all the new albums and singles on Monday and it was decided to start the official releases one day later. The rest of the world still schedules the new releases for Monday, so technically speak you can get new release officially one day ahead of time if you happened to be in another country. Dr Debug (Talk) 01:09, 8 February 2006 (UTC)[reply]

what is the percentage of people educated at a primary level in the US?[edit]

84.36.3.53 19:55, 7 February 2006 (UTC)[reply]

You need to narrow your question, make it more specific. What years? What do you mean by "primary level"? eighth grade? sixth? fifth? What do you mean by "educated"? graduated? passed? promoted? --Halcatalyst 03:53, 8 February 2006 (UTC)[reply]

There is also the literacy rate, which is generally stored in statistics by nation, such as the CIA Factbook, broken down into gender, ethicity, etc. Then there are types of literacy, such as computer literacy. User:AlMac|(talk) 05:17, 9 February 2006 (UTC)[reply]


Elementary educartion is universal in the U.S. High school graduation is extremly common, the vast majority of adults (as high as85%) have a hs diploma. Post secondary education, of some type is also common. Though only a minority of adults have BA degree, less than 40%.

Moby Dick[edit]

When reading reading Moby Dick, what chapters can a person skip in order to stick to the main narrative? ~~nickgrahamsucks~~

If you want to skip entire chapters of the book, might I suggest you go with the CliffsNotes version instead ? StuRat 21:09, 7 February 2006 (UTC)[reply]
Does pinkmonkey.com/booknotes/monkeynotes/pmMobyDick02.asp this help? WAS 4.250 21:30, 7 February 2006 (UTC)[reply]
Actually, if you know the first three words, that should be good enough for cocktail conversation. --Halcatalyst 03:05, 8 February 2006 (UTC)[reply]
I repent me of my snide remark. Melvilles's style is such that there are many digressions; this is not accidental, or self-indulgent, but rather an essential aspect of his art. You can get an idea of the story line in the Moby Dick article. This adventure-mode can be and has been captured in movies and other media. What could be more dramatic than Ahab's obsession with the white whale and the havoc/tragedy it brings about? --Halcatalyst 03:37, 8 February 2006 (UTC)[reply]

At Amazon, I found an abridged audiobook from Penguin Classics. I don't know what all was left out but that might suffice. From what I've heard, Moby-dick is an acquired taste. KeeganB

I skimmed all the zoological diversions without feeling like I was missing anything at all. — Laura Scudder 23:10, 8 February 2006 (UTC)[reply]

Nigrelli Regiment[edit]

I have long tried to find more information on The Nigrelli Regiment as discovered in a search of my Family name" NIGRELLI" . It came up in the following response to a Search "Knötel's Austrian Army of the 18th and 19th Centuries 4 Musicians of Infantry of the 18th Century, Fifer of the Deutschmeister Regiment, Drummer of the Regiment Nigrelli KA- 5 Infantry Regiment Graf Brown ...

www.uniformology.com/KnotelsAustrianArmy.html". I have not been able to find out any more info.

I wonder if any of your researchers have access to more detailed historical information than I seem to be able to get my hands on. Thank You so Much for all your future efforts , sincerely Dennis Nigrelli. Good Luck and God Bless

February 8[edit]

intravenous feeding[edit]

Was intravenous feeding available in 1939, and if so, what was used in order to do it?

Intravenous therapy has lots of good information, though nothing on the history of the procedure. But www.oley.org/lifeline/95-052.html this article states that work was done as early as 1937. This may help you answer your question. --Halcatalyst 03:03, 8 February 2006 (UTC)[reply]
It certainly wasn't terribly successful for long then. I know a significant challenge was getting the needed lipids and proteins, and as I vaguely recall that wasn't done until recently, especially for the lipids. Without both of those, one couldn't live for nearly as long. You might get better answers by moving (not copying :) this question to the science desk. - Taxman Talk 19:11, 8 February 2006 (UTC)[reply]
Take a look at www.ncbi.nlm.nih.gov/entrez/query.fcgi?cmd=Retrieve&db=PubMed&list_uids=9770991&dopt=Abstract this article, and click on "related articles" for more. Total parenteral nutrition dates from the late 1960s. Partial parenteral nutrition with amino acids dates from the 1930s; there were bizarre studies with IV milk, IV blood, and IV ethanol before that. - Nunh-huh 02:22, 9 February 2006 (UTC)[reply]

What is the Japanese equivalent of the RIAA?[edit]

As I'm a fan of J-Pop, I get curious: What department in Japan is the equivalent of the RIAA?

Since I've downloaded a lot of J-Pop off of Kazaa, and I live in the US, can their answer to the RIAA actually cross the pond and sic their lawyers on me for it?

--Shultz 01:35, 8 February 2006 (UTC)[reply]

Note that the RIAA is not any part of any government. It is a voluntary trade organization. The largest record labels, and all of them are multi-national corporations, are members, but the RIAA is like Pharma: a lobbying organization. Thus, its reach is Asian, European, American, African, and Australian. The legal battles fought in the US with the RIAA as a plaintiff are due to the RIAA using the US as its target first. This is because the US has more music downloaders than the rest of the world and because the RIAA feels that it has a friendly administration for big business interests. Geogre 01:41, 8 February 2006 (UTC)[reply]
The RIAA is an American organisation, even though some of its members are multi-nationals. There are similar organisations in other countries, such as the British Phonographic Industry (and no, you do not need to make the "I had to read that name twice" joke that is usually made whenever the BPI's full name is mentioned) and there is almost certainly a Japanese equivalent. A quick google has turned up the www.riaj.or.jp/e/ Recording Industry Association of Japan, which we don't seem to have an article about. -- AJR | Talk 00:28, 9 February 2006 (UTC)[reply]
Now we do. See RIAJ. --Shultz 18:35, 11 February 2006 (UTC)[reply]

Existence of "english revolution"[edit]

Two of my brothers are vehemently disagreeing with each other as to whether or not something called the "english revolution" ever happened (they made a bet on it). Did it? If it did, could it be considered to be in the same general category as the 'french revolution' and the 'american revolution'? I showed them a brief look at the english civil war, but the one who denies the 'english revolution's existence wasn't satisfied. 154.20.89.16 02:39, 8 February 2006 (UTC)[reply]

Probably mixing up the Glorious Revolution and the English civil war. The former isn't a revolution in the same, blood soaked way as the American or French revolutions, but it certainly was a bloodless change of government and, quite truthfully, a change in style of government. The Glorious Revolution establishes, once and for all, that the Parliament essentially elected the king. From there, the exclusion crisis (or Exclusion Crisis) would basically say that the Parliament could set up a new method of succession other than familial. That's quite a thing. I'd argue that it was a revolution, only without gunshots. Geogre 02:59, 8 February 2006 (UTC)[reply]
The Glorious Revolution is the name applied to the restoration of Protestantism in the ascension to the throne of William and Mary in 1688. It was bloodless, unlike, for example, the Wars of the Roses, the slaughters of the Tudorian times, and, of course, the English Civil War. --Halcatalyst 02:55, 8 February 2006 (UTC)[reply]
Which raises the question what constitutes a revolution. Does it have to be bloody? We often imagine a (political) revolution as such, but that doesn't mean it has to be. It just means a profound and rapid change. And now you can start arguing what exactly constitutes 'rapid' and 'profound'. So I solved the problem by making it more complex. You're welcome. DirkvdM 09:34, 8 February 2006 (UTC)[reply]
Insofar as Cromwell's leadership constituted a move from monarchy to republicanism (or military dicatorship, more accurately), you'd have to call it a revolution. Marskell 12:44, 8 February 2006 (UTC)[reply]
Quite true, and if it had stuck, I think we would be calling it the English Revolution. The point that I was making was that what actually stuck was the English Settlement of 1688-9, so I can agree that it was a "revolution" (permanent change). However, you could certainly call the Interregnum a revolutionary government. (And, like the US Civil War, the losers are generally the more romatically attractive ones.) Geogre 14:03, 8 February 2006 (UTC)[reply]

The English revolution of 1688 was very far from being bloodless. The new regime had to fight the Nine Years War, the First Jacobite Rising and the Williamite war in Ireland before it was secure. The last of these is a war whose political repercussions are still felt. Gdr 15:06, 8 February 2006 (UTC)[reply]

That's stretching a bit too far. There were two Jacobite risings, the 15 and 45, but saying, "wasn't completely secure" is miles away from saying "wasn't bloodless." The revolution was challenged, of course. The Williamite wars in Ireland were both about the Jacobite threat and not about the Jacobite threat, as English adventurism in Ireland had been a 400 year preoccupation by that point alone. Nor was Ireland a very direct threat to William, and contemporaries were split on the issue. We have this tendency to believe the voices that urged William on, but there were plenty who thought it folly. Nevertheless, none of that has anything to do with whether the revolution happened in England nor whether or not it was bloodless. Saying that it wasn't is like saying that the French Revolution wasn't over until Bonaparte died or that the American Revolution didn't end until after the War of 1812. (In fact, that is the perfect analogy, because, from the American point of view, it was fought to prevent incursions from Canada the same way that the Williamite war in Ireland was to prevent incursions from there.) If the American Revolution was over by 1778 and not 1814, then the Glorious Revolution was over by 1689 and generally bloodless. Geogre 17:15, 8 February 2006 (UTC)[reply]

I meant to refer to the 1689 rising (Bonnie Dundee's), not the the 1715 one. Sorry for the confusion (how unfortunate that "first" means "second" in this context).

My point is just that these wars were fought as direct consequences of the revolution. The English were lucky that James fled the country in 1688: this meant that all the blood was spilled abroad. Gdr 17:57, 8 February 2006 (UTC)[reply]

And the Nine Years War was of an earlier era, too. Never mind that. Of course they were extremely lucky that he fled. The blood that was spilled abroad was in putting down the Jacobites, and there were minor uprisings in the nation itself, but the most serious effect, I think, was the way that Jacobitism became the 18th century's Red Scare. The Whig ministry figured out how they could just wave the Jacobite flag to get the kings to take actions in their interests. The reason I mention this is that they were running heavy propaganda about how serious the Jacobite threat was. William was a militarist from before 1689, and he was an anti-Catholic warrior. So it didn't take much for him to want to go stamp out French forces anywhere (his prior enemy) or Catholics (ibid), and anyone saying that the Irish were forever a threat would have his ear. The French had been alledged to have landed troops (and Jesuits...the 17th and 18th century version of the communist agitator who was supposed to be everywhere in the US in the 1950s) several times. I do think it was generally bloodless for the political revolution, but it wasn't totally so. Geogre 12:43, 9 February 2006 (UTC)[reply]
I guess I need to read up on whether Robin Hood was fiction or reality based, and where the magma charter fits into all that above. As I recall, it was the compromise solution to some kind of conflict, but as we grey older, detail memories get foggy. In my opinion, in recent history there have been several "revolutions" (perhaps a different name is needed) that were relatively mild from a death rate perspective, but totally changed the world.

User:AlMac|(talk) 05:30, 9 February 2006 (UTC)[reply]

Just so I'm clear, who's right(of my two brothers)? The one that denies the "english revolution"'s existance, or the one who insists it was a real event. The exact words to define the terms of the bet were "Did something called the English revolution happen". Flea110 05:49, 9 February 2006 (UTC)[reply]

There is nothing "called" 'the English Revolution.' There was an English revolution that is called "the Glorious Revolution" and there is a war called "the English Civil War." Both brothers are wrong. Geogre 12:43, 9 February 2006 (UTC)[reply]

I disagree with you Geogre. I don't know about the history, but going on the discussion above and Flea's drafting of the bet; since there was "nothing called the English revolution" the brother who said "there isn't anything called the English revolution" should win the bet. The one saying "there was something called the English revolution" should lose. --bodnotbod 19:18, 11 February 2006 (UTC)[reply]
Argh. Yet another example of why I never enjoyed symbolic logic. I could argue that there was a thing but that it is not called by that name and therefore that both are incorrect (as the presence of the thing invalidates the spirit of the negative brother and the absence of the name negates the affirming brother), but, from a linguistic point of view, "The thought of a unicorn is a real thought" and "Saying that there is no God is impossible," so you're right. Geogre 17:00, 12 February 2006 (UTC)[reply]

Jefferson Davis Quote[edit]

I once read a book about the Civil War- I'm inclined to believe that it was from either Shelby Foote's trilogy or Bruce Catton's centennial history, but I'm not sure- containing a quote by Jefferson Davis about how he didn't want industrialization in the South. It was something like "we don't want a Lowell on the Potomac or a Waltham on the Rappahannock". Googling various permutations hasn't gotten me anywhere. Can anyone help me out? Thanks a lot. Stilgar135 03:47, 8 February 2006 (UTC)[reply]

It's likely in Foote. I have vol. I and III here at home, and I've gone through the index to both. Foote's index is pretty good, but he doesn't have "on industrialization" in his "Jefferson Davis" section in either volume. I would think it's in volume I. It might be in vol. II and I just don't have that one with me. I need to look through volume I and Davis's speeches anyway. If I find the quote, I'll post later. Geogre 11:38, 8 February 2006 (UTC)[reply]

Humanities[edit]

Forgive my ignorance.

Is Nong Doc Manh, in fact the General Secretary of "north Vietnam?'

Is it true that member of hiis "cabinet" are:


1. Tran Eurc Lurong Phan Van Kai Nguyan Van An Nguyen Tan Dung Le Hong Ann Pham Van Tran TRoroy Quary Duc (sp)


Thank you Imrana Muhammad Al Ali, esq.

See Politics of Vietnam. I've removed your e-mail address, as we do not respond to questions that way. --Halcatalyst 22:37, 9 February 2006 (UTC)[reply]

One vacation, in Las Vegas, I saw X-ing signs that read "12 MONTH SCHOOL". Wouldn't that torment children???[edit]

Seeing these signs several summers ago made me come to think of THIS scenario:

Outside, it's raining. The 4th Grade class is having "indoor recess" playing board games. The 3 computer carrels are also in use.

Jetmir: Hey, (name), I just received word that my dad got a new job in Las Vegas, as the general manager of The Stratosphere Hotel & Casino. We're moving there in a month!

Roscoe: Oh, boy. (groan) Did you know that school is all-year long in Las Vegas??

Jetmir: WHAT??? You've GOTTA be kidding me!! Where did you hear this?? What's the proof?!!

Roscoe: I was on vacation with my family last summer, and we stayed at Las Vegas for a week. Pretty fun, with all the arcades in the hotel/casinos, and waterparks. However, in a drive around that city, I saw pedestrian crossing signs that read "12-MONTH SCHOOL" on them. I thought, "Gee, these must be the most miserable kids in the world!

Jetmir: That's such a bad, offensive joke, I'm not your friend anymore.

Roscoe: Look, I can prove it to you. I'm not kidding. Lets go to a computer so I can show you these signs online.

Jetmir: Ok, you'd better be right.

The classroom computers are full, so the two boys ask for teacher's permission to head to the Library. Recess isn't over for the next 20 minutes, so she grants permission.

Roscoe Googles the term "12 month school" "Las Vegas" and out comes enough hits to convince.

Roscoe: See, that's solid evidence right there. I don't know what I can do for you, Jetmir.

Jetmir: Oh... My... God... This is something I have in my NIGHTMARES!

School Librarian: "Shh! Quiet down over there!"

Jetmir: I am a great fan of Anime, Manga, and J-Pop, so I would -much- rather run away to Japan than to get tormented by constant schooling! Why are the schools in Las Vegas so inhumane???

Roscoe: I don't know. How would you get to Japan? You can swim, but no one could swim an ocean, that's for sure!

Jetmir: Here's what I'll do- I'll sneak Dad's hiking pack up to my room, fill it up with what the hell I need, attach my bookbag and knapsack to it, fill it all up with snack food and snack drinks, and a few minutes before I leave, take $400 from my dad's desk, and sneak the heck out of there. I'll hike to the Greyhound Bus station, buy the ticket, ride to San Francisco, and stow away aboard a freighter to Japan. Once I make landfall there, no more fear for me! Heard a lot of fun things about Japan, so school there should be fun too. Hah, Dad won't come looking for me, because he won't know where the hell I am! I won't have to worry about a 12-month school EVER again!

Roscoe: (Now readers, what must Roscoe tell him? What kind of "time off" will there be for Vegas children? Or is Jetmir better off running away from an unrelenting perpetual school year? If you could give Jetmir advisories about hiking with all that equipment, attempting to buy a Greyhound ticket, and attempting to stow away on a freighter ship {after somehow making it to SF}, what would you tell him regarding this plan?)

(Note: You might think parts of this story are ridiculous, but it wouldn't be below a 4th Grader to plan a runaway like this, especially after hearing what he'd consider to be very nightmarish reports about Las Vegas's school system. --Shultz 04:49, 8 February 2006 (UTC))[reply]

I thought the push for year-round schools ended in the late 90's. I must be wrong. KeeganB
The year-round system doesn't actually have any more school days, the summer vacation is just broken up into periods of a few weeks each. This system tends to be used as a measure to deal with a temporary increase in the student population, since it allows more students to attend the same facilities, as they are spread out over a longer portion of each year. This is much cheaper than building new schools which are only needed for a year or two. StuRat 05:49, 8 February 2006 (UTC)[reply]
With all of the other details within the scenario, I am puzzled as to why "schoolboy" and "friend" could not have been given names. Creative writing criticism aside, perhaps Friend should tell Schoolboy to look at www.ed.gov/pubs/Research5/Japan/structure_j.html this page, scrolling down to the "schedule" section under "Elementary School," paying close attention to the Saturday hours and the other fun parts of the day, such as cleaning. LarryMac 14:52, 8 February 2006 (UTC)[reply]
I guess to satisfy you, I've given them the names of Jetmir and Roscoe now. Would you also care to give Roscoe things to tell Jetmir in terms of buying the Greyhound ticket at age 9 or 10, and attempting to stow away aboard a ship if he somehow makes it to San Francisco? Lets see your spin on this. And LarryMac, good point about Japan's school system. --Shultz 15:35, 8 February 2006 (UTC)[reply]
Also, make Roscoe an Internet search whiz for his age, and tell him what to tell Jetmir about attempting to enroll in a Japanese school without parents, passports, visas, etc..., and trying to find a place to live. I want Jetmir to hear about this too! --Shultz 16:35, 8 February 2006 (UTC)[reply]

Possible Scenario, if Jetmir makes it[edit]

Jetmir enters edu.educ.yonezawa.yamagata.jp/misawaw Misawaseibu Elementary School in Yonezawa

(On the way to the Principal's Office) Jetmir: Oh, those girls look nice! I think I'll like it here.

Jetmir enters the Principal's Office. The secretary's name plaque is illegible to Jetmir, as it is etched in Japanese.

Jetmir: Hi, I'd like to enroll in this school, please.

Secretary: Hello, American kid. You speak Nihongo how much?

Jetmir: Is that Japanese for "Japanese"?

Secretary: Yes, yes.

Jetmir: I only know a few words. I picked some up on my hike over here.

Secretary: Mom & Dad need to come here. They sign you for this school. Where they now?

Jetmir: Well, they're in Waukegan, Illinois, in the United States. I guess since my parents can't enroll me here, I'll have to enroll myself then.

Secretary: No, if your parents cannot sign you up for the school, you need to have adult who takes care of you to do it instead. I think the word is "Guardian". Do you have any guardians here?

Jetmir: No, I'm by myself. I ran away because my family was going to move to Las Vegas, where schools are all-year long, so there's no summer vacation. Going to school without a vacation is one of my worst nightmares, so I ran away from home, and stowed aboard a ship to Japan. That's how I got here.

Secretary: (What should the secretary tell Jetmir?)

--Shultz 16:35, 8 February 2006 (UTC)[reply]

They should tell him too move this to his own user page, as it really isn't Reference Desk material. LarryMac 16:47, 8 February 2006 (UTC)[reply]
No, not me. Jetmir. Besides, if I relocate it onto a subpage, not as many people will see it. Larry, I've had fantasies about running away when I was younger, so through this story, you'd tell me what you'd tell him if he were to go through with such a plan. --Shultz 17:04, 8 February 2006 (UTC)[reply]

Music CD formats[edit]

Will a music CD bought in Egypt play on machines in the USA? Is there a difference in recording formats throughout the world and does that make a native CD incompatability from the continent to continent. Will a CD purchased in Egypt work on standard North American equipment?

Yes, it should play on any CD player. A DVD however cannot do the same. They have specific regions that they can play in. Dismas|(talk) 10:51, 8 February 2006 (UTC)[reply]
  • I wonder why. Isn't it in the producer's best interest to make it worldwide playable as CDs? - 131.211.94.86 10:52, 8 February 2006 (UTC)[reply]
It has to do with television formats. The US and Europe use different systems from each other (different lines per inch, etc.). The DVD must be compatible with that. Many DVD's are now multi-zone, but, indeed, a Zone 1 won't work without adaptators in Zone 2. Geogre 11:32, 8 February 2006 (UTC)[reply]
Television formats are a red herring. DVD region codes are a form of regional lockout: see that article for the manufacturers' reasons. Gdr 14:06, 8 February 2006 (UTC)[reply]
Quite right. Which still leaves the question why that is not the case for music cd's. I'm not sure about the following, so take it with a grain of salt. Maybe this is something to do with who owns the standard; producers of the carrier or of the content. In stead of a red herring, cd's have a red book, the cd standard, which is owned by inventor Philips. And of course Philips just wants to sell as many cd's (of whatever kind) as they can, so they won't use any restrictions. Maybe in the case of DVD's the content-producers got their hands on the standard, which gave them the opportunity to start messinga round with it. If this is true, maybe this sort of industry needs a variation on the trias politica, separating the ownership of carrier and content. Just a thought. DirkvdM 21:09, 8 February 2006 (UTC)[reply]
CDs were designed by a consortium of Philips and Sony, both of which were content producers (see Philips Records). The difference is that at the time DVDs were designed, film studios staggered their releases in different countries (partly because of the costs of film printing); they didn't want a DVD release in one region to undercut a film release in another. With CDs there has never been this supposed conflict between theaters and home. Gdr 13:02, 9 February 2006 (UTC)[reply]

History of belt buckles[edit]

I am researching the history of clothing buckles and am looking for the earliest recorded use of clothing or shoe buckles. Any suggestions about where to conduct further research on buckles would be greatly appreciated. Thanks. KF

Given that it's a fairly rudimentary technology, buckles developed independently in various cultures across the world. As to the first instance, buckles were found in the tomb of Tutankhamun, though I'm sure they were around in earlier dynasties given the Egyptians' long history of gold-working. GeeJo (t) (c)  17:06, 8 February 2006 (UTC)[reply]
Buckles and clasps in general (including belt buckles) were among the more important archaeological discoveries in digs in prehistoric Britain as well. Sites like Wilsford (which is seemingly unmentioned in the [{Wilsford, Wiltshire]] article!) yilelded buckles. Ornamental clasps, buckles, and jewellery were seen as ornaments of the powerful (if you're interested in such things, the book Symbols of power at the time of Stonehenge by Clarke, Cowie, and Foxon is very interesting). Grutness...wha? 05:20, 9 February 2006 (UTC)[reply]
The belt buckle reached its zenith in Texas. images.google.com/images?q=belt+buckle&hl=en&btnG=Search+Images -LambaJan 17:10, 13 February 2006 (UTC)[reply]

international politics[edit]

to what extent the military might alone ensures a state's dominant position in international politics?

I would say "not much". Dominance comes from a combination of economic, military, political, and social power. An example of purely military power would be Nazi Germany, which, by 1940, was arguably the most powerful nation on Earth, when viewed strictly in military terms. However, their politics was a mess, as they had made many powerful enemies along the way. Their social influence was largely limited to a few Germanic language countries, such as Austria. Their economic might was then crippled by warfare, so their power did not last long at all.
In a reverse example, the United States tends to exert all of those methods, with varying levels of success (perhaps with less success recently), around the world. Hollywood films are seen around the world, as are McDonald's. The US has been successful in world political terms, by methods such as having the UN on it's territory (New York). The formation of NATO or the coalition the US was able to assemble for the First Gulf War would be good examples of this political influence in action. As far as economic power, the US has the highest GDP on Earth. The US also has a substantial military, but without these other sources of power the military alone would be almost useless. StuRat 18:59, 8 February 2006 (UTC)[reply]
North Korea has one of the largest armies in the world but has little international political power. Rmhermen 21:11, 8 February 2006 (UTC)[reply]
To about the same extent that physical strength alone determines your domination in social relations. In other words, it means nothing in itself, but only in how you use it or threaten to use it. If you use it to push around people who are smaller than you, you're not going to get a good rep. If you use it to protect the weak, then you might get respect, but you might also be told to butt out and mind your own business. The distinctions between good and bad use of force aren't much clearer in that case. As with social relations, things like wealth are more important for influence. Also, just like social relations, reputation and respect is more important. The Vatican may not command any significant economic or military power, but it's still vastly influential. --BluePlatypus 07:10, 9 February 2006 (UTC)[reply]
Someone once said something like "war is a continuation of politics by different means". In other words, war is the end of politics. Since war is the sole purpose for an army, it has no effect on international politics at all. Unless there's the threat of it being employed, which only works if there is some such history. So if the Netherlands use a war-threat in political negotiations, there will be little else but laughter. If, however, the USA use such a threat, it may very well work. But then the real power is not in the politics but in the threat of the past wars that have been waged, so it's really just an act of terrorism war and not politics. Just like when the bully tells you to do something you obey. But that's not politics. That's just bullying. DirkvdM 09:01, 9 February 2006 (UTC)[reply]
That'd be von Clausewitz. --BluePlatypus 17:10, 9 February 2006 (UTC)[reply]
Ah, I remember now, I heard that in the film Massacre in Rome (uk.imdb.com/title/tt0070592/?fr=c2l0ZT11a3x0dD0xfGZiPXV8cG49MHxrdz0xfHE9bWFzc2FjcmUgaW4gcm9tZXxmdD0xfG14PTIwfGxtPTUwMHxjbz0xfGh0bWw9MXxubT0x;fc=1;ft=20 imdb) (I thought I had written an article on that...strange), in which Richard Burton says it (probably quoting von Clausewitz then). Good film, by the way, especially that dialogue with the priest. "To you war is an interruption of peace, to me peace is an interruption of war." At the time that was probably a very true statement. DirkvdM 18:06, 9 February 2006 (UTC)[reply]

Mao Zedong said: "Political power grows out of the barrel of a gun." Unfortunately, it's still true in many ways, and many people around the world firmly believe in it and use it to justify military buildup. --Vsion (talk) 16:40, 9 February 2006 (UTC)[reply]

how tell time?[edit]

Since christians invented the modern calendar, how exactly did people tell time before then? Did people of the BC world just have no concept of time? Is this why their societies are old and forgoton? because of their primaitve view of the world? Is it really smart to put godless people like that in charge of the world's major sciences, like is done today? does not discovery and education require a form of faith?QbC room 15:22, 8 February 2006 (UTC)[reply]

See List of caledars a lot of these existed before the modern Gregorian calendar & even before christianity. By the way the "modern calender" isn't the only one in current use.
People in the BC world did have a concept of time (Calendars aren't necessary for this though they did have them) - just look at the jewish calendar which is at year 3760 by now. The calendar used by a people has nothing to do with the eventual fate of their society.
I have no idea who you are referring to as "godles people in charge of the worlds major sciences" I think you are assuming rather a lot (or making the mistake of assuming that modern scientists were all around before the christian era).
Does not discovery & education require a form of faith - yes faith in rational enquiry, logical reasoning & the rules of scientific experimentation & reasoning. It doesn't require religious faith which is what you seem to be implying. AllanHainey 15:45, 8 February 2006 (UTC)[reply]
  • Quite a troll of a question, and a bad one at that, since its ignorance is quite obvious. The calendar we use was not invented by christians. It was invented before Christ, even. (See Julian calendar). That "christian calendar" has 5 months (Jan, Feb, Mar, May, Jun) named after Greco-Roman gods, 2 (Jul, Aug) after pagan emperors and the remaining are numbered, except April - the origin of which is unknown. Out of the weekdays, 4 (Tue, Wed, Thu, Fri) are named after Norse gods, Saturday from a Greco-Roman god, and the remaining two are names of planets. Not one name in the calendar is christian. The only modifications to the calendar done in christian times are the setting of year 1 to the birth of Christ (which was likely wrongly done anyway), and the introduction of the Gregorian version which removed 3 leap-days per 400 years. And it's still less accurate than the Iranian calendar which was introduced centuries earlier. --BluePlatypus 16:46, 8 February 2006 (UTC)[reply]

That last sentence is quite misleading, as the Iranian calendar is observational whereas the Gregorian calendar is defined by a rule, so of course the former is more accurate. You might be thinking of the Jalaali calendar (about 6 seconds per year closer to the vernal equinox year than the Gregorian). Gdr 18:32, 8 February 2006 (UTC)[reply]

  • What a wonderful phrase in the original question: "Did people of the BC world just have no concept of time? Is this why their societies are old?" That must be it. Notinasnaid 13:33, 9 February 2006 (UTC)[reply]
Today is Magha 12, 5107. It's Kali yuga. deeptrivia (talk) 18:21, 9 February 2006 (UTC)[reply]

Quote attribution[edit]

Have I correctly identified Charles Fourier as the author of the sentence "If we could suddenly see this arranged order as it will be seen in its full functioning, it is not to be doubted that many of the Civilized would be struck dead by the violence of their ecstasy."? I have not been able to find any confirmation on the web. If I could find out where he wrote it and the original French, that would be nice also. Thanks. 128.220.220.95 18:09, 8 February 2006 (UTC)[reply]

An intelligent design ancestor ? --DLL 09:27, 9 February 2006 (UTC)[reply]
He was referring to his vision of a future utopian society. 128.220.220.95 16:08, 9 February 2006 (UTC)[reply]

Ratio of lawyers to Engineers in US and societal impact[edit]

Thanks for this great resource. I have tried to research info on above subject (Ratio of lawyers to Engineers in US and societal impact), without success.

In a nutshell, the theory goes, there is a relationship between the ratio of lawyers to engineers in any society as it relates to GNP, , general societal "health," etc. I am interested in finding out more about this theory so I can find out how it relates to the USA.

Thank you in advance; any help would be greatly appreciated. My e-mail [has been removed] Regards.

More generally, the ratio of people in a society who create wealth to those who do not, is of critical importance to the future of that society. Engineers generally create wealth by designing new or improved products. Lawyers transfer wealth (and take a large cut for themselves), rather than creating wealth. A small number of lawyers can be said to be helpful in any society, as legal remedies for conflict will prevent the destruction of wealth due to people taking the law into their own hands (think Hatfields and McCoys). However, a large number of lawyers leads to everyone suing everyone for everything, which does considerable harm to an economy, due to the time taken off work for court cases and the percentage of wealth removed from productive uses and given to lawyers instead.
Other examples of people who create wealth directly are farmers, construction workers, and architects. Others may not create wealth themselves, but may enable others to create wealth, such as doctors and teachers. Others who only transfer wealth are casino workers and sales people. Some individuals actively destroy the wealth of a country, but not any entire profession I can think of. StuRat 05:41, 9 February 2006 (UTC)[reply]
Well, an example of the latter would be the army of course. It takes away loads of potential investment in productive institutions and when it is employed destruction follows.
As for the question, not that this is directly relevant, but I just read in the Dutch Mensa Journal that an old research (don't know where or when) showed that of all the studies, the one with the lowest average student-IQ is law. The same article, however, said that another study showed that school grades are a better indicator for future career success. So so much for the relevance of that bit of info. :) DirkvdM 09:20, 9 February 2006 (UTC)[reply]
Well, I considered listing military and defense workers, as their usage in a war does, indeed, destroy wealth (although that of the enemy). However, as counter-intuitive as it may seem, I think large militaries actually prevent war and therefore the destruction of wealth. If you take the example of the Warsaw Pact and NATO nations, I believe, had either side been sufficiently weak, that war would have occurred. But, the old "balance of power" and newer "mutually assured destruction" doctrines actually seem to have worked to prevent a large-scale war. On the other hand, many countries with almost no military suffer from constant inter-tribal warfare and border wars, with Rwanda and Burundi being a good example. StuRat 19:27, 9 February 2006 (UTC)[reply]
I hope you're wrong because that would leave little room for hope. But there wasn't a constant equilibrium between the US and the USSR. The latter had only a fraction of the nuclear power the former had and that actually escalated into the Cuban missile crisis, which almost led to WWIII. But it didn't. Because the USSR backed down. But I don't want to hope for such wisdom next time something like that happens. What conclusion this leads to I'm not sure, though. DirkvdM 05:51, 10 February 2006 (UTC)[reply]
Both the US and USSR had sufficient weaponry to scare the other off of war, however. Had the Warsaw Pact nations been sufficiently weak, NATO would have forced them out of Eastern Europe when it became apparent they would not leave voluntarily. Had NATO been sufficiently weak, the Warsaw Pact nations would have invaded and conquered Western Europe. StuRat 10:29, 10 February 2006 (UTC)[reply]
If the first bit its true, then why did the US have the amount of nuclear weaponry it had? As for the next bits, that's pure speculation. If the Warsaw Pact had been weak maybe the US may have wanted to push on (although the US policy wasn't quite as bellicose as it is now) but I doubt if the European nations would have joined in. They would have been happy enough to have gotten rid of the fascists. There was some fear of 'communism' but the USSR was also an ally. And I think the same applies in reverse. The USSR had suffered about half the death toll of WWII, close to 30 million (something that has been erased from the WWII article - I'll put it back in and see what happens) and must have had little breath left. Had one side been weak and the other still strong and willing, I don't know what would have happened. I don't even dare to speculate on this. I'm totally clueless (when it comes to this specifically, I mean :) ). DirkvdM 12:15, 11 February 2006 (UTC)[reply]
The quantity of nuclear weapons in the US is hard to justify. I think it was a combo of a cheap way to increase overall firepower and it made the Soviet Union feel it had to keep up, which ultimately bankrupted them. The Soviet Union quickly became a feared enemy in Western Europe, and many there would have been grateful to drive them back, given the chance, especially West Germany. While the Soviet death toll was high in WW2, under Stalin that just wasn't a concern and he would have easily sacrificed millions if he could have conquered Western Europe. After all, he killed millions of his own citizens in peacetime for no apparent reason. As for your cluelessness being limited to this one issue, I feel you underestimate yourself (just kidding !). StuRat 06:34, 12 February 2006 (UTC)[reply]
West Germany? I thought you were talking about right at the end of the war. Are you now suggesting that the allies would have let the Germans in in their scheme to drive back the USSR? Sounds extremely unlikely. 10 or 20 years later maybe, but not right after the war. DirkvdM 08:11, 12 February 2006 (UTC)[reply]
I'm talking about the entire Cold War era. StuRat 01:55, 15 February 2006 (UTC)[reply]

Defense workers in the usa[edit]

How many people work for defense in the United States? This includes ALL workers for gun, tank, artillery, ammo, gun etc companies. Any company that holds a defense contract, or could hold a defense contract counts. How many workers are there all together?

Go to the www.dol.gov/ US Department of Labor web site. On the upper left, you will see "Topics." Click that and scroll down and you will see a section on "Statistics." Click on that and you will see subtopics, including "Employment & Unemployment." Keep on searching in this manner and you will likely get information relevant to the question you asked. --Halcatalyst 03:35, 9 February 2006 (UTC)[reply]
If you need further information, you can follow up on the references you find at DOL and other web sites, or even go into a library and do research. Librarians are there to help you. This is the way to get good answers to questions like yours.
Want a guess? I'd say about 20 million. --Halcatalyst 03:40, 9 February 2006 (UTC)[reply]
It's also worth noting that the criteria, as posted, are overly large. Any company that could hold a defense contract would elevate your total to, roughly, the entire workforce of the United States. — Lomn Talk 15:19, 9 February 2006 (UTC)[reply]
You might consider checking the Dept of Defense's statistics on web1.whs.osd.mil/mmid/military/miltop.htm military and web1.whs.osd.mil/mmid/civilian/civtop.htm civilian personnel directly employed; I don't know how it will contrast with the Dept of Labor's stats. — Lomn Talk 15:27, 9 February 2006 (UTC)[reply]

If you want something narrower, check out Category:Defense companies of the United States. Boeing and Lockheed Martin together employ over 300,000 people, and that's just two companies. Northrop Grumman has 125,000 employees, General Dynamics has 70,000, and Raytheon has 80,000. Not all of these people are involved in defence business, but you can easily get well over a million people, just manufacturing military systems. If you count logistical support and things like foodservice and gasoline, you get even higher. The military itself has a couple million, counting the reserves, and there's a lot of people out there making a lot of stuff for them. Night Gyr 09:09, 10 February 2006 (UTC)[reply]

A Secret Society Marked ROA I don't think Masonic[edit]

02/08/06

I have a very old sword that has a George Washington pummell or end on it and a guard on the handle and middle carrier marked ROA. The Blade is engraved Chas. Lee. I have located another sword similar to it and it is marked Patriotic order of the sons of America.( POSA) This is the old original Masonic Order of Pennsylvania. I have searched most older Masonic and secret Society names and abbreviations and have yet to find this ROA mark. Any help would be appreciated. Thank you Buster Burns

I removed your e-mail address -- not a good idea to post it online, and we don't respond that way anyhow. You have to come back here. In case you did, you might be interested to know that in military contexts, ROA generally refers to Reserve Officers Association. The sword's owner was probably both a military man and a Mason. Not everything about the sword would have to be masonic. --Halcatalyst 04:00, 9 February 2006 (UTC)[reply]

February 9[edit]

Which australian city acted as australia's national capital until the founding of canberra in 1927?[edit]

Marvellous Melbourne. JackofOz 01:48, 9 February 2006 (UTC)[reply]

Was Mel borne in Melbourne ? Apparently not. StuRat 21:20, 9 February 2006 (UTC)[reply]

And neither was he born there. JackofOz 21:48, 9 February 2006 (UTC)[reply]

Humanities- Law[edit]

Hi, I was wondering if you could please possibly explain to me what a statutory authority is. Thank-you for taking the time to read and research this for me.

jade155.143.220.138 03:39, 9 February 2006 (UTC)[reply]

You'll have to do the reading and research for yourself. A good place to start is Statutory law. --Halcatalyst 03:54, 9 February 2006 (UTC)[reply]
That article addresses "authority" in the sense of a proof for a particular proposition. ("Section 1411 of the statute is authority for concluding that assumption of risk is not a complete defense to a negligence action.") There's also "authority" in the sense of an entity -- a creation of government that's somewhat similar to a regular agency but also differs in some ways. If, as is often the case, such an authority is set up by a statute, then you could call it a statutory authority. We allude to this concept in the Public Authority article, but it's essentially a stub and won't help you much. JamesMLane t c 07:30, 9 February 2006 (UTC)[reply]

What is the military unit called the Nigrelli Regiment?[edit]

I have long tried to find more information on The Nigrelli Regiment as discovered in a search of my Family name" NIGRELLI" . It came up in the following response to a Search "Knötel's Austrian Army of the 18th and 19th Centuries 4 Musicians of Infantry of the 18th Century, Fifer of the Deutschmeister Regiment, Drummer of the Regiment Nigrelli KA- 5 Infantry Regiment Graf Brown ...

www.uniformology.com/KnotelsAustrianArmy.html". I have not been able to find out any more info.

I wonder if any of your researchers have access to more detailed historical information than I seem to be able to get my hands on. Thank You so Much for all your future efforts , sincerely Dennis Nigrelli. Good Luck

You've posted the same question three times in the same week. You're not going to get answers faster that way. Either people know or they don't. I'll give you a hint: Count Octavio Nigrelli. --BluePlatypus 07:35, 9 February 2006 (UTC)[reply]

Occupation Rullyman[edit]

What is a rullyman.? This occupation appears from time to time in the 1901 census in England.I have tried all the usual points of reference without success. The term is used on its own or sometimes to specify another occupation more specifically, e.g.grocer's rullyman. It is possible that this might be local to Yorkshire and the East Riding and Hull in particular, as my Google search throws up directory entries from this area. 84.51.157.186 06:40, 9 February 2006 (UTC)[reply]

www.communigate.co.uk/ne/teesspeak/page13.phtml This page gives 'rully' as slang for 'lorry' in Hartlepool, so your guess it's a northeastern term seems correct. In any case a 'rullyman' is thus a lorry driver. --BluePlatypus 06:51, 9 February 2006 (UTC)[reply]

Many thanks this is a great help. Must remember in future to breakdown a word into seperate components when doing a search that is otherwise proving elusive. 84.51.157.186 08:19, 10 February 2006 (UTC)[reply]


I think a rully was a four wheeled cart, pulled by a man rather than a horse. It was mostly seen on victorian railway lines. Hope this helps.

Sexual Timeline by pinkyblonde[edit]

Can Youpost this under "History by Topc? for me?

Sexual Timeline


Ø 1662 -- Quakers formed in England. US Quakers make significant contributions to abolition and suffrage movements.

Ø 1760 to 1820 – beginning of Industrial Revolution. Prior to this period, birth control methods were condoms, douches and certain herbs.

Ø 1790 -- New Jersey grants the vote “to all free inhabitants”.

Ø 1807 – NJ women lose the vote.

Ø 1838 – Publication of books in opposition to slavery and in support of suffrage.

Ø 1839 – invention of rubber vulcanization. Birth control methods included rubber condoms, IUDs, vaginal sponges, diaphragms and cervical caps. 1930 to 1960, these methods were declared illegal.

Ø 1848 – suffrage convention is Seneca Falls, NY publishes “Declaration of Sentiments”

Ø 1850 -- 1st Women’s Rights Convention held at Worchester, Massachusetts.


Ø 1852 -- ‘Uncle Tom’s Cabin” by Harriet Beecher Stowe is published.

Ø 1854 -- Massachusetts grants property rights to women.

Ø 1861 – 1865. The Civil War.

Ø 1865 – adoption of the 13th Amendment, prohibiting slavery.

Ø 1866 -- Merger of abolition and suffrage movements.

Ø 1867 – women lose suffrage in Washington and Utah by action of the Supreme Court.

Ø 1868 -- 14th Amendment adopted. Due process and equal protection.

Ø 1869 – territory of Wyoming grants women suffrage.

Ø 1870 – 15th Amendment adopted. Right to vote regardless of sex, color or creed. Woman are continued to be denied the vote.

Ø 1870 – territory of Utah grants women suffrage.

Ø 1873 – Comstock Law declares birth control illegal.

Ø 1883 – territory of Washington grants women suffrage.

Ø 1893 – state of Colorado grants women suffrage.

Ø 1900’s – women’s movement endorses contraception.

Ø 1909 – 20,000 women unionists strike garment industry; unionists merge with suffragists.

Ø 1910 – territory of Washington grants women full enfranchisement.

Ø 1911 – California grants women suffrage.

Ø 1912 – territory/state of Alaska and Kansas grant women suffrage.

Ø 1914 – Nevada & Montana enfranchise women.

Ø 1917 to 1918 -- WW I

Ø 1918 – Condoms become legal.

Ø 1917 -- state of New York enfranchises women.

Ø 1919 – Michigan, Oklahoma and South Dakota grant women suffrage.

Ø 1919 – 18th Amendment adopted; Prohibition.

Ø 1920 – 19th Amendment adopted, granting women suffrage in the US.

Ø 1930 to 1960 -- Birth control methods included rubber condoms, IUDs, vaginal sponges, diaphragms and cervical caps. From 1930 to 1960, these methods were declared illegal.

Ø 1933 – adoption of 21st Amendment; Repeal of Prohibition.

Ø 1960 era – Rhythm Method misconstrues a woman’s fertility, and fails as birth control.

Ø 1960 – approval of first birth control pill.

Ø 1970 – approval of Dalkon Shield IUD.

Ø 1973 – decision of Roe v. Wade, approving abortion as legal.

Ø 1975 – recall of Dalkon Shield due to pregnancy-related deaths.

Thank You.

What? DirkvdM 09:23, 9 February 2006 (UTC)[reply]
Erm, it seems that pinkyblonde has created a timeline of sexual topics, and wants us to post it to the relevant article.
Pinkyblonde, Wikipedia is a wiki; anyone (even you!) can edit any page by clicking the "Edit this Page" button at the top. If you want to create a new article, you need to create an account first (see Wikipedia:Accounts). — QuantumEleven | (talk) 09:32, 9 February 2006 (UTC)[reply]
Doesn't seem to have much to do with sex. I'd expect a sexual timeline to have things like "capital punishment for homosexual acts abolished in Lithuania" or "Missionary position invented". This timeline may be important, but isn't it more to do with women's and reproductive rights (with bits of slavery) than sex, with a heavy and unstated US bias? Notinasnaid 10:40, 9 February 2006 (UTC)[reply]
That's not a bias, it's specifically about the US. So as long as that is stated, there's no bias. Weird list, although there are some interresting bits in it. Contraceptives were illegal in the USA from 1930 to 1960? That's even weirder than prohibition. Most things would require an explanation, though (or a link), such as the last one. And indeed it's not about sex/ I'm not sure what it is about. It's a semi-random mix of issues. DirkvdM 18:13, 9 February 2006 (UTC)[reply]
It is US bias. If an article is specifically about the US, the title should reflect that and not appear as if it is timeline about sex-related events worldwide. - Mgm|(talk) 08:32, 10 February 2006 (UTC)[reply]
I think they meant "sex" as in gender, not as in sexual activities. Perhaps "female issues" timeline would be a better name ? StuRat 19:12, 9 February 2006 (UTC)[reply]
How about United States history of female issues. - Mgm|(talk) 08:32, 10 February 2006 (UTC)[reply]

"I was all in favor of women's suffrage...until I found out it just meant the right to vote." StuRat 19:12, 9 February 2006 (UTC)[reply]

I'd like to point out that the start of Qukaerism (the first entry in this list) is, when given a specific year is usually given as 1652, not 62. -- AJR | Talk 23:55, 9 February 2006 (UTC)[reply]
Just update History of feminism. But its roots can be found as far as you wish, e.g., during the Crusades, women did acquire sustancial rights due to the departing of men to Palestine. --DLL 11:04, 11 February 2006 (UTC)[reply]

Moral Decline?[edit]

I've heard some Christians say that there is alot more crime,sin, and immorality nowadays in our modern society than in the past.Is that true?

(If you believe in the Bible, read Matthew Chapter 24 and 2 Timothy Chapter 3 in the Bible and tell me if you think it's talking about about our modern world.)Bowei 08:29, 9 February 2006 (UTC)[reply]

Yes, it's true that some christians say that. --BluePlatypus 08:46, 9 February 2006 (UTC)[reply]

No.You've misunderstood what I've asked.I asked "Is it true that there is alot more crime,sin, and immorality..." NOT "Is it true that some Christians say that?"Bowei 09:08, 9 February 2006 (UTC)[reply]

I gave you the answer I felt you deserve. You've been warned to not use the Reference Desk for your religious proselytizations. You aren't asking straight questions so why give straight answers. Take your religious discussions elsewhere, this page is for questions and answers. --BluePlatypus 10:58, 9 February 2006 (UTC)[reply]
Crime is declining in America and the industrialized world in general (though U.S. crime rates remain high relative to peer countries). As of now, crime rates are about as low as they were 30 to 35 years ago. Sin and immorality are concepts generally viewed from the relative standpoint of religion, and I don't think there's a schema that can evaluate whether there's more of them. Marskell 09:20, 9 February 2006 (UTC)[reply]
Same questions, identical references on Feb. 5 ...
What can we say ? That the Bible does not give answers to all ? That truth is is the eye or the beholder ?
Did Paul imagine our great disasters ? Good luck trying to prove something about the destiny of our world. Keep hope, people told everything was bad centuries ago (it was bad for them too : they died). --DLL 09:25, 9 February 2006 (UTC)[reply]
Impossible to tell for a large number of reasons. Here are just a couple. 1) crime is probably more reported now than ever, so it appears there's more of it. Also, news is virtually instantaneous and global, so we know more and more about what happens worldwide. Both of these things make us more aware of "crime, sin and immorality". 2) what is defined as crime, sin and immorality varies over time. Some countries have move towards a more puritanical moral stance at times, and as such, to the eyes of many who live there, more things can be regarded as immoral. At other times in history, those same countries become more lax in their moral code, and as such, the list of things that seem immoral drops. At the moment, one or two of the most vocal societies in the world are in a very moralistic part of their history, and as such, "sin" appears to have increased. All in all, I would be tempted to say that immorality, crime, and sin have remained pretty constant throughout human history, and will continue to remain so as long as humans exist. It is only the moral stance of a number of people which have moved towards the rigidly puritanical in recent years that makes it seem like there is an increase. Grutness...wha? 09:32, 9 February 2006 (UTC)[reply]
Oh, and I forgot to comment on the chapters from Matthew and 2 Timothy. Yes, they do sound like the modern world. And also the early reformation. And the renaissance. And the later years of the Roman Empire. And the Victorian era. It also accurately describes people as they are described in much of the Old Testament. And - people being what they are - probably very much like humans will be in a few hundred years time. Grutness...wha? 12:20, 9 February 2006 (UTC)[reply]
After edit conflict:
What constitutes sin and (im)morality is pretty much determined by the culture you're in, so that's circular. What constitutes crime is a bit more objective. Murder, for example, is bad in all societies, but I doubt there has been little change over time in the amount of murders per capita. Just read the Bible you seem to have and see how much murder there was back then. If you believe in that, that is. Or read any other historical or history book. I'm rather inclined to believe the opposite is true. In the Netherlands about 200 murders take place per year. It won't be too long ago that the population was 1/200 of what it is now. Was there just one murder per year then? I doubt it. Alas, I haven't managed to find real info on this, but I'd say the amount of murders is a good indicator. DirkvdM 09:51, 9 February 2006 (UTC)[reply]
By the way, you say you've heard some christians say that. I hear christians say loads of things, but when I quote them it is genrally not as christians. So they said it as christians. Which instantly, literally, brings us in the realm of belief, not fact. And that is always the end of that, beacuse you can't argue with belief. There are no arguments for or against that. DirkvdM 09:57, 9 February 2006 (UTC)[reply]
When it comes to issues of sexual morality, I suggest you might want to have a look at the diary of Samuel Pepys which gives an insight into the sexual escapades of men in the Restoration. What people were actually doing and what they claimed to be doing were two very different things, and it's probably always been so. --Robert Merkel 11:12, 9 February 2006 (UTC)[reply]
I just want to repeat this so it doesn't get lost as a "hard to tell" point: crime is declining, at least recently, and at least in the United States. It's been declining for a decade and as noted is at the same level (in absolute terms) as it was 30 years ago. Check this from the census bureau www.census.gov/statab/hist/02HS0023.xls. As one example, a drop from 10.2 murders per hundred thousand in 1980 to 5.6 in 2002. Marskell 12:29, 9 February 2006 (UTC)[reply]
It's declining and at the same level as 30 years ago? That sounds like a contradiction, unless the statement is that it had risen in the 1980's (the era of Reagan and Thatcher - sounds credible). But the question is really about a much larger timescale. You're talking about decades, I was talking about centuries. But the reference to the bible suggests we should be thinking about milennea. Which is impossible. How does one compare periods and cultures fso far apart? Even if there were statistics on the old periods, a comparison would be quite impossible. And I'm just talking about murders now. Comparing mortalities is simply a matter of opinion ('belief', like I said). Of course, there were the two world wars, but after that 'peace broke out', as they say; an unprecedented amount of peace in Europe, quite unique in its history. DirkvdM 18:22, 9 February 2006 (UTC)[reply]
While I've not checked the source, here's a very quick reconciliation of the two. With the number of crimes unchanged over 30 years, and the population increasing over the same time, the crime rate has decreased. — Lomn Talk 20:22, 9 February 2006 (UTC)[reply]
You said 'in absolute terms'. I'm sorry, I didn't read carefully enough. DirkvdM 06:04, 10 February 2006 (UTC)[reply]
I suspect that there was less extramarital sex in biblical times, at least in places where they stoned women to death for getting pregnant out of wedlock. Of course, the same punishment might elicit the same rates now, so you can't really say people are "less moral" now, they just have less to fear. I suppose we can always elect Pat Robertson to be President of the United States, and thus find out exactly how stoning women to death for adultery affects the adultery rate, can't we ? StuRat 18:53, 9 February 2006 (UTC)[reply]
Other people have done an alright job of answering your question about moral decline; it's a difficult issue at best. I want to remind you that I addressed your point about those Bible passages above when you posted them before. Please don't continue posting the same material. --George 00:31, 10 February 2006 (UTC)[reply]
StuRat, Maybe there was so much stoning going on because extramarital sex was unstoppable and they had to increase the punishment ever more. :) But you're now assuming that extramarital sex is immoral. Which is disputable. I'd rather say the stoning is immoral, but then I'm condemning another culture based on the rules of my own culture. Which is the very problem I was trying to point out above. You can't do that. Each culture has its own set of rules, consistent within itself, not with the separate rules of another culture. If only people would understand that a bit more then maybe there would be a little more peace on this Earth. DirkvdM 06:04, 10 February 2006 (UTC)[reply]
It's not me assuming that extramarital sex is immoral, but rather most of the world's major religions. I, on the other hand, find most of the world's major religions immoral for their acceptance of slavery, complicity in genocide, subjugation of women, treatment of children as mere property, and their usage to give power to tyrants. StuRat 10:09, 10 February 2006 (UTC)[reply]
Totally with you StuRat! What a great summary! Way to stick it to those who suffer from the delusion that religion was at the forefront of the battle against slavery or that alleged atheist tyrants such as Hitler or Stalin or Mao or Kim Jong Il actually existed. ;-) DJ Clayworth 18:50, 10 February 2006 (UTC)[reply]
Religion was also at the forefront of the battle for slavery, was the reason for the Crusades, The Spanish Inquisition, the Israeli/Palestinian conflict, and countless other massacres/conflicts. Is it the only source of evil in the world ? No. Is it a major source of evil in the world ? Yes. Also, I believe the NAZI's had a religion of sorts, which was a mix of Christian and Norse mythology themes. StuRat 06:17, 12 February 2006 (UTC)[reply]
The Deutsche Christian group, among others, combined the NAZI party goals with Christianity. See their flag (with a swastika at the center of the cross) here: www.ethicsdaily.com/index_files/hitler.jpg. I suggest viewing Theologians Under Hitler for thorough coverage of the relationship between the NAZI party and Christianity: www.ethicsdaily.com/article_detail.cfm?AID=5817. StuRat 19:49, 12 February 2006 (UTC)[reply]
I notice a flaw in my reasoning. Extramarital sex is by definition immoral because marriage entails a promise to not have sex with anyone else ever again. So one shouldn't get married in the first place. So one could say that marriage is immoral because it means the denial of a natural drive. "Don't lead us into temptation." Well, don't get married then. Similarly, one could reason that religion is immoral. Take that, reli-whatevers. :) DirkvdM 12:32, 11 February 2006 (UTC)[reply]
I think the rise in questions like this one is kind of a moral decline, or perhaps an intellectual decline. User:AlMac|(talk) 20:54, 10 February 2006 (UTC)[reply]
I'm going with the latter. Natgoo 11:59, 11 February 2006 (UTC)[reply]
To which you are now contributors. :) DirkvdM 12:32, 11 February 2006 (UTC)[reply]
DirkvdM, please explain how you arrive at the conclusion that marriage means the denial of a natural drive. It is a natural drive to have sex; traditionally, marriage provides virtually unlimited opportunities for that, and without any of the social srictures associated with exra-marital sex but with society's full approval. How is that a denial of the drive? I would have thought it was the very opposite. JackofOz 12:56, 11 February 2006 (UTC)[reply]
Well, the real drive is to multiply as much as possible and in the case of a man that means mating with as much women as possible because they take so bloody long to produce that offspring (no offence, ladies :) ). And ever wondered why your penis is shaped the way it is and why you do all the pumping? That's to get the sperm left behind by the previous guy out of the vagina. Think if that next time you have sex. :) Promiscuity is natural. DirkvdM 08:17, 12 February 2006 (UTC)[reply]

semi-annual white space[edit]

This space is provided for anyone to write whatever they please, intelligent or stupid, relevant or not, just to get their jollies. Limit: 100 words per person. Closing date: 10 February 2006.

Let the nonsense begin! --Halcatalyst 15:41, 9 February 2006 (UTC)[reply]

Seems like the ref desks get their fair share of people writing whatever they please regardless of any specifically declared season. Of course, some do break the 100 word limit. LarryMac 15:56, 9 February 2006 (UTC)[reply]
If I'm two people, do I get 200 words?  Run!  16:19, 9 February 2006 (UTC)[reply]
Do you need that many words to describe your schizophrenia? DirkvdM 18:26, 9 February 2006 (UTC)[reply]

"There are very few problems in this world which can't be solved by a suitable application of high explosives." - Demotion expert StuRat 18:41, 9 February 2006 (UTC)[reply]

Lorem ipsum dolor sit amet, consectetuer adipiscing elit. Nunc iaculis, massa et porttitor commodo, magna purus dictum justo, id fringilla nulla felis sed orci. Sed pharetra sapien vel mauris tincidunt bibendum. Suspendisse facilisis ante. Phasellus est enim, venenatis posuere, consectetuer et, pharetra ac, tellus. Nunc fringilla ipsum sit amet risus. Fusce consequat. Vivamus tincidunt ultricies nunc. Duis facilisis nisl. Ut suscipit sapien. Cras lectus enim, molestie sed, adipiscing eget, adipiscing vitae, arcu. Etiam nibh lectus, nonummy id, tincidunt a, commodo vitae, felis. In ante ipsum, tempor et, posuere vel, malesuada at, pede. Curabitur ullamcorper neque id lorem. Etiam et tortor. --BluePlatypus 20:06, 9 February 2006 (UTC)[reply]

Bravo! --LarryMac 20:50, 9 February 2006 (UTC)[reply]

Ok, lets see how much it's possible to concentrate into a few words.

This Reference Desk has a fever. And the only prescription is...More Cowbell!


Yep, that should do it. Shame the marquee tag is disabled on Wiki :).GeeJo (t) (c)  20:54, 9 February 2006 (UTC)~[reply]

DirkvdM- schizophrenia isn't mpd. Black Carrot 02:09, 10 February 2006 (UTC)[reply]

I have nothing to say, and I'm saying it. JackofOz 02:32, 10 February 2006 (UTC)[reply]

I'm inspired to write a Limerick,
For it's the perfect nonsensical gimmick-ik.
Will it rhyme in the end?
We could all just pretend.
Or we could all say shucks! ...oops, I meant fiddlesticks!
-LambaJan 02:48, 10 February 2006 (UTC)[reply]

§§§ Why didn't Hitler invade Switzerland??? --Halcatalyst 03:08, 10 February 2006 (UTC)[reply]

The answer can clearly be found in the New Testament if you read it carefully enough. Which only goes to show that things are more like they used to be than they are now. Grutness...wha? 04:44, 10 February 2006 (UTC)[reply]
I laughed out loud. Thank you both. :-) Jwrosenzweig 06:36, 10 February 2006 (UTC)[reply]

I've changed my mind, and I still have 191 words left, so here goes. Slonimsky's definition of sesquipedelian macropolysyllabification is: Quaquaversal lucubration about pervicacious torosity and diverticular prosiliency in diatonic formication and chromatic papulation, engendering carotic carmination and decubital nyctalopia, causing borborygmic susurration, teratological urticulation, macroptic dysmimia, bregmatic obstipation, crassamental quisquiliousness, hircinous olophonia and unflexanimous luxation, often produce volmerine cacumination and mitotic ramuliferousness leading to operculate onagerosity and testaceous favillousness, as well as faucal obsonation, paralellepipedal psellismus, pigritudinous mysophia, cimicidal conspurcation, mollitious deglutition and cephalotripsical stultitiousness, resulting despite Hesychastic omphaloskepsis, in epenetic opisthography, boustrophedonic malacology, lampadodromic evagination, chartulary cadastration, merognostic heautotimerousness, favaginous moliminosity, fatiscent operosity, temulencious libration and otological oscininity, aggravated by tardigrade inturgescence, nucamentacious oliguria, emunctory sternutation, veneficial pediculation, fremescent dyskinesia, hispidinous cynanthropy, torminal opitulation, crapulous vellication, hippuric rhinodynia, dyspneic nimiety and favillous erethism, and culminating in opisthographic inconcinnity, scotophiliac lipothymia, banausic rhinorrhea, dehiscent fasciculation, oncological vomiturition, nevoid paludality, exomphalic invultuation, mysophiliac excrementatiousness, flagitious dysphoria, lipogrammatic bradygraphy, orectic aprosexia, parataxic parorexia, lucubicidal notation, permutational paronomasia, rhonchial fremitus, specular subsaltation, crapulous crepitation, ithyphallic acervation, procephalic dyscrasia, volitional volitation, piscine dermatology, proleptic pistology, verrucous alopecia, hendecaphonic combinatoriality, microaerophilic pandiculation and quasihemidemisemibreviate illation. JackofOz 07:00, 10 February 2006 (UTC)[reply]

  1. Said Yes To Drugs
  2. Puberty is a Disease State
  3. You Must Be This Tall to Ride the Wikipedia
  4. I'd Rather Be Sleeping
  5. Life Is a Waste of Money
  6. The Pathetic fallacy Wants You.

Geogre 18:45, 10 February 2006 (UTC)[reply]

I heard in the discovery channel that the bible (especially read : Matthew Chapter 24 and 2 Timothy Chapter 3) predicted a downfall of the world's economy in 2002...has this any truth to it? and what did Gödel think about this? --DLL 20:36, 10 February 2006 (UTC)[reply]

Yes, the world did end in 2002, but it was reconstituted in toto in the time it took light to travel the Planck length. Gödel issued an incomplete statement about the event, but Bertrand Russell agreed with it in principle. Douglas Hofstadter, uncharateristically, was mute. --Halcatalyst 00:10, 11 February 2006 (UTC)[reply]

Copyrights: Image to be used in ad[edit]

I just received an email from someone asking me if they could put my Photo booth image in an ad for their store. Since the guy was nice enough to email, I doubt he would use it without my permission, but I just want to check something: The image is dual-lisenced GFDL and CC-by-sa. Is there any way that I could order someone not to use it in an ad? My initial guess is that if I wanted to stop something like this I ought to have put a non-profit tag on it. Am I right? — Asbestos | Talk (RFC) 20:44, 9 February 2006 (UTC)[reply]

  • If you've licensed it as GFDL or CC-BY-SA then no, you can't stop them from using it in an ad (at least not for any reason having to do with copyrights) so long as they abide by the terms in those licenses and technically you're not even supposed to be able to change the licenses after you've released it as such. --Fastfission 21:22, 9 February 2006 (UTC)[reply]
That said, having a prominent copyright notice in an advertisement is not likely to be popular with advertisers, so I don't expect wide use of Wikipedia images in that way. --Robert Merkel 23:40, 9 February 2006 (UTC)[reply]
This copyright shit is driving me nuts. I've licensed my photos as cc-by, assuming that my name would hae to be printed alongside each photo. But then when I saw a site that didn't do that and wrote them, they said that my name would pop up on mouse-over. True, but that's not what I was thinking of. And the photo is embedded in a top bar, not looking like a separate photo. Now who is going to mouse-over over that? DirkvdM 06:26, 10 February 2006 (UTC)[reply]
Well, the license states that the credit has to "appear where any other comparable authorship credit appears and in a manner at least as prominent as such other comparable authorship credit." so it may actually be a license violation. On the other hand, there's nothing stopping you from using a more specific license. This is what lawyers are for: Making sure that what you are thinking of is actually what most people and a court would think of. (So they do have their uses.) --BluePlatypus 17:56, 10 February 2006 (UTC)[reply]

February 10[edit]

Finding Movies[edit]

I've been having trouble finding a copy of Tales from the Far Side and Tales from the Far Side II. About the best I've found is one on the English amazon.com (I had no idea there even was one) that isn't compatible with my VCR. Does anyone have suggestions? Black Carrot 03:29, 10 February 2006 (UTC)[reply]

I assume you're in America and hence use the NTSC video system, I can see two options:
  1. Buy a dual NTSC/PAL VCR - they are cheap(ish) over here (the UK), don't know how expensive they are over there!
  2. Get a friend in the UK to transfer it to an electronic copy, (convert to DVD or another digital video format) and mail both two you - I am not sure if that's illegal (IANAL!), but it would be a backup (assuming (s)he doesn't retain the copy), fair use?
Sorry if that's not useful, best I can think of, will reply if I think of anything else! --Lox (t,c) 09:22, 10 February 2006 (UTC)[reply]
Several camera/video stores can perform a transfer. You have a licensed copy, and you can transfer it. I think this is not considered copying. You should call some and make sure they are able. Beyond that, university libraries will definitely have players that can handle that zone, if you're near a college town. Geogre 11:37, 10 February 2006 (UTC)[reply]
Well, it's considered copying, but it's not usually considered copyright infringement. In most places making a copy of something you own for personal use is within the doctrine of 'fair use'. I hardly think anyone has ever been fined for copying a CD to cassette for playing in their car. --BluePlatypus 17:59, 10 February 2006 (UTC)[reply]

I'll try transferring things around or buying a new VCR if I can't find another way, but I'd rather just find a copy that started out in NTSC format. Does anyone know where I could find one? Black Carrot 20:04, 10 February 2006 (UTC)[reply]

Playing music over public address system - Copyright infringement?[edit]

At our high school in Canada, it has been a custom to play popular music over the PA system shorty before the school day begins- mainly as a cue to tell students to get to class. Recently the school board has stopped this practice, claiming it is copyright infringement. The music is generally played off of purchased CDs and normally only about 5 minutes of music is played. I suppose they have merit to the argument, but is this really copyright infringement? Thanks for any input :) Mrtea (talk) 05:53, 10 February 2006 (UTC)[reply]

I am not a lawyer (IANAL), but I think that technically it might be copyright infringement if your school does not have a license for public performance (it should have one if your school has a drama or musical society, film club or disco). However, I think it's very unlikely that your school will be prosecuted for playing five minutes of music once a day. Someone with more knowledge of performance laws might be able to give you a better answer. — QuantumEleven | (talk) 09:01, 10 February 2006 (UTC)[reply]
yes, it is. The license on the CD only qualifies you to use it for personal use, it doesn't qualify you to play it in a club, or in a school, or mix it into your hot new dance single. My college was recently threatened by www.swank.com/ Swank Motion Pictures, a company that you've probably never heard of but plays a rather important role of licensing virtually all public performances of films from major studios outside of theaters. That warning at the beginning of a movie that says it is licensed only for private, in-home performance means exactly that, you're not licensed to play it in public, and CDs are no different. They could be exposing themselves to legal liabilty if they violated the license in that way. Night Gyr 09:15, 10 February 2006 (UTC)[reply]

If you are a department store, for example, I believe you have to have a license for the 'music' that you play to keep your customers happy. This is why most department stores play cheesy covers of popular songs (rather than the originals) which they can license cheaply. DJ Clayworth 18:45, 10 February 2006 (UTC)[reply]

Sorry for the late "thank you." The responses are appreciated :) Mrtea (talk) 18:05, 11 February 2006 (UTC)[reply]

the title of a song[edit]

this song i heard has almost the same tune as that of "Uninvited" by Alanis Morisette for the first few beats. yet this is not a rock but a pop (maybe a ballad) and has a bit slower rhythm. from the sound of the tunes, it can't be anything that came out recently or even in the late 90's. can anyone figure out what this song is? oh, i heard this song featured in a TV commercial (don't remember which one, tho). please let me know. Thanks! --hoodbuster

Can you remember any of the lyrics? Click "edit" on the right handside of the heading to answer. - Akamad 14:00, 10 February 2006 (UTC)[reply]

Prohibition+Constitutional Law[edit]

why was it necessary to make prohibition an amendment to the constitution? (as opposed to just a law)

e.g., was there something in the constitution to prevent congress from passing laws on alcohol? and what's the difference with today's laws and regulations that make all kinds of drugs "probited" even worse: the alcohol prohibition was only manufacture, sale, and transportation. we now outlaw possession,too. how can we do this, but not be able to prohibit alcohol? hmmm... --70.181.177.72 06:35, 10 February 2006 (UTC)[reply]

I don't know the answer, but just want to add that you are presumably talking about the United States. StuRat 08:43, 10 February 2006 (UTC)[reply]
The subjects on which Congress is allowed to legislate are listed in Article I, Section 8 of the United States Constitution, and the 10th Amendment made it explicit that all other powers fall under state jurisdiction. You can certainly find people willing to argue that U.S. laws such as Controlled Substances Act are unconstitutional and that the need for an 18th Amendment for alcohol prohibition demonstrates the point. Evidently the U.S. Supreme Court disagrees. --Anonymous, 09:02 UTC, February 10.

Congress is limited by the commerce clause, which says that all domestic legislation must relate some how to "interstate commerce" because a complete ban on alchohol might not be considered as relating to interstate commerce, a new amendment had to be passed.

Once a big fan of the "Arthur" cartoons, what if a real 3rd grader stays the same age like him?[edit]

The first "Arthur" book (for more info, see Arthur (TV Series)) was made in 1976, eventually made its way onto the air, and is still going strong to this day. Arthur never grows up.

So (and I've been wondering this for a while), if a 3rd grader, in real life, somehow got a disease which ONLY caused him to never physically grow beyond an 8-9 year-old's body, what could/would happen after the doctor concludes that he'll stay this way for life, and how could he cope with it for the rest of his days? (Although his life could be theoretically infinite in this case, as he'll never reach the frailty and medical vulnerability of old age...) --Shultz 07:14, 10 February 2006 (UTC)[reply]

I think there are conditions that cause delayed puberty. Presumably if this happened forever, with no other problems, he would finish his schooling, get a job, learn to drive, etc., and otherwise lead a normal life. Finding a g/f or wife might be a problem, since not many women would want to date someone who looks like a kid. He would also need to carry a doctor's note for policemen who pull him over to hassle him. I don't think his life span would be all that much longer. First, a delay in the onset of puberty does not seem to mean a delay in the onset of old age, as those two things use different timing mechanisms (see this telomere articles for one old age timing mechanism users.rcn.com/jkimball.ma.ultranet/BiologyPages/T/Telomeres.html). And, old age is also due to accumulated poisons (like trans-fatty acids) and cell damage (like ultra-violet light damage). StuRat 07:57, 10 February 2006 (UTC)[reply]
Interestingly, most cartoons feature non-aging characters. Aging characters present a challenge to the illustrator, as they need to periodically revise the appearance, and can lose their fan base, who tend to like characters that never change. There are a few exceptions. The newspaper cartoon Luanne's www.jsonline.com/enter/gen/mar04/214962.asp characters appear to have aged at a very slow rate, then stopped, while the newspaper cartoon For Better of For Worse www.rag.org.au/bcc/cartoon.htm has characters aging at the normal rate. StuRat 08:36, 10 February 2006 (UTC)[reply]
Actually, those sorts of issues can lead to a whole other set of problems. The longer you have your baby teeth, the more they will wear down, and eventually will become painful. Joint problems are also more likely, as well as a higher likelihood of repetitive stress injuries. Other injuries are also likely, as the world in general is not really designed for a person who's still physically a child, just as one's body isn't meant to remain a child forever. There are also certain cognitive changes that happen at puberty; even an extremely intelligent child may get confused around adults, although a kid may not be less capable on the whole, the actual results with the same mental hardware will differ. A child that outlives his parents and still remains a child (though legally an adult) might also face other problems.. -Tim Rhymeless (Er...let's shimmy) 08:50, 10 February 2006 (UTC)[reply]
What other problems, Rhymeless?
By the way, whoever said about accumulated poisons and UV lights, assume this diseases renders the child impervious to it. Maybe this disease was concocted by an eccentric scientist. About children's teeth, assume this disease makes them automatically grow back each time. --Shultz 22:28, 10 February 2006 (UTC)[reply]
Well, if you are interested in a fictional case of boys who stay young forever, try reading Peter Pan. As I recall, they enjoyed being kids, but, at the same time, there was a certain sadness at never knowing what it's like to grow up. StuRat 05:57, 12 February 2006 (UTC)[reply]
There was a documentary on the Beeb about this recently. A man with some sort of genetic disorder didn't produce testosterone, which meant that he looked/sounded/developed as though he was ~12 when he was 30, had a course of testosterone injections, with startling results. smurrayinchester(User), (Talk) 16:21, 12 February 2006 (UTC)[reply]
Whoa, I'd like to read more about that documentary! Where can I find more info about it?? --Shultz 05:14, 16 February 2006 (UTC)[reply]

Blasphemy[edit]

I saw something on NIGHTLINE two nights ago on blasphemy and they quoted some SCOTUS case from 1952 or something. Can someone look this up for me at the ABC site? I can't access it for some reason. --Blue387 09:11, 10 February 2006 (UTC)[reply]

  • This might be the 1952 case you looking for ... is it Ok to urinate on the image of Christ? www.aclu.org/scotus/1997/22767lgl19980206.html
  • abcnews.go.com/Nightline/
  • ABC on the Danish Cartoons that have a microscopic portion of the Islamic world in an uproar, being treated by the news media like Armageddon is at hand from the Muslim street protestor gangs. abcnews.go.com/International/wireStory?id=1569192
  • ABC on other news media of Europe reprinting the cartoons because freedom of the press is more important than respecting the sensibilities of people who see nothing wrong with doing the same kind of stuff when the topic is Jews. abcnews.go.com/International/wireStory?id=1565688
  • My personal opinion is that everywhere in the world there are people who have nothing better to do that cause mischef.
    • There are people who know that attacks on religion are asking for trouble from true believers, but they do it anyway.
    • There are people who know some true believers can easily be fomented to get into these riots, so they publicize what was done in another country, make up some cartoons that are worse than the ones originally printed, and pass them off as part of the same collection. They are trying to recruit people to serve the cause of anti-Western sentiment.
    • The answer here is to teach the religious fundamentalism about secular freedoms, and at the same time review the laws on libel and hate, to see if any adjustments are needed to block this parallel to shouting fire in a crowded theatre, or hi (to my friend) Jack, on an airplane.
  • Also I think that people who burn the Christian Cross are in some way also attacking the Church, because the cross is kind of sacred, although they act like the attack is only racial.

User:AlMac|(talk) 09:50, 10 February 2006 (UTC)[reply]

  • There doesn't seem to be anything about the SCOTUS case in those articles though, although I'm pretty sure it's www.bc.edu/bc_org/avp/cas/comm/free_speech/burstyn.html Burstyn v. Wilson. --BluePlatypus 09:59, 10 February 2006 (UTC)[reply]
    • Yes, it turned out to be Burstyn v. Wilson: "It is not the business of government in our nation to suppress real or imagined attacks upon a particular religious doctrine, whether they appear in publications, speeches, or motion pictures." --Blue387 17:13, 10 February 2006 (UTC)[reply]

Chinese energy security[edit]

Does anyone have any suggestions as to where I can find some information on China's search for energy resources abroad, specifically on its increasing ties with the developing world? It is a homework question, sort of, since I am asking for sources for a short paper, but the help would be appreciated. --Impaciente 09:16, 10 February 2006 (UTC)[reply]

I don't know if this is the sort of thing you are looking for, but I happen to have recently read a few partially related online news articles:
  • www.granta.com/extracts/2616
  • news.bbc.co.uk/1/hi/business/4587374.stm
  • news.bbc.co.uk/1/hi/world/africa/4619956.stm
  • www.guardian.co.uk/china/story/0,,1694346,00.html
I hope this helps. Pissant 13:39, 10 February 2006 (UTC)[reply]

Research in Mayan culture[edit]

What is the status of current resaerch in Mayan culture? --HappyCamper 13:02, 10 February 2006 (UTC)[reply]

There is a lot and its ongoing. Is there a specific question? (The oldest copy of the Popol Vuh is being digitized at the Newberry Librarywww.chicagotribune.com/news/nationworld/chi-0602040135feb04,1,4547201.story?track=rss, the oldest known writing in Mayan was recently discoveredmetimes.com/articles/normal.php?StoryID=20060106-032042-7358r, dated to 3rd century BC, the oldest known Mayan mural has recently been uncoveredwww.mesoweb.com/reports/SanBartoloWest.html, etc.) Rmhermen 17:50, 10 February 2006 (UTC)[reply]
Not only that but, contrary to popular belief, Mayan culture never totally disappeared. You can read about the ~six million living Maya peoples. --Halcatalyst 23:59, 10 February 2006 (UTC)[reply]

Regarding eye-strain while using a computer[edit]

Is there any eyeglass available specifically used when seeing a computer? (Is there any special glass which can be used while using a computer?)

What is the name of that glass?

Can we use sunglass while using a computer? Or would a Anti-reflective coating glass be of any use?

Please don't post your question in more than one board (see the rules at the top of the page) - it only irritates the volunteers who dedicate their free time to answering it. For answers to your question, check the Science Desk. — QuantumEleven | (talk) 14:28, 10 February 2006 (UTC)[reply]
Becoming irritated but not yet biting newcomers we desk jedi are (did you try just using your eye on something else than a comp screen). --DLL 19:36, 10 February 2006 (UTC)[reply]
  • Tell your supplier of eye glasses what is the normal distance you sit from the computer screen. You can get prescription optimized for that distance.
  • Place your overhead lighting so as to minimize glare and interference.
  • Clean the monitor regularly (no dust there).
  • When buying replacement monitor, very carefuly self-educate yourselv about resolution, and seek one in which there is as little flicker as practical. The bigger the better, and go beyond SVGA.
  • There's other stuff that can be done, but it is generally too expensive for the average consumer.

User:AlMac|(talk) 21:00, 10 February 2006 (UTC)[reply]

Liking Songs[edit]

Why is it that with some songs, you can listen to them once and like them almost immediately, but with other songs you have to listen multiple times before liking them? --Reperire 16:52, 10 February 2006 (UTC)[reply]

Sometimes it's because other people said they liked it. See news.yahoo.com/s/hsn/20060209/hl_hsn/musictoyourfriendsears new research. Rmhermen 17:37, 10 February 2006 (UTC)[reply]
Also simpler songs have choruses that are picked up on right away, whereas other songs have maybe no chorus (or discernible melody) at first, because they are more complex --PopUpPirate 21:19, 10 February 2006 (UTC)[reply]
There are certain cadences that have been found to be likable, and thus transmitted over several generations in the form of folk songs and other shared music. It's precisely a level of simplicity that keeps it accessable. This is also something that's pretty universal. Most cultures have a body of music that's transmitted from one generation to the next, to youth, that is often found to be pleasant, if foreign, to the uninitiated. Songs you instantly like are most likely ones that have these qualities, or ones that have similar qualities to less accessable songs that you already brought yourself to like. -LambaJan 17:27, 13 February 2006 (UTC)[reply]

sociology( education and develoment)[edit]

currently i am working on a research paper for my sociology major and i need your help. my hypothesis is " Education is the most critical factor in the shift from underdevelopment" and the topic is "Education and Development" this paper must have a sociological foundation ofcourse and requires insite on the plight of third world countries. particularly the caribbean (west Indies) could u be of any assistance and forward whatever infomation that is relavant to this topic i would greatly appreciate that.

thank you and may god bless u richly for ur knowledge

Start with our page on Critical pedagogy, and then go and get yourself a copy of Paulo Friere's seminal work Pedagogy of the Oppressed. I'm a bit out of touch with recent theoretical developments in the field, but a search through your uni library's catalogue for 'education and underdevelopment', 'sociology of development', 'caribbean underdevelopment' etc would be a good start. Google might even be of assistance! Natgoo 20:14, 10 February 2006 (UTC)[reply]

Tobacco smoke as product placement in films[edit]

Dispite their promises not to, tobacco and film production companies are showing more and more sceenes with popular actors smoking. Now, I'm not surprised at their attempts, but what I can't figure out is why they are not being forced to comply with new legislation and the contracts they signed.

Especially, they are increasingly targeting minors. Something like 75% of movies made for the 13 to 18 year old age group have casual smoking sceenes. Even recently produced Disney movies! It makes artistic sense to have smoking in a gangster film, sure. But I have seen many very obviously contrived sceenes, very obvious tobacco advertising, inserted into kids movies. That's patently illegal. It is strictly illegal for tobacco companies to use paid product placement in any movie.

But if it is truly uncompensated, how are the tobacco companies convincing the production companies to do it? And if it is compensated, how could they possibly get so many people to to shut up?

There is not much mention of this in the general media. Some, yes, but not much at all. We hear lots about the health effects of smoking from non-profit organizations like the American Heart Association, but hardly a whisper about tobacco advertising in movies. Why? Do any of the tobacco giants own any of the major media companies. I'm not being facetious, here, or even simply rhetorical. Honestly could that be part of it?

More - how do these corporations con otherwise socially responsible actors like Julia Roberts and Sean Penn to perform as such incredibly damaging role models? Directors can tell minor or suporting actors to smoke for the camera, but a star can easily refuse the script. Don't very famous celebrities care abgout their millions of fans? Since they smoke so publicly, so normally, in movies, why are they so quiet about tobacco use in interviews and other situations outside of the studio? Even as smokers in their private lives, full well knowing what they are doing, why would they voluntarily assist in this enormous scheme unless they are actually being somehow rewarded?

Now I know this doesn't sound much like a question anymore. But honestly, it is more than just my rant. I genuinely can't figure out how all these people are able to so blatantly get away with this. I'm amazed. Any theories, anyone? --- Leah

I don't really understand your question - some people smoke in real life, so in order for films to realistically portray people, some of them will be smoking. This is particularly true given the types of characterisations we tend to see in films - social misfits, bad guys, gangsters, etc. Smoking is still accepted in all other major arenas of film production, why should Hollywood have to limit the ways it portrays life? Kids see people smoking in real life, too - I'm sure some of them even smoke themselves. If you have a specific complaint to make about a breach of the law, can you report it to someone at the MPAA (or does the US have some sort of communications ombudsman-type office)? Natgoo 20:23, 10 February 2006 (UTC)[reply]

My clarification of my question is this: Is it plausible that the tobacco companies are playing absolutely NO role in the undeniable, yet dramatic increase in on screen smoking, particularly in movies made for teanagers? What you are refering to is, I think, artistic license, which I've addressed in the original question. What I'm really asking about is the suspicious look of paid advertising. Most kids do know what smoking is of course, but unless a parent smokes, the image is not frequently displayed by a role model. How often does a kid actually see their school teacher smoking? Also, actors smoking in movies, impress tens of thousands of people with each exhalation. A stranger seen in smoking public outside the 7-11, is not any where near as influential as a Hollywood star, and certainly not seen, in each instance, by thousands of people.

It is against the law for US tobacco companies to pay US production companies to show smoking on screen. If the tobacco companies are not compensating the film makers, then why do the film makers include so many tobacco usage images, in unappropriate contexts in films (not plausably explainable as "artistic license")?

Again, I'm not really understanding - has there been an undeniable, dramatic increase in on-screen smoking? If so, it's not at all evident to me (but I haven't sat down and counted each instance of smoking in Hollywood films - perhaps you have? Or do you have some verifiable evidence to support your claim?). I just don't think it's as insidious/ conspiratorial as what you make it out to be - most instances of smoking on film most likely occur when the director/scriptwriter/actor feels it suits the scene (or when the actor wants a cigarette :). As I said in my response above, if you have a specific complaint about a breach of the law, report it. Otherwise, product placement in general is only going to increase in films and visual broadcast media as people restrict their access to advertising through the more traditional means - it's just something else we'll become inured to over time (if we're not already). Natgoo 22:11, 10 February 2006 (UTC)[reply]

Yes, you are not understanding my question. Yes, some people, but not me, count smoking sceenes in all sorts of movies, and even count how many of each age group watch the movie when it is first realeased. Yes there are think tanks that boil these things down in statistical analyses. Yes, there is loads of verifiable evidence to support the claim. I don't know how to make hyperlinks, but if you type "smoking in movies," or "smoking in films," in your browser, you will be provided with a substantial number of credible links. But, I also know that some people, once commited to an opinion will not allow themselves to reconsider.

I didn't write my initial question to generate a debate. I didn't write it to lecture anyone on the dangers of smoking. (there are still people who think smoking is not an unhealthy habit, but I know that these are people who are commited to opinion and will not allow themselves to reconsider). I certainly didn't write it to convince anyone that the film industry promotes smoking. I believe that the film industry intentionally promotes smoking, but I didn't write the question to convince anyone of that.

My question takes it as an axiom, that the film industry is intentionally showing smoking in movies for some reason other than simple artistic license. If you don't start with the belief that I have then, there is no doubt that you will be unable to answer the question which is generated from my my belief. You are arguing about the validity of my premisis. I am posing a question, based on a phenomenon I believe to be true. If you and I disagree about what color the sky is at night, I'm certainly not going to expect you to explain why it is mostly black. As I said before I didn't come here to convince anyone of anything. I came here to get a rational explaination for a social phenomenon. Some people just thrive on controversy. ---Leah

I can think of some possible innocent reasons:
  • They use smoking to identify a bad guy, or at least a controversial or mysterious one. The "smoking man" in The X-Files might be a good example of this.
  • They need the character to have something to do with their hands.
  • They need the character to have some reason to take a "dramatic pause". Lighting a cigarette, taking a puff, or putting it out can all provide such a pause. The movie Giant used this.
  • They need a source of ignition for a fire. The movie Damnation Alley is a good accidental example of this, and the Clint Eastwood western where he lit dynamite with his cig is an intentional example.
  • They need a source of conflict. Fighting over where smoking is allowed, etc., can provide that conflict.
StuRat 01:19, 11 February 2006 (UTC)[reply]

First, let me emphasise that I never wanted a debate about this.

Yes, there are alternative reasons. All listed above are examples of artistic license, as previously mentioned, and previously excepted from the question. But only so much of this can reasonably be attributed to artistic license. (By the way, its not only the "bad guy" or social misfit who's smoking; The protagonist smokes more often than the antagonist)

Anyway, Natgoo has, unintentionally I'm sure, provided me with a solution. People are gullable. People simply don't realize or care that they're being dangerously manipulated. Especially, they are unaware that they are being advertised to, and are therefore more subjected to it. Adults, who have largely decided whether to smoke or not, are not thinking - are not protecting their kids. I guess I was wrong about actors. They don't realize what's going on.

Since 2002, more teenagers (from non-smoking homes) have started smoking because they saw it in movies, than for any other reason. It really doesn't matter to me - it matters to them, whether they ever even know it or not. They are being taught to smoke before they are mature enough to form an adult decision. I didn't mean for this discussion to turn this way, I was only, genuinely looking for an explanation.

I'm only sorry that it came from you, Natgoo, subjectively instead of someone else objectively. I realized from you, by example. You don't see it, and you can't be convinced that your missing something.

As you said, you don't understand the question. Sad --- Leah

I'm sorry that you didn't receive the affirmation you were seeking when you made your post. I don't think gullibility has much to do with it at all - people (including children) are far more critical media consumers than you seem to give them credit for. The one literature review study I found (pediatrics.aappublications.org/cgi/content/abstract/116/6/1516 abstract here) (which, contrary to your statement above, seems to have received decent media coverage if all the google links referring to it are anything to go by - it just missed me) draws specious conclusions (as you quote above), and doesn't consider a wide range of variables contributing to the uptake of smoking by kids. Placing the blame on movies is a simplistic response, and one that contributes well to an emotional, simplistic argument. If I were to posit the major contributing factor to the uptake of smoking by kids it would be that their parents/society told them not to (I'm sure there would be a correlation between the population of kids who always do what you tell them not to and the population of kids smoking, if the first could be measured).
I think StuRat has given you some good answers to (what seems to be) your question, but overall I think the reason that actors, governments and people in general just don't care is because it's really not that high a priority given everything else going on in the world today. Calling people gullible and sad because they don't see the world in exactly the same way as you is disingenuous and again, simplistic. Natgoo 10:52, 11 February 2006 (UTC)[reply]
I've read only part of the above but would nevertheless like to contribute the observation that in (specifically) US movies almost every time someone smokes some (indirect) statement is made. And that statement is always negative. And that may very well have something to do with the US government sponsoring that. It's almost always the bad guys who smoke, for example. DirkvdM 12:57, 11 February 2006 (UTC)[reply]

I had just done a lot of work; I had found many sites and studies which supported what i was saying. I was trying to put hypertexts in. When I clicked on save, I saw a message that said someone else had edited this, since I began. I think my comment has been lost in Wikiland. I have not been a Wikipedia reader long enough to know how to get my work back.

Why are there smoking sceenes in Disney movies? I don't think children are stupid. But I do think they are innocent. That is why they should be protected from this (smoking in movies directed at their ages) until they are adults. I think someone has to be very clever to be able to actually get smoking sceenes into Disney movies, even though it is against the law and against plain good ethics and common child welfare sense! I was not looking for affirmation. I re-explained the question several times, yet the person who could not understand the question called me "simplistic." The actual question has since been answered.

One reason it works so well for the tobacco industry is that people don't even realize its advertising, so they are less apt to notice and think about it. ---Its all artistic expression!

Anyone can google: smoking in movies and find a great many websites that generally concur that all this smoking in movies must be much more than "artistic expression" as well as the several studies that show kids are being manipulated. More kids start smoking because they see it in film than for any other reason.

Like Natgoo, tens of thousands of teanagers are thinking Oh so what? well, this is "what."

I would appreciate it if someone knows how to get my lost wikiedit back. Thanks ---Leah

All of the websites/media reports that google returns that 'generally concur' cite the same information produced by the study I linked to above, a study (by a single issue research centre) that draws the same specious conclusion that you insist on repeating (by the way, it helps your readers if you indicate a direct quote with quotation marks, otherwise it reads as if it's your personal opinion). Leah, this is obviously something you feel very strongly about, but if you didn't want your assertions discussed then you shouldn't have posted in such a public space.
I'm not sure how your edit was lost - did you get a screen saying 'edit conflict'? If so, and if you didn't copy your additions before trying to save, then they're lost for good I'm afraid. If you'd like more information on contributing to Wikipedia there are lots of resources available at the WP:Help and Community portal pages. Natgoo 17:32, 11 February 2006 (UTC)[reply]
Why, it's already banned in news.bbc.co.uk/2/hi/south_asia/4598829.stm movies and also in news.bbc.co.uk/2/hi/south_asia/3678005.stm public places. Which country were you talking about?

I can't say whether or not there is more smoking in movies or not, or whether Hollywood is being paid off by tobacco companies...but I can say that in my own attempts to quit smoking (which I started as a kid primarily because my mother smokes), I have gone so far as to take actual courses to educate myself about smoking. Now, as I said already, I can't say what is happening in the movies - I have become incredibly more aware of smoking around me, but I haven't noticed an increase in the amount of smoking I see in movies - but I can say that the tobacco companies do hire psychological specialists to aid with their marketing (as many companies do) - or at least they did while they were still allowed to advertise - and they have conclusively found that women are much more likely to start smoking than men, and that if a person does not start smoking by the time they are 18 they are highly unlikely to ever smoke...as a result, when it was legal to advertise cigarettes in mainstream media (magazines, billboards, etc) the ads were designed to be attractive to girls between the ages of 9 and 13. bcatt 04:33, 16 February 2006 (UTC)[reply]

Yea, I think the tobacco companies actually uses what they term "anti-smoking" ads to secretly encourage smoking. For example, they might say "smoking is only for mature adults, so you should do what your parents say and not smoke". Thus, kids feel that if they don't smoke, they are immature and if they do smoke, they are brave "rebels". StuRat 23:39, 16 February 2006 (UTC)[reply]

The Camel ads are a very good example of cigarette advertising designed to appeal to young females, but all tobacco companies caught on that women were their best target www.cdc.gov/tobacco/sgr/sgr_forwomen/pdfs/factsheet_Mrktg.pdf. www.media-awareness.ca/english/resources/educational/handouts/tobacco_advertising/replacement_smoker.cfm This states the following:

A study published in the 1991 edition of the Journal of the American Medical Association found that nearly one-third of three-year-olds were able to match a picture of Joe Camel with cigarettes, and that six-year-olds were able to associate ‘Old Joe’ with Camel cigarettes as easily as they associated Mickey Mouse with the Disney Channel.

Here is a www.tobaccofreekids.org/research/factsheets/pdf/0138.pdf factsheet about ads targeting females and why. tobaccofreekids.org/adgallery/ Here, you can browse through tobacco ad campaigns by country, company, brand, and ad type. www.scenesmoking.org/ Here and www.tobaccofree.org/films.htm here are a few sites I found regarding smoking in movies...the second making direct reference to how much a tobacco company paid for exposure in particular films. bcatt 06:55, 17 February 2006 (UTC)[reply]

Interestingly, the book Runaway Jury by John Grisham is about a lawsuit against the Big Four tobacco companies, whereas in the movie based on the book, while appearing to have all the same character names and otherwise being true to the book (I have read the book but not seen the movie), the lawsuit is targetted against gun manufacturer(s) instead of tobacco companies. It is interesting to ponder the reasons for this switch. bcatt 19:54, 17 February 2006 (UTC)[reply]

Manuscript of Astorga 1624, Spain.[edit]

Good afternoon to the readership of Wikipedia.

I am a Spanish young man to the sports fishing of the trout with fly and my sleep it is to be able to contemplate in alive the Manuscript of Astorga written by Juan de Bergara in 1624, in Spain. This Manuscript treats of the manufacture of artificial flies for the fishing of the trout of the epoch.

According to informative sources in the year 1970 more or less this Manuscript was sold to a private English collector.

My sleep is to meet the owner of this Manuscript since I have intention of travelling this summer to the United Kingdom and like that to be able to admire the original one of the Manuscript of Astorga in my hands.

If some of you knows something about this Manuscript he would be grateful very much to whom was finding out about his whereabouts or other informations that should guide me even.

Receive an affectionate greeting.

Adriano Álvarez

I think you mean you "dream" of those things, not "sleep" of them. StuRat 19:56, 10 February 2006 (UTC)[reply]
See Lost work. --DLL 22:14, 10 February 2006 (UTC)[reply]

Thanks for your responses.

Sturat, you are right, I wanted to say " that I dream of these things "

DLL, thanks for your indication, but unfortunately there does not say to me * who is the English owner of the Manuscript of Astorga*

Affectionate greetings to the two.

Adriano

The book was either burned (and the owner is the ozone layer) or sold. --DLL 13:24, 11 February 2006 (UTC)[reply]
And Adriano please try editing with the edit put at the right of your question title, not the one below the last answer ... Ta! --DLL 13:27, 11 February 2006 (UTC)[reply]

Hello DLL.

Really that believed in a beginning, which had been burned but apparently another version has more force: that of which it was sold to a private English collector: do you know anything of that? In what public or private organisms might they to inform me?


Forgive me DLL, when you say that " he should try to correct position to the right of his question and not under the last response: " what do you want to say? I do not manage to answer of another form.

Affectionate greetings.

Adriano

Let's assume it was sold. It does seem to have an amount of reputability, so it seems likely the owner would let him- or herself be known, had he/she wanted to be known. You can buy things anonymously at most finer auction houses, so there's little hope if the owner doesn't want publicity. However, if it was sold, you'd expect that fact at least, to be well-established, because the auctions themselves are public. The experts at the auction house would also need to verify the authenticity. While most reputable antiquarian auctioneers do not guarantee authenticity, they are bound by a code to buy things back if they turn out to be forgeries. What I'd like to know is: What makes you think that it was sold, and what makes you think it was an English collector? If you follow those leads, then maybe you can find it. The two statements don't provide much evidence in themselves. --BluePlatypus 09:40, 12 February 2006 (UTC)[reply]

Hello BluePlatypus.

Really there are no many proofs, the only one that it was sold, it was much mentioned in his epoch precisely by numerous newspapers of then especially French and many comments of the same in different Spanish Universities, that thing about is burned was discarded since the supposed selling was realized 4 years before the Library was burned in question, since in United Kingdom there was and there keeps on being a lot of interest in everything relative to the sports fishing, with that there was very much I contact with the owners of this manuscript.

Anyway, if you know some track of shops of auctions or another place for this end would be grateful to you.

Thank you very much for your collaboration. Greetings.

Adriano

I don't know whether you'll be able to find it or not, but you can find the original scanned online www.gallosdelacandana.com/maorigi_i.asp here (and there is a transcription www.gallosdelacandana.com/manuevo_i.asp here.) I wish you luck in finding it! СПУТНИКССС Р 18:23, 12 February 2006 (UTC)[reply]
As for BluePlatypus' questions (What makes you think that it was sold, and what makes you think it was an English collector?), www.fishing.co.uk/article.php3?id=138 fishing.co.uk says:
While some claim the original handwritten manuscript was burned along with many other volumes contained in Franco's library, Pariente believes it was sold by the general's son-in-law, the Marquis of Villaverde, in a London auction. In any event, Pariente's photographs were the only thing that kept the Astorga Manuscript from total oblivion. СПУТНИКССС Р 18:28, 12 February 2006 (UTC)[reply]

Hello СПУТНИКСССР.

Really, it was Jesús Pariente Díez who rescued of the oblivion to the Manuscript of Astorga and also there was mentioned very much the friendship that Franco's relatives had with some Englishmen very keen of the fishing.

You are informed well about this manuscript: are you Spanish?

I what desire is to find some track of to where I can ask about the whereabouts of the owner of the Manuscript of Astorga, if it cannot be, about bad luck.

Cordial greetings.

      Adriano

Hedda Hopper[edit]

Can you please tell me who the name of the artist was that created Hedda Hoppers hats?

I don't think she had any one designer. www.time.com/time/archive/preview/0,10987,886945,00.html This says there were 103 (although only 3 are named). Benjamin Green-Field was another. JackofOz 23:03, 10 February 2006 (UTC)[reply]

Some help needed in interpreting UK school legislation[edit]

I'm doing some research into the laws of school discipline in England. The legislation surrounding this branch is here in two parts www.opsi.gov.uk/acts/acts1997/97044--b.htm#end and part 2 www.opsi.gov.uk/acts/acts1997/97044--c.htm. I just have a few questions on interpreting the legislation:

Is it possible to give a student detention who is 18 or over? Judging from the document, it seems that it is not. Also can you advise on how to cite legal documents e.g. the section which I think answers my question is 550b, part 2, of the education act 1997.

Am I right in thinking that the European Human Rights Act of 2002 has not been accepted in England yet? I'm aware that it is an integral part of Scottish law at the moment.

Regarding your last question, the European Convention on Human Rights was incorporated into UK law (i.e. covering England, Wales, Scotland, and Northern Ireland) by the Human Rights Act 1998, which came into force fully on 2 October 2000. I'n not sure what legislation you're refering to, there's no UK or Scottish law called the "European Human Rights Act", and the HRA has been applied in a number of English court cases.-- AJR | Talk 13:48, 11 February 2006 (UTC)[reply]

February 11[edit]

philosophy[edit]

write an article on philosophy and life

That's a good idea. Why don't you try it. Might be fun. JackofOz 00:29, 11 February 2006 (UTC)[reply]
Life is. -24.131.204.139 18:45, 13 February 2006 (UTC)[reply]

Oldest piece of music known to man?[edit]

Anyone knows which is?

Years ago my sister got this encyclopedia on a CD (Encarta, I think) and it had a buttload of MIDIs embedded on this huge file. The file also had the title of each MIDI encoded in a certain way. So, I wrote a program to extract and properly name such MIDI files, and one of them was entitled "The World's Oldest Known Music". By the time, I looked the article on that encyclopedia with such song, and it said it was for a crude sort of harp. I still have that MIDI right here (but not the CD), so I was wondering if it is indeed the oldest and etc. I can upload it if necessary (because afterall, if it claimed it's the oldest, I doubt it's copyrighted) ☢ Ҡiff 01:56, 11 February 2006 (UTC)[reply]

A particular performance of a song may be copyrighted even though the song itself is not. StuRat 03:08, 11 February 2006 (UTC)[reply]
Well, it's a MIDI file, and the song is somewhat trivial. I doubt it'd be an issue. ☢ Ҡiff 03:28, 11 February 2006 (UTC)[reply]
Samaveda (1200 BCE - 900 BCE) contains verses to be sung in specifically indicated melodies using the seven swaras or notes. You can get the MP3s www.gatewayforindia.com/vedas/samaveda.html here, though I am sure by "Oldest piece of music known to man", Encarta would mean "oldest music created by Europeans" or something like that. deeptrivia (talk) 13:17, 11 February 2006 (UTC)[reply]

Number of unemployed persons, just prior to Katrina?[edit]

I would like to know what the population of New Orleans was like before the hurricane.

I would like to find out the percentage or number of unemployed prior to the storm.

It appears to me that many persons were not employed prior to the hurricane and this would have a bearing as to how many were not paying attention to their daily life.

Are you trying to imply that if a person is unemployed, they aren't paying attention to what is going on in their lives, or are not concerned with what is happening around them? Captain Jackson 03:54, 11 February 2006 (UTC)[reply]
  • Have you studied any of the Hurricane Katrina articles here on Wikipedia and/or the associated talk pages, and their archives with respect to what the contributors there felt was appropriate to include?
    • Did you see the bit about where New Orleans population was among the poorest in the nation, with something like 1/3 of the public dependent on public transportation, unable to afford gasoline for personal car to evacuate? Do you understand the bit about how people with public assistance get paid at beginning of month, do not really get enough to live on, so towards the end of the month they have to eat out of other people's garbage because they have run out of money, so money for gas to evacuate is totally out of the question.
      • Did you know that many people on forms of public assistance, such as food stamps are also employed? Their income is not enough to live on, so they need additional financial help.
    • Did you see the statistics published right after the prior Hurricane season, where the general public was asked if there is another mandatory evacuation, will you be able to evacuate, and can you do so? There were a bunch of well to do people whose property had been stolen when they left in prior evacuations, and they did not want to leave. ** So there were three groups of population that heard the evacuation alert, but did not leave.
      • Those who wanted to leave, but were too poor to drive, or take a plane or whatever. Many of those people ended up in the Superdome and Convention Center.
      • Those who had had bad experiences in prior evacuations, returning to find their property trashed, not by the storm, but by looters.
      • Those who heard the warnings, but did not understand how bad it was going to be. They thought they could ride out the storm. Many people do ride out storms you know. They nail boards to windows, they stock up on several days worth of food. They just do not expect to be woken up by rising waters, driven up to their foof to live, unable to get at their food stocks, then be "rescued" by helicopters and boats, only to be abandoned for many days on bridges and overpasses that are above the flood waters.
  • Have you visited any of the government web sites that have labor statistics, and noted the vintage "these statistics are AS OF some date or time frame, before or after when Katrina hit?"
    • US Census www.census.gov/ for example

User:AlMac|(talk) 10:57, 14 February 2006 (UTC)[reply]

Olympics[edit]

I was just watching the opening ceremony in the Winter Olympics. During the parade of countries, the names are announced over the loud speaker, well, if the countries are just a name, known worldwide, why do they pronounce the country names in different langauges?

The names of the various countries are not pronounced the same worldwide. The delegations parade in alphabetical order according to the language of the host country, except for Greece, which leads the parade, and for the host country, which brings up the rear. The pronunciations are in the language of (1) the host country and (2) the country's own pronunciation. hydnjo talk 04:22, 11 February 2006 (UTC)[reply]
Furthermore, they read out the names of the countries in the official languages of the Olympics (French and English) plus the language of the host nation (Italian). And as to the names being the same worldwide and pronounced the same, consider that th United States is also the Etats-Unis, Vereinigten Staaten, Stati Uniti, Estados Unidos, Förenta Staterna, Te Honongo o Amerika, Amerika Serikat, Al Wilayet Al Mottahide, Soedinyonninye Shtati Ameriki, Fenua Marite, or even Amerika Birlesik Cumhuriyeti, depending on where you are. Grutness...wha? 05:32, 11 February 2006 (UTC)[reply]
Was the same in Athens two years ago with delegations in greek alphabetical order, it took me time to realize, e.g. : Virgin islands at "P" (Parthenai), Vanuatu at "B" (no "V"'s), &c. --DLL 08:44, 11 February 2006 (UTC)[reply]
This tradition is now 100 years old. It commenced with the 1906 intercalated Olympics in Athens. JackofOz 10:58, 11 February 2006 (UTC)[reply]
As someone from a country that is pronounced exactly the same in English, French and Italian I also found the three-fold repetition of the name a little odd, but I imagine it is too complicated to sort out exactly which countries are considered pronounced the same. Plus some countries might feel slighted that they were only announced once. Besides, I liked the way the Italian announcer give the name a little extra ring. DJ Clayworth 18:10, 13 February 2006 (UTC)[reply]
Maybe you can answer this one, DJ. Canada has 2 official languages which I understand have equal official status. Which language was used for the order of countries at the Montreal 1976 Opening Ceremony? and why? JackofOz 10:55, 15 February 2006 (UTC)[reply]
IIRC, it was in French, since Montreal is in Quebec, and official Quebec provincial policy is to promote French. User:Zoe|(talk) 19:46, 15 February 2006 (UTC)[reply]

Name of this piece of music[edit]

Intro of the unknown song. Can anyone recognize it? I did this crappy MIDI just now.

I had a MIDI of it before, it was on a piano and etc. Tried musipedia and didn't get anything. ☢ Ҡiff 04:12, 11 February 2006 (UTC)[reply]

It reminds me a little of Primus's Electric Grapevine, but I don't think that's it. —Keenan Pepper 20:18, 11 February 2006 (UTC)[reply]

Was it very short - say, one minute long? If so, it reminds me of...um... "Easter Woman", I think - certainly one of the "commercial songs" by The Residents. Were there any lyrics, or was it instrumental? Grutness...wha? 05:42, 13 February 2006 (UTC)[reply]

Well, all I had of it was a MIDI, and it was some classical (or so) piece on the piano. ☢ Ҡiff 06:18, 13 February 2006 (UTC)[reply]

List of Keepers of the Kings Swans[edit]

Hello, Is a list of the above from beginning to end, available? Thank You Malcolm

hm. I was going to suggest the Swan upping article - but we haven't got one :/ Grutness...wha? 23:20, 11 February 2006 (UTC)[reply]
Yes, we do. See Swan Upping. СПУТНИКССС Р 18:02, 12 February 2006 (UTC)[reply]
Swans keep themselves quite nicely. You might like to read the article; it's very entertaining. --Halcatalyst 01:49, 12 February 2006 (UTC)[reply]

Alicia Keys - Karma (on the radio)[edit]

Out of all the versions of Karma I've listened to by Alicia Keys, the version I prefer is played on the radio, but I can't seem to find it anywhere else. It's a faster version of the original, with more percussion. Is there a specific title for it, maybe found on a particular album? --Silvaran 07:30, 11 February 2006 (UTC)[reply]

You may need to purchase the single for the radio edit. -LambaJan 02:31, 14 February 2006 (UTC)[reply]

Who is "The Great Elmyra"?[edit]

Popular British band, The Stranglers, released a song in 1977 called "No More Heroes"

The song starts with these lyrics

Whatever happened to Leon Trotsky?
He got an ice pick
That made his ears burn
Whatever happened to dear old Lenny?
The great Elmyra, and Sancho Panza?
Whatever happened to the heroes?
Whatever happened to the heroes?

Lenny is sometimes quoted as Lenin, but I'm not too worried about this.

I have searched the web extensively and have not yet come across a character, in real life or fiction, in politics or any other walk of life, that seems to represent this character.

My guess is that this is a reference to a character in some old piece of literature or early 20th Century film, but this is only a guess.

Can anyone enlighten me?

Regards,

Danny --84.203.2.51 12:40, 11 February 2006 (UTC)[reply]

My intuitive hunch is that Lenny is a second reference to Leon Trotsky, although I've never heard of him being called Lenny (but then, artistic licence being what it is ...). The only well-known Lennies around at that time that I can think of were Lenny Bruce and Leonard Bernstein. Bruce was dead by then, but Lenny Bernstein had a way to go so it's unlikely to be him. I can't find anything about any Elmyra either, so maybe they're both made up names. JackofOz 13:20, 11 February 2006 (UTC)[reply]
Elmyr de Hory according to www.sticky.currantbun.com/links.htm it also suggests Lenny is Lenin as I have always interpreted it but he may morph into Lenny Bruce in the live version. MeltBanana 21:44, 13 February 2006 (UTC)[reply]

Thank you so much. It was bothering me for days.....

Danny

How has Tort overcome the restrictions involved in the law of Contract[edit]

Q:"because contract was traditionall seen a method for resolving disputes involving pure economic loss, it was originally thought that where two parties had made a contract a negligence action could not e used to fill in any gaps in that contract"


The above was quoted in a journal while i was reading about Tort law and contract law. I would like to know how has Tort overcome the restrictions involved in law of Contract? Can any law guru suggest anything? Regards Rich

The old idea was that if two people are in a contractual relationship, then that contract defines the entire relationship. So if the contract dosn't include a particular right or remedy, then you don't get it. Unfortunately that often led to unfair results. Therefore two things happened: firstly the courts implied contractual terms that weren't actually stated, and secondly they allowed people to make tort claims against each other even though they were in contractual relationships. Obviously the extent to which they went down each track varies in each jurisdiction.

Markyour words 18:33, 11 February 2006 (UTC)[reply]

Mark, Thanks for your kind reply. I really aprreciate it, however I would need a bit more of explanation. Would you be able to explain a bit more or would you be able to suggest an article or a website where I can read through relevant cases and explanations please. thanks - Rich

Well, what material is relevant will depend entirely on which jurisdiction you're interested in. If you search any legal source for 'concurrent liability' then you should find everything you need. Markyour words 17:12, 12 February 2006 (UTC)[reply]


Mark im interested in the jursdiction in the law of contract.Can you help please. Thanks Rich

Every jurisdiction (UK, Germany, Australia etc.) has a different relationship between contract and tort. Markyour words 23:12, 12 February 2006 (UTC)[reply]

Mark mine is UK...thanks

In that case take a look at www.legal500.com/devs/uk/cn/ukcn_029.htm this and the cases cited there. Halsbury's Laws of England will have the most comprehensive summary. Markyour words 23:55, 12 February 2006 (UTC)[reply]


Mark thanks...But Halsbury's laws of England, is that a book or can you send me a URL for the website/article ?

Thanks man. Been a great help. Rich

It's an enormous encyclopedia in about 30 volumes. Any law library will have it. Markyour words 22:22, 13 February 2006 (UTC)[reply]

Can you give me another example like the one you gave above abt construction industry...but this tim i would request a general example but not specified to any area like construction etc. Thanks Rich

Despite the title, the legal outline given there is not construction-specific. Markyour words 01:20, 14 February 2006 (UTC)[reply]


Hi Mark Thanks again. However, the example are mostly cases, but I would request you to send some theoritical explanatinos rather than cases. Thanks man. Rich

free markets-pro or con[edit]

What kinds of freedoms do markets provide? Are the proponents, Friedman, Hayek and Gray, correct? What are their best points? Or, are the critics, Marx, Keynes and Galbraith, the ones to side with? What good points do they make? I can't seem to find a clear understanding of what's what. Thanks for the help.-Brett M.

Unfortunately, our NPOV policy prevents us from doing your homew... giving an opinion, but there are articles here you might want to read about Karl Marx, John Maynard Keynes, John Kenneth Galbraith, Milton Friedman, Free markets and many related topics. David Sneek 14:27, 11 February 2006 (UTC)[reply]
NPOV goes for the encyclopedia, not for the ref desks, so here's my 2 bit.
Real Communism would be brilliant if it could work (if everyone would be nice to each other). State Socialism tries to create a preliminary stage to achieve such a heavenly state, but generally fails on the notion that a few guys at the top can figure out how to control this economic mess at their feet. Economic liberals concluded from this that one should accept the way things are, throw in the towel and just leave people to it. Which of course also fails on the basis that people aren't nice to each other. Luckily most governments have realised this and provided some socialist cushioning, some more than others. How much cushioning is needed where and when, however, leaves loads of room for debate.
So I might as well propose my ideal (but see later addition below). A guaranteed minimum income for everyone, no questions asked, of, say, 500 € per month, a guaranteed means to raise that to about 1000 € (either for free for age or disability or a guaranteed job, if needs be employed by the government), the next few thousand taxfree, and all that paid for by the highest incomes, possibly even an upper limit to income of, say, half a million a year and a maximum personal capital of a few million - all the rest has to be invested back into the company, not spent on personal stuff through wages for those who already have enough. That's pretty much all the government does. All the rest is left to the free market. Employers have to offer wages that compete with the government wages, so no laws are needed for that. Unemployed will have a means to support themselves on a minimal but reasonable level plus a strong incentive to find a job because the income will be in addition to the minimum income. DirkvdM 08:50, 12 February 2006 (UTC)[reply]
Three years later, and I have since dropped the minimum income. Just government-guaranteed jobs and social security for age (too high or too low), disabilities or education. This is based on the followng two assumptions:
  • Everyone has a right to a reasonable income
  • Everyone has an obligation to work for that income, insofar as possible
Combine the two, et voilà. This work can be for a free market enterprise of for a government company. In the first case, the employer needs an incentive to employ that person, which can be lower wages, supplemented by the government, financed through taxes for the rich (lower incomes remain tax-free to guarantee a completely free market at the bottom). In the case of a government company, that company operates in the free market and insofar as it is not efficient enough to generate the incomes are supplements needed, again financed by taxing the rich. Such government companies will often be needed to realise production (or services or innovation, whatever) that is socially desirable but for whatever reason not picked up by the free market. So, once again, it's a matter of combining two thngs: the need for work to be done and the need for people to have a job. If there is not enough need for work to guarantee the jobs, these government companies provide education in company-form to better prepare people for the free labour market. Only in this case is the income completely financed through taxes. DirkvdM (talk) 07:18, 10 June 2009 (UTC)[reply]

The London Euromarket[edit]

What do you think of the London euromarket? Would you best describe it as offshore finance or a money launderers haven? Thank you-Jonh Douglas

Congratulations, your question is both meaningless and pseudo-inflammatory. Which means: no answer, no flame. --Halcatalyst 01:41, 12 February 2006 (UTC)[reply]

if you don't want to answer the question, don't answer it jerkoff

has there ever been anybody with 3 arms?[edit]

If 3 arms are better than 2 then shouldn't "evoultion" have made us all have 3 arms?

What makes you think that three arms are better than two? David Sneek 16:37, 11 February 2006 (UTC)[reply]
This should be analyzed using a cost/benefit analysis. Having an extra arm would have a benefit in that more things could be held at once. However, it would require extra food to grow and supply another arm with energy and nutrients, so there is also a cost. Assuming the arm was on one side, this would also introduce an imbalance during walking. For this reason, all animals have an even number of arms. Apparently the cost/benefit ratio must work out differently for insects, arachnids, crustaceans, and other similar animals with exoskeletons, as they frequently have far more appendages. StuRat 17:48, 11 February 2006 (UTC)[reply]
Well, there are a handful of creatures with an odd number of arms, sea stars spring to mind. GeeJo (t) (c)  17:40, 11 February 2006 (UTC)[reply]
Good point. Those simple animals have a completely different type of symmetry, rotational symmetry, instead of the planar symmetry common in higher animals, like humans. This means balance is not an issue. StuRat 17:48, 11 February 2006 (UTC)[reply]
Note that many animals do have one additional appendage which serves as an arm, however. An elephant's trunk and an opposum's prehensile tail are examples. StuRat 17:48, 11 February 2006 (UTC)[reply]
Of course, a deformed individual might have 3 arms. This could happen with an incomplete conjoined twin, for example, with only the arm formed. This would not be likely to be a trait which would be passed on, however. StuRat 17:31, 11 February 2006 (UTC)[reply]
Damn, I can't find a copy online of that old Charles Addams cartoon of the harried woman ironing a shirt, frying an egg, and feeding a baby all at the same time while her bum husband grumbles at her... "Wait a minute, can't you? I've only got three hands." CDC (talk) 18:52, 11 February 2006 (UTC)[reply]
Frank Lentini had 3 legs. JackofOz 21:51, 11 February 2006 (UTC)[reply]
I'll avoid the obvious jokes which could be made about which fellows have had three legs! --Fastfission 21:59, 11 February 2006 (UTC)[reply]
Another approach might be the prehensile tails of a number of primates, which is probably as close as one gets evolutionary to a third arm. --Fastfission 21:59, 11 February 2006 (UTC)[reply]
I wish I'd thought of that...like maybe 5 comments ago ? StuRat 03:43, 12 February 2006 (UTC)[reply]

Sure. I recommend an excellent book by Armand Leroi named Mutants. Stranger things than you can imagine. ISBN 0-670-03110-0. alteripse

Even assuming the three-arms-better scenario as described above, the question is based on a common misunderstanding of natural selection and evolutionary processes. Evolution is not a process based on constantly becoming better and better, closer to perfection, but merely a process by which organisms which are more likely to survive and pass on their traits to their offspring... survive and pass on their traits to their offspring. We would all have three arms:

  • if mutations leading toward having a third limb happened,
  • and did so in healthy organisms without causing other detrimental side-effects or being physiologically tied to detrimental conditions,
  • and increased the likelihood that those with the mutation survived and reproduced more frequently, rapidly, or other way successfully than those without the mutation

This leads to lots of detrimental local maxima. Carriers of Tay-Sachs, for example, are highly resistant to tuberculosis and therefore more likely to survive and pass of their genes in heavily tubercular areas. Of course, 1/4 of those offspring won't survive to adulthood, because they will have full-blown Tay-Sachs. One of many many examples in which natural selection has led to something other than increased tendency toward perfection. (ObDisclaimer: I know this is a massive oversimplification, but it's less of one that "evolution makes us better".) Deborah-jl Talk 03:14, 12 February 2006 (UTC)[reply]

Bill Gates[edit]

Is Bill Gates a 'communist'? I know this sounds like a joke, but I read stuff like this on google, just headdings of articles but the articles didn't make anything clear...I think they used those titles just to call the attention...or is there any truth to this?--Cosmic girl 17:00, 11 February 2006 (UTC)[reply]

He seems like a straight capitalist to me. I think you should ignore any such outrageous claims unless some reasonable proof is supplied. And remember, the more outrageous the claim, the better the proof had better be. StuRat 17:22, 11 February 2006 (UTC)[reply]

haha, thnx! :D I'll try to keep that piece of advice in mind.--Cosmic girl 18:45, 11 February 2006 (UTC)[reply]

The heavily flawed argument goes like this: Gates does not want competition. In a communist society, there is no competition. Therefore, Gates is a communist. The truth is that Gates wants to compete in new areas and beat the competition. He wants Microsoft to come out on top in every business venture they pursue. He wants Microsoft to be able to choose new avenues of business and compete in those. All in all, it is pure capitalism. In fact, he is such a capitalist that he has spoken out against monopoly laws that force Microsoft to be less competitive in areas with little or no competition. --Kainaw (talk) 01:16, 12 February 2006 (UTC)[reply]
No, Gates is not a pure "capitalist." Anti-monopoly laws (which Gates opposes) force Microsoft to refrain from market predation, to which Gates and Microsoft are prone. If the field is wide open, Microsoft just charges in; there's no question of invoking any laws. Gates has a history of seeking to crush his competition, which places him in the exact opposite vein from classical captitalism, as first enunciated by Adam Smith. --Halcatalyst 01:37, 12 February 2006 (UTC)[reply]
There is an irony that unrestricted capitalism will eventually lead to one company that owns everything, in which case capitalism is defeated. Only by regulating and limiting capitalism can it be preserved. StuRat 03:38, 12 February 2006 (UTC)[reply]
What a coincidence again. I've just proposed a way to do that 3 threads up (free market - pro or con). DirkvdM 08:59, 12 February 2006 (UTC)[reply]
Well, OTOH, some people claim that capitalism is self-regulating, and that monopolies like that will eventually fail. I don't believe that personally, though. --BluePlatypus 09:26, 12 February 2006 (UTC)[reply]
  • Gates definitely believes in market intervention, philanthropy, and high tax rates for the rich. That would make him a slightly pink capitalist, maybe on line with FDR. See Gates Foundation. --M@rēino 21:45, 12 February 2006 (UTC)[reply]

I do believe that capitalism is self-regulating... my country isn't so much of a pioneer compared to the united states...but it has recieved a lot of benefits from capitalism...so I'm thankful for it. besides, the basic ideas of capitalism go hand in hand with the basic psychology of the human being...saying we aren't competitive is plain lying.--Cosmic girl 21:54, 13 February 2006 (UTC)[reply]

Indeed. So much so that it's sort of odd to thank capitalism because it's not something that is implemented. It's the normal way things are. Put people together and they'll trade (the competition bit already comes in here). Add money and you get capitalism (the accumulation of wealth by some). Socialism is an implementation of an ideology, so that's something you can thank (or not, depending). But capitalism is the natural state of affairs with money added. And for that you only need a strong government to guarantee the value of the money (or maybe not even that?). But there's always some socialism thrown in to keep things from going out of hand (see three threads up), so one could say that a bit of socialism is also part of the natural state of affairs for humans. DirkvdM 08:51, 14 February 2006 (UTC)[reply]

Agree.--Cosmic girl 12:35, 14 February 2006 (UTC)[reply]

Rape status[edit]

If I narcotize a girl with chloroform and then rape her will it be any criminal proceeding, e.g. without witnesses? (I'm just curious) Alitus, 18:55, 11 February 2006 (UTC)

Yes, it is a serious crime which can and will be prosecuted if there is enough evidence. Semen, condom residue, people who saw you drug her, and such would be used against you in court. It's called date rape, and what you are talking about is using a rufe. Captain Jackson 19:23, 11 February 2006 (UTC)[reply]
Actually I think roofie refers specifically to flunitrazepam. —Keenan Pepper 20:12, 11 February 2006 (UTC)[reply]
Captain Jackson meant to type ruse, I would say. --Halcatalyst 21:37, 11 February 2006 (UTC)[reply]
err, I meant to say roofie. Captain Jackson 00:32, 12 February 2006 (UTC)[reply]
I guess you guys are more experienced at this stuff than I am <g>. --Halcatalyst 01:23, 12 February 2006 (UTC)[reply]

Their could be a criminal proceding. If the chloriform killed her you would be up for death penalty, because murder associated with rape is felony murder. I say this as a lawyer.

I'd question your status as a (competent) lawyer, given the generalised remark you just made. The death penalty is not universal, and in a large number of countries, as well as a fair number of states within the U.S., the attacker wouldn't have to worry about being executed. GeeJo (t) (c)  02:50, 12 February 2006 (UTC)[reply]
Fox News has been railing abainst some New England Judge who only does a slap on the wrist for men who like to rape little girls. What happens depends a lot on what can be proven, whether the judge really approves of this activity, whether the children make good witnesses, whether the accused agrees to a lesser charge in plea bargaining. User:AlMac|(talk) 11:08, 14 February 2006 (UTC)[reply]

Chinese yuan pegged to the U.S. dollar[edit]

I know there's been a fair amount of debate over the Chinese yuan being pegged to the dollar; what exactly does that mean? I've seen it mentioned in regards to trade deficits (specifically with the U.S.). --Impaciente 21:11, 11 February 2006 (UTC)[reply]

It just means the the exchange rate betwen the two curencies is always the same. In this case a Chinese government decision which the US govt doesnt much like. Since the 1970's most of the major currencies have been floating - ie the financial markets decide the exchange rate according to supply and demand so they vary every day (minute). Jameswilson 00:54, 12 February 2006 (UTC)[reply]
As an odd example of a non-floating, or pegged currency, the Bermuda dollar is always equivalent to the U.S. dollar. --Kainaw (talk) 01:03, 12 February 2006 (UTC)[reply]
I should have also pointed out the problem with the yuan. The Chinese have purchased more and more crude oil. Worldwide, when they refer to "dollars per barrel", they mean "U.S. dollars per barrel". So, if you are dealing in oil, you are dealing need U.S. dollars to ensure you don't pay more due to currency exchange rates. The Chinese would obviously like to fix their rate so they pay they don't get in trouble if the value of the yuan falls. The U.S. doesn't like it because it will mean that the world could easily switch to "yans per barrel". Then, all those U.S. dollars will suddenly show up in our banks and flood the U.S. economy. That will cause a deflation, which will mean it is financially better for you to keep your money in a box under your bed than in a savings account or stock investment. That would be very bad for the economy in many ways. So, to avoid that possible chain of events, the U.S. is firmly against allowing any other major currency to be pegged to the U.S. dollar. Of note: Iraq repeatedly tried to replace the U.S. dollar as the currency for oil trade. That ended with the last Iraq war. Iran has taken the fight and is getting the backing of many who want to switch to the Euro. --Kainaw (talk) 01:10, 12 February 2006 (UTC)[reply]
Thank you for such a detailed response. But just how does a country suddenly decide "our currency will be worth this much?" Is that really up to them? I imagine there must be some mechanism at work for that to happen. --Impaciente 01:16, 12 February 2006 (UTC)[reply]
They do it the same way we do - quantity of currency and interest rates. By increasing or decreasing the amount of currency available, the value of the currency changes. By adjusting interest rates, you increase/reduce the usage of the currency (what good is a valuable dollar if nobody wants to spend it). That is very simplistic. Even though he had more experience with macroeconomics than anyone else, there are still many people who claim Greenspan has no clue how to control the value of currency. --Kainaw (talk) 01:35, 12 February 2006 (UTC)[reply]
Thanks again. I thought it had to do something with quantity, but forgot to consider the interest rates. --Impaciente 01:39, 12 February 2006 (UTC)[reply]
The other factor which Ive never quite grasped is to do with the big US deficits. The fact that every other nation has to buy dollars to pay for their oil. This somehow means that the US has been able to continually have deficits which would be unsustainable in any other country. European observers often moan about this but I dont understand quite how it works. Jameswilson 02:06, 12 February 2006 (UTC)[reply]
We are getting into a whole different topic, but this is a brief nugget of fact that you will rarely hear: The U.S. government always operates in a deficit. It always has and always will. Never once in history has it ever been out of a deficit - not matter how many anti-Republicans tell you that Clinton got us out of a deficit. Why does it work that way? The founding fathers didn't go into much recorded detail. They took out loans to pay for the Revolutionary War, so they were in debt to start with. After that, politicians have continually made comments along the lines of a government with a profit will become corrupt - as though one that operates in a deficit is not corrupt. I don't have it bookmarked, but the U.S. Treasury (www.treasury.gov) has a link somewhere that shows the deficit down to the penny. --Kainaw (talk) 02:39, 12 February 2006 (UTC)[reply]

Note that the US national debt is smaller, as a percentage of GDP, than many other nations. As far as pegging one currency to another, it can be done, for a while. After a time, though, the subsidy needed to keep the two currencies equivalent will be too expensive to maintain. This will result in the peg rate being set more in line with the current values. This will happen every few years. So, effectively, this will give a stepped exchange rate that parallels what a floating exchange rate would look like. However, the real issue isn't the pegging of the exchange rates, per se, but that the yuan is consistently undervalued relative to the US dollar. This is effectively a Chinese government subsidy on exports and should be considered a violation of WTO rules. The US hasn't yet made an issue of it, yet, perhaps in an attempt to gain political favor with China (which is needed in dealing with North Korea and the UN Security Council votes on Iran's nuclear proliferation). StuRat 03:26, 12 February 2006 (UTC)[reply]

Im not sure we are on the same wavelength here. Are we taking about the budget deficit/national debt or the trade deficit or both? Does the fact that oil is traded in dollars give the US more freedom to allow either deficit to run on without adverse consequences than would otherwise be the case? Jameswilson 03:46, 12 February 2006 (UTC)[reply]
Well, the artificially low peg rate for the yuan to the US dollar increases the US trade deficit with China. This can indirectly add to the US national debt, by reducing the amount of taxes brought in from companies doing US exports to China. It also increases Chinese national debt directly by having them supply cash to keep the exchange rate low. I don't really see how having oil traded in US dollars has much effect on anything. Since US dollars are readily converted to and from almost every other major currency, it's just like oil is traded in all of those currencies. Now if oil was traded in some weird currency, like that of North Korea, which isn't part of the global currency exchange, that would have a major impact. StuRat 05:35, 12 February 2006 (UTC)[reply]

The various states in the USA have their currencies pegged to each other by being the same one - the US$, just like it is done in the EU with the €, making the EU one big market, effectively spreading the wealth more evenly to the benefit of all. God forbid that more poor countries should also acquire our level of wealth. DirkvdM 09:10, 12 February 2006 (UTC)[reply]

Pegging two currencies together is not the same as using a common currency. The big difference is that you can force one of the two currencies to be overvalued or undervalued when pegging, but not when using a common currency, which is the real issue with the Chinese yuan and the US dollar, not the pegging per se. That said, the best way to keep the Chinese from undervaluing the yuan is to allow it to float freely. StuRat 23:30, 15 February 2006 (UTC)[reply]
You are using the common definition of "pegged" very loosely. To be able to peg two currencies, the two currencies should have separate backing capital. States do not have their own capital to back currency. The Federal gov has a small lump of gold and silver and a lot of loans. China does not share in this capital. They have their own.
As for the EU, it has caused problems by initially forcing open markets between the countries sharing the currency. It is almost like NAFTA depressing the U.S. currency while inflating the Mexican currency.
As for the earlier comment about what oil has to do with anything. The Euro article has a brief note on the use of U.S. currency for oil. If you want to buy oil, you use U.S. dollars. You do not want to risk money conversion loss, so oil purchasers have a stockpile of U.S. dollars. They do not convert the day of purchase. There is a danger of the stockpiles suddenly showing up in the U.S. if oil suddenly gets converted to euros/barrel. I think the U.S. would get through it, but it would be rather nasty. --Kainaw (talk) 19:56, 12 February 2006 (UTC)[reply]
Ta Jameswilson 00:11, 13 February 2006 (UTC)[reply]

Garry O'Connor[edit]

What did Garry O'Connor, the author that wrote Universal Father: A Life of Pope John Paul II, major in?

See www.speakersagency.com/mno/oc-ga.html his résumé. --Halcatalyst 01:17, 12 February 2006 (UTC)[reply]
That mentions where, but not what. (Not original poster, but now I'm curious.) GeeJo (t) (c)  02:46, 12 February 2006 (UTC)[reply]
From the above source I have the impresson that his focus at King's College, Cambridge was on drama; the fr.wikipedia.org/wiki/Jacques_Lecoq École Jacques Lecoq he attended was certainly theatrical, and he had a distiguished career in the theatre before becoming a writer. --Halcatalyst 05:13, 12 February 2006 (UTC)[reply]
I would wager good money that O'Connor read English, given his subsequent career (British students don't have 'majors' like US ones). History might also be possible. DJ Clayworth 17:54, 13 February 2006 (UTC)[reply]

French and German army tanks in the 50's[edit]

Does anyone know any? There's not much info here I can find on European tanks in the 50's (besides the UK).

West Germany was not able to re-arm until 1955, and I'm certain that there were serious restrictions on what they could and could not have. Both nations would have been in the process of serious rebuilding until at least 1955, so I don't think they'd have factories to build these tanks, if tanks were even a serious concern for their armed forces. I would bet that they were simply given military surplus tanks from the USA, or possibly out of date Shermans.
Here is one link I found that might be useful.
experts.about.com/q/Military-History-669/German-Tanks-WW2.htm
Captain Jackson 00:42, 12 February 2006 (UTC)[reply]
  • Plus, if you haven't seen it, our extensive article on Tanks. --Halcatalyst 01:03, 12 February 2006 (UTC)[reply]
My link really wasn't revelant to the question. Due to treaties ending World War II, Germany is not allowed to send troops outside of Germany, not even today. This was sort of a difficulty during the Iraq and Serbian crises of the 1990's. Also note that with the huge US/British presence in Germany, there wouldn't have been a need for Germany to have too many tanks. Captain Jackson 02:52, 12 February 2006 (UTC)[reply]
According to it's article the Leopard tank was designed originally for both the German and French armed forces in 1956, "to replace the Bundeswehr's United States-built M-47 & M-48 tanks". DJ Clayworth 17:42, 13 February 2006 (UTC)[reply]

February 12[edit]

is the population control affect the economy?[edit]

Do you mean, "Does the control of population affect the economy?" Population always affects the economy, and population control (as a political measure) also affects the economy, if only because it costs money to do it. Now, have you learned anything by asking the question? --Halcatalyst 00:31, 12 February 2006 (UTC)[reply]

China is an interesting example. Their "one child" policy had immense benefits early on, by allowing them to focus the countries meager resources on the few children that were born. This allowed for an increased level of education leading to a rapidly improving economy. Now for the negatives. The preference for males meant that many females were aborted or placed in orphanages to allow the "one child" to be male. The abortions led to a gender split in the 0-14 year age range of males 148,134,928 to females 131,045,415. An excess of males is likely to cause social disruption (as millions of males with no hope for marriage in any society tends to cause trouble). Also, the small population of workers will at some point be unable to support the large number of elderly Chinese. These severe demographic problems are likely to limit growth of the Chinese economy in the future. StuRat 03:07, 12 February 2006 (UTC)[reply]
Actually, there's some thought that the prevalence of Hepatitis B is also a contributor to the problem of China's "missing women". - Nunh-huh 06:21, 12 February 2006 (UTC)[reply]
You speak of the lobsided division between age groups, with more elderly than working people as something in the future. But the one child policy started in the 1950, so there should be a similar effect as that of the western post-war baby boom in the late 1940's, just a bit earlier (ten years or so?). So China has been pretty much in the middle of it for a decade or so now? All those kids that were 'supposed to' have been born in the 50's, 60's and 70's and who would now have been in their 30's to 50's (the ideal working ages) are now missing. And that must have had a serious effect on the economy. DirkvdM 09:22, 12 February 2006 (UTC)[reply]
The first attempts to limit family size in China do go back to the 1950's, but the actual limit of one child per family was only put in place in 1979. Hence, the most severe problems will happen once those born before 1979 have all retired, say around 2040 or 2050. StuRat 23:02, 15 February 2006 (UTC)[reply]
Further to what StuRat wrote, the problem that China is facing is called "the 4-2-1 problem", which means that 4 grandparents are being supported by 2 adults with only one child. As those 4 grandparents stop contributing to the economy, the 2 adults have to support them (either directly or via the economy or via the state through taxation). — PhilHibbs | talk 11:40, 14 February 2006 (UTC)[reply]

Why was Booker T. Washington's face removed from the half dollar?[edit]

You're misinformed. See Half dollar (U.S. coin). --Halcatalyst 00:28, 12 February 2006 (UTC)[reply]

Booker T. Washington was featured only on a commemorative half-dollar coin, issued from 1946 through 1951. His face was never on half-dollars intended for mass circulation. About 3 million of them were minted. They featured his portrait on the face, and the cabin where he was born on the obverse. In 1951, the coin was changed to feature both George Washington Carver and Booker T. Washington's portraits on the face, with a map of the U.S. on the obverse. These were issued through 1954. Conress had passed a law forbidding commemorative coins in 1939, but passed special legislation in 1946 for the issuance of the Iowa Centennial and Booker T. Washington half dollars, amending the legislation in 1951 for the Carver-Washington coin. These were the last commemorative coins issued before the modern commemorative coin program started in 1982 with the George Washington 250th Anniversary half dollar. www.usmint.gov/mint_programs/commemoratives/index.cfm?action=Premodern - Nunh-huh 00:40, 12 February 2006 (UTC)[reply]

Booker T? Is that the one who hangs out with the MG's, playing cool music? (Or is that 'hot music'?) DirkvdM 09:24, 12 February 2006 (UTC)[reply]

Kim Jong IL Questions[edit]

  1. Where is kim Jong Ills residence.
    He probably has several, but he'd have to have at least one in Pyongyang itself. --Robert Merkel 02:38, 12 February 2006 (UTC)[reply]
  2. How can he justify the lifestyle he lives in the face of such privation?
    Dunno. Nobody's ever had the chance to ask him. More generally, dictators have pretty much always lavished an extravagant lifestyle on themselves and their families, so you might find an answer through reading about them (some examples you might look into are Mobutu Sese Seko, Caligula,Ferdinand Marcos and Suharto). --Robert Merkel 02:38, 12 February 2006 (UTC)[reply]
  3. Is it true that he only got through kim ill Sung University because his proffesors helped him cheat?
    Trying to get any reliable information out of North Korea is pretty much impossible. As our article says, he is said to have attended Kim Il Sung University. --Robert Merkel 02:38, 12 February 2006 (UTC)[reply]
  4. Is it true that he has armies of conscripted sex workers?
    There are stories filtered through a few Westerners that spent time close to the regime claiming that Kim Jong Il and the other high officials engaged in all manner of debauchery on occasion. The accuracy of these stories is however open to question. Our article notes that he is reported to have fathered as many as nine additional illegitimate children. Typically, powerful men the world over get more sexual opportunity than others, and Kim Jong Il is the most powerful man in North Korea.
Pardon my limited command of the English language, but what are 'conscripted sex workers'? Is that a horribly complex way of saying 'whores' (well, allright then, 'prostitutes')? DirkvdM 09:36, 12 February 2006 (UTC)[reply]
Sorry if my answers are vague, but the level of reliable information about North Korea related topics is minimal. Read our Kim Jong Il article, along with the articles linked from it, to get an idea of the level of supposition required. One interesting book on North Korea Another Country by Bruce Cumings (www.amazon.com/gp/product/156584940X/sr=8-1/qid=1139711683/ref=pd_bbs_1/104-4613504-8841526?%5Fencoding=UTF8 Amazon link). However, Cumings books need to be interpreted through his anti-Americanism and his bizarre affection for the North Korean regime. --Robert Merkel 02:38, 12 February 2006 (UTC)[reply]
As far as justification, since he has total control over the press, there is no possibility he will ever be asked those questions. StuRat 02:47, 12 February 2006 (UTC)[reply]

Offical residences[edit]

  • Q: Where is fidel castro's offical residence.
  • Q: Who held title to Berchesgarden, Hitlers mountain retreat, Hitler personally, the German Governmemt, of the Nazi Party.
  • Q: What was the point of building a house as oppulent as the white house, prior to the Civil war it was the larest residence in the country. Would it not have been cheaper to build a less oppulent mansion for the President?

White house[edit]

The White House was designed to impress foreign dignitaries and dispel the European perception, at the time, of Americans as "just illiterate farmers". StuRat 02:51, 12 February 2006 (UTC)[reply]
That's probably part of it - the rebels needed a response to Buckingham Palace. In fairness, though, most of the White House is just an office building (albeit a rather opulent one). The president's private quarters, while I'm sure they're very nice, befitting the rich and powerful guys that have occupied them, are only a small part of the building. CDC (talk) 03:46, 12 February 2006 (UTC)[reply]

Castro[edit]

Fidel Castro's residence: I don't know Havana, but his residence/offices apparently face on the Plaza de Revolución, in central Havana.www.wordtravels.com/Cities/Cuba/Havana/Attractions this A 1994 Associated Press story I read on LexisNexis described this place as his official residence. Does it have a name? Anyone? CDC (talk) 03:46, 12 February 2006 (UTC)[reply]
Given the many murder attempts by the US he will probably move around a lot, in which case he would not have a residence in the literal sense of the word. DirkvdM 09:44, 12 February 2006 (UTC)[reply]

Berchtesgaden[edit]

According to www.obersalzberg.de/cms_e/content/de_ausstellung_obersalzberg/index.html this site dedicated to the history of Obersalzberg, it was owned by Hitler personally. "In 1933 Hitler acquired ownership of the Wachenfeld House, which he had been renting since 1928. In the following years the modest country house was converted into the pompous Berghof." David Sneek 14:19, 12 February 2006 (UTC)[reply]

Sale of American public lands[edit]

Noticed an article on the possible sale of public lands www.chron.com/disp/story.mpl/nation/3651464.html here. Anybody know where I could find out the areas in question? -Tim Rhymeless (Er...let's shimmy) 03:38, 12 February 2006 (UTC)[reply]

The article seattletimes.nwsource.com/html/nationworld/2002798263_landsales11m.html here says The Forest Service will publish a notice in the Federal Register around Feb. 28 requesting public comment. At that time, more detailed maps will be available for all potentially eligible lands. There's a map accompanying the article that gives amounts proposed by state. CDC (talk) 03:55, 12 February 2006 (UTC)[reply]

VERY Interesting - Please Read[edit]

www.1335.com/normyth.html

In that little article it claims that the "races" we know as the Celts (modern day Irish and Scottish people), Norse (Scandinavians, who gave "birth" to Germans), and ALL Germanic peoples/tribes (this includes of course the English, Germans, and the French) are of ancient Jewish origin. Please read for yourself- i'm not sure if all og it is accurate or not, but I think this theory is a load of excrement- the only credible "proof" is connection between the gods and religious practices of the "heathen" Jews, Canaanites, and Babylonians and such- Everything else, like when he, the author, says "The Angles, Saxons, Celts, and Goths, who overspread Europe, are said to have originated in the region of Medo-Persia, about 700 B.C., the very time and place in which the nation of Israel was lost to history." The 'are said' means he has no credible proof.

Does this seem credible or possible to anyone?

Ten Lost Tribes has a lot of background to this. Without being insensitive to anyone's religious beliefs, I think that "a load of excrement" is a very apt phrase. JackofOz 05:14, 12 February 2006 (UTC)[reply]
Agreed. The most obvious flaw is that the ancient Jews, being a Semitic people, were generally short, olive skinned, with dark, curly hair (modern Jews are a more diverse group). Does this sound like a Viking to you ? StuRat 05:19, 12 February 2006 (UTC)[reply]

To refute this, I shall say yet one more thing; Let's assume everything was true in the Bible- this theory I mentioned (of which the Bible is the primary resource) would probably be wrong: the prophecy of Jews returning to Israel wouldn't haven't happened the way it did- the way I took the prophecy, most of the Jews would return to Israel in/after 1948. There are far more people living (both alone and collectively) in the U.S., U.K., France, Germany, Scandinavia.

ALSO: Please leave your religous views out of the "picture" when you reply to this, as JackofOz did; this is sort of a cultural theory, not really a religious one.

Also, in a comparison of religions, the most basic concept of Judaism is that there is only one God, while all of those pre-Christian Northern European religions were polytheistic. The Norse, for example, had the gods Odin, Thor, Freya, etc. StuRat 05:44, 12 February 2006 (UTC)[reply]
  • Back in the middle-ages, when the Old Testament was considered a completely valid source document, quite a few (if not most) Christian peoples decided to reinvent their history to fit with it. Huns and Hungarians were supposedly the decendents of Ménrót, who Simon of Kéza identified with the biblical Nimrod. (in reality, there's no evidence of that of course. Or that Huns and Hungarians actually had much to do with each other, for that matter). The Swedes were supposedly decendants of the biblical giant Magog, according to Johannes Magnus. He'd probably be proud of that article. The Russians missed a great opportunity for revisionism, given that there's a name "Ros"/"Rosh" in the Septuagint (most other translations treat it as a word, not a proper name). Though they'd have a hard time reconciling that with what Nestor wrote. Of course, no historians regard these stories as anything other than fiction created to reconcile the Bible with pagan history. --BluePlatypus 09:11, 12 February 2006 (UTC)[reply]
If you look at the Celts article, recent research on genetics shows that the ancient inhabitants of Ireland, Scotland, etc, looked the same as they do today. Before the arrival of a few Celts who took over the country. In fact thats generally true for all of Europe. People in Scandinavia were blond before Vikings, Danes, etc. Jameswilson 00:20, 13 February 2006 (UTC)[reply]

If you think that theory is intresting, you may want to read The Seven Daughters of Eve, which talks about Mitochondrial Eve, or The Real Eve by Stephen Oppenheimer, which doesn't seem to have a page... DuctapeDaredevil 05:04, 17 February 2006 (UTC)[reply]

I have a question[edit]

Was Oprah Winfrey a teacher in Mississippi before she hit it big?

  • Unlikely, our article on her mentions a lot on her early life, but nothing about being a teacher. = Mgm|(talk) 12:29, 12 February 2006 (UTC)[reply]
  • According to www.hsph.harvard.edu/chc/wmy2006/Celebrities/oprah_winfrey.html this source, and confirmed www.acmecelebs.com/oprah_winfrey/interview.htm here, Oprah wanted "for many years" to become a fourth-grade teacher. The temperament gurus at keirsey.com/oprah.html Kiersey.com call her a "teacher/idealist." She may never have been employed as a teacher, but she is regarded as a great teacher by many. --Halcatalyst 15:33, 12 February 2006 (UTC)[reply]

Describe the Economic situation of Asia.[edit]

Vague question, vague answer. See Economy of Asia. DirkvdM 09:46, 12 February 2006 (UTC)[reply]

Financial Crisis in Asia in 1997[edit]

What trigget the Financial Crisis in Asia in 1997? What countries were affected?

Our article on Asian financial crisis has the answers. Jay 11:20, 12 February 2006 (UTC)[reply]

Tracing a Parachutist in the Free French Armed Forces during World War 2[edit]

Can anyone please tell me how can one trace a parachutist in the Free French Armed forces during World War 2. He was stationed in Glasgow Scotland, very possibly, about the end of 1943 and 1944. He was still in the French Armed Forces in August 1945. Enquiries to the Archives Militaires and the Archives Armee de l'Air have not yielded any results. Thank you.

Assuming you need a specific French parachutist it might be helpful if you could provide a name, age, physical description, unit designation, place of birth etc. However, what you are asking can't be answered without a lot of work. Celcius (Talk) Wiki be With us! 15:02, 16 February 2006 (UTC)[reply]

Thank you for the answer. If one were to know his name, unit etc., how would one then proceed, since the French Armed Forces archives have been unable to help? With thanks.

Issue-Riot[edit]

Hello i was just wondering if any one can help me, i have tried researching but have been unsuccessful about Riots. I am trying to apply international law to domestic (Australia) in my report, to present changes in the current law which will help achieve greater justice for the society and prevent riots from occuring. If anyone can help it would be very helpful Thank You

We have a good article on Riots, including a section on Australian riots, with a link there to the Sydney Riot of 1879. This overall view and history might be useful to you. There is also an extensive article on International law. You've taken on a big topic. Good luck. --Halcatalyst 17:37, 12 February 2006 (UTC)[reply]
I think riots should be viewed merely as a symptom. The underlying causes of riots can vary widely, and therefore the solutions will vary, too. StuRat 20:51, 12 February 2006 (UTC)[reply]

9 Souls song.[edit]

Hi guys. This for those who've seen the film "9 Souls".

The song at the beginning which also plays in many other parts -- the acoustic guitar song. What is it called? I know is by "Dip", by I can't find the name, is it "Nowhere I'd like to be"?

If so, i would really love to have it. does anyone know where i can get the song or does anyone have it?

thanks. gelo 11:46, 12 February 2006 (UTC)[reply]

Comic strips on computers[edit]

Can you suggest some comic strips (at least 5) available on the internet which relate to Computers or the office life in Software/Information Technology companies. I know of only Dilbert. I remember reading an interesting comic strip set in an office where all the characters are animals. If only there was a category on "Comic strips by genre". Jay 12:58, 12 February 2006 (UTC)[reply]

Well, one relatively famous one is User Friendly. --Robert Merkel 13:50, 12 February 2006 (UTC)[reply]
Ya thats a good one. I've just created a "Category:Workplace webcomics" and added user friendly to it. Jay 16:39, 12 February 2006 (UTC)[reply]
You'll be interested in www.comixpedia.org/index.php/Main_Page Comixpedia. It's like Wikipedia (as in, a wiki encyclopedia), but dedicated entirely to webcomics. Take a look, you'll probably find everything you want there ☢ Ҡiff 14:29, 12 February 2006 (UTC)[reply]
Thanks I found the Category:Workplace (but no Category:Computers) at comixpedia www.comixpedia.org/index.php/Category:Workplace. However could get only 2 comics called Cortland www.cortlandcomic.com/ and Boot Error www.booterror.co.uk/. Jay 16:07, 12 February 2006 (UTC)[reply]
Interesting that Dilbert is viewed as being set in a software/IT company. I don't think this has ever been established. I think one property Dilbert has is that everyone believes it to be about their own company (sometimes based on secret inside information). Notinasnaid 17:33, 12 February 2006 (UTC)[reply]
It seems to change from time to time, but always stay in the high-tech area. Most of the time it's a software company, but at other times they make hardware. StuRat 20:46, 12 February 2006 (UTC)[reply]
Another strip in this genre is Helen, Sweetheart of the Internet. --Metropolitan90 09:02, 13 February 2006 (UTC)[reply]

two sociological definitions of poverty[edit]

You could read the poverty article, but the answer you really want is probably in your text book. David Sneek 14:08, 12 February 2006 (UTC)[reply]

A sociologocial explanation, as opposed to other types, would view poverty as the result of marginalized minorities who are denied equal access to education and jobs due to discrimination. The effect of having parents who are themselves "victims" (say a welfare mother and an absentee father) would also diminish the chances for success of the child. I don't think the sociological explanations should be used in a vacuum, as each individual also has a substantial impact on how their life turns out due to their own decisions, like whether or not to drop out of school. StuRat 20:42, 12 February 2006 (UTC)[reply]

Picasso and Cubism[edit]

I'm doing research on Picasso's contribution to the history of cubism and was interested to hear the viewpoints of others on the matter. I'm being vague purposefully.

Well, you could start with the Wikipedia's collective view on Cubism and Picasso at Cubism and Pablo Picasso. Then you might try going to your local research library, which Wikipedia is not a substitute for. --Robert Merkel 13:48, 12 February 2006 (UTC)[reply]

Vital Records for Belz in Bessarabia, Moldova[edit]

Where can I please find the vital records (Birth, Marriage and Death certificates) for the city of Belz (Beltz, Beltsy) in Bessarabia, Moldova, for the 19th century? With thanks.

Belz is in the Ukraine nowadays, not in Moldova (which was called Bessarabia as a Russian province). As for which archives would have records, my guess is that you'd have to look in the local archives. There might be centrally archived copies in Kiev or Chişinău, but that's guessing. Personally I'd watch out for going anywhere near Transdniestr. --BluePlatypus 15:02, 12 February 2006 (UTC)[reply]
Whoops, I doubt very much that that is the same Belz. Guess it's either locally or in Chişinău then. Which still isn't very hopeful given the state Moldova is in. --BluePlatypus 15:07, 12 February 2006 (UTC)[reply]
Its probably this one - Bălţi. The mayors website is quoted so probably theyd know where the old records are archived. Jameswilson 00:25, 13 February 2006 (UTC)[reply]

Thank you for the answer. I was in fact referring to Belz (also knowm as Beltsy or Balti or Beltz) in Moldova.

Why were there no Buddhist councils between 100 and 1871? Did people forget about the concept, or was it because of the decline of Buddhism in India? deeptrivia (talk) 15:14, 12 February 2006 (UTC)[reply]

I would suggest that this happened because, firstly, as you point out, Buddhism had become rather fragmented after it died out in India, and also because, like other religions, it became steeped, not to say stuck, within its customs and traditions, and there was little need to change it until it encountered increasing western influence in the nineteenth century. However, I am only guessing. That article could do with some NPOVing, by the way. --Shantavira 18:31, 12 February 2006 (UTC)[reply]

Rommel and Hitler[edit]

Did General Rommel ever had any remarks about Hitler,I mean did he ever talked about Hitler? Also,was General Rommel member of Nazi party? Thank you

Field Marshal Rommel expressed his opinion in an even better way; he was involved in the July 20 plot to kill Hitler. David Sneek 17:18, 12 February 2006 (UTC)[reply]
Well, our article rather less than forthright on that one. Markyour words 19:12, 12 February 2006 (UTC)[reply]

Well,i red that,and Im not convinced that Rommel was part of that plot at all,thats why I asked if he ever said anything about Hitler???

Any particular reasons for your opinion? To respond to your question, of course Rommel talked about Hitler, as did everybody at that time. A book by David Fraser, Knight's Cross: A Life of Field Marshal Erwin Rommel ISBN 0060182229, perhaps among others, states that Rommel was not a Nazi. -Halcatalyst 23:07, 12 February 2006 (UTC)[reply]


Yes,because I live in Germany and I never heard that Rommel hated Hitler or was his enemy, dont get me wrong, I highly respecst both of them,thats why I cant see why would Rommel be against Hitler,it makes no sense,Rommel was a hero,not a treitor,and I learned that he was a big friend of Hitler

If, as you say, Rommel was a "hero, not a treitor" (sic), then he would have been against Hitler, since Hitler all but destroyed Germany. Had he been a real hero, he would have never have served Hitler in the first place. -- Mwalcoff 01:18, 14 February 2006 (UTC)[reply]

UN financial crisis.[edit]

Hi,

I am writing a foreign policy statement for a model united nations conference. One of my topics is the UN financial crisis. I'm representing Thailand and so i have to write about what thailand's stance is on the UN financial crisis and what thailand has done to resolve this and what future measures are being taken by Thailand. I would greatly appreciate it if someone could help me with this task by providing some info and also some references to UN resolutions regarding this topic, which thailand has supported.

Thank you, Safraz.

Google returned 14 million references, and Yahoo 6 million, to the search words un financial crisis. Try some of these to get a handle on your topic. You can also get a start at United_nations#Financing. --Halcatalyst 22:59, 12 February 2006 (UTC)[reply]

Here's www.google.com/search?hl=en&q=Model+United+Nations&btnG=Google+Search Google on the Model UN. I am happy to hear it is still going well. I was in it in the 1960's. My class represented Ghana. About 2 days before we were to show up at the MUN, Ghana had a revolution, changing to a new form of government. We had to scramble. Were we going to represent the old government in exile, and try to win international intervention, were we going to represent the new government with a whole new agenda, were we going to have some of our members secretly be agents for the other faction side ... we chose maximimum fun. The history of our nation had had a change in flag, when move from colonial to independent. The dias of MUN had the wrong flag. We wore black armbands in silent protest against the hosts refusing to fix this. User:AlMac|(talk) 11:17, 14 February 2006 (UTC)[reply]

Creation of efficient financial management responses to natural disasters[edit]

I am writing a foreign policy statement for a model united nations conference. One of my topics is "The creation of efficient financial management responses to natural disasters", (especially the tsunami). I'm representing Thailand and so i have to write about what thailand's stance is on this topic and what thailand has done to resolve this and what future measures are being taken by Thailand. I would greatly appreciate it if someone could help me with this task by providing some info and also some references to UN resolutions regarding this topic, which thailand has supported.

Thank you, Safraz.

  • The best thing to do about natural disasters is to prepare in advance. An efficient, competitive insurance industry can acheive the financial management part of that equation, but it is important for the national government to include disaster-risk assessment as part of its method of evaluating new projects. --M@rēino 21:56, 12 February 2006 (UTC)[reply]
  • You might want to limit your topic to Thailand's response to the 2004 tsunami. For the context, see the Thailand article. Googling thailand tsumani brought up 33,000 references; Yahoo produceed about 17,000. So there's no lack of material; you just have to sift through it all with a purpose in mind and try to find the gold amidst the gravel. --Halcatalyst 22:53, 12 February 2006 (UTC)[reply]

The effect created on global economies by the lack of quota system and the world oil crisis.[edit]

I am writing a foreign policy statement for a model united nations conference. "The effect created on global economies by the lack of quota system and the world oil crisis". I'm representing Thailand and so i have to write about what thailand's stance is on this topic and what thailand has done to resolve this and what future measures are being taken by Thailand. I would greatly appreciate it if someone could help me with this task by providing some info and also some references to UN resolutions regarding this topic, which thailand has supported.

Thank you, Safraz.

(Sorry i have asked 3 questions in the same format. I seperated the questions so that it'll be easier to understand.)

Though Wikipedia has a List of UN Security Council Resolutions, it's "in progress," but you might have a look at it just to get an idea of the scope of your topic. I just Googled thailand un resolutions and got 471,000 references, so it's not like there's no information out there! Of course, the first thing to do is to visit www.un.org/ the UN web site. If you poke around there, you can find relevant information; for example, a few clicks led me to this research guide for www.un.org/Depts/dhl/resguide/r60.htm Resolutions adopted by the General Assembly at its 60th session. --Halcatalyst 22:40, 12 February 2006 (UTC)[reply]

who caught abramoff?[edit]

I have heard alot in the news about the Abramoff scandal, i was just wondering who was it that caught him?

Thanks

I doubt there was one person at one moment who "caught" him. He had a lot of balls in the air and ultimately couldn't keep them all up there. See Jack Abramoff. --Halcatalyst 22:24, 12 February 2006 (UTC)[reply]
A lot of people were catching a bad odor off of his activities, so senator John McCain was one of the people to "catch" him (doing BIA work) and the justly famous Elliot Spitzer was another. Spitzer was investigating the conning of Indian tribes in New York, while McCain was investigating the taking of others. This is independently of newspapers and public interest groups investigating, in particular, Ralph Reed and Tom DeLay and their "business trip" to St. Andrews Golf Course. Geogre 20:07, 16 February 2006 (UTC)[reply]

Information: Basket Ball League Miami Chapel Champs 1920-21?[edit]

I am hoping that I can find some information on a Basketball League that dates back to 1920-21. My father has a silver trophy cup that has the engraving: Basket Ball League Miami Chapel Champs 1920-21

The cup was made by a company called Poole Silver Co. and has the date 1835 engraved on the bottom of it. Also the word Guarenteed is engraved on the bottom of it. My father found the trophy cup while doing some cleanup for a neighbor. Could this basketball team have been from Flordia or another state? I did some research and the coach at that time was M.R. Schwartz 1920-21 and then the coach became J.W. Tasker in 1921-22.

I would like to know if it could be an antique and what state it belongs to. I really think it would be a great piece of history for the school that it belongs to.

If you can help me find some information on this it would be much appreciated.

Thank You, SouthernAlabama

  • Well, there's a Miami Chapel Elementary School in Dayton, Ohio; it might have come from there if it wasn't from Florida. But if you found who the coach was, wouldn't the source for your research indicate where the team was located? --Metropolitan90 09:07, 13 February 2006 (UTC)[reply]

February 13[edit]

Movie with a fight scene in a public toilet?[edit]

I've seen a movie (maybe a TV one) last year (winter 2004-2005) in France where a guy with a sabre fight a guy with some clubs or tonfas in what seemed like public toilet (airport kind). Does it ring a bell to anyone? 83.214.23.85 00:46, 13 February 2006 (UTC)[reply]

I picture a dual between a fighter with a toilet brush and another with a toilet plunger. StuRat 03:52, 13 February 2006 (UTC)[reply]
It was Jet Li. I believe it was Unleashed, but it could have been another one of his movies. The point was that he was fighting a very tall guy. In the tiny stall, the guy didn't have room to move around, but Jet Li (being smaller) could easily fight against him. --Kainaw (talk) 04:15, 14 February 2006 (UTC)[reply]
If your talking about "Danny the Dog", it's absolutely not that.
First it was a wide public toilet like this http://en.wikipedia.org/wiki/Image:Urbeach-washroom-changeroom-architecture.jpg washroom(not a closet).
Second I don't remember Jet Li and the guy using anything like a sabre/katana or batons (not a baseball bat). But thanks anyway for you contribution. 83.214.23.85 12:12, 14 February 2006 (UTC)[reply]

Gun laws[edit]

Q: Is it legal to own a shot gun in England, for purposes of hunting?

A: Yes, although you need a firearms licence. -- Finlay McWalter | Talk 00:52, 13 February 2006 (UTC)[reply]
Actually, you need a shotgun license. A firearms license does not cover shotguns. This is only issued if you can demonstrate "good reason" and after police checks. You would also need a game license, and somewhere to hunt (there is no public hunting, with the possible exception of wildfowl on the foreshore). Note also that in England the term "hunting" mostly refers to hunting (e.g. foxes) on horseback; other kinds of hunting would be called "shooting". Notinasnaid 15:35, 13 February 2006 (UTC)[reply]
There's also the term lamping: see Spotlighting#UK and Ireland: lamping for details. The BASC webpage linked off that page has guidelines which cover the legalities of this and which refers to the use of shotguns as well as rifles. We have an article Gun politics in the United Kingdom but it seems to be in some disarray at the moment. --Telsa (talk) 18:12, 15 February 2006 (UTC)[reply]

Prostitutes and pregnency[edit]

why is that long before the widespread use of contraceptives their are few stories of prostitutes bearing children?

Prostitutes had plenty of children (if there are "few stories" about that, it's probably because stories about prostitutes having children don't sell well). They also procured abortions; you don't get stories about that much either - the market for back-alley medieval abortion stories is even poorer. -- Finlay McWalter | Talk 00:55, 13 February 2006 (UTC)[reply]
I saw a TV program a year ago about an archaeological dig of what was concluded to be a Roman brothel. The sewer underneath was packed with the bodies of male infants. Presumably the females were kept to perpetuate the trade, and the males were dangerous to keep around as the bastard of a nobleman is a political hot potato. — PhilHibbs | talk 11:29, 14 February 2006 (UTC)[reply]
PhilHibbs, do you have any more information on that? Really interesting, in a morbid way. Henriksdal 11:58, 16 February 2006 (UTC)[reply]
Sorry, I don't. It might even have been a radio program, because I don't have any vivid visual memories, and you'd expect that sort of thing to stay with you. — PhilHibbs | talk 13:58, 16 February 2006 (UTC)[reply]

Fascinating. Absolutely fascinating. --Leah

Be aware also that contraception has been "widespread" for a very, very long time. The condom alone has a venerable history, and various other forms of contraception (vinegar changing the pH of the vagina to make implantation difficult was a Roman favorite) have existed for ages. However, prostitutes did, indeed, have children, although less often than one might suppose if they had no contraception. Geogre 20:10, 16 February 2006 (UTC)[reply]

Luthansa Heist[edit]

I am looking for information on the Luthansa heist and its participants. Q: What ever happened to Jimmey Burkes bar "Roberts lounge" is it still functioning? Q: Was Jimmey Burke ever the formal owner of Roberts Lounge? Q: Regarding his associate Martin Krugman, what are his life dates? Q: Does anyone know anything about his life, or his family? What where his wig commercials like? Q: Does anyone have any documents related to Jimmey Burkes early crimminal career, or early like?

Read all about it www.crimelibrary.com/gangsters_outlaws/gang/heist/5.html here. --Halcatalyst 01:38, 13 February 2006 (UTC)[reply]

I assume that you are just going by the film Goodfellas, I believe the names of those involved in real life is different from in the film. I think Jimmy Burke was Jimmy Connelly or something else beginning with a C, can't remember fully though. AllanHainey 08:14, 14 February 2006 (UTC)[reply]
Jimmy Conway, which Jimmy Burke already redirects to. Also Martin Krugman, although I suspect the person asking the question may have already looked at these, as his questions are not answered in these articles. Chuck 22:21, 14 February 2006 (UTC)[reply]

Mafia[edit]

Is it true that in order to become a made member of mafia you must have been involved in a homicide? Or is that just ledgend? —Preceding unsigned comment added by 68.112.242.121 (talkcontribs) So, now we know where you live.

Hey you, listen to www.moviesounds.com/godfathr/offer.wav this and fuggeta bout it. hydnjo talk 02:06, 13 February 2006 (UTC)[reply]
The Mafia in the real world was primarily involved in racketeering. In the movies, it was involved in everything else. However, it is primarily a gang, like any other gang. One way to ensure gang unity is to have some leverage against each member. By forcing a member to commit a crime and keeping evidence of the crime, you have leverage over that member. --Kainaw (talk) 04:12, 14 February 2006 (UTC)[reply]
"Was"? User:Zoe|(talk) 22:28, 14 February 2006 (UTC)[reply]

Napoleon?[edit]

Q: Because Napoleon kidnapped the Pope because of the Popes use of his temporal authority in the papal states, is not the popes attempt to ex communicate him against papal law?

You can read all about it in the Italian campaign section of the Napoleon article. The Pope's power to excommunicate is part of his office. --Halcatalyst 01:34, 13 February 2006 (UTC)[reply]

elevated art[edit]

what is elevated art? is it historic art that has to do with biblical themes? or is it a genre of art that's thought to be superior? Whenever I search for it it just says it's elevated art, and gives no definition or on the net it just talks about subject matter but not specifically elevated subject matter. --thanks

Well, there's Elevator music, which is pretty much the opposite of what the Highbrow population would consider good taste. "Elevated art" is pretty much what those with breeding, education, and money declare it is. It can be literature, music, painting, sculture, dance.... --Halcatalyst 05:07, 13 February 2006 (UTC)[reply]
    • Ok, what would elevated subject matter be when Reynolds and Gainsborough were painting?
      • Well, (1) it isn't the subject matter that makes the art elevated, but (2) if you think that's important for these two painters, I suppose the most elevated part would be the wigs. --Halcatalyst 04:46, 14 February 2006 (UTC)[reply]

"three alarm fire"[edit]

When reporting on fires, US media often uses a term like "three alarm fire", where the number of alarms seems to be roughly correlated with the fire's severity. What, actually, does the number represent? Fron context, I've not been able to distinguish between it being:

  1. the number of times people call the fire department
  2. the number of premises affected
  3. the number of different fire stations which send appliances to the fire
  4. the number of fire appliances which are needed to tackle the blaze

Which, if any, of these is right? -- Finlay McWalter | Talk 02:13, 13 February 2006 (UTC)[reply]

the third and fourth. more stations = more appliances, and vice versa. —Charles P._(Mirv) 02:27, 13 February 2006 (UTC)[reply]
  • Each fire district defines they're own specific meaning. In most districts the "number" refers to the type and number of equipment that responds. There is no universal or standard meaning to the number(#) alarm fire. In general, the higher the number then the more equipment is dispatched in response. hydnjo talk 03:07, 13 February 2006 (UTC)[reply]
Agreed. They convey the severity of the fire to the firefighters by sounding the alarm that many times, so a three alarm fire has them ring the alarm at the fire station 3 times. The firefighters respond with a specific level of staff and equipment. StuRat 03:48, 13 February 2006 (UTC)[reply]

Weapons legality[edit]

Do we have an article on legality of various sorts of weapons, in various jurisdictions, etc? I am specifically interested in if we have a n article on legality of martial (i.e. non-firearm) weapons. -Tim Rhymeless (Er...let's shimmy) 09:32, 13 February 2006 (UTC)[reply]

Which raises the question what constitutes a weapon. I have several machetes and someone once told me that is illegal (in the Netherlands). But they aren't any more dangerous than a kitchen knife. Or several 'normal' tools in an ordinary toolbox. 80.126.178.133 08:42, 13 February 2006 (UTC)[reply]

I know what you mean. I was recently told that the thermonuclear devices I keep in my basement are even considered by some to be weapons. StuRat 09:02, 13 February 2006 (UTC)[reply]
Which are just as utilitarian as home heating appliances as my long bladed hedge trimmers. Seriously though, machetes are great for coconuts, not that it's easy to get fresh coconuts in the continental U.S.—WAvegetarianCONTRIBUTIONSTALKEMAIL22:11, 13 February 2006 (UTC)[reply]

Stressed out[edit]

I am totally stressed out. i have so much homework and i can't seem to get any done. Japan is pretty coo. It's hard to speak Japanese though. I think we should all learn English.

Is there any question here ? StuRat 17:53, 13 February 2006 (UTC)[reply]

yeah I'm kinda stressed out too, and I think Japan is cool too, though I've never been there...but... what is the question? --Cosmic girl 21:55, 13 February 2006 (UTC)[reply]

He said Japan is coo, not cool. He's too coo to use lame words like cool. --Kainaw (talk) 04:07, 14 February 2006 (UTC)[reply]
Oh, but of course. -Tim Rhymeless (Er...let's shimmy) 08:36, 14 February 2006 (UTC)[reply]

The question is, what to do when you're stressed out? The answer is, find some way to rest or relax. -- (Dr.) Halcatalyst 00:04, 14 February 2006 (UTC) Expect a bill at the end of the month[reply]

I go hang out at the store with my friends and bug the ppls there. -Tim Rhymeless (Er...let's shimmy) 08:36, 14 February 2006 (UTC)[reply]
Ah, typo police here: What Tim actually wanted to say was that he is like sooo coo he goes to the store with all his friends and buys them apples. Now, that would be a real stress-breaker, no? Plus it's healthy, think of all those vitamins and the gratitude of his friends' moms... Ah, really heart-warming!-[www.stillnewt.org/library DJ Vollkasko] 19 Feb 2006

should law follow social change within a society or should it create social change?[edit]

After we write your essay for you, should we deliver it directly to your teacher, too ? StuRat 17:55, 13 February 2006 (UTC)[reply]
I believe the answer is "yes". Deltabeignet 23:02, 13 February 2006 (UTC)[reply]
How many words does the essay have to be? Can the answer be, "Law should follow social change within a society when the social change is created by a law that is following the change within the society." --Kainaw (talk) 04:06, 14 February 2006 (UTC)[reply]

Joan Didion, "The Year of Magical Thinking"[edit]

To what do you think the last sentence in this novel refers: "No eye is on the sparrow but he told me that."

Joan Didion's The Year of Magical Thinking (ISBN 140004314X) is not a novel but rather a meditation on her year of grief following the death of her husband, John Gregory Dunne. Though I don't have the book at hand, the sentence you refer to is clearly a complex allusion to the traditional gospel spiritual, "His Eye is on the Sparrow," which was also the title of a well-known book (ISBN 0306804778) by blues singer Ethel Waters. The idea in the titles is that God cares for even the least of his creatures (see Matt. 6:25-34). Didion and Dunne were not religious. Hence, Didion writes, "No eye is on the sparrow"; but what she means by "he told me that" must be understood from the sentences which precede this final sentence. --Halcatalyst 04:17, 14 February 2006 (UTC)[reply]
P.S. For proper citation, refer to MLA style manual. The model and example given there are as follows:
  • A website: Author of webpage."Article Title." Title of webpage. Date of publication. Institution associated with. Date of retrieval <url>.
  • "Plagiarism." Wikipedia: The Free Encyclopedia. 22 Jul. 2004, 10:55 UTC. Wikimedia Foundation. 10 Aug. 2004 <http://en.wikipedia.org/wiki/Plagiarism>.
--Halcatalyst 04:41, 14 February 2006 (UTC) If the advice was not needed by you, it's needed by others.[reply]

Actually, Didion did mention in the book that she and her husband were religious, even going to church.

Retaliation towards Germany post World War 2[edit]

After WWII finished, were there acts of retaliation towards Germany from citizens of nearby countries? Is there a list anywhere? When was the last act committed? Thanks. 218.101.92.204 22:21, 13 February 2006 (UTC)[reply]

Football hooligans tried to murder a Russian in England at Euro 96 because they mistook him for a German. The last act of retaliation is certainly yet to be committed. Markyour words 22:26, 13 February 2006 (UTC)[reply]
When I was stationed in Germany in the 1970s, a friend of a friend (so it might be an urban legend) got out of the Air Force and decided he wasn't ready to return to the United States yet, so he got a house in a German town near to the base, but he had to replace his US Forces license plate with a German plate. He decided to take a driving tour of parts of Europe, and he claimed that when he was driving in Yugoslavia, people threw rocks at his car because of the German plates. User:Zoe|(talk) 22:32, 14 February 2006 (UTC)[reply]
For a start, have a look at the following articel, where you will find some links to other related articels.: Expulsion of Germans after World War II--nodutschke 15:36, 15 February 2006 (UTC)[reply]

Actually, Didion mentions in the book that she and her husband WERE religious. Take that.

Prime Ministers of the UK.[edit]

Primary occupations of UK Prime Minister: Does any one have any comments? Tony Blair: Lawyer. John Major: Banker Margret Thatcher: Lawyer Jim Callahan: Union offical. Edward Heath: Civil Servant Harold Wilson: Academic Douglas Home: Polititian, Aristocrat Anthony Eden: Politition. Clement Attlee: Lawyer Winston Churchill: Writer, politition.

Nevil Chamberlain: Planter Stanley Baldwin: Industrialist Ramsey MacDonald: Polititian David Lloyd George: Lawyer

Comments about what? That they had jobs? A lot of people have jobs... Do you have a more specific question? Dismas|(talk) 23:13, 13 February 2006 (UTC)[reply]
ISTR they were all politicians at some stage, which seems like an unlikely coincidence to me. Grutness...wha? 06:01, 14 February 2006 (UTC)[reply]
I'd say their primary occupation was as politicians as they all got to be PM. By the way you've got the beginnings of a nice list, perhaps List of British prime Ministers by occupation, why don't you make a start on it & no doubt others will fill in the blanks over time. AllanHainey 08:24, 14 February 2006 (UTC)[reply]

Somewhat related; I've tallied the studies done by members of parliament in the Netherlands in 1995 and these were the most common ones:
law 33
politicology 22
economics 21
social studies 16
education 14
agrarian (incl biology) 10
languages 7
math/physics/chemistry 7
technical studies 5
The ones at the top make sense, but note the lack of hard sciences. Isn't it about time things were balanced out a bit more? Do we need a new party for this? How is this in other contries? DirkvdM 09:12, 14 February 2006 (UTC)[reply]

I've no hard data on this, but I think that in New Zealand at least, there are a handful of "hard science" people in our 125-member parliament. Our current Minister of Transport was formerly a veterinarian, for instance. Strangely, my local M.P. (also a cabinet minister) taught German at High School. Mind you, our Prime Minister was a Political Studies lecturer, her Deputy was an Economics lecturer, and the leader of the Opposition was a senior civil servant (Manager of the Reserve Bank, no less). Other prominent NZ MPs include a Red Cross spokesperson, an electrician and technical science teacher, a carpenter, a zoologist and oil-rig worker, a lecturer on environmental studies, any number of lawyers, political scientists and economists, and - believe it or not - a transsexual prostitute and stripper. Grutness...wha? 11:03, 14 February 2006 (UTC)[reply]
You should check those facts. For example Margaret Thatcher was never a lawyer - she had a degree in Chemistry and worked as a research chemist before becoming a politician. Also Churchill was a soldier as well as a writer.
I don't find the figures odd at all. People skills are generally not highly valued in the 'hard sciences', and so people with those skills don't tend to favour them as careers. On the other hand people skills are extremely valuable in politics. The people who manage to make a success of both must be pretty exceptional. DJ Clayworth 18:13, 14 February 2006 (UTC)[reply]
What is politicology? User:Zoe|(talk) 22:33, 14 February 2006 (UTC)[reply]
DJ, you seem to focus on the rhetorical side of politics, the presentation to the public and the building up of a social network, whereas I was thinking of the jobs at hand, which are often technical. Such knowledge and insight are important for the members of parliament (and the people behind them). The ministers can't do and think of everything. That's one thing parliament is for. The party members (whether in parliament or not) to help them work things out and members of other parties to check on them. And if the job at hand is some construction thing (which it often is) then some technical knowledge would be very handy. In several fields. So you need a broad orientation in parliament, not a focus on a few studies that happen to all sit in the same little corner. True, technicians and 'hard scientists' don't usually have the best social skills, but of course some do, and I think it's time they went into politics. However, if Margaret Thatcher is the only example, one might wonder if that is such a good idea after all. :) DirkvdM 08:23, 15 February 2006 (UTC)[reply]

why did japan want to go war with the usa in both major wars ww1 and ww2?[edit]

  • Japan's participation in World War I was on the same side as the US, see the Triple Entente article. In World War II, Japan signed an alliance with Germany and the attack on Pearl Harbor which brought the US into the war was part of an Imperial expansionist drive on the part of Japan in the Asia-Pacific region. --Canley 03:01, 14 February 2006 (UTC)[reply]
Initially, the public was taught that Japan was evil and joined the evil Germans against the US. But, at some point, the Zimmerman Telegram was made public. It showed a slightly different scenario. To be brief and skip over all the facts (because it is late and my brain isn't functioning)... Germany sank the RMS Lusitania with a uboat. The US threatened to join against Germany if it ever used a uboat to sink ANY ship without surfacing first. Germany cut a deal with Japan and Mexico. If they attack the US on the same day (Japan from the west and Mexico from the south), then Germany will help from the east and the US will be defeated right away. Then, Germany can sink all the ships it wants without surfacing. The US intercepted the telegram to Mexico, found out about the deal, and talked Mexico out of the deal. Then, the US prepared to defend against a German attack from the east that never materialized - but Japan did attack from the west. They didn't attack Los Angeles as expected. They attacked Pearl Harbor. So, the US public, that was dead set against the war all along, suddenly wanted blood. The US joined the war and the rest is history.
Now, I want to point out that, for all we know, the Zimmerman Telegram could all be a hoax set up by the US government. But, it does make the whole story a bit more interesting. --Kainaw (talk) 04:02, 14 February 2006 (UTC)[reply]
Note that the Zimmermann Telegram incident took place during World War I, and the attack on Pearl Harbor during World War II. The Mexican decision not to participate in this alliance seems to indicate that Japan was not even informed by Mexico (or Germany) of the proposal. Japan was also subject to the Anglo-Japanese Alliance with Britain, and hence entered the war on the side of the Allied Entente Forces. --Canley 05:46, 14 February 2006 (UTC)[reply]
Also, Germany did attack the US by sinking ships in harbours (New York harbour I believe, but I'm not sure), in other words in US territory. And Germany did officially declare war on the US (after Japan had done so), which gave the US the perfect excuse to fully enter the war. Before that, they had their own Monroe doctrine stopping them. It is generally believed that Hitler's biggest mistake (apart from starting the war in the first place) was the attack on the USSR. But he might just have pulled that off if he hadn't pulled the US into the war as well. But that's mere speculation. DirkvdM 09:20, 14 February 2006 (UTC)[reply]
Do we really think that the US would have fought a one-front war with Japan if Germany hadn't declared war? User:Zoe|(talk) 22:35, 14 February 2006 (UTC)[reply]
For a time, yes. The US would have preferred to focus it's military on Japan until it was defeated (maybe by 1944 in the case of a one-front war) and then declare war on Germany. StuRat 21:05, 15 February 2006 (UTC)[reply]

Note that both Japan and Germany had reasons to declare war on the US during WW2:

  • The US and other nations had placed an embargo on Japan after the invasion of Manchuria and other aggressive acts by Japan. Since Japan is an island with limited resources, this embargo was starting to hurt. This left them the options of withdrawing troops or attacking those nations (including the US) which were blocking their access to the resources they needed. StuRat 21:05, 15 February 2006 (UTC)[reply]
  • In the case of Germany, the US's Lend-Lease program with England was supplying their enemy with the war matériel needed to hold out against the German military. StuRat 21:05, 15 February 2006 (UTC)[reply]
  • Also, why did not Japan invade Switzerland or Sweden ??? --DLL 22:09, 15 February 2006 (UTC)[reply]
I assume you mean Germany...
They didn't have to. Sweden was a tempting target though due to the essential iron ore exports to the German war machine. However, Germany needed Norway for strategic reasons and on the road to Norway Germany needed to use the airbase in Aalborg in northern part of Jutland (Denmark) to ensure efficient Luftwaffe operations. Almost as an afterfought Hitler decided to take all of Denmark. So now Sweden is sorrounded by German occupied territories to the west and south while the Soviet Union is harassing Finland to the east. Germany could negotiate pretty much any deal with Sweden including iron ore shipments and transportation of German troops across Swedish territory to Finland - which they did. Attacking Sweden was needless as they already had everything they needed.
In the case of Switzerland - Hitler wasn't really interrested in a war with the western powers. He wanted Chechoslovakia, Poland and pretty much most of Russia. The French and British declarations of war wasn't Hitlers real agenda - Hitler wanted peace with France and England and was extremely frustrated concerning the British determination. Hitler had assumed that when France surrendered he could starve Britain to surrender so he could focus on the real objective - the Sovjet - and Lebensraum.
An attack on Switzerland was 1) Not Hitlers agenda. 2) Would further complicate matters if peace were to be negotiated with France and England. 3) Switzerland wasn't of any strategic importance to Germany as the German ally Italy lay to the south, the French Vichy government lay to the west and Germany lay to the East, in now Austria.
In hindsight - France and England declared war on Germany to protect Poland and Eastern Europe from German assimilation. The end result became Sovjet occupation instead... Celcius (Talk) Wiki be With us! 15:37, 16 February 2006 (UTC)[reply]
Is a "Sovjet" a jet made by the Soviet Union ? LOL StuRat 23:57, 16 February 2006 (UTC)[reply]

Movie title[edit]

I need help recalling the title of a movie (comedy, perhaps 1980s). The premise was that two youngish guys, not wanting to have their vacation (or something) spoiled by the death of their host/benefactor, pretend that he is still alive by proping him into a wheelchair (with sunglasses) and bringing him along to various events. Groaning allowed. Thanks, --hydnjo talk 23:26, 13 February 2006 (UTC)[reply]

Sounds a bit like Jeremy Bentham. JackofOz 23:41, 13 February 2006 (UTC)[reply]
Nah, this one was set in contemporary times.  ;-) hydnjo talk 23:44, 13 February 2006 (UTC)[reply]

Was it www.imdb.com/title/tt0098627/ this? 69.142.89.10 00:30, 14 February 2006 (UTC)[reply]

We should at least provide a Wikipedia linkPhilHibbs | talk 11:18, 14 February 2006 (UTC)[reply]
There was even a sequel. User:Zoe|(talk) 22:37, 14 February 2006 (UTC)[reply]

Is their there a hell?[edit]

  • If you mean a physical place with lots of fire and a guy called Satan. There's no scientific proof that place exists, but that doesn't mean anything. If you take it a less literal and use hell to describe a place where people have to relive bad experiences for the rest of eternity, earth could easily be hell. It also depends a lot on your religion and what your personal beliefs are. - Mgm|(talk) 23:45, 13 February 2006 (UTC)[reply]
Not to mention here, and I've been to Hull and back! СПУТНИКССС Р 23:50, 13 February 2006 (UTC)[reply]
No, not unless you count Dundee. Markyour words 01:23, 14 February 2006 (UTC)[reply]
Working at Mc Do's ? see below --DLL 22:05, 15 February 2006 (UTC)[reply]
  • "Do I know where hell is, hell is in hell-o. / Heaven is good-bye forever, it's time for me to go." (Lee Marvin: Wand'rin' Star, 1951)--www.stillnewt.org/library DJ Vollkasko 19 Feb 2006

Why would God want someone to work at McDonald's?[edit]

A long time ago (back in '04), when I told a friend about how I wished for a better job, he told me something like (can't remember exactly), "Good luck. You'll find a better job. Or maybe God thinks you're better off working at McDonald's."

Well gee, how does working at such a menial place serve God in any way? How does that make my life better, how does it make me tithe more, and how does it further God's cause? I don't know why God would want for anyone to work over there. In fact, I think all humans deserve better than to work there. Androids should replace the employees, while the employees work at better places. --Shultz 23:43, 13 February 2006 (UTC)[reply]

  • "God works in a mysterious way, His wonders to perform". JackofOz 23:49, 13 February 2006 (UTC)[reply]
  • God works in mysterious ways. It may not directly make your life better, but by serving people in a friendly manner, you may affect their lives in ways you can't possibly imagine. Knowing first aid when a customer drops to the floor during your shift could save their life. Your friendly service (despite your working conditions) may make customers reflect on themselves and decide they don't have it so bad and shelf their suicide ideas. You working there could influence a lot of people in a good way. - Mgm|(talk) 23:49, 13 February 2006 (UTC)[reply]
  • No need to see God as a placement counselor, but you could take the attitude that no honest work is demeaning and that all work can be sanctifying. --Halcatalyst 23:54, 13 February 2006 (UTC)[reply]
Well you see, I was the lobby person, the most mindless job there ever was. McDonald's wasn't my bag, so I resigned in a hurry and left on 2-2-2005. I understand that lobby people could save the lives of those who have heart attacks, but it's not customer service, and when androids replace employees of menial jobs, they could still be programed to save lives, and much, much more. (Again, human employees deserve better.)
I hope to design and program androids once I'm out of college. I could make a MUCH bigger impact this way. What majors do you suggest that I take, that fits what I want to do? --Shultz 23:57, 13 February 2006 (UTC)[reply]
Engineering. --User:68.112.242.121
There are several engineering majors at my university, K-State. Civil Engineering, Chemical Engineering, Mechanical Engineering, Electrical Engineering, and the list goes on and on. Therefore, which majors are appropriate for a future android designer & programmer? --Shultz 00:16, 14 February 2006 (UTC)[reply]
Android construction would span several disciplines, but mechanical engineering or computer science would be the best choices. GeeJo (t) (c)  09:06, 14 February 2006 (UTC)[reply]
  • Every life serves a purpose. It may just be that yours is to serve as an example to others. — PhilHibbs | talk 11:14, 14 February 2006 (UTC)[reply]
You could start with artificial intelligence and follow all the links. That'll take you to just about all the articles and sources we have in that area. The majors would be harder, but like they said above, start with basic mechanics, electronics and computer science and see if you can handle it. Also go for job shadowing if you get the chance, to see what it's like. As for the religion - If you really trust the big G to make you happy, you shouldn't keep questioning him. That can only lead to eternal damnation and teenage pregnancy. Black Carrot 12:31, 14 February 2006 (UTC)[reply]
Engaging in honest work, learning to serve others, understanding humility; I can think of many reasons why God might want people to work in McDonalds. However I can't think of any reason why he would want people to eat there. DJ Clayworth 18:05, 14 February 2006 (UTC)[reply]
My understanding of Christianity is that God gave man free will to work wherever he was able to get employment. In other words, God doesn't give you bad jobs, you get them all by yourself. (If I was Christian, this is what I would probably think, anyway. I don't think God cares where every person gets a job, in the same way He really shouldn't care who wins the Superbowl.) --Fastfission 00:12, 15 February 2006 (UTC)[reply]
No God, christian or non-christian, does such menial works as choosing a job for you. We are born free and grow slaves of caricatural Gods. Just do your best. --DLL 22:02, 15 February 2006 (UTC)[reply]

If you can demonstrate career progress in your job at McDonalds, that is a very worthwhile jumpstart to any career. For example, if you can advance to shift supervisor, that demonstrates ambition and leadership. At many companies, that is more important than specific technical skills. So for example, a graduating student who was a shift supervisor at McDonalds will almost certainly be hired ahead of someone who has had more "relevant" technical experience but has not demonstrated any ambition or leadership. Samw 02:12, 19 February 2006 (UTC)[reply]

Saddams Step Father[edit]

What ever happened to saddams abusive step father? Did he live to see his step son come to power? Did he gain an office? did Sadam take retribution on him? The article on Saddam mentions little about Saddams step father.

According to www.boston.com/news/packages/iraq/globe_stories/102702_hussein.htm this source, "Even after Saddam became the strongman in Iraq, and all the family lived in tremendous affluence, Saddam's stepfather was still living in a mud hut in the tiny old dusty village of Uja." Saddam got away from his step-father as early as he could, at age 17. There's no evidence I could find that indicates he ever did more than scorn and ignore him. --Halcatalyst 14:26, 14 February 2006 (UTC)[reply]

World leaders[edit]

World leaders ages. Are their any siting heads of state/ government born before 1918? Are their any sitting heads of State/ government born after 1980?

Confusing question. There's only one head of government in the British parliament, Tony Blair. Is he a "world leader"? That's a matter of opinion. JackofOz 23:57, 13 February 2006 (UTC)[reply]
I think the question is "Are there any leaders (heads of state or government) in the world aged over 88 or under 26?" In the case of heads of state there might well be some oldies, since that can include kings and queens, who don't need to do much beside be present at occasions and one can do that at an age. Other than that, I doubt it. DirkvdM 09:26, 14 February 2006 (UTC)[reply]
DirkvdM, maybe next time you can strike the original title through, rather than replacing it entirely. The way it reads now, one would be as confused by my answer as I was about the original question/title. (The original title was "British parliment" [sic]). JackofOz 10:58, 14 February 2006 (UTC)[reply]
The question was posted twice and I merged them. Don't recall replacing the title. DirkvdM 08:27, 15 February 2006 (UTC)[reply]
The software seems to be having a tough time. The "show changes" and the updated page show different things. Very confusing. Sorry if I defamed you. JackofOz 10:59, 15 February 2006 (UTC)[reply]
Nah, I'm not easily defamed. DirkvdM 08:18, 16 February 2006 (UTC)[reply]

As far as monarchs are concerned, Malietoa Tanumafili II of Samoa is the only one who is older than 88 - he's in his mid 90s. Of the others, Taufa'ahau Tupou IV of neighbouring Tonga was born in 1918, and Abdullah_of_Saudi_Arabia is 80, just ahead of 79-year-old Elizabeth II of the United Kingdom and 78-year-old Pope Benedict XVI. Most of the current reigning monarchs were born in the 1950s. As for the youngest none are nearly as young as your question, with Mswati III of Swaziland the youngest at 37. There may be younger elected or military leaders, but I don't know. BTW, I was surprised to notice that there are currently two ruling Albert IIs, one in Belgium and one in Monaco, and that the heads of state of Kuwait and Qatar have almost identical names! Grutness...wha? 11:24, 14 February 2006 (UTC)[reply]

There was a Final Jeopardy question on Jeopardy! last week about which two currently ruling European monarchs have the same name.  :) User:Zoe|(talk) 22:42, 14 February 2006 (UTC)[reply]
Bet you wished you had had the Wikipedia ref desk at hand then. :) DirkvdM 08:27, 15 February 2006 (UTC)[reply]

Famous woman who have had abortions.[edit]

Their are many famous woman known for their permiscuity. Such was case before birth control became widespread, yet many never bore children. Is their any evidence famous promiscuious woman who aquired back ally abortions?

  • It might help if you could narrow your question down to identify the women who didn't have children that you are thinking of, and the time period. I assume you mean pre-20th century? My understanding is that in ancient Greece & Rome, rather than practicing abortion people practiced infanticide. (Note, however that the Greek hetaira Aspasia kept her child. But the hetaira weren't necessarily "promiscuious" in the sense you are probably thinking of.) Abortion methods were known, but in ancient times were very dangerous. Even until the early modern period, even though we know the techniques that were used, I doubt there is any record of specific women who used them. It is much easier to figure out if someone had a child 500 years ago than it is to figure out that she didn't have one. As a side note, birth-control methods were known in ancient times (the ancient Eqyptians did something with a cervical cap made from dried aligator dung among other things), they just weren't as effective as the ones we have today and were comparatively more expensive. Crypticfirefly 06:17, 14 February 2006 (UTC)[reply]

Kim Jong IL[edit]

Would society not be better off if somone would kill Kim Jong ILL?

Let's see. leaving aside the question of whether killing a person is ethical or "good", we have several scenarios, incuding among them:
  1. Kim dies, communism collapses, Korea is reunified, everyone lives happily ever after
  2. Kim dies, someone even more despotic takes his place, claims that "the west" was responsible for the killing of his predecessor, and unveils that yes, they really do have nuclear weapons.
  3. Kim dies, there is a power vacuum, civil war erupts in North Korea, spilling over into both South Korea and China. South Korea calls on its allies in the U.S., and we end up with American and Chinese troops eyeballing each other across the no-longer-demilitarized zone.
I suppose your answer depends on whether you feel lucky. Well do ya, punk? Grutness...wha? 05:57, 14 February 2006 (UTC)[reply]
Good reasoning. But then why do (or did) the US try to kill Castro? That would certainly make him a martyr. Then again, the US have made a similar misjudgement in the Bay of Pigs, so I suppose hatred, like love, blinds people. DirkvdM 09:30, 14 February 2006 (UTC)[reply]

Easiness[edit]

Who is easier French woman or Latin woman?

I would say that it depends very much on the woman in question and not her ethnicity... But why don't you do some field tests and let us know of your results.  ;-) Dismas|(talk) 05:34, 14 February 2006 (UTC)[reply]
Original research is frowned upon in Wikipedia. David Sneek 07:53, 14 February 2006 (UTC)[reply]
A) It was a joke, see the smilie. B) I didn't say they should write an article about it. C) It was a joke. Dismas|(talk) 09:34, 14 February 2006 (UTC)[reply]
Heh, I think David Sneek meant his statement as a joke as well. :) (It was funny, too!) zafiroblue05 | Talk 10:45, 14 February 2006 (UTC)[reply]
So, what's the answer? I need to make travel plans. --Halcatalyst 13:26, 14 February 2006 (UTC)[reply]
For me, French. I don't speak Latin. DJ Clayworth 18:00, 14 February 2006 (UTC)[reply]
  • Definitely French women. I don't speak much French, but Latin is a dead language anyway. :) - Mgm|(talk) 18:45, 14 February 2006 (UTC)[reply]
I wonder why people keep saying that. It might not be lingua franca anywhere, or any country's official language, but it is far from dead. JackofOz 20:27, 14 February 2006 (UTC)[reply]
It would be considerably easier to have sex with a French woman, since all of the Latin women are dead. User:Zoe|(talk) 22:44, 14 February 2006 (UTC)[reply]

femme vs. femina—I think the one syllable of the French is slightly easier than the three of the Latin, especially because you don't have to remember which vowels are short and which are long, or where the stress goes. —Charles P._(Mirv) 04:49, 15 February 2006 (UTC)[reply]

If by Latin, however, you mean Spanish - then the answer is French women. Not only are French women easier, Spanish women are also uglier, all look the same, and have the worst dress sense in Western Europe. This isn't a personal attack, but it is original research. Proto||type 14:02, 15 February 2006 (UTC)[reply]
Oh, wow. This is no personal attack either, but you do sound quite ignorant to me. --RiseRover 14:00, 19 February 2006 (UTC)[reply]
It is a rather unpleasant impersonal attack, though. Superm401 - Talk 06:28, 20 February 2006 (UTC)[reply]

UFO abduction[edit]

How do extrateresials abduct people from cities and other densely populated places? Would they not be witnessed?

Your question assumes that the abductions themselves are real and the stories about them true when in fact they have not been even remotely proven. See Alien abduction for more info. Dismas|(talk) 05:32, 14 February 2006 (UTC)[reply]
You're talking about creatures that are (probably) totally and utterly different and of whom we know nothing apart from some wild stories and you expect us to be able to tell you how they do things? Thanks for giving us so much credit. You're too kind. :) DirkvdM 09:34, 14 February 2006 (UTC)[reply]
It's easy. They use an application of Clarke's Third Law. Notinasnaid 11:21, 14 February 2006 (UTC)[reply]
If abductions were conducted by people from our own world and same technology available as we have, then in big city it is easier to hide after the abduction. Just have a get away vehicle (like an auto or van) that a few blocks away will drive up a famp into a semi, so that the get away vehicle as witnessed by people at site of abduction, is no longer visible to any police looking for it. This is called a successful kidnapping plot, and there is plenty of evidence that people know how to do it. Actually sneaking up on people in a sparsely populated rural area is more difficult, because strangers are more likely to be spotted.
When abductions are to be done by visitors from another time (time travelers collecting samples from our time, because when history gets changed, they no longer have reliable evidence for science analysis), dimension, or wherever, they have the technology that lets them move between wherever and us reality, so obviously real easy for them to take passengers back with them. Their motivations must be quite different from present day humanity, because if our people had this technology, the bad people among us would be busy removing all the gold from Fort Knox, the crown jewels, emptying the Smithsonian, raping starlets, you name it, but they are not, which tends to imply that either there are no such visitors, or if they are, their value systems are totally alien to us. User:AlMac|(talk) 11:33, 14 February 2006 (UTC)[reply]
Thankfully this business is on the decline because the aliens have "learned all they can from the anal probe." Marskell 17:24, 17 February 2006 (UTC)[reply]

February 14[edit]

British Museum[edit]

I think my question didn't get saved so I'll ask again. In the Victorian Era around the late 1800's does anyone know what the floor layout was, did it include an egyptian exhibit, if so on what floor and what was included, and if anyone knows what other exhibits this would include? —Preceding unsigned comment added by 198.188.255.2 (talkcontribs)

Perhaps read British_Museum#History. Markyour words 01:27, 14 February 2006 (UTC)[reply]
  • I can't give any details, but given that Ancient Egypt was very popular in the late 1800s (hence mummy unwrappings) it's only logical for the British museum to exhibit Egyptian artefacts, although a lot less mummies (the famous ones still had to be discovered). - Mgm|(talk) 18:48, 14 February 2006 (UTC)[reply]

Religion in Sri Lanka[edit]

Is the Multi-religious Federation www.tamilnet.com/art.html?catid=13&artid=17183 a specific organization in Sri Lanka? What is its local name? ᓛᖁ♀ 00:31, 14 February 2006 (UTC)[reply]

Monotheism/Religion[edit]

Was polytheism the first type of religion or was there also monotheism during the same time period?

71.98.96.188 02:03, 14 February 2006 (UTC)[reply]

  • See the article History of religions, which ties the growth of "city states" to the development of monotheistic religions. --Canley 03:07, 14 February 2006 (UTC)[reply]

That's history. But religion goes back way beyond (written) history. Probably at least tens of thousands of years. So the question is impossible to answer. DirkvdM 09:36, 14 February 2006 (UTC)[reply]

The Abrahamic religions are generally thought to have been the first monotheistic religions. But see the article on Monotheism for a somewhat different view. Polytheism is mostly ancient but can still be found today. --Halcatalyst 13:17, 14 February 2006 (UTC)[reply]

People worship whatever seems powerful to them. Early on, that's mostly animals. Then, once animals have been tamed, comes the Sun, Moon, etc. as in the Egyptian gods. Next, come people, so you get gods that are a lot like people, such as the Greek and Roman gods. Finally come the fundamental forces of nature, like gravity, and one omnipotent god is sufficent to explain those. StuRat 22:34, 14 February 2006 (UTC)[reply]

classical music piece[edit]

I am trying to find the name of a piece of classical music I know only as "Fire Engines". Thank you —Preceding unsigned comment added by 66.15.123.8 (talkcontribs)

Can you give us some representation of the melody? —Keenan Pepper 04:56, 14 February 2006 (UTC)[reply]
Uh... Perhaps he means Chariots of Fire theme by Vangelis? It is a classical in a way, but not "classical" ☢ Ҡiff 09:29, 14 February 2006 (UTC)[reply]

Name The House Wreckers[edit]

This was on British TV but I believe the incident is American and would have been shown there too.

Two or more teenagers videoed themselves causing massive vandalism to a residence. I don't believe any of the teens lived there. It was a large house. Much of the vandalism was done with baseball bats.

The video footage includes shots of; a celing fan, with a light in the centre of it, being smashed down; the toilet seat being kicked off and the bowl being hit with a bat; filmed from below, one of them kicking through the ceiling from above; many, many windows being smashed; there is an outside shot of the house from some distance away revealing that all the windows that can be seen from the view had been smashed. I understand they were punished for this with their video, of course, making their case impossible to defend.

Anyone able to name the kids or add any details I could plug into a search engine? --bodnotbod 04:25, 14 February 2006 (UTC)[reply]

I saw the footage, it was quite a while back, over a year ago, I think. I also recall that it was a new housing development under construction at the time. StuRat 22:23, 14 February 2006 (UTC)[reply]
Yes, it is at least a year old, I'd say. Hmmm, from what you say then, perhaps the house was a show-home rather than owned by anyone at that time. --bodnotbod 03:52, 15 February 2006 (UTC)[reply]
In the US it is common for developers to build houses in a subdivision ahead of time, in the hopes that they can sell them once built. I believe it was one of those, under construction (nearly finished). StuRat 04:39, 15 February 2006 (UTC)[reply]
Could this be it ? It's from 11 years ago: query.nytimes.com/gst/fullpage.html?res=990CE5D81E38F932A25750C0A963958260 StuRat 05:00, 15 February 2006 (UTC)[reply]

Musical charts[edit]

Why do some music recordings feature on charts for more than one style of music? For instance a musical release may feature on both the pop and country charts etc?

As I understand it, this is because of the radio stations that a track is played on. Sometimes the pop stations might play a country song, for example. -Tim Rhymeless (Er...let's shimmy) 08:37, 14 February 2006 (UTC)[reply]

Killed By Trousers[edit]

I see statements like "more people are killed by trousers than by aeroplanes", but how do I find out the statistics for the number of people killed by trousers? I have done a few web and wikipedia searches but all I can find are uncited quotes like this. — PhilHibbs | talk 09:47, 14 February 2006 (UTC)[reply]

Uh-oh. I can see the TV reality show now: "When good trousers attack" Grutness...wha? 10:45, 14 February 2006 (UTC)[reply]
87.2% of statistics are just made up on the spot. Notinasnaid 11:01, 14 February 2006 (UTC)[reply]
73.2% of people making that claim have watched The Wrong Trousers. —WAvegetarianCONTRIBUTIONSTALKEMAIL14:11, 14 February 2006 (UTC)[reply]
You know, it's interesting. My psychology and statistics textbooks claim people who make up statistics make up round numbers, like 10% of people being gay, but it seems to me pseudo-random numbers are much more popular. They sound like you know exactly what you're talking about. Black Carrot 20:18, 14 February 2006 (UTC)[reply]
Realistically we will never know the number of trouser-related deaths, because the countries where most people live (India, China etc.) have bigger worries than keeping those statistics. In the UK, however, RoSPA publishes amusing statistics on accidents (presumably most are not fatal) involving various items. In 2002 these included 9,410 accidents involving trousers, 17,159 involving cats, and 656 for 'sex or marital aid e.g. condom or vibrator'. You can laugh at other people's misery www.hassandlass.org.uk/query/MainSelector.aspx?Reset=T here, though the interface is a tad cumbersome. Markyour words 20:46, 14 February 2006 (UTC)[reply]

I have a hard time picturing how people are killed by pants, but there are an amazing number of tree related deaths each year, including the following:

  • Cars that run into trees.
  • Trees that fall on people or houses, usually during storms.
  • People who fall out of trees while climbing them.
  • Objects that fall out of trees on people, like coconuts and treehouses.
  • Lightning that hits trees and kills people hiding under them from the rain.
  • Trees can spread a fire that kills people.
  • Trees, once cut down to form houses, can kill peeople by collapsing, catching fire, etc.

StuRat 22:18, 14 February 2006 (UTC)[reply]

Don't forget that time a guy had sex with a tree, cut himself, got an infection and died.  freshgavinΓΛĿЌ  06:38, 15 February 2006 (UTC)[reply]

Greatest Sitting world leader[edit]

Who is the greatest sitting (current) world leader

The Buddha is generally depicted as sitting. --Halcatalyst

Yes, he is the greatest, sitting idol. David Sneek 14:16, 14 February 2006 (UTC)[reply]
http://en.wikipedia.org/wiki/Image:Lincoln_statue.jpg Abraham Lincoln is also sitting, and pretty great. Seriously though, entirely subjective questions are not going to get a sensible answer here. DJ Clayworth 17:57, 14 February 2006 (UTC)[reply]

I nominate: Tony Blair.

  • How about the Dalai Lama. Who is the greatest depends on your interpretation of great. - Mgm|(talk) 18:51, 14 February 2006 (UTC)[reply]
I nominate maddox.xmission.com Maddox Black Carrot 20:15, 14 February 2006 (UTC)[reply]

Lonest serving legislators?[edit]

Who is the longest serving current member of the French National assembly? Who is the longest serving current member of the Japanese Diet?

Because I want to write a wikipedia article listing longest, currently serving legislators.

There's a list of the 577 deputies on www.assemblee-nationale.fr, you may also try asking them. David.Monniaux 13:39, 20 February 2006 (UTC)[reply]

Jews in power[edit]

Outside of Isreal are their any jews who currently serve as president or Prime minister of a nation?

Well, www.greatestjeneration.com/archives/000837.php this admittedly non-authoritative link states that Job Cohen would have been the first Jewish PM outside of Israel had he won election in the Netherlands two years ago. Alas, he did not. Michael Howard former leader of the Conservative Party (UK) is Jewish, but of course he never made it to power either. Finally, there's Joe Lieberman who almost became American Vice President in 2000. I think it quite possible that the answer is no at the moment.
Benjamin Disraeli is probably the best known person of Jewish descent to hold Prime Ministerial office outside of Israel (though that's going back more than a century). Marskell 13:46, 14 February 2006 (UTC)[reply]
Léon Blum who was prime minister of France was jewish. Béla Kun, the communist leader of Hungary (for a short while) was somewhat jewish (as a hard-line communist he was of course against all religion). On that note, Samuel Aba, a medieval king of Hungary, was likely jewish. (Which is rather remarkable). --BluePlatypus 14:22, 14 February 2006 (UTC)[reply]
New Zealand's 19th century prime minister Sir Julius Vogel was also Jewish. ISTR that there has been another Jewish PM in New Zealand, but his name eludes me. Not that that answers the "currently" part of the question. Grutness...wha? 21:49, 14 February 2006 (UTC)[reply]
Julius Vogel also wrote the first New Zealand science fiction novel. Lisiate 21:52, 14 February 2006 (UTC)[reply]
and the NZ science fiction awards (of which I am a past recipient :) are named in his honour. He may not have been the first NZ sf novelist though... the case is still out on that one. Oh, and as far as Jewish heads of state are concerned, though he's not a national head, Michael Bloomberg is the next best thing. Grutness...wha? 22:14, 14 February 2006 (UTC)[reply]

Janet Jagan, president of Guyana from 1997 to 1999, was of Jewish descent, although nonpracticing. Bruno Kreisky, chancellor of Austria from 1970 to 1983, was also Jewish by birth, but I don't know if he practiced the religion. According to the www.worldjewishcongress.org/wjcreport/05fall/atw_p23.html World Jewish Congress, five presidents in the Central America-Caribbean region have been Jewish, including two practicing presidents of Panama -- Mwalcoff 04:51, 15 February 2006 (UTC)[reply]

The first Australian-born Governor-General, Sir Isaac Isaacs was Jewish. He wasn't the head of state in my opinion or that of Wikipedia, but a large number of Australians do consider the G-G the head of state. JackofOz 07:46, 15 February 2006 (UTC)[reply]

The Lawyer Chenny shot[edit]

Is it just me or does this guy look like president Bush? Could he possibly be his (President Bushes) biological father?

I don't know the lawyer you're talking about, but the Bush family has been around in the Texas oil biz for a long time. Gee Dub is most certainly Aitch's son.

I suggest you have a close look at —WAvegetarianCONTRIBUTIONSTALKEMAIL14:08, 14 February 2006 (UTC)[reply]

  • I've got no idea who this Chenny is. Perhaps you're referring to Dick Cheney. - Mgm|(talk) 18:54, 14 February 2006 (UTC)[reply]

No, it's Harry Whittington. See Dick Cheney#Hunting accident. StuRat 21:58, 14 February 2006 (UTC)[reply]

"I always said that US Vice President Dick Cheney was evil (Halliburton, the Iraq war, US energy policy, the Valerie Plame incident, etc.). But, at least he used to be subtle about it. Now he's apparently decided to just go out and start shooting people." 21:58, 14 February 2006 (UTC)

Dubya sure looks like his father's (George H. W. Bush) son to me, though I think he does choose slightly nicer ties:

Frankly, if we're going to be throwing nasty aspersions round, the favourite one going round the lefty blogs at the moment is: what were Cheney and Whittington going out shooting with two women who weren't their wives...;) --Robert Merkel 22:52, 14 February 2006 (UTC)[reply]

KFA[edit]

Is their any credible evidence that KFA members have, used conscripted sex workers, in North Korea?

See Korean Friendship Association. Given the tight control on the media in North Korea, it would be difficult to confirm or deny any such allegations. --Halcatalyst 13:23, 14 February 2006 (UTC)[reply]

Blair's politics[edit]

Is Tony Blair right of center, or left of center?

  • It's kind of a matter of opinion: as a Labour Party (UK) politician, he'd usually be considered left of centre. However, as the article Tony Blair attests, he has come under considerable criticism from left-wingers in his own party for being too right-wing, and forsaking the party's traditional priorities. --Canley 13:15, 14 February 2006 (UTC)[reply]
  • He is right of centre. In some aspects of policy he has gone further than Thatcher, especially with regard to introducing private sector involvement in public services. Of course, if the two main parties drift right you could argue that the centre drifts right with them. Have a look at this: www.politicalcompass.org/ Political compass. --bodnotbod 17:05, 14 February 2006 (UTC)[reply]
Left, right and centre are attempts to apply simple labels to complex, ambiguous things like political positions. Worse, their interpretations are entirely subjective, with different people understanding them in different ways. Deciding what label to apply to his policies is very unlikely to be helpful. To take one simple example if Tony were a US politician his policies (public healthcare support, gun control, same-sex civil unions) would put him on the extreme left wing. DJ Clayworth 17:51, 14 February 2006 (UTC)[reply]
  • Not forgetting the fact someone can have extreme left views on one subject and an extreme right one on others. The first three answers here, should give you an idea how useless those labels really are. - Mgm|(talk) 18:57, 14 February 2006 (UTC)[reply]
It's also the case that the "left/right" business varies according to where you are, since "centrist" tends to refer to the accepted middle-ground in whatever country you're in. To use an example - traditionally in US politics the Democrats are to the left and the Republicans are to the right, but the Democrats would be considered well to the right of centre compared with politics where I am in New Zealand. Britain's politics lie somewhere between the US's and NZ's so from a US viewpoint, Blair would be considerably to the left, whereas he's far closer to the centre or over towards the right by British or New Zealand standards. Grutness...wha? 21:56, 14 February 2006 (UTC)[reply]
See Left-Right politics. Frankly, the terms are near-useless in trying to place politicians in a historic or global perspective; they are useful only in a gross approximation of their views within a country's political debate at a particular time. --Robert Merkel 22:43, 14 February 2006 (UTC)[reply]
As an illustration of some difficulties with the left/right classification in an international context, here are my impressions of the political compass bodnotbod linked to. That test seems designed to push me into a left wing corner. I'm left-wing with respect to the questions asked. But there is no nuance on the left wing side. Just like IQ tests come in different forms depending on the intelligence measured, there should be different versions for different countries here. Simply put, the questionnaire states that if you're Dutch, you're a commie. ::Take this one: "Although the electronic age makes official surveillance easier, only wrongdoers need to be worried." I disagree only with the word 'only' and therefore with the whole statement. Had that been 'mostly' I would have agreed. Or this one: "The businessperson and the manufacturer are more important than the writer and the artist." I disagree. But had it been asked in reverse I would ahve disagreed as well. But that sort of question is never asked.
And I found this one interresting: "A significant advantage of a one-party state is that it avoids all the arguments that delay progress in a democratic political system." I agreed with that, but wonder if that is taken as a right-wing or a left-wing thing. It could refer to Lenin or Hitler. Or this one: "Some people are naturally unlucky." I don't see any link with left/right-wing (I disagreed because I don't see a scientific basis for it). Finally, I often missed the option 'haven't a clue'.
Oh, and then there's this one: "Sex outside marriage is usually immoral." As I explained in a previous thread, I disagree because I consider it by definition immoral (but interpreted the intention and filled in 'agree').
And the result? Apparently, I'm a mild left wing libertarian, putting me right next to Gandhi, Nelson Mandela and the Dalai Lama (I can live with that) and opposite George Bush (phew, that's a relief :) ). DirkvdM 09:36, 15 February 2006 (UTC)[reply]
Whee! Interesting test. It put me out beyond Gandhi on both axes...and currectly predicted my Green Party vote at last year's NZ election. Grutness...wha? 04:49, 16 February 2006 (UTC)[reply]

Sex[edit]

Do female wikipedian's between the age of 18 and 28 like to have sex?

Some do, some don't. Why don't you ask in your favorite Wikipedian bar? --Halcatalyst 13:19, 14 February 2006 (UTC)[reply]

Something wrong with 29+? Marskell 13:48, 14 February 2006 (UTC)[reply]
  • I imagine quite a few of those female Wikipedians like to have sex depending on who they're with. Kinsey found out sex is a hard area to research reliably. If you're looking for a sex partner as your earlier questions make it appear, I suggest you check out your local bar/cinema/disco/club etc. - Mgm|(talk) 18:40, 14 February 2006 (UTC)[reply]
  • Meet women at the cinema? Some bugger starts prowling around my local fleapit trying to draw random women into conversation whilst I'm trying to watch the film, they'll end up with a jumbo hot dog wrapped 'round their neck. --bodnotbod 18:51, 14 February 2006 (UTC)[reply]
  • How about break time, or in the foyer before the film? - Mgm|(talk) 18:58, 14 February 2006 (UTC)[reply]
What are these "females" of which you speak? Angmering 22:29, 14 February 2006 (UTC)[reply]
Why are you limiting the ages? User:Zoe|(talk) 22:48, 14 February 2006 (UTC)[reply]

Because I am 24, and do am not a gerophile. Seriously I am a 24 year old law student, is their woman in my age bracket living in the north east who wants to have sex?

I'm sure there is. At least one. User:Zoe|(talk) 03:05, 15 February 2006 (UTC)[reply]
But, there probably aren't any women who read this page who have the least desire to have sex with a random man who posts such a question here. StuRat 04:34, 15 February 2006 (UTC)[reply]

Sex: "The pleasure momentary, the posture ridiculous, and the expense damnable" -- Evelyn Waugh. Personally, I don't have any objection to the posture or the expense. Geogre 18:47, 15 February 2006 (UTC)[reply]

British supporting actors 1950s[edit]

where can I get a list detailing which actors - e.g. Victor Maddern, Sam Kydd - made the MOST films in the 1950s. I have been commissioned to write a book about the top ten but don't know how to find the top ten! help. Dee Gordon

Presumably, since you've been comissioned to write this book, you must have some idea who they are likely to be - say narrowing it down to the top hundred or so. It should be no more than a couple of days work to go through www.imdb.com IMDB and find out which ones were in the most films. DJ Clayworth 17:46, 14 February 2006 (UTC)[reply]
  • It seems a strange commission, in a way. One is tempted to ask "who cares who made the most?" A book might more entertainingly cover those actors who had the most interesting life stories or individual anecdotes. But I'd second the recommendation of wading, rather laboriously probably, through the IMDB which will give you a good at a glance view of likely contenders. Asking on the IMDB forums is liable to get you more knowledgeable answers than here.
You could also try asking on the talk.guardian.co.uk Guardian newspaper talk boards which I recommend because it has an older, British, clientelle than many other forums out there, many of them interested in film and media. Try the Notes and Queries folder there (linked to on the left of that page) --bodnotbod 18:38, 14 February 2006 (UTC)[reply]
  • How did you get commissioned to write a book about which you don't know the basic outline/facts? - Mgm|(talk) 19:00, 14 February 2006 (UTC)[reply]

First Amendment Protections[edit]

Were sit-in's during the Civil Rights Movement in the 1960's protected by the first amendment (freedom of speech)? It's always said that they were, but I thought that the Constitution only protected people against the government - sit-ins occurred in private restaurants who could choose to accomodate whoever they want? What am I missing? Thank you. Mickey 16:55, 14 February 2006 (UTC)[reply]

Assuming you mean in the US, then sit-ins, which are typically designed to cause a disruption, are not protected, whether on private or public land. Then again, being arrested may actually be the goal, for maximum press coverage. A sit-in in a government office might be such an example.StuRat 04:31, 15 February 2006 (UTC)[reply]
Racial discrimination by businesses that serve interstate commerce was prohibited in the Civil Rights Act of 1964. The constitution had nothing to do with it. -- Mwalcoff 04:41, 15 February 2006 (UTC)[reply]
The fact that restaurants can't legally discriminate based on race doesn't make it legal for those who accuse them of violating this law to sit-in and disrupt their restaurant. StuRat 06:59, 15 February 2006 (UTC)[reply]
In the early 1960's the protestors merely sat down in all available tables/seats and waited to be served, which didn't happen. Is that disruptive? Rmhermen 20:04, 15 February 2006 (UTC)[reply]
I suppose that would depend on what they ordered and how long they stayed. If they ordered a cup of coffee then stayed for 10 hours, yes, that would disrupt their ability to make money. StuRat 00:08, 17 February 2006 (UTC)[reply]

Stalins almamater[edit]

What ever happened to the Tiflis Theological Seminary, where stalin studied? Is it still in operation?

The building that housed it is now the Georgian State Art Museum rustaveli.tripod.com/sakartvelo/tbilisi/tbilisi.html. The seminary apparently was closed down at some point. www.patriarchate.ge/biograf/1e.htm This page says that the only seminary open in Georgia in the 1960s was at Mtskheta. The Mtskheta seminary became the current Tbilisi Theological Academy and Seminary www.osgf.ge/tta/ in 1988 www.patriarchate.ge/biograf/1e.htm. --Cam 05:27, 15 February 2006 (UTC)[reply]

Images of Flags of Countries Projected by NBC on the ice at the Olympics[edit]

I've been watching the Speed Skating coverage by the NBC network in the United States. It seems like NBC is somehow projecting the flag of the country the competitor is from on the ice during some speed skating races. How are they doing this? Is it happening within the stadium at the Olympics or is it being done by NBC?

It's a video effect, not something you'd see if you were actually present. www.courant.com/sports/hc-olytvcol0210.artfeb10,0,650815.story?coll=hc-headlines-sports - Nunh-huh 20:53, 14 February 2006 (UTC)[reply]
The same way that ads are projected onto the background behind a catcher during televised baseball games. If you were in the stadium, all you'd see would be a green background, but at home you see the ad. Sometimes, if you watch really closely, you can see that, as the camera switches from some other shot back to the catcher, the ad takes a fraction of a second to be redisplayed. User:Zoe|(talk) 22:50, 14 February 2006 (UTC)[reply]

Law Concurrent Liability[edit]

hi all, can someone give me an idea on concurrent liability...detailed explanation is required....not cases... I would like to relate it with Tort and Law of contract... please help.

Matt

You need to specify what jurisdiction you want to know about. Oh, and your almost certainly going to have to get to grips with the cases - they're the primary sources.Lisiate 21:56, 14 February 2006 (UTC)[reply]


Hi, I wanted to know Concurrent Liability in Tort and Contract law in UK. i would appreciate if you could give me a lot of theoritical explanation rather than cases. Thanks


Hi All, I really need a help on this. Didnt get a response yet. Can someone please help me? Thanks Matt

British Politics[edit]

Can a king or prince or queen or any of the royalty become prime minister?

No. But the nobility can now they are no longer allowed to sit automatically in the House of Lords.
Technically, I'm not sure that members of the royal family are even allowed to vote. As to the House of lords rule, there have been instances (before the modernisation of the UK Upper House) where hereditary peers have renounced their titles (i.e., "become commoners") in order to become MPs - most famously Tony Benn. It also gives a cluse as to the reason for the name of the "House of Commons". Grutness...wha? 00:41, 15 February 2006 (UTC)[reply]

Except for the reigning monarch legally yes. However, it would considered a violation of tradition for a member of the royal family to run for political office, or even vote.

I agree with that. Some members of the Royal Family are peers and entitled to sit in the House of Lords, eg. the Prince of Wales has done so. They would not be able to stand for the House of Commons unless they disclaimed their peerage (whether this would also entail somehow renouncing their royal status is something I'm not competent to say). If they were eligible to stand for office, they could be elected, and could ultimately become PM. Grutness, whether a member of the Royal Family is allowed to vote or not has nothing to do with whether they could, technically, stand for election. Voting is non-compulsory in the UK, so for all I know Tony Blair might never have voted in his life. (Actually, I'm sure he has, but he's not forced to.) JackofOz 07:41, 15 February 2006 (UTC)[reply]
I don't remember saying that it did - although I suspect if someone is legally barred from voting then they would be unable to stand for office. Whether the royals are actually barred or simply don't vote is another matter entirely. Traditionally, though, members of the royal family never even indicate their support of any particular political viewpoint, let alone a particular party, and similarly never launch legal proceedings against anyone - although Prince Charles in particular has bent these unwritten guidelines considerably with his views on environmentalism. Grutness...wha? 10:52, 16 February 2006 (UTC)[reply]
A monarch in a parliamentary democracy that has any dealings with politics (usually in an advisory function) is (or else should be) supposed to be politically neutral, something one cannot be when in parliament. I don't know how this is in practise, but by this reason it should be illegal (by constitution). And of course the same goes for anyone who might somehow become the monarch and that might include a lot of members of the royal family, though that may vary a lot between countries. DirkvdM 10:00, 15 February 2006 (UTC)[reply]
Note also that the idea that the Prime Minister must be a member of the Commons, like so much of British constitutional practice, is only a matter of tradition and not law; and in fact it is a tradition no more than 100 years old, arising after Lord Salisbury's terms as PM. See Prime Minister of the United Kingdom for discussion of how the office and its traditions have gradually evolved. So theoretically I don't think there's anything stopping a monarch from breaking tradition appointing himself or another royal as PM; however, the conditions for that to happen and for the Commons to give him the necessary vote of confidence are hard to imagine. It would certainly be viewed as undemocratic in any normal circumstance. --Anonymous, 10:10 UTC, February 15, 2006.
NB that reform of the House of Lords means that only 92(?) peers (one in ten) now sit in the Lords. They can however now stand for election to the House of Commons instead without having to disclaim their title. Viscount Thurso became the first to take advantage of this change, winning Caithness for the Lib Dems in 2001. Jameswilson 23:53, 15 February 2006 (UTC)[reply]

February 15[edit]

Russia[edit]

Is Russia a democracy or a dictatorship?

Is this a homework question? If so, have you read democracy, dictatorship, Politics of Russia (though that article pulls its punches), and Vladimir Putin? Hint: it's not a simple question, and neither "democracy" nor "dictatorship" are unambiguous or rigidly defined. --Robert Merkel 03:31, 15 February 2006 (UTC)[reply]
Doesn't seem too much like a homework question to me. GeeJo (t) (c)  10:06, 15 February 2006 (UTC)[reply]
  • Are we talking now or historically? - Mgm|(talk) 09:18, 15 February 2006 (UTC)[reply]
The question is in present tense, so it refers to now. In stead of burning my fingers on this specific case let me point out that a dictator can assume power through a democratic process. Hitler is a prime example. And Saddam Hussein also claims to have been democratically elected. DirkvdM 10:03, 15 February 2006 (UTC)[reply]

Soccer Hooligans[edit]

Is it true that many European soccor hooligans have links to radical right, Nazi, and neo fascist groups.

See our article Hooliganism. --Halcatalyst 04:55, 15 February 2006 (UTC)[reply]
Define hooligan. An illustrating anecdote: I've been a football supporter with the 'Angel Side' (MVV) for a few years, and when in Mexico I went to a football game. At the counter I asked where the home supporters were located (usually the most lively part of the stadium). The guy didn't understand, so I explained, after which he indignantly exclaimed "We don't have any hooligans here." Which was quite an understatement. The entire stadium was totally dead, with the response to a goal being a friendly applause. DirkvdM 10:09, 15 February 2006 (UTC)[reply]
I'm sure if you ask politely they will arrange a violent riot so you will feel right at home. StuRat 04:53, 16 February 2006 (UTC)[reply]
Nah, I'm a pacifist hooligan. At least if one is to believe that test I referred to above, which put me in league with Gandhi and the Dalai Lama. DirkvdM 08:26, 16 February 2006 (UTC)[reply]

David Cammeron[edit]

From the U.S. prospective who would be easier to work with, David Cameron or Gordon Brown?

Is this a homework question? Have you read David Cameron and Gordon Brown? Furthermore, you might read Conservative Party (UK) and Labour Party (UK) for some more hints, particularly the mention of Atlanticism in the Tory article. Think also about what you mean by the question - do you mean the US generally or the current White House, which are not one and the same thing (as Australia may well discover in February 2009). --Robert Merkel 03:38, 15 February 2006 (UTC)[reply]

Hereditary peers[edit]

Why did britan give them so much power until very recently? How could a modernist justify giving so much power to someome, just because of what their great grandpa did? Is it true a member was caught selling drugs at Westmister, in the late 90's?

See Conservatism. See also Tourism and the thought of what getting rid of the biggest hereditary office in the land would do to the UK's inbound tourist trade... --Robert Merkel 03:42, 15 February 2006 (UTC)[reply]
It's likely that all countries have constitutional leftovers that seem unfair nowadays. In the U.S., residents of DC have no representation in Congress, California has 62 times more people per senator than Wyoming does and you can be elected president even if someone else gets more votes than you. And Canadian senators are appointed by the governor general on the advice of the prime minister. -- Mwalcoff 04:38, 15 February 2006 (UTC)[reply]
Which is ironic considering the US want to 'bring democracy to the world'. Maybe they should start with getting a proper one themselves. :) Irritatingly, some people here in the Netherlands now say we should also get ourselves a district system. Oh horror. DirkvdM 10:16, 15 February 2006 (UTC)[reply]
Well said. In fact the US is not a democracy at all. I think it is actually a Federated Republic, or something like that. I'm not sure what it is, but I know that it is is not a Democracy. Imagine that Dubya, undemocraticly imposing democracy all over the world, when he doesn't even represent one! (how can I get this to not put a big blue box around whatever I write, but still enable me to indent? ----Leah
I fixed the indent problem.. In the future, write it as a single line of text (don't hit return between lines) with colons in front to control the indent level. As for the US not being a democracy, it is most definitely a form of representative democracy. While it is true that power is not shared equally in any representative democracy, this does not mean it's no longer classified as a democracy. All people in a democracy do have some power, however slight, versus a totalitarian system where they have no power at all. To actually give everyone equal power, direct democracy would be needed in the US, and everywhere else. StuRat 20:17, 18 February 2006 (UTC)[reply]
Oh, ok then. So She is a democracy. I looked up Federal Republic. It appears that the two terms are not directly related. Is there any such critter as a Totalitarian Federated Republic? What would that be? ---Leah
A direct democracy is not needed for everyone to have equal power. A representative democracy in its most representative form does that too, unless one nitpicks. The parliament of the Netherlands is sometimes called the most representative democracy in the world because it uses direct elections without an electoral threshold. So if a party gets the number of votes required for one seat (number of voters divided by number of seats), it actually gets that seat. Which is just under 65.000. Of course, if you vote for a party that doesn't reach that number of voters, you won't be represented. The chances of this happening decrease with the number of seats, until it reaches one, in which case you have a direct democracy. But then that doesn't work on a national scale, so the Dutch system would be the best approximation (if only we'd get rid of the bloody queen and upper house).
Actually, I think world peace would be best served if the US had a directly elected parliament. I'll raise this question in a new thread. DirkvdM 08:08, 19 February 2006 (UTC)[reply]
In any representative democracy, their exist a huge conflict of interest between the electors and the elected. The temptation for a representative to take money from individuals, corporations, and special interest groups, and then do their bidding, rather than representing the interests of their constituents, is enormous. This is equally true of both parliamentary and presidential systems. I am far more concerned about this than any of the minor factors you've mentioned, since this undermines the entire basis for democracy, "a government of the people, by the people, and for the people". StuRat 05:27, 20 February 2006 (UTC)[reply]
True. But the cost of fighting that should be minimal compared to the cost of everyone having a say in every subject, as in a direct democracy (there would be no time left to do any actual work, for one). The main factors seem to me to be a high enough income for politicians to reduce the temptation (this is also the reason for the high salaries of judges, I believe), a big enough punishment if one is found out (ie jail in stead of a slap on the wrist) and as system that makes checking easy (don't ask....). By the way, in "a government of the people, by the people, and for the people" what does "of" stand for? "By" and "for" already say it all, don't they? DirkvdM 10:16, 21 February 2006 (UTC)[reply]
In my interpretation, a representative government is created "by the people", via voting, but is not composed "of the people", at least not ALL of the people. And whether the government is run "for the people", or rather for the benefit of the representatives, is this issue we're discussing here. StuRat 22:35, 21 February 2006 (UTC)[reply]
In the US, we've tried repeatedly to limit the influence of money on government, to no avail. Note that most of this influence is legal. Individuals and corporations are allowed to make campaign contributions to their favorite candidates. Since election campaigns cost millions, the only other alternatives are to have taxes pay for campaigns, which sounds dangerous to me, mandate that broadcasters provide free advertising during elections, which brings up the issue of how to decide which candidates are "valid", or we could only allow the rich the run by forcing them to pay for their own ads. I suppose banning such ads entirely would also even the playing field, but this would require a Constitutional Amendment, since that would violate the Freedom of Speech. Of course, the whole issue of how to finance campaigns for representatives goes away in a direct democracy. StuRat 22:35, 21 February 2006 (UTC)[reply]
And no matter how much you pay representatives, it can't compete with the riches they can get by selling their votes, which are worth billions of dollars to corporations. Also note that highly compensating representatives makes them non-representative of the population. Why would they care about the issues of the poor if they are all rich ? StuRat 22:42, 21 February 2006 (UTC)[reply]
As for the issue of it taking too much time for everyone to vote on everything, I would expect people only to vote on things that interested them. For example, childless adults wouldn't be likely to vote on school curriculum requirements. It might be good to require a quorum, so that laws wouldn't be created with only a few people voting for them. StuRat 22:35, 21 February 2006 (UTC)[reply]
Of course, I doubt if the EU government is elected based strictly on population, either, but rather by "districts" (countries). StuRat 04:50, 16 February 2006 (UTC)[reply]
Ah, but the EU is not a country. I used to be all in favour of giving all power to a parliament with equal representation because that is more democratic. But if the constitution referendum did one thing for me it was make me think about what the EU really should be. And I'm now convinced it shouldn't be a country, little more than a collection of (trade) agreements (which may spread as far as it wishes - Turkey - Iraq - Russia - India - Brazil - wherever). That's why I voted against the constitution - there should be no constitution at all. DirkvdM 08:34, 16 February 2006 (UTC)[reply]
Britain didn't give peers their power, the peers took their power from the king, starting with the Magna Carta. "Modernists" didn't "give so much power" to them, modernists took it away. - Nunh-huh 13:07, 15 February 2006 (UTC)[reply]
There are a number of good reasons for having a non-elected revising legislative chamber, briefly summarrised they are that
  1. It allows the ability to appoint people of real merit/knowledge on particular subjects.
Who is to decide that? DirkvdM 08:43, 19 February 2006 (UTC)[reply]
  1. It allows experienced politicians to continue to play an active (& beneficial) part in public life, even (or especially) in areas where they have particular interest/experience.
  2. There is an in-built conservative (with a small c) majority which ensures that no temporarily elected party can do away with long held rights & liberties (though this has broken down since the 'reform' of HoL & the large number of Labour peers now in place).
  3. Generally it ensures that well educated people have the opportunity to debate the issues, & the fact that they didn't need to sit in the HoL except when they wanted to usually meant that only those who actually were well educated & concerned about the issues did speak or vote.
Who is to decide that? DirkvdM 08:43, 19 February 2006 (UTC)[reply]
  1. It allows the government of the day to reward old politicians & make room for new ones in the HoC (more so nowadays since life peerages became common).
  2. It prevents both houses of the legislature being controlled by the same party, in the sense that members of the HoL, while some nominally belonged to a party, tend to be far more independent than members of the HoC as they're not afraid of deselection from their seats/the party machine not booking them paying speaking engagements/not getting appointed to Cabinet or other career benefit.
  3. Members of the HoL don't need to worry about getting voted out so they could discuss politically unpopular issues which will inevitably offend/upset some people/vested interests in a way which elected political parties & members are reluctant to do.
Ah, you've actually got a good argument here. But it's pointless if they have no power and undemocratic if they do. DirkvdM 08:43, 19 February 2006 (UTC)[reply]
On the guy caught dealing drugs I believe it was the son of a peer & frankly thats not uncommon as Jack Straw's son was also caught for dealing drugs, as have a number of other people. Even if it was a peer it wouldn't matter as you can't use this as an argument against a whole group. AllanHainey 12:13, 16 February 2006 (UTC)[reply]

Augustine of Hippo...Just War[edit]

Please help.I have been hunting for the specific text(s) in which Augustine puts forward the idea of 'just war.'Could someone help me to locate it/them? Thanks. Paul Delaney

www.ccel.org/a/aquinas/summa/SS/SS040.html This segment on war from the Summa Theologica of Thomas Aquinas cites Augustine by chapter and verse. --Halcatalyst 04:53, 15 February 2006 (UTC)[reply]

Minoan food and clothing[edit]

What did the ancient Minoans eat? I can't seem to find it anywhere in wikipedia... gelo 05:55, 15 February 2006 (UTC)[reply]

Minoan civilization#Agriculture has the information you want. To that I would add that, being island-dwellers and noted seafarers, they probably ate a fair amount of seafood. What about clothing? —Charles P._(Mirv) 06:23, 15 February 2006 (UTC) 06:17, 15 February 2006 (UTC)[reply]
Based on the images I've seen, they didn't wear much.  :) User:Zoe|(talk) 19:27, 15 February 2006 (UTC)[reply]

philosophy of man>phenomenology of love[edit]

man is a social being and therefore he is capable of loving and being loved. They say that the experience of loving starts in the experience of loneliness, how did it happen? Explain.

Unfortunately, we can't do your homework for you. - Akamad 06:56, 15 February 2006 (UTC)[reply]
Have you read our article on Love? GeeJo (t) (c)  10:01, 15 February 2006 (UTC)[reply]

The greeks(plato and aristotle), the scholastics, and the mercantilists each built their analysis around a specific economic objective(or goal) for society. Describe these objectives[edit]

When is your homework due? Notinasnaid 08:15, 15 February 2006 (UTC)[reply]
  • These objectives are probably in the course material you had to read/watch before answering this question. - Mgm|(talk) 09:19, 15 February 2006 (UTC)[reply]

You seem to have forgotten at least one Greek philosopher, Socrates.StuRat 21:28, 15 February 2006 (UTC)[reply]

Damn fine soccer player, too... Grutness...wha? 10:54, 16 February 2006 (UTC)[reply]

And he is particularly missed:

Immanuel Kant was a real pissant
Who was very rarely stable,
Heidegger, Heidegger was a boozy beggar
Who could think you under the table,
David Hume could out-consume
Wilhelm Friedrich Hegel.
And Wittgenstein was a beery swine
Who was just as schloshed as Schlegel.
There's nothing Nietzsche couldn't teach ya
'Bout the raising of the wrist,
Socrates, himself, was permanently pissed.
John Stuart Mill, of his own free will,
On half a pint of shandy was particularly ill,
Plato, they say, could stick it away,
Half a crate of whisky every day.
Aristotle, Aristotle was a bugger for the bottle,
Hobbes was fond of his dram,
And Rene Descartes was a drunken fart,
"I drink, therefore I am."
Yes, Socrates, himself, is particularly missed.
A lovely little thinker,
But a bugger when he's pissed.

JackofOz 12:03, 16 February 2006 (UTC)[reply]

17 century painting styles[edit]

Please advise me on how to find the most popular painting styles during the 17 century. Thank you. Rachel

The rather poor article History of painting suggests Baroque which is a better article. This only discusses Europe, scattered articles may give wider details e.g Japanese_art#Art_of_the_Edo_period. Unfortunately fine art is rather a black hole here on the wiki so if you learn anything more it would be great if you can come back and contribute it. MeltBanana 16:26, 15 February 2006 (UTC)[reply]

"I had a beautiful Rennaisance vase, but then I dropped it, and now it's Baroque." StuRat 21:23, 15 February 2006 (UTC)[reply]

Hume[edit]

Hume said we can't be certain of any kind of knowledge besides mathematical knowledge...but can we be 100% certain even of that? if yes, what explanation did he give?.--Cosmic girl 15:25, 15 February 2006 (UTC)[reply]

When mathematicians talk of a "proof" they do mean something absolute, and this is something not available in other forms of study (unless it follows the same principles, see below). But what do mathematicians really mean? Mathematics is built on axioms. These are things we have to take for granted, things we assume. A "proof" is actually a proof using axioms as the original assumptions, and using axioms in the steps of the proof, and nothing else added. You can therefore say that if the axioms are valid, then the proofs are valid. Most maths is built on axioms that make sense in the real world such as x = x, but mathematicians can have funs by using illogical, made up, or plain wrong axioms. The proofs using these axioms are still proofs. And you can prove other things outside maths, if you start with a different set of axioms. Some theologians have done this, starting with truths that they hold to be self evident. I guess you could use axiomatic proofs in physics too: if s = u + a . t is an axiom, then we can probably prove things about distance travelled as related to time passed. Doesn't mean that what is proved is right. Does that make sense? Notinasnaid 16:46, 15 February 2006 (UTC)[reply]
As a quick interjection, non-numerical proofs are possible as well and (I think...) Hume would accord them the same designation as "knowledge" as he would anything else, so long as you do not attempt to induct on the basis of the proof. Sure, "this is true" insofar as the top of the page is consistent with the bottom--and beyond the page nothing can be said. He wouldn't accord any veracity to an applied proof (numerical or otherwise) as justifying a future certainty. Math says: "Venus' orbit will do X." Hume says: "I don't know that." He's not declaiming math as untrue; he's refusing to draw an inference where the proof says he should. Marskell 22:20, 15 February 2006 (UTC)[reply]
From another perspective, this is explained in critical thinking as the difference between a fact and a truth. It is a factual statement to say that there is intelligent life on Mars. It is factual because it can be proven to be true or false. We know it to be false because we have explored enough of Mars to know that there is no intelligent life there. Now, claiming that there is intelligent life on some other planet is also a factual statement. We don't know if it is true or false, but it could be proven by inspecting every single planet. Math works with factual statements, as explained above. So does everything else. The Declaration of Independence is based on the factual statement that all people are born with inalienable rights. It assumes this to be true because it is self evident. That is not a proof that it is true. It is assumed to be true and then all factual statements based on it are assumed to be true as well. Should you disagree about the self evidence of inalienable rights, the rest of the Declaration of Independence is a flawed proof. --Kainaw (talk) 18:16, 15 February 2006 (UTC)[reply]

ok I see. I knew that math is based on axioms, but they are axioms acording to US humans...let's imagine that 'God' exists, maybe he can make irrational things (for us) happen and be 'true metaphysically'...so, what I'm saying here is that we would be really rational to accept even what we take to be axioms have a posibility, however slight, of being wrong 'metaphysically', thus, not 'absolutely true', am I making a point? or is there an argument that inmunizes math to the irrationality of metaphysics and so called 'ultimate truth'. oh and also, that comment notinasnaid said about math making proofs out of irrational and untrue axioms for 'fun'..is it just for fun? or has anything useful ever been developed (like technology or an important theory) using 'false axioms'? my guess is it hasnt but...what do I know?. :P --Cosmic girl 22:26, 15 February 2006 (UTC)[reply]

First, the easy answer... There are areas in logic where an expected false fact is used as a true axiom in a sort of reverse proof.
Now, the harder answer... When we, as humans, state something as 'true', we imply that it is true within the confines of time and space as we know it. It is a well accepted truth that all laws of math and physics as we know it cease to exist at the big bang. Most feel that they cease to exist in the middle of a black hole. So, it is perfectly reasonable to claim that beyond our existence, there is one in which our math is completely wrong. But, who cares? We use math and physics to understand the universe that we live in - not one that we have no connection or comprehension of. --Kainaw (talk) 23:54, 15 February 2006 (UTC)[reply]

Totally agree, who cares? science is a great tool (I repeat,tool), and I also 'know' that there is the possibility that our math isn't absolute for 'everything' even things 'outside' our existence...and now you said it too, so I'm not the only one...I thought I was crazy for reasoning this way actually, but now that you say that our math and physics break down soon after the BB...I know maybe I'm not that crazy.--Cosmic girl 02:53, 16 February 2006 (UTC)[reply]

The meaning of "Cory" as in Chilean or Polish Cory[edit]

This is in political science text: "The Third Wave, Democratization in the Late Twenthieth Century," Samuel P. Huntington, U of Oklahoma Press. It reads: "There is no Chilean Cory." "Lech Walesa was a Polish Cory." p. 158.

Thanks Janet

Maybe Corazon Aquino AnonMoos 19:50, 15 February 2006 (UTC)[reply]

Mona Lisa[edit]

Why were panels at both sides of the Mona Lisa removed?

A mystery. The sides of the panel were cut not long after Leonardo's death, but that's about all that is known. David Sneek 23:11, 15 February 2006 (UTC)[reply]

How is it known that the panels were cut soon after his death? ---Leah


early christian letters[edit]

Can you tell me the name of the early christian letters that are read at mass?--70.251.111.227 19:25, 15 February 2006 (UTC)[reply]

Do you mean epistles? AnonMoos 19:48, 15 February 2006 (UTC)[reply]

How many car accidents happen a day in the USA?[edit]

Question in title only

by a remarkable coincidence, this question was just answered over on the Miscellaneous desk. GeeJo (t) (c)  19:38, 15 February 2006 (UTC)[reply]
Also Here. Have they put this as a sample question somewhere or something? GeeJo (t) (c)  21:01, 15 February 2006 (UTC)[reply]

Humanities (I think) People with same birth and death dates[edit]

I can't seem to find an article that lists people who have the same birth and death dates...surely there must be some prominent people with this unusual occurrance? Is there already an article and I'm just not finding it? If there isn't an article on this already, can anyone suggest what might be the easiest way to find out who had the same birth and death dates? Thanks in advance. bcatt 19:35, 15 February 2006 (UTC)[reply]

Try List of people who died on their birthdays. By the way, it's not all that uncommon. It happens to one in every 365 people. GeeJo (t) (c)  19:38, 15 February 2006 (UTC)[reply]
I admit that for somebody with a degree in math, I am horrible at probability and statistics; so kindly indulge me and explain how it can happen to 1 in every 365 people. I can sort of understand that any individual person has a 1 in 365 chance of dying on his birthday (excepting those born on 29 Feb, but let's ignore them for now). --LarryMac 20:19, 15 February 2006 (UTC)[reply]
I don't understand your question. Since any individual person has a 1 in 365 chance of dying on his birthday, then 1 in 365 people will do so. Markyour words 20:37, 15 February 2006 (UTC)[reply]
Hey, I said I was horrible at this stuff. There is some kind of brick wall in my head that prevents me from seeing those as the same thing. I always preferred the easy stuff, like differential equations, although most everything is lost in the haze of 20 some years of disuse :-) --LarryMac 20:57, 15 February 2006 (UTC)[reply]
Maybe think of it this way: 1 in 365 people will die on 21st June, Christmas Day, or any other particular day of which there is one every year. That 'particular day' can be defined in any way you want, as long as there is only one each year. Since each person has one birthday per year, 1 in 365 people will also die on his birthday. Markyour words 21:28, 15 February 2006 (UTC)[reply]
The number of people who die on their birthday might even be higher than 1/365, if you consider surprise parties given to elderly people with heart conditions. :-) StuRat 21:31, 15 February 2006 (UTC)[reply]
I appreciate the help, and I will try to get my mind to suitly emphazi the explanation. --LarryMac 01:54, 16 February 2006 (UTC)[reply]

Hehe...thanks for the point in the right direction...and for the laughs. bcatt 23:18, 15 February 2006 (UTC)[reply]

Actually I've seen it said that a person is statistically more likely to die in the months just after their birthday than those just before. This is claimed to be for psychological reasons: wanting to reach one more birthday promotes the will to live. I don't know if it's an accepted theory today, though, or how it would relate to deaths actually on one's birthday. --Anonymous, 00:30 UTC, February 16, 2006, not my birthday.
There is some evidence for people dying slightly faster after major events - Christmas and New Year are the big ones, but then early January is also the time that winter really begins to bite - but I'm not sure if a study has linked it to birthdays or not, and "the months" is very vague... that's covering half the year! Someone actually won an IgNobel for related work, come to think of it...
2001 Economics prize: Presented to Joel Slemrod, of the University of Michigan Business School, and Wojciech Kopczuk, of the University of British Columbia, for their conclusion that people find a way to postpone their deaths if that would qualify them for a lower rate on the inheritance tax.
That classic work of accountancy is here www.columbia.edu/~wk2110/bin/dying-final.pdf PDF - Page 5 summarises related research, including a one-third (!) drop in Chinese mortality the week before the Harvest Moon festival and an equal peak the week after, and a claim (though unreliable) the same effect happens for birthdays. You might want to chase up the cited paper there.
Bear in mind that, especially with birthdays and major religious events, the actual strain of the event may help induce the death itself; they cite post-Passover deaths as being greater if the holiday was over a weekend, where the celebrations tend to be more highly observed. Shimgray | talk | 17:41, 17 February 2006 (UTC)[reply]

Diaspora?[edit]

Could the peopling of Oceania/Polynesia be called a diaspora? 64.198.112.210 21:07, 15 February 2006 (UTC)[reply]

That is more of a scattered settling. They did ultimately split up and migrated into new territories on their own. I thought of diaspora as it pertains to the Jewish scattering across Europe as teh result of persecution. When the American west was settled, it wasn't called a diaspora; they settled and migrated on their accord.--192.160.130.12 21:28, 15 February 2006 (UTC)[reply]
The defining characteristic of a diaspora is not persecution, it's spreading out from a specific place. The settling of America included several diasporas, such as the Irish diaspora, but since the settlers came from many different places, it was not itself a diaspora. The peopling of Polynesia is not really a diaspora because we don't know where they started out from. In a trivial sense, of course, they must have started from somewhere in Africa, which leads some people to include them in the African diaspora. But that's just silly. Markyour words 23:24, 15 February 2006 (UTC)[reply]
Could it be? It is! It's usually referred to as the Pacific Diaspora. Grutness...wha? 00:31, 16 February 2006 (UTC)[reply]
Most of the Google hits for that seem to refer to Pacific islanders moving away from the islands, to NZ or the US. Which would be in line with the usual terminology of naming the diaspora after the place left, not the place gone to. Markyour words 00:55, 16 February 2006 (UTC)[reply]
Let me re-word my reply: The Pacific Diaspora is the name used by anthropologists studying the spread of the Polynesian, Melanesian, and Micronesian races across the Pacific from about 100 BC to 1200 AD. Or at least it is here in the Pacific. Grutness...wha? 05:02, 16 February 2006 (UTC)[reply]

I think it would be helpful to first think about the concept of diaspora. In any kind of social reality, applying any kind of category has limits and these limits are often a result of the categories themselves. The concept of diaspora, if understood to mean the dispersal of a group of people from their original homeland, assumes that human beings were somehow some time back in history immobile and rooted. Within anthropology, that is a conceptual remnant that doesn't really apply any more. Human beings have always been on the move. Not that they've been nomadic, but people have been fluid in their movements over time. The second assumption is about "original homeland". If there was an original homeland, where was it? In applying social categories such as diaspora, it's better to see what the people themselves are calling it. Do the people of Oceania/Polynesia call themselves diasporic? (Conalho 22:11, 16 February 2006 (UTC)conalho)[reply]

I can't speak for all Pacific Island people, but here in New Zealand, the Maori still regard the legendary Hawaiki as their spiritual home, and the most important event in Maori (pre-)history is the arrival of the great fleet of canoes in New Zealand from Hawaiki. Grutness...wha? 01:09, 17 February 2006 (UTC)[reply]

Who said it?[edit]

Questioning authority is like a gangreous toe about to kill the whole body.

I can't find this through google, help would be appreciated.

Do you mean "cutting off a gangrenous toe" ? StuRat 00:40, 16 February 2006 (UTC)[reply]
A google search for -questioning authority gangrenous toe- turned up this www.nancho.net/kipower/kisoma.html along with a bunch of medical sites. Bartleby.com doesn't have it. Black Carrot 20:01, 18 February 2006 (UTC)[reply]
  • "Question Authority" is a bumper sticker from the 1960s. The quote above sounds to me like a bad joke, maybe an original joke by the poser of the question. I say bad joke because... what is it supposed to mean, anyhow?!? --Halcatalyst 19:30, 22 February 2006 (UTC)[reply]

Mentalism[edit]

I need to learn some basic techniques of mentalism before monday (It's a private issue...meant for nothing harmful though) is there any website I can get any tips from? please, I really need this for monday.I've searched in google but haven't found any useful site, or if anyone wants to give me some advide on this, please post it on my talk page,I'd be so thankful.--Cosmic girl 22:16, 15 February 2006 (UTC)[reply]

Try doing a search on cold reading. StuRat 00:21, 16 February 2006 (UTC)[reply]

thanx :)...but I'm trying more to actually influence someone to my favour than to guess his life... because...well it's a long story, but I'm the victim and I need to gain this key person to 'my cause' or else I'm doomed.--Cosmic girl 04:20, 16 February 2006 (UTC)[reply]

We need the specifics to help. Who are you trying to influence to do what ? StuRat 04:30, 16 February 2006 (UTC)[reply]
Short answer - as far as science has been able to tell, it doesn't work. Whatever it is you want from this person, you'd be better off learning a bit more about "conventional" persuasion techniques than pinning your hopes on the supernatural. That said, if you have a religious bent, some religious friends claim that prayer helps them find answers to difficult problems, though not necessarily in the way that they anticipate. --Robert Merkel 05:50, 16 February 2006 (UTC)[reply]
  • Mentalism is not supernatural perse. It's also the name for the use of psychology in magic type mind reading performances. I would recommend reading the section on Derren Brown on the website of the British Channel 4 for some useful techniques. - Mgm|(talk) 08:44, 16 February 2006 (UTC)[reply]

Mentalism isn't supposed to be suprenatural... it's just psychological persuasion, that's all... and I want to have an influence because my parents want me to go to psychological therapy for not being a catholic... it's not only something stupid but the psychologist is gonna agree with them since they are the ones that pay and I don't live in a 'free country' like america... here, the family has it right always, no matter how twisted are the things they ask for.--Cosmic girl 17:51, 16 February 2006 (UTC)[reply]

If you're just looking to learn about psychological persuasion, there's plenty of books that cover that. Since you don't have much time, though, the Wikipedia articles on How to Win Friends and Influence People and Influence Science and Practice might be good places to start. I'd dispute your claim that mentalism "isn't supposed to be supernatural" and is "just psychological persuasion," on the grounds that psychologists themselves study influence and persuasion all the time--and they don't call it mentalism. Chuck 23:03, 16 February 2006 (UTC)[reply]
Well yes, but psychologists don't do it for the purposes of entertainment. If they did, then they'd be mentalists. GeeJo (t) (c)  13:57, 17 February 2006 (UTC)[reply]

Thank you! :) ...well maybe it's just another name for the same thing, then, because 'mentalists' say they have no paranormal abilities...they say it themselves, guys like derren brown, or banachek.--Cosmic girl 03:10, 17 February 2006 (UTC)[reply]

LOL, I thought you meant Banacek, the 1970's TV mystery series: [www.imdb.com/title/tt0068044/] StuRat 16:21, 17 February 2006 (UTC)[reply]

haha :D --Cosmic girl 17:07, 17 February 2006 (UTC)[reply]

One guerilla technique that could help is - say to the shrinker (psy man) that you had a supernatural dream and that you are deeply catholic, but the angel in the vision told you not to go to church but only to pray in the silence of your heart. Some did try it and found it useful. --DLL 19:20, 17 February 2006 (UTC)[reply]

=P haha... Ok, I will but I'll avoid the dream part since I don't wanna be locked up.--Cosmic girl 21:34, 17 February 2006 (UTC)[reply]

Lift Out Quotes[edit]

Could you please tell me what a lift out quote is? It relates to the packaging of news in media. Thanks

I think the idea is that it's a line that can be quoted and make sense alone, hopefully without distorting the original meaning. Take the following paragraph, for example:
Many people spend their lives trying to find the
secret of happiness.  I believe the secret of
happiness lies in setting achievable goals,
working hard to realize those goals - then, once
they have been achieved, defining a new set of
attainable goals.  Unhappiness comes from failing
to set any goals, setting impossible goals,
setting overly simple goals, or failing to define
new goals once the first set has been attained.
The "lift out quote" here, is the only sentence which can stand alone to summarize the paragraph:
"I believe the secret of
 happiness lies in setting achievable goals,
 working hard to realize those goals - then, once
 they have been achieved, defining a new set of
 attainable goals." StuRat 00:10, 16 February 2006 (UTC)[reply]

Recent bills in congress[edit]

Can anyone please tell me a few bills that have been proposed by congress? More specifically, bills that have been proposed recently by the house or the senate.

Assuming you mean the US Congress, that's not any more specific, since the Senate and the House of Representatives, together, are the exact same thing as Congress. For recent bills, how about the proposed extensions of the Homeland Security Act ? StuRat 00:17, 16 February 2006 (UTC)[reply]
Go to thomas.loc.gov. The House bill to be introduced was H.R. 4753, so if you search for, say, H.R. 4650, H.R. 4651, etc., you'll see what members of the House have been proposing. For the Senate, they're now on S. 2286, so you can search for S. 2200, S. 2201, etc. -- Mwalcoff 01:13, 16 February 2006 (UTC)[reply]

February 16[edit]

Early Scopes For Weapons?![edit]

Where can I get information on the first and early scopes for guns!?

Early guns weren't accurate enough to benefit from a telescopic sight, so, if anything, probably only had a simple sight such as a vertical bar above the end of the barrel. StuRat 00:28, 16 February 2006 (UTC)[reply]
Scopes were not required until rifles were accurate beyond at least 500 yards (the minimum distance required for Marines with plain iron sights). That was just before WWI. So, it is a 20th century invention. Previously, scopes were literally telescopes or binoculars attached by the user himself. For snipers, scopes are not well-liked. Yes, they can help increase the distance you can aim, but they attract a lot of attention to the sniper. Night scopes are a completely different entity. They aren't used specifically for distance. They provide ability to see targets with very little light. --Kainaw (talk) 02:22, 16 February 2006 (UTC)[reply]
You'd neeed more support for the statement that snipers don't like rifles with scopes. Military snipers generally are attached to a regular unit and fire from considerable distances, not solo from trees or the tops of buildings, as the movies might picture it. At least, so I gathered from Anthony Swofford's Jarhead : A Marine's Chronicle of the Gulf War and Other Battles (ISBN 0743287215). --Halcatalyst 04:51, 16 February 2006 (UTC)[reply]
Scopes existed long before WWI. Some of the earliest scopes were long thin tubes aligned with the barrel. See www.davide-pedersoli.com/accessoriDettaglio.aspx?CategoriaId=1317&lang=en for U.S. civil war era reproductions. Telescopic scopes date back at least to the 1880's. Rmhermen 06:34, 16 February 2006 (UTC)[reply]
Sorry for not being clear. I meant that the "rifle with a scope attached from the factory" did not become popular until shortly before WWI. Before that, the user took a scope and put it on his own rifle. As for snipers, there are two problems with scopes. They reflect light and they are awkward to use from a prone position. Keep in mind that a sniper does not want to be detected in any way and you can see why snipers are wary of using a device that makes them more detectable and is not necessary. They will use scopes, GPS devices, laser tracking, and all that stuff if required, but not if it isn't required. --Kainaw (talk) 14:25, 16 February 2006 (UTC)[reply]
Sniper scopes are absolutely necessary. In keeping with the "one shot, one kill" motto, a scope and a range-finder [preferrably carried by a 'spotter'] are essential to the job. As for light reflection, a simple shade mounted on the front lense can offset the problem. The benefits far outweigh the risks, in my opinion. --67.142.130.19 22:13, 22 February 2006 (UTC)[reply]

Actors/actresses who were never nominated for an Oscar[edit]

I'm wanting to compile a list of film actors/actresses who were never nominated for an Academy Award despite being considered serious contenders. Any suggestions for (a) names and (b) particular performances? JackofOz 01:11, 16 February 2006 (UTC)[reply]

  • Being considered serious contenders by who? - Mgm|(talk) 08:42, 16 February 2006 (UTC)[reply]
Nitpickers are excused from answering. Anybody else? JackofOz 08:48, 16 February 2006 (UTC)[reply]
  • That doesn't sound like nitpicking. Behind every nominee stand tens of thousands of actors who were never nominated. To be encyclopedic, exactly what criteria would the article define and enforce for "being considered a serious contender"? Without a rigorous definition, the article isn't likely to survive. Notinasnaid 08:52, 16 February 2006 (UTC)[reply]
    • It wasn't intended for a Wikipedia article (although now that you mention it, that's not a bad idea). The 'serious contenders' was my shorthand way of eliminating most of the tens of thousands, and I was really hoping not to have to get into an exhaustive definition of my terms before I even got to the first name. (No such luck apparently). Most of the tens of thousands would be in nobody's list of potential Oscar nominees, so they're not even considered to begin with. A lot of others would be names that are well-known to the movie-going public, but they never managed an "Oscar-standard" performance (or maybe were never given a sufficiently meaty role). That leaves a fairly small number of people who put in at least one "Oscar-standard" screen performance, but for whatever reason never had an Oscar nomination. Don't ask me to define "Oscar-standard", it's inherently subjective. I don't want discussion, I want names. JackofOz 09:19, 16 February 2006 (UTC)[reply]
Would your list include actors/actresses who did or do not usually appear in English language films? If they are accepted, I nominate Isabelle Huppert and Monica Vitti. David Sneek 09:20, 16 February 2006 (UTC)[reply]
Yes indeed, and thank you. Actors in any film eligible for Oscar consideration would be acceptable.
Joseph Cotten is another. If his roles in Citizen Kane, The Third Man, Portrait of Jennie and Shadow of a Doubt weren't good enough for the Academy, what would be! JackofOz 11:11, 16 February 2006 (UTC)[reply]
And Cary Grant! But what about actors whose careers were in their latter years when the academy awards were started up? Charlie Chaplin, for example? And do you simply mean for best actor/actress, or are you including supporting roles (several top names, such as Sean Connery, have only won for supporting roles)? A few names that spring to mind include Samuel L. Jackson, Lauren Bacall, and Sigourney Weaver, to name but three (and Ralph Fiennes has been overlooked yet again this year...). It's also worth noting that Henry Fonda had to wait until his last role for his only acting Oscar. Grutness...wha? 11:21, 16 February 2006 (UTC)[reply]
Cary Grant was nominated twice, for Penny Serenade and None But the Lonely Heart, plus they gave him an honorary award. How about Sterling Hayden - The Killing, Dr. Strangelove, Johnny Guitar, The Asphalt Jungle... David Sneek 11:27, 16 February 2006 (UTC)[reply]
I like Sterling Hayden. He's on the list. That makes 4 now.
A nomination in any acting category means they're off the list. Sorry, Grutness, but none of your names qualify. Henry Fonda did win for On Golden Pond, but he had also been nominated in 1944 for The Grapes of Wrath. Sigourney Weaver was nominated for Gorillas in the Mist and Working Girl - in the same year. Chaplin was nominated for Best Actor for The Great Dictator. Samuel L Jackson was nominated for Pulp Fiction. Ralph Fiennes was nominated for Schindler's List. JackofOz 11:42, 16 February 2006 (UTC)[reply]
ah sorry - misread your question as relating to winners, not nominees.In that case, I can still suggest Peter Lorre, though. And Richard Attenborough's only Oscar was for directing. Grutness...wha? 01:11, 17 February 2006 (UTC)[reply]
Lon Chaney, Sr.; His incredible performance in The Unknown would have been eligible in the first year of the Academy Awards. David Sneek 11:53, 16 February 2006 (UTC)[reply]
Jeanne Moreau. David Sneek 11:56, 16 February 2006 (UTC)[reply]
Yep, and Lon Chaney Jr would qualify (Man of a Thousand Faces). Most of Jeanne Moreau's movies were French-language films made when only English-language films were accepted for acting awards. The rules are different these days. She's pencilled in. Thanks for these great names. I want more. JackofOz 12:30, 16 February 2006 (UTC)[reply]
It's difficult, because "Awards & Nominations" loads very slowly at the IMDb right now. But let's not forget Marilyn Monroe (I'm not really a fan, but she deserved at least a nomination for The Misfits). David Sneek 12:58, 16 February 2006 (UTC)[reply]
John Barrymore. When did they first allow foreign language films for acting awards, btw? I'm afraid I might have to withdraw Monica Vitti's nomination. David Sneek 13:17, 16 February 2006 (UTC)[reply]
Monroe and Barrymore, great. It wasn't that long ago, perhaps 20 years tops. I'll have to check it out. Two more: Vincent Price and Errol Flynn. JackofOz 13:30, 16 February 2006 (UTC)[reply]
The Oscars have allowed foreign language films to get acting nominations at least since 1961 when Sophia Loren won for Two Women in Italian; I don't have any evidence that foreign language performances were prohibited from nominations before then, either. See awardsdatabase.oscars.org/ampas_awards/help/statistics/actingForLang.html. --Metropolitan90 02:59, 17 February 2006 (UTC)[reply]

Here are some more. I got these from www.filmsite.org/noawards3.html here. Some of these are still acting so they may yet manage a nomination: Dana Andrews, Lucille Ball, Jacqueline Bisset, Dirk Bogarde, Noel Coward, Douglas Fairbanks, Sr, Douglas Fairbanks, Jr, Mia Farrow, W.C. Fields, Glenn Ford, Jean Harlow, Rita Hayworth, Alan Ladd, Myrna Loy, Ida Lupino, Joel McCrea, Roddy MacDowall, Malcolm McDowell, Fred MacMurray, Maureen O'Hara, Tyrone Power, Edward G Robinson, Meg Ryan, Donald Sutherland, Robert Taylor, Robert Young. JackofOz 10:09, 17 February 2006 (UTC)[reply]

You forgot to suitly emphazi them. Myrna Loy got an honorary Oscar, by the way; I looked her up yesterday. David Sneek 10:34, 17 February 2006 (UTC)[reply]
Yes she did, and so did Edward G Robinson and Noel Coward, but they were never nominated in a competitive acting category. Honorary awards are by definition non-competitive. JackofOz 01:11, 18 February 2006 (UTC)[reply]

If I founded a new city named "Hoeryong" in Pennsylvania or North Carolina, how would people react?[edit]

Every so often, new cities are built. Therefore, if I were to build a city in Pennsylvania or North Carolina and name it "Hoeryong", how would people react? What would they say about my naming decision? --Shultz 05:10, 16 February 2006 (UTC)[reply]

Depends on how good their Dutch is, I suppose. Or wasn't I supposed to give this away? DirkvdM 08:47, 16 February 2006 (UTC)[reply]
The wikilink will give a hint. It's not Dutch. --Shultz 13:42, 16 February 2006 (UTC)[reply]
He means the wikilink of Hoeryong, that he added above. It appears that you didn't notice. --129.130.117.8 16:41, 16 February 2006 (UTC) The preceding unsigned comment was added by Shultz. Markyour words 16:55, 16 February 2006 (UTC)[reply]

Well, if the Canadians managed to get away with naming a town Dildo, how bad can your suggestion be ? www.fallingrain.com/world/CA/5/Dildo.html StuRat 21:35, 16 February 2006 (UTC)[reply]

Canadians didn't. Dildo, Newfoundland, was founded long before 1949, and therefore was named by Newfoundlanders. For fun, see also... huh, it's been moved pending possible deletion. Silly idea. Anyway, for fun, see also Wikipedia:List of interesting or unusual place names. --Anonymous, 23:13 UTC, February 16.
Most of them would probably not care, even if their politics are different. There is a medium sized city in OH called Medina (pronounced with emphasis on the long 'i') which is named after the city that pilgrims go to in Saudi Arabia. Most haven't made the connection, and the ones who have don't care enough to have an opinion one way or another on the matter. -LambaJan 04:29, 17 February 2006 (UTC)[reply]
Mecca is the city Muslims visit during that Hajj, but I suppose some might stop by Medina, too. StuRat 06:09, 17 February 2006 (UTC)[reply]
Verily. "Like Mecca, the city of Medina only permits Muslims to enter. Both cities' numerous mosques are the destination for large numbers of Muslims on their annual pilgrimage." JackofOz 07:15, 17 February 2006 (UTC)[reply]
Didn't someone point out in a thread a while ago that there is a Chinese town with a Chinatwon (wasn't that Hong Kong?). In that case Mecca might just have a Medina quarter too. DirkvdM 09:01, 17 February 2006 (UTC)[reply]
Didn't know Labradors had dildos. Apparently they had to find new land for that. DirkvdM 09:01, 17 February 2006 (UTC)[reply]

In an attempt to get something serious out of this. In my first comment I was referring to the Dutch word 'hoerejong', which means 'son of a whore'. I thought that maybe Pennsylvania and North Carolina had a large Dutch community. I now wonder if the reason for the question is they have a large Korean community. DirkvdM 09:01, 17 February 2006 (UTC)[reply]

Pennsylvania does have a large Dutch community. StuRat 16:06, 17 February 2006 (UTC)[reply]
Michigan and New York have large Dutch communities. Pennslyvania and North Carolina have large German communities. Rmhermen 17:36, 17 February 2006 (UTC)[reply]
I see. Apparently Pennsylvania Dutch really means Pennsylvania Deitsch, AKA German. My bad.StuRat 05:35, 18 February 2006 (UTC)[reply]

What is George W. Bush's IQ?[edit]

What it said.

Real intelligence quotient, or faulty intelligence quotient? Grutness...wha? 11:25, 16 February 2006 (UTC)[reply]
A closely guarded secret? AllanHainey 12:32, 16 February 2006 (UTC)[reply]
Not applicable :) deeptrivia (talk) 12:40, 16 February 2006 (UTC)[reply]
Well, one way of making a guess is to assume that his SAT scores correlate with his IQ (not an uncontroversial assumption, but we'll make it for the purpose of argument). According to this article, he was in the 88th percentile on verbal scores, and 86th percentile on mathematics. So, very roughly, if we assume he has an IQ greater than 87% of people, with a mean of 100 and a standard deviation of 15 that works out to an IQ of about 117 (assuming I've done my calculations correctly).
What conclusions should be drawn from this information are of course open to the reader; even assuming that rough guesstimate is accurate, IQ is by no means the only thing that determines one's suitability for the presidency. Aside from having political convictions acceptable to enough of the American populace, there are qualities such as personal integrity, appropriate education and life experience, the concept amphormously described as emotional intelligence, and the quality of advice they receive (some of which is under the President's control, some of which ultimately comes down to luck). You may also draw your own conclusions on these aspects of Bush's presidency so far. --Robert Merkel 13:32, 16 February 2006 (UTC)[reply]
This reminds me of a question about Bush I asked awhile back, to which I received no answer. I'll try again, below. --Halcatalyst 17:31, 16 February 2006 (UTC)[reply]
By the way, see www.snopes.com/inboxer/hoaxes/presiq.htm this widely-circluated parody on the topic, which was at the time reported as fact in The Guardian.--Pharos 21:07, 16 February 2006 (UTC)[reply]
IQ is indeed not quite the only important thing and even itself only an indication of intelligence. It is, however, a fairly good indicator of intellectual powers in the sense of achieving things (the tests are tested against school grades). And one can expect the leader of a country to have a certain intellectual level. Let's say university level or at least something close to that. So at least 120. A score below 100 would be below average (one might even say substandard) for your average job. For the leader of a country such a low IQ would be so bad it might even be a good idea to make it illegal. More so if that person is a president who almost single-handedly holds all power (and what a power in this case). Well, I know I exaggerate now, but I mean that in a parliamentary democracy powers are more evenly spread and the stupidity of one person will be levelled out by the others in the government and in parliament. DirkvdM 09:17, 17 February 2006 (UTC)[reply]
I don't think such legislation is necessary - voters would be smart enough to recognise that somebody with a mediocre IQ simply isn't up to the job and not elect them. Democracy means trusting the voters to get it right. --Robert Merkel 00:14, 18 February 2006 (UTC)[reply]
I admire your trust. Democracy is the best form of government by virtue of the fact that all other forms of government are even worse. The problem in the US system (not only there, though) is that people vote for a party but do that by voting for a single person who usurps a large part of the power. People want a single strong leader only because they don't know what's good for them. Checks and balances are needed. DirkvdM 07:30, 18 February 2006 (UTC)[reply]
A comment for both of you, if I may. Robert, in a democracy where the electoral system is transparent, the people choose whomever they choose, and that choice is by definition "the right decision", so it's not even a question of trust. Dirk, nobody can argue that their personal wisdom is greater than the collective wisdom of an entire country. Individuals may not know what's best for them, but countries do, in my opinion. "Usurpation" is hardly an accurate way of describing the system that the USA has accepted for a very long time. :-) JackofOz 10:40, 19 February 2006 (UTC)[reply]

Greek Historical Period 500-440[edit]

Hello i was just wondering if any one can help me in explaining the significance of the battles of Thermopylae and Artemisium Thank You

Have you looked at the articles on the Battle of Thermopylae and the Battle of Artemisium? David Sneek 11:08, 16 February 2006 (UTC)[reply]

Yeh i have but they arent giving me information in a great detail.

Women becoming porn star[edit]

How rich can one get by becoming a porn star or some sort of posing nude in magazines? Once decided that I am going to get into it, which area should I try to get maximum rich? That is should one try porn movies or should one try porn TVs or simply which would give me more fame and money How rich are people who are pornstars? Have anyone earned more than 100 million dollars?

what are the main reasons that business' carry out market research? --82.37.56.231 14:03, 16 February 2006 (UTC)--82.37.56.231 14:03, 16 February 2006 (UTC)--82.37.56.231 14:03, 16 February 2006 (UTC)[reply]

thanks

I know not much of this industry but it seems basic principles can applied. However, additional information is required to adequately answer your question.
  • The quality of the product you are selling.
    • Do you feel your product is of high or low quality?
    • Is the product heavily used or barely used?
  • The consumer base for the product you are seeling.
    • Can significant consumer demand be established for the product?
    • Do you think the product can be sold universally or only in few specialized communities?
    • Does your product support a multi-user interface?
  • The possible applications of the product you are selling.
    • Is the product versatile or can it only be used for very specific purposes?
    • Can the product be applied to previously uncharted areas of commerce in the relevant industry?
    • Can the product be bundled with similar products?
  • Time frame in which the product can be sold.
    • For how long is it feasible to sell the product?
    • Does the product have en expiration date?
  • Your investments in and maintenance of the product.
    • Does your product correspond to modern standards and needs?
    • Do you maintain your product well or do you sell it "as is"?

Celcius (Talk) Wiki be With us! 16:01, 16 February 2006 (UTC)[reply]

  • The people making the most money in porn (as well as any media) aren't the actors, it's the producers. (some of which have backgrounds as actors). I think it's safe to say no porno actor has ever come anywhere near $100 million on acting alone. They'd probably do well to reach $1 million. --BluePlatypus 16:13, 16 February 2006 (UTC)[reply]
  • Furthermore, their average careers are about 12 months. They might make $5,000 - $10,000 per film, if starring and getting certain types of penetration, and the women might work in 3 films a week. That's a lot of ripping and tearing and infecting (normal infections, not AIDS). Careers are short, drug use high, savings minimal, so no, it's not a great career for getting rich. On the other hand, the producers and distributors can make $250,000 on a $20,000 "film" investment. If they, too, make 3 films a week, they get exceptionally wealthy in the same year. Geogre 16:50, 16 February 2006 (UTC)[reply]
Katie Price has done well from modelling topless - she's created a successful media personality, sold a lot of books, had a reality show, and made quite a bit of money - this money.uk.msn.com/MyMoney/Insight/Well_Heeled/article.aspx?cp-documentid=145397 msn money piece states she and Peter Andre made £5 million in 2004 alone. That's definitely comfortable. Natgoo 21:11, 16 February 2006 (UTC)[reply]

The harder the porn, the more exploitative of the actors it becomes. So, posing nude or doing soft porn has the most potential to pay off. However, other skills, like acting ability, are probably needed, in any case. One way to "cut out the middlemen" (who steal all the profits) is to create your own web site. It would likely need to offer something special to succeed, though, like catering to a specific fetish. StuRat 21:21, 16 February 2006 (UTC)[reply]

Chasey Lain, Asia Carrere, and Jenna Jameson made quite a bit of money, though not $100 million probably, but then people in the porn business are a lot like standard actors. There are a handful who make millions per picture but most of them don't. In the end, porn is just another job. You either get very lucky, or you work at it like you would any other job. Dismas|(talk) 21:28, 16 February 2006 (UTC)[reply]
I disagree that all jobs are the same with respect to salary distributions. Jobs in sports and entertainment (including porn) have a dramatic difference between top salaries and bottom salaries, so much so that most people fail to make a living in those fields. Many non-sports and entertainment jobs, like teachers, doctors, engineers, etc., have a lot less difference between the top and bottom. Pretty much anyone who can hold down a job in those fields can make a good living. StuRat 00:18, 17 February 2006 (UTC)[reply]

Bush's MBA record[edit]

It's well-known that President Bush had "gentleman's C" grades as an undergraduate at Yale. He also received a Harvard MBA in 1975. How good was his academic performance at Harvard? Grade point average? Rank in class? Any distinctions? --Halcatalyst 17:37, 16 February 2006 (UTC)[reply]

Student records are sealed, generally. Unless the student makes them public, even if he becomes a public figure or a government official, the school has neither an obligation nor a likelihood of giving out his transcripts. Now, if he used his transcript for a job somewhere, that company might violate privacy regulations and make the information known, but the reason no one has reported his grades is that, honestly, Harvard won't tell. However, he reminds me of something Calvin Trillin said about Dan Quayle: "He's going around the country giving C students a bad name." Geogre 20:22, 16 February 2006 (UTC)[reply]
Harvard is very protective of student records and their policy is usually to not release them to researchers until 80 years have passed.hul.harvard.edu/huarc/holdings_02.shtml --Fastfission 22:59, 16 February 2006 (UTC)[reply]

Thank you both. I don't suppose academic achievement is highly valued in those political circles anyhow. --Halcatalyst 23:40, 16 February 2006 (UTC)[reply]

If someone applies for a job, it would be normal for the future employer to ask for (and get) the various grades that applicant has reached, right? And in a democracy the employers of a politician (or at least the recruitment officials, or what do you call that?) are the voters, the general public. So it would make sense if all future presidents would have to make their grades public knowledge. DirkvdM 09:25, 17 February 2006 (UTC)[reply]
In theory yes, in practice, as Halcatalyst said, grades probably aren't the deciding factor in a job like President. Plus, would you vote for a politician who got all As in his degree, but can't speak coherently to save himself, and wants to bring back corporal punishment for keeping a canary in your apartment (insert-any-objectionable-law-proposal-here)? — QuantumEleven | (talk) 12:26, 17 February 2006 (UTC)[reply]
Richard Hofstadter's Anti-Intellectualism in American Life (1966) ISBN 0394703170 is still very much in print. --Halcatalyst 16:31, 17 February 2006 (UTC)[reply]
Of course that is not the only basis to select an employee, but it's still one of the more important ones. DirkvdM 07:33, 18 February 2006 (UTC)[reply]
A few years ago... OK, 40 years ago, I recall ads in business publications saying, "Send me a man who reads." And they weren't referring to reading the stock tapes. --Halcatalyst 14:38, 18 February 2006 (UTC)[reply]

Beacon Hill, Tulare County, California.[edit]

Could you give me the history on how "Beacon Hill", located in Tulare County, California, got it's name and is it a protected landmark?

Thank you,


Russell Austin

Jesus Quote[edit]

There's a biblical quote I'm trying to track down, something to the effect of "what you hold true on earth, I'll hold true in Heaven." Does anyone recognize it? Was it addressed just to a particular person (like the Pope), or to all Christians? Black Carrot 20:33, 16 February 2006 (UTC)[reply]

I think it's a paraphrasing of Matthew 18:18, Verily I say unto you, Whatsoever ye shall bind on earth shall be bound in heaven: and whatsoever ye shall loose on earth shall be loosed in heaven. (KJV) --BluePlatypus 20:39, 16 February 2006 (UTC)[reply]
I do not disagree that it is likely a paraphrase of Matthew 18:18. However, it is a terrible paraphrase. Matthew 18:18 is discussing "saving" people. By bind, it is referring to the souls that you bind to Jesus. By loose, it is referring to the souls you let loose from Jesus. So, a better paraphrase would be, "Those who are saved will be in Heaven. Those that aren't, will not." --Kainaw (talk) 20:47, 16 February 2006 (UTC)[reply]

I'm not sure. The version of the quote I have above is from Dogma_(movie), and when I tried to find a reference to it on the web all I got was chat sites repeating it as though it were already gospel. In the movie, it claimed someone (I believe Jesus) said to someone (I belive called Paul) just before he left that whatever they decide on earth would be binding. This was apparently proof that something with a papal sanction (in this case an arch that cures sin) is absolutely reliable. I got onto this because I'm not sure whether the Vatican's recent move to eliminate Limbo, or its original decision that it existed in the first place, in any way represents the actual netherworld in their religion. Black Carrot 03:47, 17 February 2006 (UTC)[reply]

You could take a look at Limbo, and here's a link to a Catholic News Service article, www.catholicnews.com/data/stories/cns/0506867.htm Closing the doors of limbo: Theologians say it was hypothesis. As for Catholic teachings on the "netherworld" -- scroll down in Hell for a decent discussion. The idea of Limbo had to do with Original sin: if all human beings were infected with sin as a result, and baptism were the only way to erase it, what about the innocent souls (especially infants) who through no fault of their own were unbaptized? Did they go to hell? Theologians who thought this was unthinkable invented the idea of Limbo, which was not bliss with God in Heaven but was definitely not the fires of Hell. The idea was popularly accepted but never became church doctrine. Nowadays most theologians would frame the entire question differently. "Limbo" in theology has about the same status as the aether in physics. --Halcatalyst 05:01, 17 February 2006 (UTC)[reply]
"something with a papal sanction is absolutely reliable" sounds like Papal infallibility; that article has some potentially interesting readings on the scriptural grounding of that bit of dogma. Shimgray | talk | 17:27, 17 February 2006 (UTC)[reply]

That could be interpreted to mean that succesful people on earth will be succesful in heaven and vice versa...and in other occasion it's said that the last will be the first...thus the bible has inconcistent metaphysics.--Cosmic girl 17:41, 17 February 2006 (UTC)[reply]

The Bible inconsistent? Never. :) Superm401 - Talk 06:44, 20 February 2006 (UTC)[reply]

...totally... :D--Cosmic girl 22:47, 23 February 2006 (UTC)[reply]

We have an interesting article on Biblical inerrancy, a belief held by, oh, 40% of Americans today, a people known for their Religiosity as well as their passion for inerrant morality, many of whom believe, as he himself has implied, that the current President is a man sent by God to save the world. I guess these folks think it's all déjà vu, or something fundamentally like that. Leaves me kinda mystified. --Halcatalyst 19:48, 22 February 2006 (UTC)[reply]

February 17[edit]

How do I determine the taxes due on employer pension payments?[edit]

How is it possible for me to determine the taxes due on pension payments? Thank you Leah Roach

In the US, usually the money placed in the pension plan by the employer was not taxed. Therefore, when it is paid out, it is taxable, usually as ordinary income. --Halcatalyst 03:09, 17 February 2006 (UTC) For informational purposes only. For tax advice, consult your tax professional.[reply]
It's a little vague to give you the answer too. You didn't say where you live, so again for the US, the entity responsible for paying the pension is also responsible for withholding taxes from the payments. See www.irs.gov/pub/irs-pdf/p15.pdf publication 15 if you really want to know how that is done. Other than that, file a tax return to see if the withholdings covered the tax bill enough or to excess. Probably a 1040 would work, though you might be able to get by with others. See www.irs.gov for more. - Taxman Talk 05:17, 17 February 2006 (UTC)[reply]

suppressing a PhD[edit]

In some contexts, people tend to suppress the fact they have PhDs, especially those in the humanities. I've noticed this in business and journalism especially. What gives? I know that some people say P.H.D. means "piled higher and deeper," but.... --Halcatalyst 00:52, 17 February 2006 (UTC)[reply]

I would think in fields where a PhD is considered to be excessive this would be the case. It comes across like you're appending your high IQ to your title, just to impress people. In fields where a PhD is considered more of a requirement, like college profs, I would expect them to use the title more. StuRat 08:23, 17 February 2006 (UTC)[reply]
I'm not sure what you mean by "suppress the fact". If you mean that they don't list themselves as "Joe Schmoe, PhD" -- that's because everybody teaching and writing at the academic level in these fields (not all, of course) is pretty much assumed to have a PhD or some other high-level degree, so pointing it out doesn't help anyone. Additionally, the people who usually tend to point such things out explicitly are often those with some sort of problem establishing authority (i.e. "This book was written by Joe Schmoe, Ph.D." is a clear sign that Joe Schmoe thinks his academic training is necessary for you to take him seriously), and in fields where PhDs are the norm then you definitely don't want to look like one of those fellows. In many fields, though, PhDs are less common "requirements" and so noting them probably does not have this effect (or has a slightly different effect -- noting that you have a PhD in computer science will make it clear that you are an "academic" computer scientist, something somewhat different than most people working in on the "business" end of computer science. If that makes sense. --Fastfission 19:10, 17 February 2006 (UTC)[reply]

Small countries at the Olympics[edit]

Has a micronation ever competed in the Olympics? What is the smallest country by population that has ever won a gold medal? —Keenan Pepper 01:35, 17 February 2006 (UTC)[reply]

Probably Luxembourg. Won 2 golds, and has less than 500,000 people. See Total Olympics medal count. JackofOz 01:48, 17 February 2006 (UTC)[reply]
There are some problems in the data at Total Olympics medal count. According to Winter Olympics medal count, Liechtenstein has won two gold medals at the Winter Olympics, and it's considerably smaller than Luxembourg. Most of the micronations have competed in the Olympics, although not necessarily successfully. --Metropolitan90 02:47, 17 February 2006 (UTC)[reply]
I thought the original poster was talking about microstates and forgot to look up the definition of micronation. No micronations have ever competed in the Olympics. --Metropolitan90 02:49, 17 February 2006 (UTC)[reply]
Micronations don't compete at the Olympics. The smallest country with medals is almost certainly Liechtenstein in the Winter Olympics, and ISTR Bermuda won a medal at a Summer Olympics (possibly in the long jump or triple jump?). Nauru has won gold in Weightlifting at the Commonwealth Games, and the same competitor finished 8th in his event at the 2004 Olympics - not bad for a country with 6000 people. Grutness...wha? 10:13, 17 February 2006 (UTC)[reply]
Bermuda has one a medal according to the listed article. I didn't notice any with smaller populations. Rmhermen 17:23, 17 February 2006 (UTC)[reply]
They've won one bronze medal. The question was about gold medals. JackofOz 01:05, 18 February 2006 (UTC)[reply]
To further emphasise: despite its name, a micronation is not a nation, certainly not a way of saying a "small country", so it cannot possibly compete in the Olympics. Notinasnaid 18:27, 17 February 2006 (UTC)[reply]

Don't you guys remember Anthony Nesty from Suriname (pop: 438,144, lower than Luxembourg's 468,571 ), who upset the incredible Matt Biondi (by 0.01 second) to win Gold in 100 m butterfly at 1988 Summer Olympics. I never forget the race, Matt Biondi was targeting for the co-record of 7 Golds that Olympics games, but lost in two events, one of which to Anthony Nesty. --Vsion (talk) 09:38, 24 February 2006 (UTC)[reply]

The act of reading/studing a restaraunt menu and ordering[edit]

After many years of working in restuarants and observing patrons, it has come to my attention that there is no specific verb for the act of seating/reading/and ordering of a menu. Because millions of people do the exact action accross the world everyday there must be a verb to describe the action as one?

thankyou for your time Allyn Laing

  • You just gave them. Just because they usually follow each other doesn't mean there should be one verb to describe them all. - Mgm|(talk) 09:36, 17 February 2006 (UTC)[reply]
  • Not every idea or thing needs a single word for it. Different languages will vary in how they treat the same concept, and where one language takes several words to define a given idea another may have a single word that covers it. For example, in Spanish, there is a word for the sticky film left on the floor the morning after a party. I doubt many other languages have such a word, and no, I have no idea what the word is anymore. - Taxman Talk 17:32, 17 February 2006 (UTC)[reply]
lol! Maybe something related to costra or costrilla... --RiseRover 14:12, 19 February 2006 (UTC)[reply]
  • I suggest adopting menusing as the word and see if we can get it into common usage. DJ Clayworth 18:12, 17 February 2006 (UTC)[reply]
I will oppose the creation of such a "word" with every resource available to me. JackofOz 00:59, 18 February 2006 (UTC)[reply]
  • I propose calling it the Digester's Read :) Grutness...wha? 02:21, 18 February 2006 (UTC)[reply]

LOL!!! I'm for all of those. -LambaJan 06:27, 19 February 2006 (UTC)[reply]

Love[edit]

What is love?

Allen Matthews --67.173.157.34 02:13, 17 February 2006 (UTC)[reply]

Yes! I question I can answer with a YTMND: datalove.ytmnd.com/ —Keenan Pepper 02:41, 17 February 2006 (UTC)[reply]
(I think the original is whatislove.ytmnd.com/) —Keenan Pepper 02:44, 17 February 2006 (UTC)[reply]

Listen to the song L.O.V.E. by Ashlee Simpson...XD --Cosmic girl 03:36, 17 February 2006 (UTC)[reply]

See Love. -LambaJan 04:50, 17 February 2006 (UTC)[reply]

Assuming you mean romantic love, this is a mechanism to enforce pair bonding in humans and certain other species of animals. <satire>But then again, I'm just a romantic fool.</satire> StuRat 08:28, 17 February 2006 (UTC)[reply]

<friendly riposte not meant to be taken seriously> You couldn't possibly be. No true romantic would ever talk about "mechanisms to enforce pair bonding". </friendly riposte not meant to be taken seriously> JackofOz 09:03, 17 February 2006 (UTC)[reply]
That was a joke, son. I guess I need to use the <satire> </satire> flags for you. I've now added them. StuRat 15:52, 17 February 2006 (UTC)[reply]
No true evolutionist either. There are only mechanisms that have been selected for because they enforce pair bonding and this brings some reproductive benefit. --Anonymous, 09:51 UTC, February 17.
I've had this discussion before. Saying all that every time is about as painful as saying "when the Earth rotates into a position such that the Sun is now visible", instead of "at sunrise". If it was actually a discussion of evolution, I would go into that level of detail, but feel free to shorten it when not discussing evolution directly. StuRat 15:52, 17 February 2006 (UTC)[reply]
Love is nothing. At least, in tennis. User:Zoe|(talk) 18:11, 17 February 2006 (UTC)[reply]

I'm an Ignostic regarding 'love' because you can mean a lot of things by the word.--Cosmic girl 18:25, 17 February 2006 (UTC)[reply]

Thanks for expanding my vocabulary. JackofOz 00:56, 18 February 2006 (UTC)[reply]
You want your vocab expanded ? Ok...a freemartin is a lesbian cow. Seems odd that they have a specific name for it, doesn't it ? StuRat 05:23, 18 February 2006 (UTC)[reply]
The riches are inexhaustible. Night, Ralph. JackofOz 06:18, 18 February 2006 (UTC)[reply]

I could've sweared a 'freemartin' was a guy called martin who was a former prisoner of some sort! =P .--Cosmic girl 19:15, 18 February 2006 (UTC)[reply]

I'm storing Freemartin away in my little book of insults :) And don't forget that a podshaver is someone who hand-makes cricket bats! But I digress. Love can be many things, and has been classified and categorised in many different ways. A few of these include the following.
Love is:
  1. real ("real love")
  2. feeling ("feeling love")
  3. wanting to be loved
  4. touch (and vice versa)
  5. reaching ("reaching love")
  6. asking to be loved
  7. free (and vice versa)
  8. living ("living love")
  9. needing to be loved
Grutness...wha? 00:48, 19 February 2006 (UTC) (with apologies to J.L.)[reply]
You forgot #knowing (we can be)! --RiseRover|talk 19:58, 23 February 2006 (UTC)[reply]
Speaking of classifications for love, there is a theological doctrine that places it in four catagories:
1. God's love for Himself
2. God's love for mankind
3. Mankind's love for God
4. Mankind's love for man
I didn't think this fit the conversation earlier, but now I think you might find it a little bit interesting. -LambaJan 06:24, 19 February 2006 (UTC)[reply]

Which reminds me that some other languages have separate words for these different kinds of love. English lumps them all together. JackofOz 10:17, 19 February 2006 (UTC)[reply]

Israel page sources?[edit]

In the page on Israel, under Historical Roots, it states(summerized) that the Jewish people lived there, were kicked out by the Roman Empire, then the Roman Empire was beaten by the Arab Nations. As one of the arguments against Israel's Existance is that the arabs owned the land first then the Jewish people took it away from them, this statement is significant. My question is, has this fact been verified, and if so, how and by whom. If this fact is true, it shows the the Jewish people were the first people too live there. 72.57.173.62 02:33, 17 February 2006 (UTC) CuriousReader[reply]

This seems like an oversimplification, as there were other conquerors between the Romans and Muslims. However, the claim that the Jews were there first is easily verified, as Mohammed actually conquered Jerusalem from the Jews. The Jews had to be there first, then, didn't they ? Especially as Islam was basically founded by Mohammed. The Jews did take the land from the pagans who lived there before them, though. StuRat 04:14, 17 February 2006 (UTC)[reply]

There's a book called the Tanakh (or O.T. to most of us) that outlines the original claim. You're not from around here, are you? alteripse 02:50, 17 February 2006 (UTC)[reply]

Is this a sterotype or is it true?[edit]

Seriously now, do black people in the southern US really prefer fried chicken and watermelon or is it just a common sterotype? A Clown in the Dark 03:31, 17 February 2006 (UTC)[reply]

Well, I'm white and I like both. In fact, that makes a pretty good picnic. Just add a soft drink and some sort of amateur sport, and you're there. Black Carrot 03:39, 17 February 2006 (UTC)[reply]
Make up your mind. Or are you only black when you're a carrot? DirkvdM 09:28, 17 February 2006 (UTC)[reply]
Those are common foods, particularly for picnics. I'm not sure those foods are much more popular amongst blacks than whites, though. They are cheap, however, so would be more popular among the poor, many of whom are black. StuRat 04:06, 17 February 2006 (UTC)[reply]
I think the watermelon stereotype comes from the unfortunate fact that many blacks were forced to labor on watermelon farms. This was treated in Bamboozled. The fried chicken stereotype is probably more factually based, but it should be thought of as simply a visable portion of a stereotypical diet that also includes many other hearty southern recipies. Here it's important to note the southern connection. If you go to Georgia, you'll find that the white diet is nearly identical to the diet that northern whites often attribute to the stereotypical black. -LambaJan 04:56, 17 February 2006 (UTC)[reply]
Don't forget the cornbread and collard greens! —Keenan Pepper 05:42, 17 February 2006 (UTC)[reply]
I eat all of these things: soul foodDlayiga 07:17, 17 February 2006 (UTC)[reply]
Who the hell doesn't like watermelon? Marskell 10:35, 17 February 2006 (UTC)[reply]
Fructophobes, perhaps? JackofOz 10:44, 17 February 2006 (UTC)[reply]
I don't like watermelon. Thryduulf 11:45, 17 February 2006 (UTC)[reply]
Any reply will probably will probably violate NPA... I didn't think it was possible to actually dislike watermelon but I have been corrected :). Marskell 23:24, 17 February 2006 (UTC)[reply]
Eating too much fruit can give you a urinary tract infection. Especially sweet fruit and watermelon certainly classifies there. Alas, I speak from experience. DirkvdM 07:36, 18 February 2006 (UTC)[reply]
Watermelon and okra are foods with African origins although, for watermelon at least, its history in the Americas is complex. Rmhermen 17:17, 17 February 2006 (UTC)[reply]

India[edit]

How many metropolitan cites are there in India? Is Banglore or Hyderabad a metro.61.17.240.135 05:40, 17 February 2006 (UTC)[reply]

There is no clear definition of metropolis, so I'm afraid it's not possible to give a good answer to your question. - ulayiti (talk) 17:40, 17 February 2006 (UTC)[reply]
The table near the end of Delhi classifies Ahmedabad, Bangalore, Chennai, Delhi, Hyderabad, Kolkata, and Mumbai as metropolises though how this was arrived at is not obvious. Tintin (talk) 00:19, 18 February 2006 (UTC)[reply]

Outsourcing[edit]

To Reference Desk,

Here is a business question, I have been noticing the recent buzzword "transformational outsourcing", what does this mean? What does mean in connection to "oursourcing"?

Perhaps they are talking of outsourcing to such an extent that only upper management remains in the company. StuRat 08:17, 17 February 2006 (UTC)[reply]
After subjecting myself to a maze of buzzwords, all alike, I seems that "transformational outsourcing" is concerned with using outsourcing to change the way a company does business. While originally outsourcing was seen primarily as a way to cut costs, this transformational stuff is less about costs and more about finding ways to quickly achieve some type of business goal. A simple Google search with the term "transformational outsourcing" yielded "about 239,000" results. Just beware of the buzzwords. --LarryMac 17:36, 17 February 2006 (UTC)[reply]

Doctrine of Estoppel[edit]

A person who is not a party to a contract cannot sue to except where for very good reasons,The common law allows him to do so.state and explain at least four instances in which statutory power or authority has conferred on the individual the power to sue to enforce contract to which he/she is not a party

Do your own homework. Thryduulf 11:43, 17 February 2006 (UTC)[reply]
Seriously, you didn't even try to phrase this as a normal question. If you're going to try to deceive your teacher about who did your homework, you may as well practise on us first. GeeJo (t) (c)  13:53, 17 February 2006 (UTC)[reply]

Absurdism vs. Faith[edit]

Why should I choose Absurdism as opposed to Faith according to absurdists? And why should I choose Faith as opposed to Absurdism according to theists or theologians?, what would Camus, kierkeegard,kafka,tertullian,and all those guys tell me?What was Kant's oppinion on absurdism? did he have any? ( I know, this question can make anyone uneasy...but I ask those kinds of questions...=P ) --Cosmic girl 17:11, 17 February 2006 (UTC)[reply]

Why do you assume Absurdism and Faith are mutually exclusive? Our own definition "Absurdism is a philosophy stating that the efforts of man to find meaning in the universe will ultimately fail because no such meaning exists (at least in relation to man)" is unfortunate and misleading I think. Kierkegaard for one, viewed faith as absurd. A better definition than Absurd = meaninglessness is Absurd = acting despite meaninglessness and this action is itself a kind of faith. This, IMO, is how Kierk. and later existenialists viewed it. Marskell 17:44, 17 February 2006 (UTC)[reply]

You are right, acting in spite of the absurd is a kind of faith, but I meant the faith of the theists that not everything is meaningless, I mean, the faith of the theists that there is an absolute which is self consistent and provides meaning, not an absurd 'truth'. Absurdism is in fact opposed to the faith of the theists because the later implies meaning exists. maybe kierk. meant that faith is absurd because we have no reason to have it, BUT I'm asking why should someone choose either one. (faith or non faith)--Cosmic girl 18:18, 17 February 2006 (UTC)[reply]

Credo quia absurdum. David Sneek 19:52, 17 February 2006 (UTC)[reply]

XD thnx! but the question is still open if anyone wants to contribute...=P. --Cosmic girl 21:41, 17 February 2006 (UTC)[reply]

Ms. Cosmic:
I think if you want an answer as to why traditional, doctrinaire, "this-is-the-reaveled-word" believers adhere to a particular faith, you will be somewhat disappointed. I would say (with a touch of prejudice) that they have this faith because their fathers had this faith. There is no grander reason than that. If you believe that the Qur'an is the revealed word or that Christ is the son of God, well, you believe that--and I suspect you believe that because you were raised with that truth. If there were a definite reason to believe it there would not be a debate. But there is no definite reason. You can't ask for one with faith in this strict sense.
The point of my post was to underscore the fact that pious monotheisits cannot corner faith and call it their own. The existentialists, particularly Kierkegaard (peace be upon him :), helped re-define faith as absurd, for people who would never think the concept relevant otherwise. Faith is a big word. It's bigger than any -ism you can present. Marskell 23:44, 17 February 2006 (UTC)[reply]

wow...I'm wordless,I can't counter that anymore, good job!..you are right,faith is a big word, but skepticism can be bigger! Also...I didn't mean faith too much in the sense of this is the revealed word I meant it more in the sense of absolute truth exists = faith as opposed to we can't know if absolute truth exists or absolutes don't exist = skepticism and maybe absurdism. --Cosmic girl 00:12, 18 February 2006 (UTC)[reply]

Part of a litany: "Sustain us in belief, protect us with doubt" (Patricia Storace). --Halcatalyst 03:43, 18 February 2006 (UTC)[reply]

Well, as someone who comes from a rather agnostic/materialist family who became a rather faithful member of an organized religion, I'd have to say that a big attraction for me is the logic of it. I mean this in terms of the teachings that are common among the major ones, which are consistant with the ideal actions that would maintain order and tranquility in the world. Sure, certain teachings have been changed or blown out of proportion so that some selfish individuals could use religion for their own personal motives, but those people who really acted in a manner consistant with what the major religions teach, whether they claim membership to those religions or not, have all been nearly universally recognized as great people who have done great things to furthur society. I don't think that's absurd. -LambaJan 06:44, 18 February 2006 (UTC)[reply]

It's not a coincidence that the Abrahamic religions have a lot in common, as both Christianity and then Islam branched off of Judaism. StuRat 19:43, 18 February 2006 (UTC)[reply]

Journalists helping their topic pieces[edit]

If journalists, say from National Geographic, write a piece on a suffering family in some part of the world do they financially assist the family or just write the piece and then move on? More generally, to what extent do journalists become involved with helping to improve bad situations on which they are reporting (like newscasters saving pets during Katrina)

I recently saw just such a discussion, this time with PBS news reporters following up on the Pakistan-India earthquake. They said they feel they can do far more good by bringing the world's attention than they can by helping them directly. That said, they did help directly a bit, but not much. StuRat 02:21, 18 February 2006 (UTC)[reply]

Congo Free State and Tippu Tip...?[edit]

Can it be argued that Tippu Tip inspired Congolese nationalism and independence from Belgium?

Just about anything can be argued. But what do you think happened? (Hint: You would only get some sort of opinion on that by doing your homework.) JackofOz 00:43, 18 February 2006 (UTC)[reply]
Meh. Big thug got shoved out by the bigger thugs. Read King Leopold's Ghost. --Brasswatchman 06:15, 22 February 2006 (UTC)[reply]

February 18[edit]

Mosques[edit]

Hello i was just wondering if there are any muslims that would like to complete my survey.. it would be a great deal of help.. thank you... Name:(Optional)... Gender:... Age:... Do you attend your mosque regularly?.... Do you know what the architecture of the mosque means? If yes please explain... Does the architecture of your mosque personally to you, reflect or inspire religious beliefs? If yes what are they?... What does the architecture personally mean to you?... As an individual does the architecture capture you in any form (such as spiritually, emotionally and e.t.c)? Thank You For Completing the survey —Preceding unsigned comment added by 210.50.176.166 (talkcontribs) 01:09, 18 February 2006

I'm sorry to criticize, but I don't think this is the best place for this particular survey... not if you want your results to have any scientific validity, anyway. For instance, the method of transmittion (wikipedia reference desk) invites the Selection bias. See Bias (statistics) and List of cognitive biases. Also the short length of the survey and the openendedness of the questions may be problematic. I think the best way to collect this data would be to visit a local Mosque and interview a few people. -LambaJan 06:58, 18 February 2006 (UTC)[reply]
But only if you want to interview church-going muslims specifically and even then there will be a bias towards the more church-going ones, if you'll pardon my crooked English. DirkvdM 07:40, 18 February 2006 (UTC)[reply]
  • Don't you mean mosque-going? - Mgm|(talk) 08:54, 18 February 2006 (UTC)[reply]
Ehm, whatever. What is a general term? Temple? Ok, temple-going then. DirkvdM 08:46, 19 February 2006 (UTC)[reply]

I know my survey is got openendedness but im just trying to gather data for a wide range, this will just help me in my summing up of the topic i have choosen.. Thank You If you could complete the survey it would b great :D —Preceding unsigned comment added by 203.134.71.159 (talkcontribs) 08:42, 19 February 2006

From Islamic architecture:

Common interpretations of Islamic architecture include the following:

  • The concept of Allah's infinite power is evoked by designs with repeating themes which suggest infinity.
  • Human and animal forms are rarely depicted in decorative art as Allah's work is matchless. Foliage is a frequent motif but typically stylized or simplified for the same reason.
  • Calligraphy is used to enhance the interior of a building by providing quotations from the Qur'an.
  • Islamic architecture has been called the "architecture of the veil" because the beauty lies in the inner spaces (courtyards and rooms) which are not visible from the outside (street view).
  • Use of impressive forms such as large domes, towering minarets, and large courtyards are intended to convey power.
-LambaJan 04:29, 21 February 2006 (UTC)[reply]

Graduate and post graduate students.[edit]

what is the differene between a post grauduate student and an under graduate student?

Graduation. GeeJo (t) (c)  12:36, 18 February 2006 (UTC)[reply]
Undergraduate and Postgraduate education explain it better. JackofOz 12:47, 18 February 2006 (UTC)[reply]
In the U.S. educational system, you have:
  • Undergraduates. These are people attending college for the first time. Usually 4 years of instruction.
  • Graduate students. These are people who are pursuing master's or doctoral degrees. They have usually received an undergraduate degree. Programs vary by degree type and discipline, usually between 1 and 6 years. Sometimes these are apparently referred to as "postgraduates" (though I haven't heard it much in the United States).
  • Postdoctoral appointments. These are usually not considered students, but people who have received their master's or doctoral degrees who get an appointment for a year or two (or more) at a university or research institution, often to do research and sometimes to teach. The ramifications of these vary by discipline; in the humanities they are often a way to "kill some time, get some experience" if one has difficulty getting a job the first time around.
Hopefully that clarifies the terminology a bit. --Fastfission 19:25, 18 February 2006 (UTC)[reply]

Art[edit]

This came up during a discussion with a friend last night: at what point in history did artists start signing their works? I'm thinking of western art here. I know Michelangelo carved his name on one of his Pietàs, but didn't add a signature on any other work. When did signing a painting or sculpture become the normal thing to do? David Sneek 12:55, 18 February 2006 (UTC)[reply]

This has to do with the nature of the artists, themselves. In the middle ages, art was treated largely as a commodity, with each king or noble employing a staff who were told what art to create. The emphasis was on the art looking the same, so the work of any one artist wouldn't stand out. Signatures under this scheme served no purpose. Later, artists like Michelangelo developed a style all their own, and worked for a number of employers, such as the Medici family. Under this new paradigm, signing works of art now made sense. Still, there are times a work of art remains unsigned, such as when the artist considers it unfinished or not up to par. StuRat 19:19, 18 February 2006 (UTC)[reply]

Thank you, StuRat, but it's not exactly the answer I was hoping for. I understand that a new paradigm and different ideas about the role of the artist caused this change, but nevertheless people like Tintoretto, Titian or Michelangelo hardly ever signed their work; regardless of whether it was finished or not, of the highest standard or not. I'm looking for something more concrete: at what point did it become customary to finish every work with a signature? I realize that it's maybe unanswerable, because this custom probably emerged gradually... David Sneek 11:32, 19 February 2006 (UTC)[reply]

Is Polygamy possible with written consent?[edit]

I just want to know whether having two or more wives is possible in USA for US citizens provided all wives give written consent for that. And if possible, tell me whether it is possible in other western countries and is it possible in Singapore?

Not sure what you mean -- U.S. governmental entities don't legally recognize any marriage after the first. "Letters of consent" have nothing to do with it. Whether they choose to prosecute for bigamy in any particular case is a different matter. AnonMoos 13:26, 18 February 2006 (UTC)[reply]
IANAL. To my knowledge, letters of consent or any other similar thing cannot change laws. The only way to legally marry more than one person, if it's illegal in that area, is to get the law changed. Dismas|(talk) 16:24, 18 February 2006 (UTC)[reply]
I agree that it's not currently legal in the US, but suggest it could be, under a system similar to database locks:
  • An EXCLUSIVE marriage (the regular type) would only be allowed if neither partner was already married, and would prevent EITHER form of marriage by either partner until the marriage is ended with a divorce.
  • A SHARED marriage would be allowed so long as neither partner had an existing EXCLUSIVE marriage and would only prevent future EXCLUSIVE marriages, until it ended with divorce.
Such a system would prevent the current discrimination against religious minorities, such as a faction of Mormons, and other cultures where polygamy is normal. StuRat 19:00, 18 February 2006 (UTC)[reply]
But, such a system would cause problems for the economic purpose of marriage. Married people may file taxes jointly. All joint forms ask you to list the husband and wife. There is no form for second, third, or fourth wife. We know how much government wastes on paperwork - imaging how many billions of dollars it will take to change the tax law and forms to allow for multiple spouses. Then, there is insurance. In insurance, a spouse is often covered and/or liable. This is set for one spouse. They don't want to deal with the situation that one guy gets insurance and suddenly his 14 wifes have to be covered as well. In death, a spouse has many legal rights to the estate. Imagine how this would play out with multiple spouses. All in all, marriage is far more a legal rights agreement than a bond of love. So, in allowing multiple spouses, the legal ramifications should be taken into account. --Kainaw (talk) 20:06, 18 February 2006 (UTC)[reply]
Just because it's difficult doesn't mean it can't be done. Note that many of those issues arise now, when a rich old man dies with young wife like Anna-Nicole Smith (who apparently only married him for the money), several ex-wives, and perhaps current or former girlfriends, some of which he has had children with. StuRat 20:41, 18 February 2006 (UTC)[reply]
One of the b*i*g difficulties is the law of contracts - see the consequences when the marriage ends, either by death, divorce or other existing (?) impeachments. To whom belongs the car, the dog or the house ; what may children expect of the wealth of the married parents, who cares for them, &c.
Some countries may find nothing to oppose to marital (unmarried) life with more than one partner. --DLL 20:15, 18 February 2006 (UTC)[reply]

I'd suggest having a look at some of the websites linked at the bottom of Polyamory - something on one of them might give you some advice. IANAL, but personally I'd suggest a non-marriage partnership (such as de facto), with agreements similar to pre-nups. Legally polygamy is barred in most western countries, but any combination is possible in de facto (I know a FMM group and a FMF group living quite happily here in NZ). Grutness...wha? 00:06, 19 February 2006 (UTC)[reply]

Allowing people of the same sex to marry at all would be a more just cause than inventing ways of letting straight people have more than one legal spouse. I congratulate those countries (alas, not mine) that have already taken this step. JackofOz 00:17, 19 February 2006 (UTC)[reply]

In Singapore, under the Women's Charter (Singapore), polygamy is illegal. However, there is an exception; marriages for Muslims can be registrated under the Shariah law, which allows polygamy "under special circumstances", but these cases are considered to be rare. --Vsion (talk) 09:21, 24 February 2006 (UTC)[reply]

Country with no Army[edit]

What countries of the world do not have a standing army?

According to www.dailytexanonline.com/media/paper410/news/2004/02/10/WorldNation/Death.Toll.In.Haiti.Uprising.Reaches.40-602419.shtml?norewrite&sourcedomain=www.dailytexanonline.com this article from the Daily Texas Online, Haiti has no army, the www.cia.gov/cia/publications/factbook/geos/ha.html CIA factbook states that "the regular Haitian Armed Forces (FAdH) - Army, Navy, and Air Force - have been demobilized but still exist on paper until or unless they are constitutionally abolished." - Akamad 14:22, 18 February 2006 (UTC)[reply]
Nauru, for one. I suspect a few other Pacific states. Also various European microstates - Andorra, Monaco, &c? Shimgray | talk | 14:31, 18 February 2006 (UTC)[reply]
Costa Rica (www.cia.gov/cia/publications/factbook/geos/cs.html#Military). David Sneek 16:19, 18 February 2006 (UTC)[reply]
Yeah. See Military of Costa Rica. Also, Japan is forbidden by its own Constitution from having "land, sea, and air forces, as well as other war potential", though the "self-defense forces" seem like an army. Iceland also has none. Likewise Liechtenstein and Palau. --jpgordon∇∆∇∆ 16:53, 18 February 2006 (UTC)[reply]
I'd also include the Vatican, if only because it's impossible to take an army in uniforms like these seriously. :) GeeJo (t) (c)  18:20, 18 February 2006 (UTC)[reply]
You do realize that the Swiss Guard are trained in machine gun usage as well as halberds? Rmhermen 20:26, 18 February 2006 (UTC)[reply]
Yep, which is why I qualified the statement. They are a defence force, but it's pretty hard to take them seriously wearing Ren fair clothing. GeeJo (t) (c)  23:01, 18 February 2006 (UTC)[reply]
Traditions have their uses. JackofOz 00:00, 19 February 2006 (UTC)[reply]
Yes. Imagine these guys running across the battle field at you. You'd be laughing too hard to fight back. --Kainaw (talk) 04:19, 19 February 2006 (UTC)[reply]
Maybe some pope watched Monty Python? DirkvdM 08:57, 19 February 2006 (UTC)[reply]
Anyone remember the pope character that John Cleese played so brilliantly? He summoned Michelangelo, who had painted a Last Supper with kangaroos, red and green jellies, and three (!) Christs. The sketch ended with Pope Cleese shouting "Look mate, I'm the bleeding pope, I may not know much about art, but I know what I like". Maybe it was he who designed the Swiss Guards' uniforms. JackofOz 10:12, 19 February 2006 (UTC)[reply]

And what prevents these countries from being invaded? DirkvdM 08:57, 19 February 2006 (UTC)[reply]

You want an honest answer? I could say, "who wants them!?" But, the honest answer is that they have defense agreements of one kind or another with larger, well militarized countries. For example, Kuwait has practically no military force. So, Iraq invaded and Kuwait called upon the United States to defend it. From a political standpoint, Kuwait is protected by the UN. From a practical standpoint, Kuwait is protected by the United States. --Kainaw (talk) 13:08, 19 February 2006 (UTC)[reply]
Iceland is also protected by the U.S. Rmhermen 20:50, 19 February 2006 (UTC)[reply]
In the case of Kuwait, that makes sense; it has something that most other contries want to preserve, namely oil supply. What interrest does the USA have in Iceland that they would protect that country? And what if the US wanted to invade Iceland? Sounds unlikely, but just for the sake of the argument? DirkvdM 10:27, 21 February 2006 (UTC)[reply]
Iceland has Bjork! Seriously, Iceland has a monstrous fishing market. They also have a large stone market. The key is that they will ship stone to you a water-bound project (like a levee or bridge support) and drop it right where you want it. If you buy it from a U.S. supplier, you have to pay to ship it by truck or rail, which is much more expensive. (This came up two years ago when the new Charleston, SC bridge was built.) Iceland also has a lot of aluminum, but that market has been getting smaller as most usage of aluminum is being replaced by other products - like plastic bodies in cars or steel cans for drinks. Finally, Iceland is part of NATO. Why? It would just look weird if you left it out. --Kainaw (talk) 14:41, 21 February 2006 (UTC)[reply]

You know, there's an article on this. List of countries without an army. Enjoy. --IvanP/(болтай) 14:20, 23 February 2006 (UTC)[reply]

royal academy of arts[edit]

what were some of the events leading up to the foundation of the academy besides the foundling hospital? Who else attempted to form an academy? or let me know where i can find the info-that would be just as good. thanks

Have you had a look at the Royal Academy article? David Sneek 16:46, 18 February 2006 (UTC)[reply]

Legislation and Special Needs Code of Practice[edit]

Could anyone tell me as a result of the following quote which Law came about. Providers should make `reasonable adjustments` to include children with SEN - e.g provide/attend training. A written SEN policy is needed. A SENCO should be identified Sorry i forgot to put it is in the UK I have looked through the SEN and Disability act but, cant find it.

Thank you for your help Cazzy 23:53, 18 February 2006 (UTC)[reply]


Do you know which country ? StuRat 20:32, 18 February 2006 (UTC)[reply]
SEN's are used in the UK - could it be the www.opsi.gov.uk/acts/acts2001/20010010.htm Special Educational Needs and Disability Act 2001? Natgoo 20:51, 18 February 2006 (UTC)[reply]
Part IV of the www.opsi.gov.uk/acts/acts1996/96056-zd.htm#p4c1 Education Act 1996 also discusses SEN - it might be what you're looking for. Natgoo 01:41, 19 February 2006 (UTC)[reply]

Japan Questions[edit]

I was wondering if anyone could help me out with this question.

When was the mortar and petsal invented in Japan? Also, when did gem mining (esp. diamonds and rubies) first start in Japan?

I am using this info to date a folk tale which I cannot find a time period for.

Thank you very much! ^_^

Raven

Appearently, the Japanese used to use a pestle to grind things on a flat stone until the mortar was introduced from China in the 11th or 12th century. www.gourmetsleuth.com/suribachis.htm I couldn't find anything on gem mining. BTW, what is this tale? -LambaJan 22:10, 18 February 2006 (UTC)[reply]

The folk tale I am researching is called the Jeweled Sea. --Raven

Identify this food stuff[edit]

I visited my favorite oriental food store the other day and bought what I though was a container of hummus. When I opened it I discovered something else. The text on the container says it is from Lebanon, called "Halawa extra" and contains grounded sesame seeds, sugar, natural roots and pistachios. The taste is very sweet. What is it and what can I do with it? Thuresson 20:23, 18 February 2006 (UTC)[reply]

Sounds like tahini. StuRat 20:29, 18 February 2006 (UTC)[reply]
It's Halva (the second meaning), which is known in various places as Halwa, Halawa, Halava, etc. -- Finlay McWalter | Talk 20:32, 18 February 2006 (UTC)[reply]
Thanks for the help. But what do I do with it, can I eat it straight from the container with a spoon? Thuresson 07:55, 19 February 2006 (UTC)[reply]
You betcha! I can eat way too much of that stuff, given the opportunity. If there's oil sitting on top of the halvah, you might want to try to stir it in -- otherwise the halvah can be somewhat chalky. On the other hand, some people like it chalky; experiment. Note: it can be addictive. A pal of mine visits her family home in Nablus every couple years, and they usually bring me back some of the local stuff. Mmm. --jpgordon∇∆∇∆ 08:09, 19 February 2006 (UTC)[reply]
Sounds like a halva good treat. StuRat 05:06, 22 February 2006 (UTC)[reply]
(slaps StuRat with a wet haddock) ;-)
You can eat it straight out with a spoon, but beware that you will likely finish the container before you know it. Alternatively, you can spread it on bread and eat it - rather a nice combination, too. Bon appetit! — QuantumEleven | (talk) 13:28, 22 February 2006 (UTC)[reply]

sanskrit words[edit]

hello ,

can anyone give me the sanskrit equivelant to the following words in hindu philosophy ( in sanskrit letters ) : Agni, Varuna वरुण, Rta, Soma, Rudra, Vishnu विष्णु, Prajapati, Samhita, Brahma Sutra ब्रह्मासूत्र, Yoga Sutra, Yoga Vasishta, Atman, Samsara, Kshatriya, Shudras, Artha, Kama काम, Karma Yoga, Bhakti Yoga, Jnana Yoga, Raja Yoga, Samadhi, Devadasis, Samnayasin, Sadhu, Swami, Yogin, Mudra, Mantra, thank you

i have an open proxy so i cant edit - but what is that writing they use after they say sanskrit ( .... ) in many wikipedia articles , thats what i need , but maybe this isnt the right place . Hhnnrr 21:05, 18 February 2006 (UTC)[reply]

Those look like Sanskrit words already. —Keenan Pepper 21:27, 18 February 2006 (UTC)[reply]
yeah, but the questioner's asking for them in Sanskrit script. Grutness...wha? 00:00, 19 February 2006 (UTC)[reply]
There isn't such a thing as "Sanskrit Script". Westerners most often see Sanskrit written in Devanagari, but traditionally, Sanskrit was written in different regional scripts in the different parts of India (Devanagari happens to be the regional script for the Hindi language, basically). AnonMoos 00:34, 19 February 2006 (UTC)[reply]
I'm just saying what the questioner asked. Devanagari would be the most logical script to use, since it is the...erm... graphia franca for siuch languages. Mind you, the question should have been over at the Languages Desk rather than here anyway. Grutness...wha? 00:42, 19 February 2006 (UTC)[reply]

I gave you a few above, and I could do the rest according to how they would be in Hindi, but I'm not confident enough that they would be correct Sanskrit. Some of the Wikipedia articles have the devanagari, so try putting the terms in the search box at the upper left to look for the article. You can also start from Hindu deities that has links to many of our articles. You could also get all your answers by spending some time google searching. - Taxman Talk 13:31, 19 February 2006 (UTC)[reply]


Have you looked up the woed Brahmi ? Devnagari (as well as I can remember from my schooling)came as a descendant to Brahmi. I can not use the four tildes. ~ ~ ~ ~ because my IP address is not permanent. 202.161.131.69 18:22, 23 February 2006 (UTC)[reply]

Comment: Question has long been answered on Language reference desk. Please don't double post questions. deeptrivia (talk) 04:55, 24 February 2006 (UTC)[reply]

Vatican & heliocentric theory[edit]

I have heard that the Catholic Church did not officially recognize that the Earth revolved around the Sun until the 1920's. Is this true?

Thanks, Andrew4549

I don't know about that, but they only apologized for their conviction of Galileo for saying so, in 1992. StuRat 23:45, 18 February 2006 (UTC)[reply]
When did they decide it wasn't a sin to use anasthetics? User:Zoe|(talk) 07:01, 19 February 2006 (UTC)[reply]
The Catholic Church never thought it was. Jesuit missionaries learned the virtues of cocaine from those they conquered in South America; following its use by the Countess of Chinchon, Regent of Peru, in 1638, it was introduced to Europe. It was the Protestants who abjected to its use as an anaesthetic, calling it an invention of the devil, or the Catholics (it hardly mattered which). And in England, when the use of (better) anaesthetics was introduced, it fell to the Protestant churches to denounce them as contrary to Holy Writ, it being self-evident that the use of chloroform to avoid "the primeval curse on woman" was impious.
As to Galileo, like all history it is rather more complicated than one likes. It wasn't "Earth center, Galileo bad" followed by "Oops, Sun center, Galileo good." The first response the Church had to being proved wrong about heliocentrism was to evade its own history (i.e., lie: to claim that Galileo was condemned not for affirming the Earth's motion, but for supporting it from Scripture, or that he was condemned not for heresy, but for contumacy, or that the condemnation was provisory, and that popes as popes had never condemend Galileo's theory, and that therefore infallibility was not involved. All the while, quietly removing Copernicus from the Index Librorum Prohibitorum, etc. I don't know that they ever "officially" said "the Earth revolves around the Sun" in the same way, or with the same force, with which they once said "The Sun revolves around the Earth". - Nunh-huh 07:50, 19 February 2006 (UTC)[reply]
In this depiction of the Tychonic system, the objects on blue orbits (the moon and the sun) rotate around the earth. The objects on orange orbits (Mercury, Venus, Mars, Jupiter, and Saturn) rotate around the sun. Around all is sphere of fixed stars.
After the observation of the Phases of Venus was more or less accepted by even the top Jesuit astronomers as invalidating the Ptolemaic system, the Church switched to the Tychonic system as their primary cosmology. It's not quite as indefensible -- it is basically the same thing as the Copernican system, except all from the point of view of a stationary earth. The choice between the Copernican system and the Tychonic one was, for many years, primarily a matter of philosophical choice; there was no real empirical way to distinguish between the two until the parallax was observed in the 19th century. So they aren't quite bone-headed as they usually are made out to be, though I admit I don't know all of the details of what their "official" position meant or whether or not Jesuit astronomers really stuck with it for so long (I doubt they did). --Fastfission 22:09, 20 February 2006 (UTC)[reply]

February 19[edit]

Is pain in the mind?[edit]

Is pain in the mind? Hence, is it possible to condition our minds to NOT feel pain? gelo 01:25, 19 February 2006 (UTC)[reply]

Have you checked the useful online resource Wikipeda? They have an article about pain and it is not too painful to read. MeltBanana 01:32, 19 February 2006 (UTC)[reply]

Nuclear Superpowers[edit]

Although the Soviet Union collapsed in 1991, Russia today would still have nuclear power. So between the United States and present-day Russia (not the Soviet Union), who would have stronger nuclear weapons today according to the most recent reliable info? Did the collapse of the U.S.S.R. even have an effect on their nuclear weapons programs? --Swang 04:16, 19 February 2006 (UTC)[reply]

  • "Stronger nuclear weapons" is a bit of a null term given the state of current warfare - let's just say they're both strong enough. In terms of overall number of warheads (a far more important thing) the US would have a significant edge. But when it's a difference between being able to destroy the planet ten times over or twenty times over, these figures don't really make that much difference. Grutness...wha? 06:12, 19 February 2006 (UTC)[reply]
  • If you take a look at our articles "List of countries with nuclear weapons" and Nuclear arms race" it will give you some idea of the comparative nuclear forces in terms of warheads. Of course there is more to it than that (delivery systems, etc.), but it is somewhat of a metric, and both of those articles have references to more detailed discussions of both the U.S. and Russia's nuclear forces as of the end of 2005. The collapse of the USSR did have a very powerful effect -- both countries soon scaled back their production programs, stopped all nuclear testing, and generally shifted their weapons development programs into what is called in the U.S. "stockpile stewardship" for the most part. A somewhat detailed discussion of all of this from the U.S. perspective is in our article "Nuclear weapons and the United States". Hope that helps you. --Fastfission 16:40, 19 February 2006 (UTC)[reply]
    • By the way, Russia always had and still has more warheads than the U.S. (since the 1950's anyway) Rmhermen 20:42, 19 February 2006 (UTC)[reply]
      • oops - my bad. ISTR that the US does lead in therms of number of delivery systems, though, which is important if you're planning to bomb somewhere. Grutness...wha? 00:42, 22 February 2006 (UTC)[reply]
        • Strictly speaking that depends how you define "delivery system". Suitcase bombs would be the delivery system of choice for i.e.. terrorists. You could slap a nuke on anything that moves i.e.. mules and make perfectly reliable and deadly nukemules. Celcius (Talk) Wiki be With us! 14:54, 24 February 2006 (UTC)[reply]

Order of the Friars Minor in Peru[edit]

Was the "Order of the Friars Minor" persecuted or oppressed by the civil authorities or the Church at the beggining of the Eighteenth Century in Peru?

-- M. M. S.

I scouted around some for information on this question and came up empty. However, given the general historical circumstances, the answer is likely that there was significant conflict among the Franciscan (OFM) and Jesuit missionaries, the Spanish authorities, Conquistadors like Pizzaro, and the church authorities both local and back in Rome. See Spanish conquest of the Inca Empire and, more generally, Spanish colonization of the Americas. --Halcatalyst 20:41, 22 February 2006 (UTC)[reply]

World peace and US democracy[edit]

I've got this notion that the world would be a much more peaceful place if the US had a parliamentary democracy (preferably with direct elections, so more like the Dutch than the British system, but that's not my point here).

The present system is 'winner-take-all'. After the president has been elected, the votes for the other contenders are thrown out the window. Which effectively means there will only be one other contender. If a left-wing party would participate, it would mainly draw votes from the Democrats. Assuming that party doesn't win (a fair assumption) the only effect will be that the Republicans win. So a left-wing vote actually gets the right wing in power. For the same reason there is no need for the Democrats to have a left wing agenda. Left wing voters have no choice. So the Democrats stay nice and close to the Republicans on their right side, just a bit to the left of them. By the way, I wonder if the system could have turned out such that there were now two left-wing parties.

A parliamentary democracy would not only give all denominations (not just the left but all sorts of mindsets) a vote but it would also force all those different groups to talk to each other. Which would mean the decision to go to war is not that easily made anymore. So my questions are 1) does this reasoning (apart from the last sentence) hold up and 2) is it a good idea? It seems like a good idea to split these two things, so I've made two sub-headers (later addition: that had little effect - the entire discussion moved to the second header after a few postings). DirkvdM 08:29, 19 February 2006 (UTC)[reply]

Is above reasoning correct?[edit]

To some extent, yes. The current system does prevent the extreme left (think American Communist Party) and the extreme right (think Pat Robertson) from gaining much power and also prevents an indecisive government that is incapable of making decisions. I think it's good to prevent those things, however. StuRat 08:59, 19 February 2006 (UTC)[reply]

In practise, in a parliamentary democracy the more extreme parties usually get only a small percentage of the votes (if we forget about the Nazis, but there was something different at work there as well, namely two causes of poverty). Even in the Netherlands at a time when communism was in vogue (after the Russian revolution and after WWII) the Communist Party got only about 10% of the votes, after which they waned and finally dissolved. So yes, extremist parties will probably emerge, but they will remain small and the 'anger' of those voters will be channeled and appeased to some extent. DirkvdM 09:20, 19 February 2006 (UTC)[reply]
I agree that each individual extremist party would only gain a small portion of the votes, but collectively, all of these extremist parties might even manage a majority. StuRat 09:27, 19 February 2006 (UTC)[reply]
For that to work (I mean for them to work together) They'd have to be on the same side of the political spectrum, which sounds very unlikely. There is quite a history of parliamentary history. Can you think of an example of this ever happening?
Most countries that run a proportional type of voting system have a threshold that a party must reach before they gain representation (here in NZ, for instance, a party must either win a constituency or gain 5% of the vote overall). That stops the vast majority of the lunatic fringe parties from getting near parliament. It's worth noting that this restriction was lacking in Germany in the 1930s, but has been in place since the end of WWII. Grutness...wha? 10:53, 19 February 2006 (UTC)[reply]
The Netherlands have no electoral threshold and that has worked out fine so far. By the way, LPF, a party that some consider extremist, had a huge success in the Dutch general election, 2002 that no threshold would have stopped. However, just one year later they were effectively thrown out of the forced coalition and lost loads of votes in the ensuing elections. At the moment they're marginalised to just one seat in the polls. The political message has gotten through and things are almost back to narmal now. DirkvdM 13:09, 19 February 2006 (UTC)[reply]

The main flaw in your reasoning is that you ignore the party primaries. Democrats or Republicans who are dissatisfied with their party's incumbents can mount a primary challenge. If the dissatisfaction is great enough, they can succeed. For example, in 1980 the Republican right disliked the liberal Senator Jacob Javits; the more conservative Al D'Amato beat Javits in the primary and then won the general election. Similarly, in 1992 Carol Moseley Braun mounted a left-wing challenge to a conservative Democrat, Alan Dixon, in the Democratic primary; she won the nomination and the election.

At one time, party bosses exercised much greater control over the selection of nominees. The rules that enforced that control have largely been rescinded or declared unconstitutional, though. Today, for the most part, the bosses can't keep you out. People who aren't in tune with the prevailing views can indeed have an impact on the system, provided that they have the sense to run in the appropriate primary instead of acting like complete nitwits. JamesMLane t c 09:47, 23 February 2006 (UTC)[reply]

Would it be a good idea? (would it bring world peace?)[edit]

There seems to be the assumption buried within this question that the US is the source of all conflict in the world. I don't think so. If we look at the Muslim terrorism issue, they don't just attack the US, but have also attacked the Russians over Chechnya and Dagestan, the French oil tanker Lindburg, etc. Russia and France both opposed the US invasion of Iraq.

There are many other wars that occur with no action from the US, like the Darfur conflict in Sudan and the genocide in Rwanda. In fact, it might well be argued that these wars began or continued because of the failure of governments around the world to act to oppose them. Putting a less decisive government into power in Washington is not the cure needed here, but rather we need more decisive governments elsewhere in the world. The few cases where war has ended due to military intervention have recently been led almost exclusively by the US, such as in Kosovo and the first Gulf War (liberation of Kuwait). It certainly seems to me that Europe could have led, and should have, in their own backyard (the former Yugoslavia), but failed to do so. If this inability to act is the result of a parliamentary system, then this weakness seems like a detriment to all of mankind, to me. StuRat 08:59, 19 February 2006 (UTC)[reply]

I'm not claiming that the US is the only warring nation in the world, but it certainly has been the most prominent one since WWII. Muslim terrorism is a very recent thing, but the fact that it is mostly aimed at the US is an indication of what I mean. If the US hadn't meddled so much in foreign affairs maybe the whole thing wouldn't have arisen. As usual, it is probably caused by a combination of things and if one cause had not been there .... I don't know, I don't have a crystal ball, but it sounds plausible. And be careful of what you wish for. If other contries were as decisive and willing to go to war as the US, they might very well decide to gang together and go to war with the US. DirkvdM 09:14, 19 February 2006 (UTC)[reply]
I disagree that the US has been the most prominent warring nation since WW2. The Soviet Union carried out a massive 45 year occupation of Eastern Europe, including numerous invasions to prevent the formation of democracies. They also funded and supplied the North Vietnamese in the Vietnam War and invaded Afghanistan in 1979 as well as funding/supplying the Arab nations during the Arab/Israeli wars and funding/supplying many other combatants around the world. I'd bet the Muslim attacks on the Russians in that war killed far more than the 5000 or so Americans killed during and since the 9-11 attacks. The Russians continue their war in Chechnya, which gets little world press relative to the Iraq war. Also note that Muslim extremists have attacked the Spanish, British, Australians, India, and even the Netherlands (Theo van Gogh). StuRat 09:52, 19 February 2006 (UTC)[reply]
The US government's foreign policies certainly haven't helped, and have been a major contributing factor to political instability in several parts of the world. Ironically, a country which prides itself on being anti-imperial seems to have a government that goes out of its ay to treat other countries as if it was a colonial power, assuming that its views are the only possible right ones and ignoring local wishes. However, it is far from the only culprit, and it may be a common factor in any country with enough clout to be a superpower. Mix a bit of religious fuundamentalism into the mix and you've got a lot of different factors in any upswelling of military belligerance. BTW, I'd argue the point about Theo van Gogh - his killing was a one-off assassination, related to his own personal work and unrelated to the views of the people or government of the Netherlands. It certainly wasn't of the same sort as the attacks on the US and UK. Grutness...wha? 11:16, 19 February 2006 (UTC)[reply]
Right, dont' give me that van Gogh bull. That was just one extremist killing another over some insults. That's not terrorism. And even terrorism (ie war without the backing of a country) isn't the issue either. The issue is if a certain form of government (eg presidency vs parliament) is more likely to get a country to go to war. For example, do you think Russia would still be so much at war if it had a functioning parliament in stead of a new dictator? From the Politics of Russia article: "A new constitution, creating a strong presidency, was approved by referendum in December 1993." Sounds like that's when things went wrong. With the strong presidency. Actually, Russia may be a better example than the US to show how bad the idea of a presidency is. First it was a complete nut, Yeltsin, and now a dangerous one. What will follow?
To generalise this a bit more, it would be interresting to make a list of countries by type of government (centralisation of power in more or fewer hands) and see how bellicose the countries in the various categories are. DirkvdM 13:27, 19 February 2006 (UTC)[reply]
I believe the assassination of Theo van Gogh was in intentional attempt to influence the political climate in the Netherlands by murdering those who present an unflattering view of Muslims. To lump him and his murderer together as "extremists" is wrong, he didn't kill anyone. Are you arguing that people in the Netherlands who express views critical of Islam should be equated with murderers ? Terrorist have tried to gain control via a campaign of assassinations in the past, although this is not a typical method of al-Queda. Your other recent political assassination (Pim Fortuyn), although not committed by a Muslim, makes me wonder if your political institutions are somehow responsible. StuRat 04:20, 20 February 2006 (UTC)[reply]
One doesn't have to be a killer to be an extremist. Now you are lumping things together. :) And the murder of van Gogh wasn't political. The murder of Fortuyn is usually not regarded as terrorism, which is odd since murdering a politician for his views has much stronger terrorist effects. The fact that the murderer wasn't a muslim probabluy has something to do with this. (Mind you, I'm not defending Islam, just attacking bias. I'd say "fuck all religions", which would make me an extremist as well, just a very generalised one :) ) How do you figure Dutch political institutions were responsible for these murders? DirkvdM 10:30, 23 February 2006 (UTC)[reply]
The U.S. system is actually designed to disperse power, not concentrate it. The Constitution says only Congress can declare war. Although Congress doesn't really declare war anymore (it hasn't since WWII), it still must approve the deployment of troops and the military budget. Congress agreed by wide margins in 2002 to give the president authority to invade Iraq (see Authorization for Use of Military Force Against Iraq Resolution of 2002). In a system like that of the UK, Bush would only have had to convince his Cabinet. -- Mwalcoff 18:35, 19 February 2006 (UTC)[reply]

If you're looking for a structural reason why, at least to western europeans, US politics seems extreme (some would say "resolute", your mileage may vary), I don't think you can really blame the presidential system. Indeed, the distribution of powers between the legislature and the presidency should, at least in theory, make the president weaker - and indeed, in domestic matters where his writ is particularly circumscribed the president really is comparatively weak. If you want a political-structure explanation for US policy (and frankly I'd be wary of the need for one, as there's plenty of evidence that most US citizens are perfectly happy with most US foreign policies) I'd look to the way electoral boundaries are drawn. In the UK, by way of a contrasting example, boundaries for the Westminster parliament are drawn up by the Boundary Commission (United Kingdom), a largely non-partisan quango. This tries to make constituencies roughly map to communities, and in general its findings aren't very controversial, and there are rarely accusations that it's engaged in gerrymandering. As a consequence (yes, I'm asserting that its a consequence, not just a coincidence) there were members from 12 different parties elected to the westminster parliament. Of those elected, many are either politically centrist (often those in fairly marginal constituencies) or (e.g. Dennis Skinner) politically independent minded, able to take a nuanced, defiant, or contrarian position. Electoral boundaries in the US are (generally) drawn by highly partisan committees, and shamefaced gerrymandering common. This has two (in my view deleterious) effects on US politics: firstly it largely squeezes other parties out of contention (the house and senate each have only one independent, so an essentially perfect duopoly prevails). Secondly the large number of safe seats that this redistricting produces means there's no need to put a moderate up for election (to sway floating voters and the soft flank of the opposition). I'd guess (it'd be an interesting theory to try out) that if boundaries were drawn on roughly geographical lines (or by some other means, even a random scheme) that you'd see both a greater variety of parties and a greater number of rebellious moderates. A system of proportional representation would (probably) enhance this effect. You likely would see one or several parties of the genuine left, but you'd see one or several parties to the right of the republicans (including, perhaps, right-libertarians and a close-the-borders-splendid-isolation party, and quite possibly an outright nationalist/fascist party). I wouldn't be at all surprised if you didn't get a conservative religious party, and parties representing the larger ethnic groups (California's Mexicans, Florida's Cubans, and urban African-Americans are all rather taken for granted by the Republicrat duopoly, and some will welcome the opportunity to try for enhanced negociating muscle). I absolutely don't think this would result in a wholesale shift of US politics to the left (or the right), but you'd see a marked change in its character; even then, the new congress would seem a lot more like the Knesset than Westminster. -- Finlay McWalter | Talk 18:49, 19 February 2006 (UTC)[reply]

Hold on, I've made a mistake. I started focusing on the presidency system, when the real issue is the winner-take-all effect. Which, of course, one has with a president. But I once understood that the same goes for congress (or was that the senate specifically?), but I now see in the relevant articles that it has representatives from both parties. But now I don't understand why the US has only two parties in congress when it does not use a first-past-the-post system. I suppose I'll have to read the US politics articles (should have done so before I asked this :) ). Thanks for the word duopoly. That led to the relevant articles Two-party system and Duverger's law, which in turn led to first past the post.
By the way, I wasn't suggesting that there should be a shift to the left in the US. Just more parties that have to form coalitions, which would mean that they would have to talk to each other and find ways to accommodate the other parties. That would not just have an effect in the number of representing parties but also in the way of thinking. Not having to listen to others can cause tunnel vision (if that's the word for it).
And the UK may also have a district system, with in the last elections the result that the liberal democrats got 22% of the votes but only 10% of the seats, which is highly unrepresentative, but at least they got some seats (or rather bums in them :) ). DirkvdM 20:32, 19 February 2006 (UTC)[reply]
The US does have a first-past-the-post system. Whoever gets the most votes for a House or Senate seat wins. Unlike in Canada and the UK, there is not really much of a push for proportional representation in the US.
Remember that in the US, parties don't matter nearly as much as they do in most other countries. Every vote in Congress -- with the exception of the vote to elect the speaker of the House -- is a free vote. In order to get approval for something, you have to convince 218 House members and 50 or 60 senators individually. Just because the leadership of the party wants something passed doesn't mean they necessarily can count on their caucus's members voting for it. -- Mwalcoff 21:02, 19 February 2006 (UTC)[reply]
What I meant was that I had understood that members in the congress or senate were all of one party. But I think I understand now. There's a first past the post at state level, with each state delivering their representative, so you do get a mix of the two parties, but at the same time you have only two parties. This feels rather compicated for someone used to a simple one man one vote representative democracy. Sort of like trying to understand the imperial unit system. I'll ponder on this some more tomorrow when I'm awake (bedtime now). DirkvdM 21:51, 19 February 2006 (UTC)[reply]
Members of the House are elected from Districts. It's winner take all per district, not per state. The Senate is per state, however. User:Zoe|(talk) 00:12, 20 February 2006 (UTC)[reply]
I think its just a consequence of being "top nation" to have an active military policy. If a country has that power, it will use it. Britain did it, now its the US's turn. When you are top nation, the country gets into a sort of mindset that says "Our way is best" which means it wants to spread its values/system abroad. Whether that is a heroic effort or just a waste of resources that could be better used at home is open to question. Im not convinced any change to the parliamentary rules would change that impulse. Jameswilson 02:44, 20 February 2006 (UTC)[reply]

Note that you can also force different parties to compromise by requiring a supermajority. The minority party can stop a bill from passing currently using a filibuster, so long as the opposing party can't muster a 60% vote to invoke cloture. It's a really clumsy mechanism, but does give the minority party some power. The Republicans were threatening to eliminate the filibuster, but seem to have decided against it, perhaps sensing that they will likely be the minority party soon. Constitutional amendments require a 2/3 majority, so those definitely require the approval of both parties. StuRat 03:46, 20 February 2006 (UTC)[reply]

Hold on again, I'm awake again now and realise that if decisions (apart from changing the constitution) require a simple majority and you have only two parties then whichever party has the majority calls all the shots, right? So there might as well be just that one party, unless enough members of that party decide to 'vote with the other party'. Does that happen a lot? And if all members decide for themselves, then what is the purpose of having parties in the first place? This can be asked of the politics of most countries, I suppose.
As for a 2/3 majority to change the constitution, with just two parties it isn't inconceivable that one of them gets such a large majority. Unless the other party sees it coming and adapts its policy to win votes back. That would make for very opportunistic politics. And again, something similar seems to be happening in the Netherlands right now. DirkvdM 07:47, 20 February 2006 (UTC)[reply]
Is there? What are you referring to?
Within American politics, one of the most important questions (in terms of power distributions) always seems to be which party controls the House, and whether its the same as the Presidents. If not, its always harder to govern.
But returning to your original question, you can only check whether this is the case if you compare with other cases. Certainly, if you compare with Proportional representation it certainly seems to work (although you should note that the Dutch have pacified their system since 1917 (no nl article on that yet! Should get to work on that) and are traditionally somewhat of an exception). But try comparing it to the UK, which has a winner-takes-all system. If your idea were true, would then not the UK also be a war-mongering nation?
My two cents is that the electoral system does have something to do with it. But more importantly is that hegemony does strange things to a country. It tends to think that it can - indeed even has the repsonsibility to - "protect" the world. This goes doubly so for a Hyperpower as the US is now. And of course, usually it does (Pax Romana, Pax Britannica, Pax Americana). Until they overplay their hand. The Minister of War (Peace) 23:13, 20 February 2006 (UTC)[reply]
In Congress, just because a party controls at least half of the seats doesn't mean the party leadership can pass anything it wants. Individual members can (and often do) vote however they want, regardless of party, and often other considerations come ahead of party ones. For example, if your party leadership is pushing a bill that negatively affects the zinc industry, and you come from a leading zinc-mining state, you would likely vote against the legislation. This would be completely expected. Also, control of the legislature is far more dispersed than it is in a parliamentary system. The really important people are the chairmen and chairwomen of the committees that deal with the issue at stake, not just the party bigwigs. And in the Senate, it's easy for a determined minority of senators to block legislation, even if a slim majority of senators supports it.
You'll rarely see a case in an American legislature where a monolithic group controls the body entirely and passes any legislation its leaders want. Even when a single party controls a large majority of seats, you tend to see fractures between factions within the party. -- Mwalcoff 00:45, 21 February 2006 (UTC)[reply]
Minister of War (what an appropriate name here :) ), in the Netherlands, the major left-wing party, PvdA, is said to have moved too far to the right, probably as a result of the recent success of right wing parties. As a result, voters have started to move to more left wing parties (Green Left and Socialistische Partij, resulting in a possible majority for the three left wing parties in the elections next year. What do you mean by the Dutch systmen hav0ng been pacified in 1917? Is the change from a district system to direct elections called pacification? (Can't find an article on that). The difference between the UK and the US is that the UK have a parliament with many parties who have to talk to each other to get things done. So it's not in essence winner-take-all. It just happens that Labour got 55% of the seats during the last elections (despite the fact that they only gfot 35% of the votes, so that system is also seriously flawed). And note that the UK is the other nation that was eager to invade Iraq. And lastly, I think the power of the US is something the rest of the world needs to be protected against.
Mwalcoff, I already understood that members of Congress (why is the article usually dropped here and not with the Senate?) can 'vote with the other side', but is that common or does it just happen once and again? Put differntly, what comes first for them, their legislature or their party? If there are fractures within parties, then wh don't they split off to start a new party, as is common in the Netherlands? Then again, I relealise the answer lies in the original question of this thread - they would just lose all power. So could one say that the answer of the two-party system to its rigidity is that members are less loyal to their parties? That does seem to make some sense. DirkvdM 11:52, 21 February 2006 (UTC)[reply]
Party members quite regularly vote at odds with their party, for the reasons listed previously. Their first loyalty is to getting reelected, and that means voting however their constituents want. Also, a few politicians accidentally snuck in who actually have personal convictions, and vote that way, like John McCain. StuRat 05:24, 22 February 2006 (UTC)[reply]
There have been a few times in which sitting legislators have left their parties to sit as independents because they are so upset with the direction of the party. But it is very rare. There is no reason to leave the party when the party is usually quite willing to tolerate different views. For example, Connie Morella served in the House as a Republican for years despite having somewhat liberal views. The Republicans continued to support her because they knew a more-conservative candidate wouldn't get elected in her district, and because they knew Morella would vote to continue Republican control of the House. Had Morella left the Republian caucus (and not switched to the Democrats), she would have lost her important committee seats, since those are distributed between the two caucuses. Also, she would have lost the benefit of Republican national campaign dollars for her re-election effort. So it makes far more sense to pick one of the two parties and stick with it -- despite any reservations a legislators has about the party's views -- than to try to go out on your own.
When a large group of legislators is dissatisfied with the direction of the party, they generally try to change it from within rather than bolt the party. For example, "centrist" Democrats formed the Democratic Leadership Council in the mid-1980s to try to move the party to the right. The leftish wing of the party later formed the Congressional Progressive Caucus to try to avoid such a change of direction. Today, the centrist and left-of-center wings of the party sometimes fight in primary elections and battles for party leadership positions. But for one of the wings to leave the party would simply ensure more victories to the Republicans under the winner-take-all system. See, for example, the [[|United States presidential election, 1912|1912 presidential election]], which took place after Republican Theodore Roosevelt had left the party to run as a Progressive. The split in the Republican ranks allowed Woodrow Wilson to become president with only 42% of the popular vote.
Here's an interesting tidbit for this discussion: Abraham Lincoln, Woodrow Wilson, Richard Nixon and George W. Bush were all elected largely due to a split opposition; none of them won 50% of the popular vote the first time around. Lincoln, Wilson and Bush each led the U.S. into a war, while Nixon expanded the Vietnam War. -- Mwalcoff 18:14, 26 February 2006 (UTC)[reply]

How can a minority stop legislation if a simple majority is needed?

Let me repeat: The minority party can stop a bill from passing currently using a filibuster, so long as the opposing party can't muster a 60% vote to invoke cloture. It's a really clumsy mechanism, but does give the minority party some power. StuRat 05:24, 22 February 2006 (UTC)[reply]
Ah, I'm sorry, I forgot to follow those links. My loss. Turns out fillibuster is originally Dutch. And indeed we had a prime minister like that in the 1970's, Joop den Uyl, who needed very little sleep and kept on talking until the others decided to give him what he wanted so they could go home. Not the same, but similar. DirkvdM 07:16, 22 February 2006 (UTC)[reply]

Who is she?[edit]

Another Wikipedian and I are minorly puzzled as to commons.wikimedia.org/wiki/Image:SF_Chinese_new_year_p1060726.jpg who this woman is at a Chinese New Year celebration in San Francisco. We both think it may be Dianne Feinstein but we aren't completely sure; if it is, we're going to add the picture to her article, but I don't want to run the risk of having the wrong picture on her article. Can anyone else give insight? Mike H. That's hot 10:40, 19 February 2006 (UTC)[reply]

I don't know who it is, but it is certainly not Dianne Feinstein! - Nunh-huh 11:18, 19 February 2006 (UTC)[reply]
I'm pretty sure that's Nancy Pelosi. --jpgordon∇∆∇∆ 17:12, 19 February 2006 (UTC)[reply]
What would she be doing there? At least with Feinstein there is a resemblance and there is a reason for her to be in San Francisco. My bad, I confused Pelosi with someone else. I didn't know she was a representative from California; somehow I thought New York. Mike H. That's hot 02:41, 20 February 2006 (UTC)[reply]
I thought it looked more like Feinstein first, actually. But there's a picture www.house.gov/pelosi/press/releases/Jan06/lunar.html here of Pelosi at the New Year parade in a red convertible, and that had me convinced for a while, but then I noticed the lady in the picture isn't wearing the same clothes as Pelosi. Her jacket has lapels and pin on it and she's wearing red pants. Pelosi is wearing black pants. Strange. --BluePlatypus 16:13, 20 February 2006 (UTC)[reply]
Ah wait! That's the 2005 parade. No wonder she's not wearing the same clothes. --BluePlatypus 16:15, 20 February 2006 (UTC)[reply]
Well, she still has the black skirt -- it looks like she's got a lap blanket (it's not exactly warm in SF in January.) --jpgordon∇∆∇∆ 18:23, 20 February 2006 (UTC)[reply]

Politics of Nepal - the current issue[edit]

What are the Maoists in Nepal upto?

Thanks. I will also see the article. I hope it is not biased in favor of the government.

I hope it's not biased against it. Superm401 - Talk 07:09, 20 February 2006 (UTC)[reply]

What does Purchasing power parity mean? If that figure for Nepal is at US $ 1500 per year, how does it compare to our official per capita income of around $300? [Should this question be in the Newbie section?]

Most things are much cheaper to buy in Nepal - on average five times cheaper than in the US so 300 x 5 = 1500. If you look at international statistics you always have to make sure which figure is being used, GDP or GDP PPP. If you want to measure economic power use GDP, if you want to compare living standards use GDP PPP. Jameswilson 23:38, 20 February 2006 (UTC)[reply]

Thanks, it does make sense. I looked up the PPP article but it had some tag thing on the 'disputed' or something Anyways, thanks again ... (My IP adress is not a permanent one.)

I hope I am not overdoing this thing but what do you (the wikipedians with an interest in Nepal) think the result of the current political standoff will be? I would use four tildes ~ ~ ~ ~ but as I said earlier (My IP adress is not a permanent one.) 202.161.131.69 18:30, 23 February 2006 (UTC) KH[reply]

I do not own a trademark to the phrase "(My IP adress is not a permanent one.)" but I intend that to be my signature line when I become a 'full' user.

I know this is no place to take surveys or speculate. I just wanted to know what you think will happen in the near future in the politics of Nepal? What should happen? Thank you for your vision.

[Those who have not read the articles Nepal, politics of Nepal or are not well-versed with contemporary Nepal are exempt from answering.] (My IP adress is not a permanent one.) 202.161.131.69 19:00, 25 February 2006 (UTC)[reply]

I'm not qualified to speculate on your question, but you can http://en.wikipedia.org/w/index.php?title=Special:Userlogin&type=signup get a permanent user account now. No personal information is required. Superm401 - Talk 22:23, 26 February 2006 (UTC)[reply]

Kings of Malta[edit]

Which kings knights fought in the great siege of malta?

John Bridges

The Knights Hospitaller were an independent military group although Malta was nominally a fief of the King of Sicily at the time of the Siege of Malta (1565). Rmhermen 20:32, 19 February 2006 (UTC)[reply]

1972 Virginia House of Delegates Campaign[edit]

I would like to know the vote count for the 1972 Va. House of Delegates Campaign between Hal Lackey and A.L. Philpott. Thanks.

The information on the legis.state.va.us/ Virginia General Assembly web page goes back only to 1994. This means you will need to go to a library, preferably the state library, to get the information you seek. --Halcatalyst 16:38, 22 February 2006 (UTC)[reply]

96-Year-Old GED Graduate[edit]

What is the name of the 96-year-old farmer in Georgia, U.S. who recently received a high school diploma? I think the name I heard was Thurman Barnes, but I haven't seen its spelling and did not find an article on him.

--66.81.192.26 15:42, 19 February 2006 (UTC)[reply]

Here you go -- www.westgatech.edu/News/news2005.htm#barnes --LarryMac 18:30, 19 February 2006 (UTC)[reply]

Architectural firms during the depression[edit]

Hi, I'm new to this, so please excuse any errors. Also I'm rather computer illiterate - don't know a mouse from a megahertz.

I would assume that during the Depression architectural firms had to close for lack of business. Are there any statistics regarding this? Is there a book I could read to find out this information? Any help is greatly appreciated. Thank you. Sue Grawey

I don't have any statistics on this (if you find them tell us or put them in an article). But I assume you are assuming that no investments were made because of the depression and therefore no construction work was done. However, one economic policy was to actually invest to get the economy back up it's feet. Governments did that (the Dutch government to some extent), but it was also a good idea for businesses. At least that's what my grandfather thought. During a depression labour is cheap, so construction necessarily also (being rather labour intensive). So he built what was dubbed the Glass Palace in Heerlen, a big, highly modern department store. A visiting minister declared him nuts for undertaking something that big during a depression, but he was put in the right because his business continued to flourish (and expand further) after that. Actually, this makes so much sense to me that I cannot understand why some people think differntly. A depression is the best time to invest. And constuction is a fine example for that. Actually, Bauhaus did rather well during the depression, I believe, until Hitler decided their architecture was degenerate or something. DirkvdM 20:44, 19 February 2006 (UTC)[reply]
The exact opposite is easier to document. In New York, both the Empire State Building and the Chrysler Building were built during the depression. If you want to go to architecture of things other than buildings, there is the Hoover Dam and Mount Rushmore. So, it is certainly true that some architectural firms closed, but others continued to do very well. --Kainaw (talk) 20:48, 19 February 2006 (UTC)[reply]
Note that some depression era New Deal agencies created in the US under FDR, such as the Works Progress Administration and the Tennessee Valley Authority, used government money to fund construction, in a deliberate attempt to end the depression, or at least reduce it's severity. StuRat 02:37, 20 February 2006 (UTC)[reply]
You might be interested in the architecture timeline for buildings constructed during this period. Warofdreams talk 04:22, 20 February 2006 (UTC)[reply]

Seeking information on a copy of The Storm[edit]

I have a (24" X 36") oil painting copy of The Storm by Pierre Auguste Cot. I can not find an Artist name. However, there is a small brass plaque on the frame with the name A Vateriano. Were there many oil copies and how can I find more information about tnis copy?

There are several coats of varnish over the painting which seem to make the painting very dark. It is in need os restoration and I wonder if it worth doing.

Hoping you can help me with this request. Thank you for your time.

Gene Pisciotta

Wikipedia has a short article on Pierre Auguste Cot, which links to the much more scholarly and thorough Academic art. You might find some references there which will help you with your search. --Halcatalyst 16:31, 22 February 2006 (UTC)[reply]

Bach's Missing Partita?[edit]

I can't find the names of Bach's G Major Partita's movements. It was written for the viola, any takers?

I haven't found any reference to a partita for viola in G major. But is it possible you mean the Partita for solo violin in G minor, BWV 1001? The movements are: 1. Adagio 2. Fuga (Allegro) 3. Siciliana 4. Presto. JackofOz 01:40, 20 February 2006 (UTC)[reply]

what problems did spain and poland have with the IGC?[edit]

Can you please suitly emphazi which IGC you are referring to? Grutness...wha? 01:55, 20 February 2006 (UTC)[reply]

February 20[edit]

Composer(s) of "Unknown" Tunes[edit]

In some years kno, I have heard these melodies on TV and Cellphones without the knowledge of the name of the composers and the name of the tunes. I would be forever grateful if someone could help me find out the name of these melodies and their composers. I have tried to write out a part of the tune from my memory if that might be to some help.

Melody 1

Melody 2 File:SONG02.mid

Thank you for you time. I will forever be grateful. -(Funper 20:57, 20 February 2006 (UTC)).[reply]

1 - Beats me, but I'd like to know as well
2 - Johann Sebastian Bach's Toccata and Fugue in D minor, BWV 565Ҡiff 04:26, 21 February 2006 (UTC)[reply]

The mysterious first song is The Volga Boatmen's Song (Эй, ухнем - Ey, ukhnem), a Russian folk song that originated with these guys. --IvanP/(болтай) 14:38, 23 February 2006 (UTC)[reply]

Your contributions will never be forgotten. Thank you once again. -83.250.90.119 20:11, 23 February 2006 (UTC)[reply]

Counter arguments[edit]

Are there any 'conclusive' or somewhat conclusive arguments against philosphical skepticism?...I think Wittgenstein gave one but I don't understand it.--Cosmic girl 01:23, 20 February 2006 (UTC)[reply]

Wittgenstein's point was that if you doubt everything, the concept of doubt is no longer meaningful. "If you tried to doubt everything you would not get as far as doubting anything. The game of doubting itself presupposes certainty." On Certainty (115) and "A doubt without an end is not even a doubt" On Certainty (625). David Sneek 08:17, 20 February 2006 (UTC)[reply]
You mean like this? Put it in one statement "Everything is uncertain.", now ask if that statement applies to itself. If yes: Then everything may not be certain. If no: Then everything is not uncertain and you'll have a hard time explaining why other things cannot be as well. This is the paradox of scepticism, and also why scepticism is worthless, in the opinion of at least one philosophy professor I know. You can destroy any argument through large-scale skepticism, but it applies to everything. It's a philosophical nuclear bomb; it destroys everything, including itself, leaving nothing but rubble on which you can't build anything. --BluePlatypus 16:02, 20 February 2006 (UTC)[reply]
Humbug. That's why everyone should be a nihilist. And also not a nihilist simultaneously. Philosophy may be a footnote to Plato, but Pyrrho's the one who actually had it all figured out. (And not. Simultaneously. Whee!) --George 06:52, 21 February 2006 (UTC)[reply]

thanx! :).--Cosmic girl 18:32, 20 February 2006 (UTC)[reply]

A philosophy professor told us in one of the first classes that philosophy is dangerous. When I started thinking about how nothing is certain I understood what he meant. If you reason through that everything is uncertain, you might find yourself in a big scary void. You need some basis to keep your sanity. Alas, for a lot of people that basis is religion. You want to believe in something, so you're tempted to believe in anything, whatever is close at hand. I suppose that's why, after havong come up with his 'cogito ergo sum', Descartes continued with religious nonsense, because he couldn't find any other certainties and apparantly didn't want to accept that there simply weren't any. DirkvdM 12:03, 21 February 2006 (UTC)[reply]

Interesting! did your professor actually say :everything is uncertain??? awesome,that has never happened to me...haha...and I think you are right about needing some basis to keep your sanity...--Cosmic girl 22:54, 23 February 2006 (UTC)[reply]

Religious faith is by definition something about which there is no certainty. If what religions espouse could be proven, no faith would be necessary, and it would be called science. This is not an argument for religion; but equally, religion is not rendered nonsense merely by its lack of proof. JackofOz 12:32, 21 February 2006 (UTC)[reply]
This definition limits religion to a discipline that covers ontological and epistemological issues. Most religious people would contend that an accurate definition of religion would need to include, at a minimum, sociological issues as well. -LambaJan 03:11, 23 February 2006 (UTC)[reply]

UK co.'s and the S. Asian MKT[edit]

What are some good business journals or newspapers in S. Asia that could help with research I;m doing on UK companies trading in the emerging S.Asian market?

I don't know whether it's what you're looking for, but the Far Eastern Economic Review is the best known business news magazine focussing on Asia generally. --Robert Merkel 05:33, 21 February 2006 (UTC)[reply]

Roman / Classical Age Protectorates[edit]

How did the Roman Empire manage to make defeated states productive members of the Empire? Surely there was resentment on the part of the conquered. What incentives were employed to keep them from rebelling once the armies had moved on? --Demonesque 12:53, 20 February 2006 (UTC)[reply]

Yeah, apart from the sanitation, the medicine, education, wine, public order, irrigation, roads, a fresh water system, and public health, what have the Romans ever done for us? But seriously, the Romans were not employing incentives, they had garrisons. They would usually find enough collaborators to run things, and if this didn't go as planned they used force. 213.201.189.242 13:14, 20 February 2006 (UTC)[reply]
  • Well, first they had garrisons of troops there. But most importantly, there wasn't much incentive for people to rebel. Romans allowed people their own rulers, their own religion and customs, and their own laws. (within certain bounds, of course) There was little reason to rebel against the Romans simply because they were foreign rulers, because nationalism and the idea of nation-states didn't really exist yet. A lot if not most of the peoples conquered by the Romans had already been under foreign rule previously, by the Greeks or Persians or Carthaginians, for instance. --BluePlatypus 13:55, 20 February 2006 (UTC)[reply]

They allowed the local elites to keep their privileges and encouraged them gradually to become Roman in their way of life. As that happened, some later Roman Emperors were even not from Italy. The peasants just carried on as before. Jameswilson 23:46, 20 February 2006 (UTC)[reply]

Most soldiers were at the borders, with only a few in the previously conquered areas. The advantages that 213 mentions would take time to develop. But the expansion of the empire also took centuries. So I imagine they conquered a bit, let things settle and then went on. Also, peoples just across the border would see what advantages being in the Roman empire brought, so when the Romans came there may not have been so much resistance. And indeed the freedom of religion and other customs will have been very important. Once you were in the Roman empire you were even considered a Roman, not a slave or other lesser person (unless you had resisted, which would have resulted in being enslaved). People from the provinces could even rise to high positions and some even became emperor, including a black African. After that Europe has never seen a black leader (as far as I know), so in a sense the Romans were ahead of us.
Alexander the Great, however, conquered an area of about the same size in just a few decades. And indeed I asked a similar question a few months ago. As is often the case, I supplied many of the answers to my own question myself. :) DirkvdM 13:56, 21 February 2006 (UTC)[reply]
Another important fact was that the Roman empire wasn't a homogenous block, different parts of the Empire (even at the level of individual towns & villages) had different levels of rights & responsibilities (to have a market, to have certain holiday days, etc, etc). The more loyal & trusted a particular town the more rights it got & if it rebelled they restricted the rights of its citizens & increased its responsibilities. So there was more incentive to behave & less to rebel (on top of the factthat if you did rebel the Romans would try to kill you). AllanHainey 15:55, 21 February 2006 (UTC)[reply]

Proper use of pardon me and excuse me?[edit]

When is it appropriate to use pardon me and excuse me?

As in, "What is the difference between the two phrases"? The answer to that is just your personal preference. I use excuse me but other people might say pardon me which both mean the same thing. If you mean what situations would you use the phrases in, then it would be any time that you need someone else to inconvienence themselves for your benefit, like moving to the side in order for you to pass through a passageway or aisle. Or when you do something that is thought to be slightly socially improper like when you yawn while holding a conversation with someone. There are just too many instances to list them all. Does this answer your question though? Dismas|(talk) 13:51, 20 February 2006 (UTC)[reply]
Every use I can think of is interchangeable. At first, I thought "I beg your pardon" had no "excuse" equivalent, but then I realized it is "Excuse you". I think it is just like the difference between trousers and pants. One is primarily British and the other is primarily American - but, they aren't exclusively either one. --Kainaw (talk) 15:08, 20 February 2006 (UTC)[reply]
There is one important case where you should always use pardon me and not excuse me, and that is when you are trying to persuade the governor not to send you to the electric chair. DJ Clayworth 18:27, 20 February 2006 (UTC)[reply]

I always thought that pardon was more of a british or more proper way of speaking. Normally I just use excuse me.--God of War 05:25, 21 February 2006 (UTC)[reply]

"Pardon me" would be considered formal and old fashioned in the UK. I normally only use "excuse me", though "pardon?" is a common response in the case of not having heard what someone has said. --Shantavira 10:50, 21 February 2006 (UTC)[reply]
There's also a not so subtle distinction in that case. If you simply didn't quite understand what they said, you might say "pardon me". That would be the "neutral" case. However if you thought they said something offensive, but you wanted to be certain before launching into a broadside, you might be more likely to say "excuse me" (with appropriate inflexion). JackofOz 11:23, 21 February 2006 (UTC)[reply]
"Pardon" is non-U. —Blotwell 08:01, 26 February 2006 (UTC)[reply]

Widening of a Country's Tax Base.[edit]

What are some of the significant measures that a third world country could undertake to widen its tax base?

Education is one. But that requires that people do their own homework. --BluePlatypus 16:17, 20 February 2006 (UTC)[reply]
Hahaha! The Minister of War (Peace) 22:58, 20 February 2006 (UTC)[reply]
Some peasant-farmers may earn more than they declare (if they declare at all), so carefully-chosen taxes on property, chemicals, cars, petrol, consumer goods, etc should enable the government to get some money out of the better-off farmers without hurting the dirt-poor subsistence farmers. Jameswilson 00:01, 21 February 2006 (UTC)[reply]
See informal economy and underground economy. On the business tax base side, as a very rough rule of thumb, the incentives to register businesses (legal protections, codified property rights) ought to outweigh disincentives (1001 forms to fill out, burdensome business registration fees). I read recently that registering a business in Peru costs as much as two years income. Naturally, many people will simply take their businesses underground in such cases. Countries also need to pay fairly/supervise properly their civil servants to avoid corruption. Marskell 11:15, 21 February 2006 (UTC)[reply]
Tax the rich. Typically, in third world countries there is little in the way of a middle class (which may very well be their biggest problem in the long run because it's the middle classes that keep an economy healthy, not the rich). The vast majority are poor. And you can't pluck a bald chicken (Dutch expression). Any general taxes will affect everyone and thus kill the poor and weaken the middle classes (which would kill the economy). So all that is left is to tax the rich. Which of course will not happen if those in power are the rich. In which case the country is doomed forever. The only solution I can think of is democracy, but how does one bring that about. External intervention is generally not received well (think of Iraq). Sometimes a king will give up power, as the king of Spain did after Franco left the country to him. But that doesn't quite happen often enough. So a revolution would make sense. And if that happens a mixed external force (ie UN forces) could be welcomed and will probably be necessary to avoid a different kind of chaos. Sometimes a country manages to build up something after a revolution, though, like in Cuba, although people aren't too well off there, but at least they're not as poor anymore. Which brings me back to taxation. The accumulation of wealth is not allowed there, so Cuba sort of does what I propose, except they nip it in the bud and consequently don't have any rich. Effectively, the State has all the wealth, so there is less of a need for taxation. DirkvdM 14:13, 21 February 2006 (UTC)[reply]
There's a limit to how much you can tax the rich before they're not rich and won't bother to produce anything anymore or they move somewhere else. But of course you knew that. Better is to invest in education and job training and make it easier to start small businesses. Microfinance schemes and cooperatives that bring local products more directly to market have been extremely successful in India and other places in lifting people out of poverty. More investment in those types of things and infrastructure does much more to alleviate poverty than welfare systems. Unfailingly welfare systems increase unemployment and increase those receiving welfare. More specifically to the question, is once wealth creation is promoted, efficient tax collection and enforcement schemes are needed. Better enforcement of tax collection with simpler tax systems would likely increase tax collections much more than increasing tax rates. Marskell makes very good points too. - Taxman Talk 14:55, 21 February 2006 (UTC)[reply]
Spoken like a true taxman. :) I agree with most of what you say, except for two things. You claim that welfare systems increase unemployment. And you don't just make a theoretical claim. You say 'unfailingly'. Not only is that a way too strong statement to be true. It also suggests that you have statistics to back this up. Can you produce these? They should show a huge unemployment for countries like the Netherlands and Sweden. Or let me produce some statistics for you. What about List of countries by unemployment rate. Do we see the typical socialist countries (Scandinavia) at the bottom? Hell no, Africa has gathered there, so if welfare is the determining factor it's the opposite of what you say. On a list of 159, the Netherlands and Sweden are at 39 and 42 (and Cuba is even at 13, but the situation is just too different there to make this a fair comparison).
In your first sentence you assume that the rich are the ones who produce stuff and you seem to find this so natural that you assume I would agree. Of course the manufacturers are those who do the actual producing. Aka the workers (by which I also mean white collar workers and researchers and whatever). These have to be managed to coordinate their efforts. In western tradition it just happens that the managers of companies are usually the owners, who happen to be the rich. But there is no natural reason why it should be that way. Take Cuba again. No rich there (well, a few maybe, but not like 10% of the population or something) and for a third world country it is doing quite well. You do have a point though, in that if you don't take all away form them in one fell swoop you have to walk a thin line and let them profit from what you do with the money too, like nicer roads to drive their cars on (good infrastructure is essential for the development of a country). Take the Netherlands. Very high taxes and a few rich have moved abroad. But only a few and where did they move? To Belgium, just across the border. Literally, just a few km a way from their beloved home country because they don't really want to leave. DirkvdM 19:54, 21 February 2006 (UTC)[reply]
Well if I was the eponymous Taxman from the song my answer would have been the opposite wouldn't it? :) I probably should have skipped that point if I wasn't going to elaborate on it because it wasn't nearly my most important or strongest one. My point on welfare was a ceteris paribus one, where a country with a given level of resources adds a richer benefit to a welfare system, more people will utilize it. You can't compare it to countries with vastly less resources and infrastructure. Statistics won't necessarily work because there is no ceteris paribus in the real world. So yes, I was probably a little overenthusiastic and should have said overall, rather than in every case. While we're at looking at unemployment rates though, why not also look at tax rates and growth rates? And you should read my first sentence more literally. I wasn't saying the rich are the only ones that produce, just that there's a limit to how much you can tax them. My overall point stands that raising the tax rate on the rich won't work without the other more important elements of smart investment, etc that have already been mentioned. If you do those right you can afford to increase the taxes on the poor too if you want. Take a poor person that makes $1 a day and give them a microloan that allows them to invest in what they need to quintuple their output and they can afford a 50% tax rate and still be better off. - Taxman Talk 15:32, 22 February 2006 (UTC)[reply]
Yes, this assumption that its the rich who create the wealth is a neo-liberal point of view, not an undisputed fact. Its especially dubious in countries where the rich elite have mostly inherited their wealth and make laws which keep the entrepreneurs in their place. That led to political crises in Britain in the 19th century (Reform Act, Corn Laws, etc). Today some Latin American countries, for example, are in the same position. You need to break the stranglehold on power of the traditional landowning elite (and taxation is one way of doing it) before other groups can progress. Microfinance initiatives are fine but those who succeed come up against the same brick wall. This is not a problem that the US, Canada, etc, ever had (though I supoose you could argue the Civil War had elements of that). But remember that in the context of Britain for example, new right leaders like Mrs Thatcher also had little time for the traditional upper classes, both in finance and politics/the civil service, regarding their influence as an obstacle to her dream of an entrepreneurial Britain. Jameswilson 00:12, 22 February 2006 (UTC)[reply]
Yes, breaking obstacles to entrepreneurship is key, but I don't agree that taxing the rich is the only way to do that, or even the best. If you create policies and investments that get out of the way of the entrepreneurs, they will create businesses more successful than those of the lazy wealthy. - Taxman Talk 15:32, 22 February 2006 (UTC)[reply]
I'm not saying its the only way, but in some countries it is a necessary precondition (I mentioned Britain at the start of the Industrial Revolution, also before the French/Russian Revolutions, etc) the obstacles to development were those at the top not the bottom. They were perfectly happy with the status quo after all. I was arguing the structure of many Third-World countries is similar. Its not that their elites are lazy, its that they are very good at keeping their monopoly on economic and political power which hinders the wider development of the country as a whole. You couldnt get large-scale entrepreneurship off the ground without some levelling (down) of the playing field to start with.
BTW, I dont agree with your welfare assertion at all even on the revised definition, and I'm sorry that you seem to take it as a given, but thats a different debate. Jameswilson 23:52, 22 February 2006 (UTC)[reply]
May I summarise: fuck tax the rich, keep the poor healthy enough to be productive and let the middle classes run the economy. We seem to agree at least on the last bit, but that means not taxig them too heavily. And I suppose you will also agree that the second bit is a minimal requirement for a healthy economy, but that will cost money. Which leaves the question where tha money is to come from. Which brings us to the first bit because third world countries have too smal a middle class to cough up the money.
In anticipation on a possible response: as sook as keeping the poor healthy does not cost money because they can do that for themselves, they are not poor anymore and effectively middle class. DirkvdM 10:46, 23 February 2006 (UTC)[reply]

length of months of indian solar calendars[edit]

Please kindly tell me the precise interval of time between two samkrantis of various months in indian solar calendar (exactly the Vikram Sambat).What is the phylosophy behind the mathematics of Vikram sambat.

If I were you, I'd have a look at Hindu calendar, Samvat (calendars) and the articles in Category:Months of the Hindu calendar. They may give you the information you need (if not, come back here and try again!) Grutness...wha? 23:16, 20 February 2006 (UTC)[reply]
A new Hindu month starts precisely when the sun enters a fresh "sun" sign. This event is known as Sankranti. A zodiac is 30 degrees of the celestal sphere. Knowing the exact speed of the sun, you can calculate the exact duration between two sankrantis. It comes out to be 30 point something days. Of course, a Hindu day doesn't start at midnight, but at sunrise, so it doesn't start at the exact same time everyday, and therefore a new month also doesn't start at midnight. deeptrivia (talk) 03:43, 22 February 2006 (UTC)[reply]
Thanks deeptrivia. I know that the indian months statt and end the way you said.What I actually wanted to know is how long are the months exactly(from sankranti to sankranti)?
If you're talking sunrise to sunrise, it would depend whereabouts you are in India (or elsewhere in the world), since sunrise times vary with latitude. I wouldn't be a big difference in summer, since the Tropic of Cancer passes through central India, but in winter, dawn would come far earlier to Tamil Nadu and Kerala than to Punjab and Himachal Pradesh. If you're talking the moment of entering the sun sign, then it's simply 1/12 of the exact length of the year, surely? Grutness...wha? 05:19, 23 February 2006 (UTC)[reply]
You did not understand my question.What actually is the time taken by the sun to traverse each rasi while the earth revolves in an elliptical path around the sun.

World with happiness[edit]

In today's world most of the people are living in poverty whereas some of the people of developed nations have accumulated a significant amount of world's wealth.If there were a one world government and just distribution system, would the present day wealth of the world be sufficient for living happily for all ?

I suspect you think just distribution is equivalent to even distribution. However, it is arguable whether just distribution is the same as even distribution, and who gets to decide what is just?
Just is a subjective value assessment, like beautiful. Even is an objective assessment, measurable quantitatively.
Poverty is a relative term, just like tallness or shortness. It only exists if one person has more than an adjacent person. It does not exist until the two are compared.
Does wealth equal happiness?
Does just distribution equal happiness?
Does equal distribution equal happiness?
No government known to exist has ever achieved even distribution of wealth, and all that have tried have produced misery and injustice rather than happiness.
In other words, your underlying assumptions are too unrealistic to generate a question which can be meaningfully answered. alteripse 17:55, 20 February 2006 (UTC)[reply]
  • You are overgeneralizing. First you're equating "even" with "absolutely even", which is a quite different thing. Secondly: Communism tried to create an even distribution of wealth and failed, creating misery in the process. I'll agree to that. But you are then making the assumption that Communism is the only method to achive a more equal distribution of wealth, and thus that it has always failed. Which is wrong. See for instance List of countries by income equality. In the top 10 you'll find all Nordic countries, which all have had a strong Social Democratic political history in the 20th century, and led a quite active policy of redistribution. I'd say it's been successful at redistributing wealth and hasn't created any significant misery or poverty in the process. Whether the Scandinavians are happier people for it, I can't say. But they did keep re-electing the politicians who did it, so most people probably agreed with it. --BluePlatypus 00:28, 21 February 2006 (UTC)[reply]
My answers were generalized in an appropriate response to an incredibly generalized question. Are you trying to quibble over what degree of generalization the question warranted? Go find something more constructive to do. alteripse 03:37, 21 February 2006 (UTC)[reply]
What's with the personal attacks? --BluePlatypus 13:12, 21 February 2006 (UTC)[reply]
He did do a good job of generalizing the issues that the questioner didn't mention in their very broad question and I can see how your statement starting off with "you are overgeneralizing" would be offensive. You made good points too, and they would have done just as well as an expansion instead of telling him he was wrong which is essentially how it came out. It is also interesting to note that substantially all of the countries with the most even wealth distribution (such as the Nordic countries) their societies are far far more homogeneous than most. I'm not sure there's a causal effect, but it is interesting nonetheless. Further, the Lorenz curves would be much more valuable information than a summary statistic like the Gini coefficient. - Taxman Talk 14:46, 21 February 2006 (UTC)[reply]
  • I can only agree that the questioner is mixing two different concepts: wealth and happiness. What do happy poor people and miserable rich people demonstrate? Notinasnaid 18:00, 20 February 2006 (UTC)[reply]

I think living 'happily' is subjective... since for example, now food is something everyone is supposed to have, but in the future maybe it will be like if you don't own a pc you won't be having the basic human rights or something, like the right to information...so...even if the world progresses more, I think we will always have something to complain about..and I guess that is something good, since if we didn't see the bad side, we wouldn't work for solutions... and as for the 'just distribution' thing, I don't believe in it...since it can make economy fall apart, from my point of view. Also, I completely agree with Alteripse, I couldn't have said it better.--Cosmic girl 18:20, 20 February 2006 (UTC)[reply]

To go back to the original question. If the rich parts of the world redistributed money to the poorest parts, yes, you could end starvation and high infant mortality. Yes that would increase the sum of happiness. Its more doubtful that the world is currently rich enough to be able to afford decent education, housing and health care for all its inhabitants. Probably not. Jameswilson 04:20, 21 February 2006 (UTC)[reply]
I agree. but isn't it funny how we tend to think the world cannot "afford" education, housing and health care? As if capitalism is a law of physics. ultimately, everything in the world, including capitalism, is man-made, and we could create housing, health care and all that, if we just chose to do so. Just a philosophical side-note. The Minister of War (Peace) 14:01, 21 February 2006 (UTC)[reply]
I haven't looked at the numbers but I think if world GDP were naively spread evenly to all people there would be at least enough for enough food and clean water for everyone. But that would mean liquidating income producing assets, and removing incentives to produce so it may not work for long. Also you would instantly create a population growth explosion by reducing mortality (especially infant) and supporting the ability to have more children. Of course we don't have room here to fully debate the possible outcomes of all economic systems. - Taxman Talk 14:46, 21 February 2006 (UTC)[reply]
Of course, really ending poverty would mean everyone in the world could finally run that SUV they wanted. Or do we not want to end poverty quite that much? Notinasnaid 09:25, 21 February 2006 (UTC)[reply]

List of countries by GDP (nominal) says the world's total income is 40.000.000.000.000 (40 trillion or 40 million million) USD. Divide that by a population of 6 billion and you get about 6.600 USD per person per year. So based on this one would have to say no, won't work. However, it's not as simple as that. In the west, this would be just enough to live off (it's actually less than the lowest unemployment benefit in the Netherlands). But in a third world country one can feed and house oneself quite decently with one or two thousand a year, so it would mean an unnecessarily high income. So if rich countries would give away just a fraction of that (say 10% of it's GDP in stead of the 1% or so that is given now) that would mean a huge impulse to third world economies, provided it is distributed well. Meaning evenly, so people can start up their own businesses (and thus get a strong middle class - the decisive factor that is often missing in third world countries). But there are probably many other factors to take into account, like knowledge (ie education) and birth control. And those things takes much longer, so this is not going to work overnight.

Agree. It isn't that simple... besides, I don't want to sound mean, but if everyone was somwehat 'equal' in economic matters...then the 'capitalism pyramid' would fall appart... this is really simplistic, but it's the way I think things could go wrong, so if we want an equilibrium and thew progress of humanity as a whole ( like scientific progress for instance) not everyone can be 'at the top' this seems pretty obvious to me.--Cosmic girl 00:39, 22 February 2006 (UTC)[reply]

In response to some earlier answers, an almost even distribution has been achieved in Socialist States (often misnamed Communist States), and the effects, compared to what they had before (!) was quite positive. Other aspects of those countries may have been less favourable, but those were largely to do with other than economic measures, so that doesn't reflect on this principle per se. And Cuba has a decent health care for all of its citizens (apart from a lack of supplies). The standard of living in Cuba is a lot higher than that in many other third world countries, with a Human Development Index placing them on number 52 on a list of 177 countries hdr.undp.org/reports/global/2005/pdf/hdr05_HDI.pdf. That's above Mexico, Malaysia, Saudia Arabia and some Eastern European contries. Don't believe the hype. DirkvdM 15:15, 21 February 2006 (UTC)[reply]

Maybe Cuba is better off than maby 3rd world countries...but why do you think it is a 3rd world country in the 1st place...from what I've seen, here in Perú, the situation is way better than in cuba...you should hear some jokes about cuba...they are so cruel, yet so true.--Cosmic girl 00:42, 22 February 2006 (UTC)[reply]

How do you know they're true? Jokes are usually based on exaggeration or even making things up. DirkvdM 07:31, 22 February 2006 (UTC)[reply]
To raise the population of China to the living standards/level of consumption of the U.S.A. we would require the natural resources of 4 more planet earths'. Given that China has only one sixth of the worlds population it is extremely unlikely that everyone in the world could have living standards/levels of consumption anything close to that of the U.S.A. or Western Europe (where levels of consumption & energy usage are surprisingly a lot less than the U.S.A.). AllanHainey 16:02, 21 February 2006 (UTC)[reply]
Yeah, but the question was about redistributing the wealth, not raising everyone to the level of western countries. DirkvdM 20:02, 21 February 2006 (UTC)[reply]
Ok, then the answer is certainly not since if wealth were redistributed to that degree, Americans would be very unhappy indeed. Notinasnaid 20:26, 21 February 2006 (UTC)[reply]
Which again raises the issue of the relation between richess and happiness. USians would certainly feel unhappy at first, but they might be able to adapt to the new situation (well, maybe the next generation) and be just as happy. In my experience (and I've heard other people say they noticed this too) people are actually happier when they are poorer. Maybe something to do with having nothing to lose, nothing to worry about. Provided they have the basics, that is, namely enough nourishment and protection against the elements( food, water, clothing, housing) and social contact. Maybe that last bit is the essential happiness bit and the rest is just a basis. We can provide that basis and they themselves can then take care of the friendship bit. DirkvdM 07:31, 22 February 2006 (UTC)[reply]

Most of these answers have been about redistributing money. The logic has been to give each person the same amount of money and then assume that their economies would be the same as now. This is not following the thought through to the end. A one world government and just distribution would mean no tariffs, no product dumping, no overproducing of crops and things and letting them sit and rot instead of sending them around to where they're needed, no travel costs besides fuel and general overhead, etc ad nauseum. Would the resources of the world be enough to more than cover the needs of the world's population? I think yes, if it were correctly utilized. If you define economic happiness as having a disposable income without needing to work more than 30-40 hours in a week, which isn't unreasonable, I would guess that yes, the present day wealth of the world would be sufficient. -LambaJan 07:04, 22 February 2006 (UTC)[reply]

Well said. DirkvdM 07:31, 22 February 2006 (UTC)[reply]
Thank you. -LambaJan 18:47, 22 February 2006 (UTC)[reply]

US Steel Industry pioneer[edit]

Who was this guy who started out as a telegraph writer and then one day did something by sticking his neck out and ordering something on behalf of his employer of a railway firm who saw the growing emergence of steel as a track laying metal and invested on it and became rich?

You must be thinking of Andrew Carnegie. --Halcatalyst 01:34, 21 February 2006 (UTC)[reply]



Thanks that is the case. Thanks once again. (My IP address is not a permanent one.)

Sleep[edit]

Will sleeping help u lose the calories. I read it on a website. Thks

Calories are burned all the time but some activities will burn more calories than others. According to www.fitwatch.com/phpscripts/viewexercise.php?descr=sleeping&mets=0.9 fitwatch.com an person weighing 160 pounds will lose 69 Calories from sleeping 1 hour. If the same person was running at 5mph for 1 hour he/she will lose 610 Calories. Thuresson 23:11, 20 February 2006 (UTC)[reply]
Another way to look at is that you lose weight while sleeping because you can't eat while sleeping. Thus, the small number of calories burnt are not replenished. StuRat 23:23, 20 February 2006 (UTC)[reply]
Not really an argument. You don't really need to eat while doing anything in order to "not-burn" calories. Calories are burnt all the time, and eating is just the process by which we replenish them. ☢ Ҡiff 04:18, 21 February 2006 (UTC)[reply]
For people who are overweight, eating is the process by which they replenish burnt calories, and then some. Hence the problem of weight gain. So, if the person was asking if getting lots of sleep helps them lose weight, the answer is that it might. The only way this might not be true is if they consume more calories in their waking hours to compensate for their lack of calories consumed during a longer sleep. StuRat 05:53, 22 February 2006 (UTC)[reply]
Though not directly related to your question, sleepdisorders.about.com/od/obesity/a/sleepweight.htm it has been suggested that lack of sleep can lead to weight gain. - Akamad 09:14, 21 February 2006 (UTC)[reply]
Right, most likely because more waking hours mean more opportunities to eat. StuRat 05:46, 22 February 2006 (UTC)[reply]
That's illogical. Very few people use all their current opportunities (i.e., very few spend every waking minute eating). Therefore, giving them more waking time wouldn't increase consumption. The article suggests a different, common-sense explanation: Lack of sleep causes fatigue, which reduces physical activity, so the person burns fewer calories. The article also mentions an endocrinological effect. JamesMLane t c 09:59, 23 February 2006 (UTC)[reply]
Your reply was illogical, by assuming it could only be true if they ate every waking minute. It is only necessary that they eat in proportion to the number of waking minutes, for example, during 10% of their waking minutes. StuRat 05:42, 24 February 2006 (UTC)[reply]
I see no reason to assume that people eat in proportion to the number of waking minutes. From my own experience, that isn't true. JamesMLane t c 11:36, 26 February 2006 (UTC)[reply]
That would depend on why they are eating. If people only ate because their body correctly tells them they need nutrients, then I would agree. However, people eat for many other reasons, like boredom, enjoyment of the taste, to be social, etc. If one is awake twice as long, one would reasonably expect to be bored twice as much and eat out of boredom twice as often, for example. StuRat 08:08, 27 February 2006 (UTC)[reply]

Song "Sarah Dawn"[edit]

When I was a kid, my parents would play a song called "Sarah Dawn" or "Sara Dawn." It was written by a divorced dad and directed to his daughter, Sara(h) Dawn. I've tried to find some record of the song but have had no luck, and my parents can't remember the name of the man who sang it. I remember some of the lyrics: "Sarah Dawn, Sarah Dawn, I love the little girl With the bright smile on. And I can't wait to see her, My darlin' Sarah Dawn." Does anyone know of the song I'm talking about or who sings it?

February 21[edit]

Panzer Spahwagon periscpoe /whats it worth & how do i find out more about the id numbers?[edit]

I have a german Panzer Spahwagen Ause IV periscope need to know what its worth. I also need to know what the ID numbers are and mean.

You can get some background by reading the Periscope and Tank articles. Then try Googling PanzerSpahwagen periscope and check out the 30 or so hits. --Halcatalyst 05:05, 22 February 2006 (UTC)[reply]

What evidence is there that Minoan society/civilization was a thalassocracy?[edit]

This is part of my research on Minoan society. I cannot seem to find very much EVIDENCE that shows or tells that Minoan civilization was a thalassocracy...

any help would be HIGHLY appreciated. gelo 05:59, 21 February 2006 (UTC)[reply]

Largely circumstantial and hearsay, I would suggest, but all pointing towards their sea-power. Herodotus described them as a thalassocracy (check our article on thalassocracy) - and there seems to be a strong correlation between Minoan power and the use of bronze in the eastern Mediterranean. The Minoans are known to have been traders, and would easily have managed to get copper in nearby Cyprus, but for tin they would have had to have ventured farther ashore or relied on visitors from tin-mining lands. Given that these probabl;y couldn't have been relied on arriving on a regular basis, it would have made sense if it was the Minoans doing the travelling - possibly as far as southwest England, which was the major tin-producing region at the time. Also, let's face it, we're talking about a civilisation based on an island (Crete) which had strong links with other peoples throughout the Mediterranean. That would have required regular maritime travel. Small island + wealth built on sea trade = thalassocracy. Grutness...wha? 07:14, 21 February 2006 (UTC)[reply]
wha?: thanks! oh, and where did you happen to find that information? i seem to not be able to find much anywhere...
well, I started by going to our article on thalassocracy. Then I followed the link to Minoan civilization, and from there to tin, and... just keep following the links! Grutness...wha? 00:50, 22 February 2006 (UTC)[reply]


hindu doctrine[edit]

hello , i've read most of the articles here related to hinduism , plus many books at home . but it seems that the more i read the more i get confused . i really cannot understand their creed in regards to deities and worship especially how this world came to be . is it brahman that created it .. do they actually believe it was created ? vishnu . krishna , shiva ... are they manifestations of brahman .. are they all one ? what is prajapati ? is he the creater ? or is he brahman .. or did brahman create all the deities ?! i must admit i am completely comfused ! are they all one , or are there different beliefs within the hindu doctrine? i have enough information about hinduism that i would understand a brief explanation from an expert ... please if anyone has information i would appreciate it deeply .. thank you .. ( i cant post this in the hinduism talk page since i have an open proxy an i am blocked from editing , thats why its here ) Hhnnrr 13:02, 21 February 2006 (UTC)[reply]

Okay, I'll attempt to answer your questions with some of my views:
  • Over the millenia, many Hindus have given their views on things like origins of universe, etc., and each view has won a certain number of adherents. Some views have dominated at some point of history, and some at other. Anybody who has gained sufficient credibility and respect can put his/her own view. The oldest one we know of is from the Rig Veda:
Neither being (sat) nor non-being was as yet. What was concealed? And where? And in whose protection?…Who really knows? Who can declare it? Whence was it born, and whence came this creation? The devas were born later than this world's creation, so who knows from where it came into existence? None can know from where creation has arisen, and whether he has or has not produced it. He who surveys it in the highest heavens, he alone knows-or perhaps does not know. (Rig Veda 10. 129)
For other creation views look at Hindu cosmology.
  • Brahman is not a "being" anyway. "So is 'he' Brahman?" doesn't make sense. Brahman just means something like "ultimate reality." Check out the article.
  • According to the concept, not only vishnu, krishna, shiva, etc. but you and I are also manifestation of Brahman. So are all other living and nonliving entities, space, time, mind, et al. Brahman is considered without any form in Advaita (Identical to Buddhist concept of Shunyata) and without material form in Dvaita schools of philosophy. Other schools hold that it doesn't matter.

Hindus are free to hold any of these views (or have their own). I hope this was of some help. deeptrivia (talk) 04:04, 22 February 2006 (UTC)[reply]

Thats a very wide range of possible beliefs with a lot of potential for disagreement. Have their ever been wars between the different types of Hinduism? As between Catholics & Protestants, Catholics & Orthodox, or Sunnis & Shias? Jameswilson 00:27, 24 February 2006 (UTC)[reply]
I am not aware of any wars taking place among difference sects, but there is a long tradition of shastrartha (public debates) on the relative merits of their own school of thought. People are free to choose between existing thought-streams based on their own logic or even to develop their own independent one. There is nothing like finality or absoluteness of anything in Hinduism, not even Brahma since some schools of thought are agnostic. There is agnostic Buddhism with its spiritual richness at one end and Charvaka school with its pure emphasis on materialism at the other. But more important than these differences is the almost universal idea that there are infinitely many ways of attaining enlightenment/reaching God, and they are all equally valid. Religious texts often emphasize ideas such as "Truth is One, but sages call it by many names", "no matter in what form people worship Me, they reach Me", etc., and non violence is an important ideal. People belonging to different Hindu denominations generally accept that their differences do not matter. Buddha is very highly respected by all Hindus and is considered to be an avatar of Vishnu who came to rescue Dharma, when it was getting degraded. deeptrivia (talk) 03:47, 24 February 2006 (UTC)[reply]
Interesting. Jameswilson 23:53, 24 February 2006 (UTC)[reply]

conflict of the orders[edit]

what were the results of the conflict of the orders?

Which particular orders did you have in mind? --Shantavira 14:36, 21 February 2006 (UTC)[reply]
Disorder. Markyour words 14:50, 21 February 2006 (UTC)[reply]

Like in the kitchens of a restaurant? Dan l'homme

Maybe everyone started thinking for themselves and lived happily ever after. :) DirkvdM 15:17, 21 February 2006 (UTC)[reply]

See Conflict of the Orders. —Charles P._(Mirv) 17:37, 21 February 2006 (UTC)[reply]

China[edit]

1. Can China become a military superpower within 3 decades from now? and if not why?

Absolutely, as long as it does it's own homework. DJ Clayworth 15:00, 21 February 2006 (UTC)[reply]

The question suggests it isn't right now. What constitutes a military superpower? They've got the numbers. And I suppose they;ve got some guns too. What else does on need? To prove it by sending out troops? Maybe they can't be bothered, as has been their policiy for the last, oh, say, few thousand years. DirkvdM 15:23, 21 February 2006 (UTC)[reply]

See Military of China (biggest army in the world, nukes, etc.) Sound super enough for you? Rmhermen 15:38, 21 February 2006 (UTC)[reply]
Power projection is a major factor for being a superpower, rather than a regional power. Perhaps that should play a role in your answer. — Lomn Talk 16:18, 21 February 2006 (UTC)[reply]
Inline with power projection, how is China treated by other "super powers"? You can use Tibet as an example. Contrast that with Kuwait. It doesn't take much to make an argument there. --Kainaw (talk) 19:53, 21 February 2006 (UTC)[reply]
Read superpower.--Robert Merkel 03:11, 22 February 2006 (UTC)[reply]

To be a military superpower, I would think they would need a larger navy. In particular, more aircraft carriers. StuRat 03:32, 22 February 2006 (UTC)[reply]

To borrow Bill Clinton's campaign slogan: "It's the economy, stupid". Once China become an economic superpower, it will, by extension, become a military superpower. Europe will sell arms to China, other countries will establish military ties. China already has the 2nd largest GDP (PPP), and its economy shows no sign of slowing down. --Vsion (talk) 10:34, 24 February 2006 (UTC)[reply]

Laws against Christianity[edit]

Hi there,

I have to write a sermon on christian martyrs for my church and I was wondering if you guys could help me find out where in the world it is illegal to openly practise christianity, distribute christian literature or attempt to start a church etc, and what the pentalties for breaking these laws are. This would be really helpful!

(Edit) I have done a sermon on the history of Christian influences (good or bad) already, so I'm really looking for information on Laws against Christianity, obviously America and western countries (Most of europe, Australia and America) aren't really relevant to this particular sermon. I have also been commisioned to write an article on this and various other subject.

Also, example of Christian martyrs would be quite helpful, as long as they're not already stated on wikipedia!

Cheers {Yin 00 07:11, 22 February 2006 (UTC)}[reply]

  • For starters, practicing Christianity is still illegal in Mecca. If you want a historical view, don't forget Bogumil and Heretic -- before the 1800s, Christians were quite happy to make martyrs out of each other. --M@rēino 16:29, 21 February 2006 (UTC)[reply]
openly practicing Christianity, or indeed any religion except Islam, is illegal in all of Saudi Arabia. private practice is usually (not always) tolerated, but never endorsed. evangelism is strictly forbidden. the punishment is usually deportation (as just about every Christian in the country is a foreigner) or imprisonment.
China has state-sponsored and controlled Catholic and Protestant churches, the members of which are well-treated. Christians are not permitted to practice or evangelize outside these official churches, and may face legal consequences (such as imprisonment) if they try. Vietnam has a similar system. North Korea also maintains official state churches, and unauthorized religious practice may (reports are difficult to confirm) lead to torture or execution.
then of course there are innumerable examples from history, but that ought to be enough to get you started. —Charles P._(Mirv) 17:01, 21 February 2006 (UTC)[reply]
See freedom of religion. By "practicing Christianity", you are being vague. Can I be a practicing Southern Baptist in the Vatican? To my knowledge, the United States has the most freedom of religion due to extensive Supreme Court support of the separation of Church and State. Then, you go to the opposite extremes of No Religion (ie: Communist Russia and China) or One Religion (ie: Iran and Saudi Arabia). What about Israel? You can be a different religion in the country, but can you practice something other than Judaism? --Kainaw (talk) 17:15, 21 February 2006 (UTC)[reply]
Iran discriminates against religions besides Twelver Shiism (the official state religion) to some degree, but does tolerate them www.state.gov/g/drl/rls/irf/2005/51599.htm. Israel's religious minorities are free to practice as they please, but face certain legal and social disadvantageswww.state.gov/g/drl/rls/irf/2005/51601.htm. —Charles P._(Mirv) 17:32, 21 February 2006 (UTC)[reply]

To make your sermon more balanced, you should also mention the many times when Christians persecuted non-Christians, or where one sect of Christianity persecuted others. These include the Spanish Inquisition, the Crusades, pograms, witch trials, etc. Fortunately, these are mostly in the past, and hopefully will remain there and be joined by such activities from other religions. StuRat 20:59, 21 February 2006 (UTC)[reply]

This is in Bloomberg's today :
"Feb. 21 (Bloomberg) -- The U.S. Supreme Court, saying law enforcement goals in some cases must yield to religious rights, ruled that the Bush administration can't block a New Mexico church from using a hallucinogenic tea.
In a unanimous opinion written by Chief Justice John G. Roberts Jr., the court today said the church, a 130-member branch of a Brazilian denomination, is protected by the 1993 Religious Freedom Restoration Act. The justices upheld a preliminary injunction barring federal prosecution of church leaders ..."
What a delicious country! --DLL 21:31, 21 February 2006 (UTC)[reply]
And this unanimous vote is even with the most conservative Supreme Court in years. Just goes to show you that the US takes Freedom of Religion seriously. Not everyone, mind you. I doubt if Pat Robertson would allow any religions other than his to practice, if he could prevent it. StuRat 22:57, 21 February 2006 (UTC)[reply]
In the Netherlands,. there's the SGP, protestant party that strives to exterminate false religions (following an old version of the bible). They have been in goernment for decades thanks to a rigid following in extreme religious communities, but luckily they will never get enough seats to initiate anything. DirkvdM 07:43, 22 February 2006 (UTC)[reply]

Algerian poet[edit]

I'm looking for the name of an Algerian poet that sounds similar to "Bonalan Kanfa". The only references I've seen are a Vietnamese translation of his/her work called "Beliefs" (or something like that) in 1965. His/her name must have been written differently. I need verification that this person exists. Thanks. DHN 18:45, 21 February 2006 (UTC)[reply]

Is this a modern poet or a medieval/ancient poet? The nearest name for a poet to this I can find in Wikipedia is Al-Khansa, who lived in the 7th century. Grutness...wha? 00:59, 22 February 2006 (UTC)[reply]
I'm not sure, but I'm leaning towards modern. DHN 01:44, 22 February 2006 (UTC)[reply]
Sorta sounds like Boualem Sensal, but probably not. Look through the Embassy of Algeria in Vietnam website's "ambalgvn.org.vn/site_web_eng/CULTURE/litteraire.html Evolution of the literary and artistic expression" section and see if any prospective names turn up. El_C 08:06, 23 February 2006 (UTC)[reply]
Thanks! Boualem Sansal sounds very promising! DHN 18:20, 23 February 2006 (UTC)[reply]
My pleasure. Good luck! El_C 04:12, 24 February 2006 (UTC)[reply]
Now that I have more time to do look up this person, it appears that he's not the one I am looking for (born 1949). Although the name Boualem might lead me closer to the real poet. DHN 06:27, 24 February 2006 (UTC)[reply]

Mongols or Romans[edit]

Which empire had a stronger military, the Romans or the Mongols> Please include which one had better military tactics. Also, were the Romans that much of a influence in Western military tactics?—Preceding unsigned comment added by 69.9.8.178 (talkcontribs)

If you are in general unfamiliar with the history of warfare in the pre-rifle era, I recommend that you read these two questions from earlier today.

They're important because Rome's greatest innovation in warfare was learning how to beat a thalassocracy without becoming one (see Roman Navy). The Mongol Empire had no need for boats (although their Korean and Slav vassals were quite adept). The Mongols, on the other hand, had the best cavalry that the world had ever seen. Mongols were not only excellent riders, but also excellent breeders -- horses with the endurance of Mongol horses literally did not exist in Roman times. The genetic improvements to the horse is arguably their greatest innovation. --M@rēino 19:14, 21 February 2006 (UTC)[reply]

One feature both had in common was the ability to "turn" conquered peoples into Roman or Mongol Empire citizens, who then provided soldiers to further the spread of each respective Empire. Other conquerors, such as the Spanish in the Americas, massacred the existing population and/or enslaved them, and had only contempt for native languages, religions, and customs. The Romans and Mongols, however, could actually improve the lives of conquered peoples, by allowing them to keep their old languages, religions, customs, and even leaders, in some cases, while improving the standard of living with new technologies, protected trade routes, etc. This was a "carrot and stick" approach, however, meaning that those people who did rebel were treated brutally. StuRat 20:43, 21 February 2006 (UTC)[reply]
That's rather generous Stu as far as the Mongols are concerned. They massacred 30 million people, particularly in China, which would not be matched until WWII. Marskell 10:09, 22 February 2006 (UTC)[reply]
That would be the "those people who did rebel were treated brutally" part. StuRat 05:48, 24 February 2006 (UTC)[reply]

Convincing someone that corporal punishment is bad[edit]

There's a girl in my school (around 17 18 years old) whom I very much respect. I was, however, shocked when in a class conversation she mentioned that she believes that there's nothing wrong with adults hitting their kids for doing something wrong. She says she thinks it's necessary in order to 'teach them respect'. She's from the middle east, I don't know whether or not that background would have any effect on these sorts of beliefs.

I've always been taught that spanking is debatable, but outright hitting your kids is a big line to cross over. Doesn't a parent hitting their kids create serious bad side effects? Basically what I'm asking is, is there any way I can talk to her to convince her that it's a bad thing? If there is, what should I say and/or what should I show her? Flea110 23:33, 21 February 2006 (UTC)[reply]

Have you looked at the references under spanking? --Anonymous, 00:06 UTC, February 22.
Spanking or slapping is a very temporary pain that is not intended to cause injury. As with any physical contact, accidents can happen. I went to school with a kid who lost an eye in an accidental slip with an old woman who was trying to shake his hand, missed, and jammed her big ring right into his pupil. Just as it would be ridiculous to claim hand-shaking should be banned for accidental injuries, spanking should not be banned for accidental injuries.
The problem with spanking is psychological. If you smack a kid repeatedly without reason, no cause-and-effect is made. So, the child is raised to be fearful at all times. Spanking is not required for this. If you lock a kid in a room, put a kid in the corner, yell at a kid, call a kid bad names, or simply walk away and ignore a kid repeatedly without reason, no cause-and-effect is made. The child will be raised to expect the punishment at all times.
A smack that is hard enough to get a child's attention, but not hard enough to cause pain is highly effective when done while a child is doing something wrong. It is far more effective and less psychologically damaging than spending 10 minutes sitting in the corner in timeout. --Kainaw (talk) 00:14, 22 February 2006 (UTC)[reply]

Well said. And, all in all, I'd figure it's more important whether you use punishment with wisdom than which punishments you use. My parents used everything from spanking with a brush to shouting to grounding, but only when it was deserved. Mostly. For a different viewpoint (and it's satire, don't send him hate mail): www.thebestpageintheuniverse.net/c.cgi?u=beat Black Carrot 02:42, 22 February 2006 (UTC)[reply]

Also, if the child is endangering their life or the life of others with their behavior, a more drastic punishment is needed than if they accidentally put their elbows on the table. StuRat 03:28, 22 February 2006 (UTC)[reply]

I think that the proper thing to do is start where you both agree. You both will most likely agree that for the training of children a just discipline is necessary, one that requires reward and punishment. You can then move on to how behavior that deserves punishment can largely be averted through constant parental attention, education, distraction and consistant praising of good behavior. To this she will also probably agree. Then you can talk about how the most effective punishments are the ones that logically flow from the behavior. Ex: the child says something mean to you and you act like they hurt your feelings. Then you can say that the child will not learn to respect adults if they use corporal punishment. What they'll learn is that, 'even though it's not ok to hit, grown-ups can hit kids because they're bigger and grown-up.' This would prove to be a mis-education that is really not benificial to the development of the child.

Presented this way, I think she'll at least appreciate your argument, even if she doesn't end up completely agreeing. And I think these previous posts have shown you that her view is not confined to her culture. It's actually represented by cultures in most corners of the world. -LambaJan 05:49, 22 February 2006 (UTC)[reply]

I think it may also be worth harping on the word respect. This is a little unfashionable in some circles because of its implications of blind following of authority, but in other circles respect is considered extremely important, yes even to the blind following of authority, and lack of respect is in some cultures considered as serious as putting your hand in the fire. (For instance, the ten commandments are held to mandate respect for parents). Try to see how this affects the initial statement by looking at both views of "respect" before applying your own view. Notinasnaid 08:54, 22 February 2006 (UTC)[reply]
LambaJan- Are you sure that hitting kids teaches them 'even though it's not ok to hit, grown-ups can hit kids because the're bigger and grown-up'? A quote from Jeff Foxworthy: "My father took 'time out' of his busy day to whup our asses." I know it's not representative, but every single comedian I've ever heard mention it(except maybe George Lopez) has taken it, like Foxworthy does, as a point of pride that their parents had the balls to hit them when they misbehaved. Are you sure it teaches them that grown-ups are hypocrites? How do you feel about the 'it toughens you up' argument? Black Carrot 02:40, 24 February 2006 (UTC)[reply]

Read this somewhere some years ago: "If father beats up mother, what sense does it make to a child if his mother beats him up for beating up his little sister?". Speaking from first-hand experience, using corporal punishment only teaches kids to be afraid of their parents, keep secrets, and sometimes lose respect for, or even despise, authority in general. I have to agree with LambaJan on this one...a child's discipline should be directly related to the behaviour (hitting them because they swore or stole, etc. does not make any sense), making a point of praising good behaviour and ignoring "bad" behaviour whenever possible will make "bad" behaviour less attractive (a large number of children misbehave because punishment is the only way they get any significant amount of attention)...it is clear that hitting is still a widely accepted method of "discipline" for children, and yet we wonder why there is so much violence, particularly, we wonder why teens are so unruly...the problem is that the people asking this aren't bothering to ask how they themselves have contributed to it. bcatt 03:10, 24 February 2006 (UTC)[reply]

February 22[edit]

how many different celebrities have portrayed the third "roxbury guy"[edit]

See A Night at the Roxbury. Amazingly, this encyclopedia has information in the articles, not just on the forum! --Kainaw (talk) 01:02, 22 February 2006 (UTC)[reply]
Actually, that article only names two while mentioning there were six. While the OP only asked for the number, presumably the identities were wanted as well.

Star+gate[edit]

In the book about Star+Gate (a fortune-telling system), it mentions a game that can be played on the diagram itself, but never really describes it. Does anyone know how it works? Black Carrot 02:14, 22 February 2006 (UTC)[reply]

Has anyone even heard of Star+Gate? Black Carrot 23:57, 22 February 2006 (UTC)[reply]

Is Arabic taught in Israeli schools?[edit]

Hey - I was wondering if an Israel native would mind answering a question I had. Is Arabic taught in the Israeli school system? Is it particularly emphasized over other languages? Thanks. --Brasswatchman 06:18, 22 February 2006 (UTC)[reply]

Good question. Let me add the complementary question: Is Hebrew taught at schools in bordering Arab countries? DirkvdM 07:47, 22 February 2006 (UTC)[reply]

Arabic is taught in Israeli schools, but I doubt whether it's generally a compulsory subject for students of non-Arab ethnic background. Hebrew is taught in some Arab universities (there was a long New York Times article in 2003 about the effect of the invasion on the Hebrew department of an Iraqi university), but mainly for "know your enemy" reasons, or for Old Testament studies in Christian theological seminaries. I would be very surprised if any significant amount of Hebrew were taught in primary or secondary education in any Arab country. AnonMoos 12:44, 22 February 2006 (UTC)[reply]
Ah, yes, muslims (which most Arabs are) would be interrested in Hebrew to read the bible (which is also a muslim holy scripture). But wouldn't that be a dead version of Hebrew? That would then probably not be relevant to what the questioneer is interrested in. DirkvdM 10:56, 23 February 2006 (UTC)[reply]
Actually, the Bible is not particularly a "Muslim holy scripture" in the sense that you have in mind. Some Muslims do look into the Bible out of curiosity, but there's absolutely no Islamic religious obligation to do so, and the consensus view among Muslim religious-legal scholars is that the Qur'an and Muslim traditions contain everything which it is really needful to know; that the pronouncements of the Bible are not binding on Muslims; and that if anything in the Bible disagrees with anything in the Qur'an, then the Bible is ipso facto "corrupted". That's why I specified "Christian theological seminaries" (where ancient Hebrew would be taught). AnonMoos 14:41, 23 February 2006 (UTC)[reply]
All I meant is that the bible is a holy book to Muslims (and Jesus is an Islamic prophet, by the way). DirkvdM 08:46, 26 February 2006 (UTC)[reply]
Why would you be surprised if Hebrew were taught at secondary schools in Arab countries, at least the ones that border Israel? DirkvdM 10:56, 23 February 2006 (UTC)[reply]
Because it would be granting the subject an established legitimate position and official government seal of approval which would not correspond to the feelings of most Arabs in the matter. Also, only a rather small number of the inhabitants of Arab countries will come into contact with Israelis on any regular basis, and even among those who do, English would often be just as useful as Hebrew. If you only learn one language, English is certainly more widely useful as a general international language than Hebrew. AnonMoos 14:41, 23 February 2006 (UTC)[reply]
German and Freanch are taught in the Netherlands (although not entirely compulsory). DirkvdM 10:56, 23 February 2006 (UTC)[reply]
That's nice -- Germany and France have a much larger population than the Netherlands, while the Arab countries have a much larger population than Israel. AnonMoos 14:41, 23 February 2006 (UTC)[reply]
Which is the point I was making (read on). DirkvdM 08:46, 26 February 2006 (UTC)[reply]
Of course it's also the language of an enemy, but that works both ways. Then again, there's also the importance of the language. French and Arab are much more important on a world scale then Hebrew (the differnce in importance is quite large). So maybe you're right after all to be surprised. DirkvdM 10:56, 23 February 2006 (UTC)[reply]


Suzanne Bing[edit]

I am seeking information on the French actress Suzanne Bing (early 20th century). I would like to know her dates (birth/death), and anything else about her!

She was born in Paris, March 10, 1885 and died in Neuilly-sur-Seine, November 22, 1967 www.lesgensducinema.com/affiche_acteur.php?nom=BING%20Suzanne&from=dvdtoile. She married the composer Edgard Varèse on November 5, 1907, gave birth to their son daughter Claude in 1910, and divorced him in 1913 phillal.club.fr/PAGES/bio.html. She played the governess in the 1934 film www.imdb.com/title/tt0242343/ Le calvaire de Cimiez. Most of the google hits I get are for the French translations she made with Jacques Copeau of Shakespeare's plays. David Sneek 13:22, 22 February 2006 (UTC) p.s.: Lupo has created an article about her, Suzanne Bing.[reply]
Darn, you beat me with the announcement. :-) Lupo 13:56, 22 February 2006 (UTC)[reply]
But you beat me with the article... Good work. David Sneek 13:58, 22 February 2006 (UTC)[reply]


looking for information on the myth women who fell from the sky[edit]

I specifically need to locate information that can assist me with writing a contrasting analysis between Women who fell from the sky by and Out of the Blue

Are you referring to the Iroquois folk-tale "Women Who Fell From the Sky"? There are several stories titled "Out of the Blue". Will any of them do? --Kainaw (talk) 02:26, 23 February 2006 (UTC)[reply]

Who was Rosa Parks?[edit]

You know, funny enough, an online encyclopedia actually has articles on very important and significant people, like Rosa Parks. Maybe you should try searching first next time. СПУТНИКССС Р 22:28, 22 February 2006 (UTC)[reply]

In Detroit, there are 2 parks named after her, the "Rosa Parks parks". StuRat 02:28, 24 February 2006 (UTC)[reply]

February 23[edit]

Date for Albert Ellmenreich's Spinning Song[edit]

Does anyone "out there" know in what year Albert Ellmenreich composed the "Spinning Song"? I would appreciate the answer ASAP as I am putting this information in our piano recital 2-26-06 and need to type up the program by 2-25-06. I couldn't find him in my encyclopedia (VERY old!) or my music history books. THANKS! ---rkwhitney

I looked in my music theory books and found this song in the index. It doesn't say much except that nothing is known about Ellmenreich. He produced an opera in 1856 (Der Schmied von Gretna Green) and wrote "Spinnliedchen" (the Spinning Song). --Kainaw (talk) 02:19, 23 February 2006 (UTC)[reply]

United States[edit]

As America is a continent consisting of many countries, can you tell me why the the people of one of these countries, the United States, call their country America and refer to themselves as American?

Doesn't the United States of America mean, the United States of the Continent of America?

Thank you very much in advance.

Ceetee

Words do not always evolve in logical ways. Americans would say that it's wrong that there is a continent "America" --- rather, there are two continents "North America" and "South America" -- and so there is no confusion such as you describe. Also that if "Republic of Peru", for example, is commonly shortened to "Peru", then it makes just as much sense to shorten "United States of America" to "America". Obviously, other people see it differently. See also United States#endnote_America and Use of the word American. --Anonymous, 04:01 UTC, February 23, 2006.
That and one can say "Peruvian" or "Mexican" and so forth; "United States of American" is ridiculous as is "United Statesian" (ridiculous in English, of course; in other languages it is not necessarily so silly sounding). There just aren't a lot of other options which sound right in English. --Fastfission 04:25, 23 February 2006 (UTC)[reply]
  • Because "America" is the only proper name in the name of the country and the full name is too unwieldly. People in the Union of Soviet Socialist Republics weren't actually "Soviets" either (a "Soviet" being an elected assembly), but that's what they were called. --BluePlatypus 10:00, 23 February 2006 (UTC)[reply]
Also, to the English speaking world, the word 'America' will probably have been used for the English speaking part, which is almost exclusively the US. Just as to Spanish speakers, 'America' will mean mostly the Spanish speaking part (of the continent 'América'!). The es.wikipedia.org/wiki/América Spanish Wikipedia article on 'América' wastes few words on it. Calling the US 'América' is a misnomer coming from the Anglosaxon world. Here's the complete section:
En la actualidad, el nombre de América se ha convertido de manera errada en un sinónimo para los Estados Unidos de América (EE.UU.). En el mundo anglosajón ha surgido el término de Las Américas para referirse al continente y así distinguirlo de los EE.UU., cuyos habitantes son denominados americans. En el mundo hispano, el término "americano" corresponde al gentilicio referido al continente mientras que los habitantes de los EE.UU. son denominados estadounidenses, aunque también se utiliza el término "norteamericano", igualmente errado.
DirkvdM 11:10, 23 February 2006 (UTC)[reply]
I don't think it's a misnomer. That's when you give something an erroneous name. 'America' has a long list of meanings (click the link), and the USA is one of them. Some may argue that should not be the case, but it's purely academic now. The name is here to stay. JackofOz 11:32, 23 February 2006 (UTC)[reply]
Apparently the Spanish Wikipedia doesn't believe in NPOV. User:Zoe|(talk) 23:29, 23 February 2006 (UTC)[reply]
Except we're talking about the English word "America" and its valid meanings in that language. Other languages would have a different set of valid meanings for the word. JackofOz 23:37, 23 February 2006 (UTC)[reply]

Also note that there are other countries named for their continent, like South Africa and Australia. StuRat 02:14, 24 February 2006 (UTC)[reply]

Well, as for Australia, that's because they are their continent. It'd sound a bit silly to call them one thing and the land they sit on something else. Black Carrot 02:25, 24 February 2006 (UTC)[reply]
That's an interesting one. Australia as a unified nation didn't exist till 1901, and prior to that the continent was occupied by six British colonies. The British Matthew Flinders was the first to prove it even was a continent, by circumnavigating it in 1803. The British Admiralty (bless them) decided in 1824 that the continent was to be named Australia (nicely continuing the tradition of giving all non-European continents names that both start and end in A). But what would have happened if the continent had been named well before Europeans ever inhabited it? If the continent had been named eg. Utopia, the country would probably have become Utopia too, not Australia. (DirkvdM, the Dutch didn't go far enough when they named the west coast as New Holland. They should have kept going to Queensland, all the way to Bundaberg. Or perhaps they would have left in disgust too.) JackofOz 09:57, 24 February 2006 (UTC)[reply]
The Malagasy Republic (old name ?) on the island of Madagascar didn't feel the need to use the name of their island for their country. StuRat 06:16, 24 February 2006 (UTC)[reply]
Yes they did. Malagasy is simply an adjectival form of the word Madagascar, in their language. Same principle as the Argentine Republic = Argentina. JackofOz 09:57, 24 February 2006 (UTC)[reply]

Rabat[edit]

Article states cassock was replaced by a black shirt with rabat. Can you tell me what a rabat is?

  • "A piece of cloth fitted to the collar and covering the shirt front, worn chiefly by Roman Catholic and Anglican clergy" dictionary.reference.com/search?q=rabat dictionary.com. Noting to do with the cities. --Halcatalyst 05:08, 23 February 2006 (UTC)[reply]
  • Presumably the word has the same derivation as cravate (a very interesting derivation involving travelling mediaeval merchants from Croatia, IIRC). Grutness...wha? 05:28, 23 February 2006 (UTC)[reply]
The dictionary says it has the same etymology as "rebate". Where did you hear about medieval merchants? The accounts I've heard claim that the cravat/cravatte goes back to the 30 years war, which seems right to me since the cravat didn't come into fashion until the 18th century. --BluePlatypus 09:26, 23 February 2006 (UTC)[reply]
The Holy Roman Empire used lots of Croatian cavalrymen. It could have come from them. User:Zoe|(talk) 23:32, 23 February 2006 (UTC)[reply]
Hm. I read it a long time ago... sad if it was just another urban myth. The story, approximately, was that many of the mediterranean's finest seamen were Croatian, and as such the Dutch - at the time the world's top merchants - employed them on their trade routes. Because of the risk of severe sunburn on African and other tropical voyages, Croatian sailors wore strips of cloth across their shoulders and necks as protection from the sun. The Dutch merchants started wearing a stylised form of these strips of cloth to identify themselves as traders working with fleets travelling in the tropics, calling them Hravats. In time this identification spread to anyone working in financial professions as a whole, with the hravat itself developing into the modern business tie. Grutness...wha? 00:38, 24 February 2006 (UTC)[reply]
Rabat is the capital of Morocco. deeptrivia (talk) 04:03, 24 February 2006 (UTC)[reply]

Sabbath Law in the UK/Europe.[edit]

Is there a law that makes holding events in the UK on Sabbath days (of any religion [Christians = Sunday, Jews = Saturday, etc.]; which therefore means that members of relgious faiths cannot attend) - illegal?

Thanks in advance

129.11.76.215 11:46, 23 February 2006 (UTC)[reply]

There are many laws with single out Sunday for special treatment. For instance, I think shop workers cannot be forced to work on Sundays. But I don't think there is any law referring to sabbaths or religious holidays in other ways. Notinasnaid 11:48, 23 February 2006 (UTC)[reply]
There are, in the UK, restrictions on Sunday trading, i.e. shops opening on Sundays. In England and Wales, the Sunday Trading Act 1994 allows smaller shops to open unrestricted, but limits large shops to 6 hours on Sundays. There are also restrictions on opening on christmas Day and Esater Sunday. Also, before the recent change in the licensing law, pubs closed at 10:30pm rather than 11:00pm on Sundays.

Traditionally football matches in the UK were played at 3pm on Saturdays. About 20(?) years ago when Darlington decided to play a match on Sunday in the hope of attracting a bigger crowd, they found that they were not allowed to insist on an admission charge. If somebody didnt want to pay they had to let them in anyway. (PS The law was soon changed). Jameswilson 00:36, 24 February 2006 (UTC)[reply]

In many other parts of Europe, most shops will be closed on Sundays and have different/restricted opening hours on Saturdays. To what extent this is tradition and to what extent determined by law I'm afraid I don't know. — QuantumEleven | (talk) 13:33, 24 February 2006 (UTC)[reply]

Minoan palaces -> economy?[edit]

Hi everyone.

Could someone PLEASE explain and outline to me the ROLE that the palaces of Minoan civilization played in the economic life of Minoan Crete?

I have searched "Minoan Civilization" quite thoroughly, yet without any result in relation to this question.

This is part of my research which I am having trouble with.

Any help would be GREATLY appreciated! gelo 12:37, 23 February 2006 (UTC)[reply]

Economy : the rich build palaces to give employment to the poor. Mostly in a great town or nearby the sea, in every conceivable period of humanity. Politics : do not change anything as long as it works. --DLL 21:15, 24 February 2006 (UTC)[reply]

Law in the American Old West: federal/state/regional divisions[edit]

Hi all, Working on a project on the American Old West, and while I know that the sheriff was responsible for enforcing the law on a town-by-town basis, was the law still "broken down" back then the same way it is now, with various federal, state and municipal laws, or did individual towns have a lot more control over laws and punishments regarding what are now considered federal crimes?

  • Interestingly enough, we do have an article on the Old West. You just never know what all you can get when you type in a topic in that Wikipedia search box over on the left, there. And then there are all those links you can follow up... curiosity heaven. Beats Google and Yahoo any day for signal-to-noise ratio. --Halcatalyst 22:10, 23 February 2006 (UTC)[reply]
And astoundingly enough, I DID read the article. But there wasn't enough depth, and I was hoping an Old West scholar might be able to save me a hella lot of digging. Not a lot of articles on legal jurisdiction in the American Old West, oddly enough. --MattShepherd 14:16, 24 February 2006 (UTC)[reply]

I believe the relationship was the same, in theory, for states. For territories, however, the Federal Government had more direct control. Also note that, as a practical matter, isolated areas had to make and enforce their own laws. There has also been a steady erosion of states-rights by federal authority since the formation of the US, with the biggest jump being perhaps during the Civil War. StuRat 01:55, 24 February 2006 (UTC)[reply]

That's a good starting point, thanks. Would researching the history of States' Rights be more fruitful then fumbling around Old West literature looking for legal concepts, d'you think? --MattShepherd 14:16, 24 February 2006 (UTC)[reply]
  • I would say this would be a more sober approach. Really, the American push westward was just an example of European expansionism, which happened to gain the attention of Hollywood, which happened to gain the attention of the world. --Halcatalyst 18:23, 24 February 2006 (UTC)[reply]

Diabetes and fasting[edit]

Diabetics need to eat regularly to control their blood sugar. What, then, do Jewish diabetics do for Yom Kippur, and Muslim diabetics do for Ramadan? —Keenan Pepper 14:28, 23 February 2006 (UTC)[reply]

  • In the case of one Jewish woman that I happen to know quite well, her rabbi absolved her years ago from any fasts that would be a serious risk to her health.--M@rēino 15:16, 23 February 2006 (UTC)[reply]
similarly, Orthodox and Catholic diabetics are exempt from fasting during Lent. Muslims who cannot fast due to medical reasons are encouraged to feed a needy person insteadwww.islamtoday.com/show_detail_section.cfm?q_id=234&main_cat_id=9. —Charles P._(Mirv) 15:36, 23 February 2006 (UTC)[reply]

Even if there is the slightest danger by one's fasting, then one is absolutely forbidden to fast on Yom Kippur. If one would do so and die as a result, it would be like committing suicide.

The[edit]

The article you're looking for is here. David Sneek 15:57, 23 February 2006 (UTC)[reply]
Although maybe here. User:Zoe|(talk) 23:38, 23 February 2006 (UTC)[reply]
Ten bucks says the big smoke person submitted it instead of hitting space. Black Carrot 02:20, 24 February 2006 (UTC)[reply]
  • Anybody know the current status of that particular technical problem? --Halcatalyst 18:14, 24 February 2006 (UTC)[reply]

The Big Smoke of 1138 to 1140[edit]

My sister found two postcards that our father had sent when he was in the army. He is now dead. He was born in 1901 and served in the army in Siberia and the Phillipines. On these two postcards are the letters I C O R O T Bodies after the Big Smoke in 1138 to 1140.

Can you tell us anything about this event in history and where it took place?

Thank you for researching this question?

Bbbboltz

So is that what George Lucus is all on about with all those 1138 references--205.188.117.12 06:10, 24 February 2006 (UTC)[reply]
The word might be Igorot... AnonMoos 16:31, 23 February 2006 (UTC)[reply]
Which army did he serve in ? I can't think of any offhand that were stationed in both Siberia and the Philippines. I suppose the Japanese army might, but the Russo-Japanese War was before he would have been old enough to serve. Also, I don't think the Japanese occupied Siberia during that war. StuRat 01:34, 24 February 2006 (UTC)[reply]
The British, French, US, Canadian, Czech governments sent troops to fight in the Russian Civil War. Was it that? Jameswilson 02:55, 24 February 2006 (UTC)[reply]
I suppose that's possible. If from the US, that might explain why he was also stationed in the Philippines, which was US controlled at the time. StuRat 06:27, 24 February 2006 (UTC)[reply]
Australia sent some troops too. JackofOz 11:12, 26 February 2006 (UTC)[reply]
"Taal Volcano is an active volcano in the Philippines." Situated on the island of Luzon, as Igorot. The Big Smoke may refer to any volcanic episode : we do not have a comprehensive history here. Please describe more of the postcards : sights, garments ... --DLL 21:09, 24 February 2006 (UTC)[reply]

black swedes[edit]

My family came from Sweden, and have always been refered to as Black Swedes because of a darker complection and dark hair. I know my family existed in Sweden prior to 1640, but not where they came from. I have heard there are black Swede societies in Sweden. Can you help with info.Wayne

  • The best I can do is point you to the analogous situation of the Black Irish. Remember, if you go back 10 generations (about 220 years) you could have as many as 1024 10th-generation grandparents (less if some cousins married each other). With a little bit of genetic luck, as little as one non-blonde ancestor could leave you with swarthy skin and brown or black hair, because the several genes involved in that process are somewhat recessive to blonde hair/blue eyes/white skin and dominant to brown and black.--M@rēino 18:30, 23 February 2006 (UTC)[reply]

Their are tones of reasons why you may be dark skin. Sweden had some american colonies, perhaps one of your anscestors took a native wife. You may also have European jewish blood. Furthermore you may have mediterainan features. Historically throughout europe their where dark skin white people.

Not to forget the prolific viking slave trade and excursions to Spain and Italy. In any event, it could have happened at any period of your ancestral tree. It could also be a genuine genetic mutation/anomaly/variation - stranger things have happened Celcius (Talk) Wiki be With us! 15:33, 24 February 2006 (UTC)[reply]
  • I've lived in Sweden and never heard the term 'Black Swede' in any context, modern or historical. The only ethnic groups of any substantial size in the 17th century which looked significantly different from the Swedish population would be Sami (who are less blond than Swedes), Wallons (a large number of which immigrated to Sweden in the 16th century) and Gypsies (the first mention of them in Sweden was in the 15th century). Although it seems likely to me that the term is simply a neologism based on the stereotype that all Swedes are blond, which just isn't true and hasn't ever been. --BluePlatypus 10:21, 28 February 2006 (UTC)[reply]

Movie character Walter Fielding[edit]

What movie has the character Walter Fielding in it, and who plays the character? --Jeanne

The Money Pit, Tom Hanks. —Charles P._(Mirv) 18:47, 23 February 2006 (UTC)[reply]

Politics; history of Dick Cheney's history of violating commonly held democratic values[edit]

I would like to compile a history for personal study on Dick Cheney's political background with regard to actions he has taken in former administrations and in the present one, that would be seen as anti-democratic by rational liberals/progressives. Can you help me with where/how to look in wikipedia for detailed info on this? Thanks - Gerry

Have you tried Dick Cheney. I have to warn you, Wikipedia has a strong non-point-of-view policy. So, the article is not written stricly by "rational liberals/progressives". It is written by your average normal people. --Kainaw (talk) 19:14, 23 February 2006 (UTC)[reply]
Well, actually it's written by a diverse bunch of wackos, but they avoid presenting their wacko opinions as facts. =P —Keenan Pepper 00:14, 24 February 2006 (UTC)[reply]
I represent that statement. --Halcatalyst 05:19, 24 February 2006 (UTC)[reply]

Thanks for the tips -- simplicity prevails again! searched Dick Cheney and ta-da, enough facts to make a Texas oil man swear off Lone Star -- or just help a very average citizen connect with tools to navigate the daily shell game - Chinrest

If you want a presentation of Cheney's crimes that's unhindered by Wikipedia's policies and not edited by the right-wing wackos here, you might try these sources:
  • www.commondreams.org/ (enter "Cheney" in the search box and you'll be set for days);
  • demopedia.democraticunderground.com/index.php/Dick_Cheney Cheney article in Demopedia;
  • www.dkosopedia.com/index.php/Dick_Cheney Cheney article in dKosopedia.
There's plenty about Cheney's pre-VP career that should be added to our article. After you've done your research, I hope you'll be able to edit Dick Cheney. Remember not to comment on how heinous a particular action was. The NPOV approach is that you simply report the facts (and cite your sources), leaving the reader to make the judgments that would be obvious to any decent person. JamesMLane t c 13:05, 26 February 2006 (UTC)[reply]
There is one more note on NPOV. You should not delete cited facts that you do not like. For example, a child pornographer was arrested and convicted of 100 counts of child pornography. The USA PATRIOT Act was used in the investigation. One user was extremely upset about having the fact that he was convicted added to the article. He wanted it to imply that anyone can be arrested at any time - not that someone who was arrested was later convicted. --Kainaw (talk) 15:44, 26 February 2006 (UTC)[reply]
Thanks, James ... Thanks, Kainaw; this has been my first venture into the wikipedia world; maybe after I've absorbed what I can from the sites you pointed out, my wife and I (who's actually a better researcher and writer)will take a shot at adding to the article -- Chinrest
Small note: don't start a line with spaces; that makes the line run on to the right into infinity (well, if you keep in typing that is :) ). If you want to indent, use colons (:). Done so for you. DirkvdM 08:53, 27 February 2006 (UTC)[reply]

I want to find out the author of All the way, a biography of Frank Sinatra[edit]

Michael Freedland. www.amazon.com/gp/product/0312191081/ Chuck 19:58, 23 February 2006 (UTC)[reply]

Italian Artist Fillippo Indoni[edit]

I am trying to find out information on this artist I have one of his water colors. Can you help get more info on him. Or let me know where I can look him up thank you--68.194.131.37 21:06, 23 February 2006 (UTC)[reply]

  • Google came up with some 300 references, including www.redferngallery.com/bioFilippoIndoni.html this blurb. --Halcatalyst 21:57, 23 February 2006 (UTC)[reply]


Thank you, I feel foolish I forgot all about Google!!!! --68.194.131.37 14:24, 24 February 2006 (UTC)[reply]

British-French concorde[edit]

The British and French were antagonists during the Napoleonic era, and by World War I they were allies. What were the reasons for, and the process and main dates, of their reconciliation? What did it have to with, for example, the Franco-Prussian war of 1870 and the rise of German nationalism? --Halcatalyst 21:52, 23 February 2006 (UTC)[reply]

The short answer is that when Germany showed that it had become more powerful then France, the traditional English fear of Continentals switched focus to Germany. In previous centuries it had been the Spanish (even for a short while the Dutch). Jameswilson 00:47, 24 February 2006 (UTC)[reply]
Britain and France have been fighting each other since those 2 countries existed. Infact the last centuary is the only one where they haven't gone to war (& even that is debatable in view of the Destruction of the French Fleet at Mers-el-Kebir & Vichy France). However for various periods of time 'between the wars' Britain & France have collaborated when it suited their interests. In the 19th C for example on the Suez Canal & the Crimean War. Usually it was to present a united front against some other power such as Russia or the Ottoman Empire but since 1870 the other main power in Europe was Germany. This was always very loose though & nothing really formal until just before the Great War & even then their co-operation was considered an entente cordiale rather than a strong alliance. Interestingly after that war Georges Clemenceau was talking with David Lloyd George and said something like "and now you will go back to opposng us" Lloyd George replied "that has always been the policy of my country". However the resurgence of Germany again intervened & they became allies again. Though after WWII traditional squabbling & disagreement returned,, though without war between the two. AllanHainey 08:34, 27 February 2006 (UTC) (sorry, I forgot to sign earlier - AH)[reply]
--DLL 20:51, 24 February 2006 (UTC)[reply]

Music[edit]

Guys, I remember seeing a TV show which, as I recall, showcased a strange Mongolian singing style. The singer would sing a sort of scratchy bass note, but then add a harmony on top of that, all with one voice at the same time. the same principle as, say, a bagpipe, but with only one man's voice. I know this is a poor description, but I really am confused about what it is in the first place. I'm looking for the name of the style, and any information you might find. Thanks, --67.142.130.32 22:19, 23 February 2006 (UTC)[reply]

You're probably thinking of throat singing. Chuck 22:59, 23 February 2006 (UTC)[reply]
It's not the same principle as a bagpipe, but it is the same principle as harmonics on a guitar or violin. The article overtone singing is very detailed. —Keenan Pepper 00:18, 24 February 2006 (UTC)[reply]

Rabat[edit]

Requested information on Rabat on 23 February, 2006. Wanted to say thank you for the information. It was extremely helpful and the response was amazing.

Thanks Again Marion

Carrie Underwood management[edit]

I was hoping someone could help me find out who Carrie Underwood's management company is? I know she is with Arista records but I don't think they are also her management company.

Thanks for your help!

23:03, 23 February 2006 (UTC)

Since she won American Idol, I think Simon Fuller is still in charge of her management, but I may be wrong. Mike H. That's hot 02:33, 24 February 2006 (UTC)[reply]

Christian influence in the works of C.S. Lewis[edit]

I'm currently reading the Chronicles of Narnia. After I finish each of the books in the series, I've been reading the Wikipedia entry that book. Most of the entries for the individual books has a section on the Christian teachings or biblical references in that book. The article for The Silver Chair doesn't though. {{spoiler}} At then end of TSC there is a point at which, after having died in Narnia, King Caspian is under water in a stream in Aslan's world. Aslan, a lion, has Eustace, a boy from our world who is visiting Narnia/Aslan's world, go to a thorn bush and pick the largest thorn. Aslan then has Eustace stab his paw after which he drops one drop of his blood in the stream. When this happens, King Caspian grows young again and emerges from the water. It is then explained that Aslan's world is some sort of heaven.

So, what I'm wondering is if anyone can tell me what biblical verses this scene may reference. I seem to remember one about drinking the body and blood of Christ to be saved or something. Thanks for your help, Dismas|(talk) 23:22, 23 February 2006 (UTC)[reply]

  • sounds to me like a cross (no pun intended) between that and the ascension - Matthew ch.28, Mark ch. 16, Luke ch. 24. Grutness...wha? 00:52, 24 February 2006 (UTC)[reply]
Yes, I'm sure that's the crux of the biblical refs. StuRat 01:17, 24 February 2006 (UTC)[reply]
Thanks! Dismas|(talk) 14:55, 24 February 2006 (UTC)[reply]
You should remember that Lewis did not intend the Narnia books to be allegories. In other words there was not intended to be a one-to-one correspondance between things in Christianity and things in Narnia. The blood of Jesus is powerful in Christianity, whether you take that symbolocally or not, and that's probably at least part of the derivation of this scene. DJ Clayworth 18:58, 24 February 2006 (UTC)[reply]

February 24[edit]

Early or first gun scopes?????[edit]

Where can I find info about the above? Thanx 01:02, 24 February 2006 (UTC)

probably here, or more specifically here, if you're completly stumped--205.188.117.12 05:02, 24 February 2006 (UTC)[reply]

This question was asked a few days ago. StuRat 01:13, 24 February 2006 (UTC)[reply]

And you can find that discussion here. --Halcatalyst 05:12, 24 February 2006 (UTC)[reply]

Legalize Sports Gambling?[edit]

I would like to know the Pros and the Cons of Legalizing Sports Gambling across the United States.

You should really do your own homework, but nice I'm feeling nice here are some possible effects. You can work out which ones are pros and which are cons.
  • More government income on betting tax
  • More gambling addicts because of easier access to betting
  • More opportunities for corruption in sports
  • reduced income to criminal gangs who can't make money running illegal gambling
  • less money spent on tracking down illegal gambling
  • more money spent regulating gambling
  • Wayne Gretzky won't have a gambling scandal to come back to after the next olympics.
  • People don't have to go to Canada to bet on sports
DJ Clayworth 18:52, 24 February 2006 (UTC)[reply]

KFA[edit]

Does any one have any information about the Korean friend ship association?

Ireland[edit]

Which parties in Ireland are conservative? and which paries are left leaning?

  • Yes, but then he'd stop spamming this page, and I wouldn't have all these fun questions to answer--205.188.117.12 04:59, 24 February 2006 (UTC)[reply]
  • Actually, it's not spam, since he has to type it in himself. It's merely child's play. 205.188.117.12, why don't you come out of hiding? --Halcatalyst 05:06, 24 February 2006 (UTC)[reply]
  • Behind your IP number. Why not opn up a free account with benefits? - Mgm|(talk) 08:53, 24 February 2006 (UTC)[reply]

historical leader perspecives[edit]

Would Churcill be Blairite? Would Lincoln be Bush supporter?

The 1800s versions of BOTH US parties would probably be about on par with today's "conservative" party, since apaprently while conservative doesn't apply to fiscal policy, it does stand for "we haven't changed our party's platform since both US parties were bible thumping twits"--205.188.117.12 04:58, 24 February 2006 (UTC)[reply]

Reference desk is not place for speculation. We're here to answer factual questions only. ☢ Ҡiff 04:27, 24 February 2006 (UTC)[reply]

Is that a fact? DirkvdM 10:12, 25 February 2006 (UTC)[reply]

Springfield Mass[edit]

What is the cause of this cities organized crime problem? Why is the city so poor? Cara Hyder, do you still live in springfield?

  • Please get back to your homework. --Halcatalyst 04:43, 24 February 2006 (UTC)[reply]
Try reading our Springfield, Massachusetts and organized crime articles. --Shantavira 13:02, 24 February 2006 (UTC)[reply]

George W Bush[edit]

While not perfect, Bush is clearly well meaning; why is this charming farmer-president so hated.

Farmer? Didn't know connecticut had farms, learn something new everyday--205.188.117.12 04:55, 24 February 2006 (UTC)[reply]
I thought the Bushes were from Texas/Maine (and Jeb from Florida). StuRat 05:07, 24 February 2006 (UTC)[reply]
If by texas you mean connecticut, than yes.. but you can't be a connecticut cowboy, or new england good ol' boy, so it doesn't hurt to just make stuff up--205.188.117.12 05:24, 24 February 2006 (UTC)[reply]

The Reference Desk is not for rhetorical questions. —Keenan Pepper 04:09, 24 February 2006 (UTC)[reply]

He was born and educated in conneticute, but he spent his preteen years, and most of his adult life in Texas!
Yes, if you use enough exclamation points, it must be true!!!!1!!1!!--205.188.117.12 06:09, 24 February 2006 (UTC)[reply]
If you don't believe he's a Texan, read our article: George W Bush. Here is an excerpt:
"The eldest son of former President George H. W. Bush and his wife Barbara Bush (née Pierce), George Walker Bush was born in New Haven, Connecticut. He moved to Texas with his family at the age of two and he identifies himself as a native Texan." StuRat 06:45, 24 February 2006 (UTC)[reply]
Also see Kennebunkport for a description of the Bush home there. StuRat 06:49, 24 February 2006 (UTC)[reply]
Getting back to the original question. I can only speak for myself but mostly its due to the persistent lying, hipocracy, corruption & support for shady businesses of his administration together with his record of passing restrictive legislation, seeking more & more power for the executive branch, disregard & discrediting of science he doesn't agree with, persistent flouting of international law, the geneva convention & basic human rights. His administration's record of mis-management, control & restriction of the media, ignoring important information, attempting to link every action they're in favour with to fighting terrorism, harnessing foreign & domestic aid for the purposes of supporting Bush's religious beliefs, excessive links & influence of big business, support for foreign dictators & other torture supporting regimes, unilateral military action & starting wars on a false premise. I'm sure there are more reasons but I'm just going off the top of my head. Take a look at news reports of the political record of his administration (probably not mainstream American news reports though - certainly not Fox) & you'll see many reasons why people hate it, & its most prominent figurehead. AllanHainey 11:32, 24 February 2006 (UTC)[reply]
  • I agree with you. So big deal. Why don't you go discuss it on some talk page, or some discussion room or chat room somewhere, and don't clutter up the reference desk. --Halcatalyst 18:06, 24 February 2006 (UTC)[reply]

While some countries look for 'well-meaning' in their leaders, others look for competence and honesty. DJ Clayworth 18:41, 24 February 2006 (UTC)[reply]

I dont know what it is about Bush really. Its partly an image thing. Two unscientific examples. Most of my right-wing English friends dont bother to defend him personally even when they defend American foreign policy, for instance. He just doesnt seem to inspire their respect. Some of them think hes just a puppet and the real power must be exercised by Cheney and others. My very right-wing Australian aunt cant stand him either. Thinks he's thick, out of his depth, dangerous, certainly not well-meaning. The appeal he has to American people just doesnt seem to translate automatically to right-wingers in other countries, even English-speaking ones. Its different from Reagan who you could see was quite amiable as a chap even if you thought he wasnt really bright enough to be a world leader. I dont know - these are just random observations, but Bush just doesnt come across somehow. Jameswilson 00:24, 25 February 2006 (UTC)[reply]

  • Since when have we started calling a former oil CEO (Arbusto) and owner of Texas Rangers a farmer? Owning a ranch makes one a farmer as much as owning a Hummer makes one a laboring mechanic. --66.81.192.102 09:56, 25 February 2006 (UTC)[reply]
    What has a Hummer to do with being a mechanic? :-) Lupo 20:26, 25 February 2006 (UTC)[reply]


  • But in the first place, you may want to believe that he is "well meaning", but many, many, many people truly believes exactly the opposite. LG, 2313H EST, Feb 27, 2006

American culture[edit]

Why is the U.S. family farmer so revered; they usually are to inept to get off the farm or are rich landowners made even richer by gov subsidies?

Because every super power needs it's cash crops, otherwise we here in the evil communist pinko north east (home of the stock exchange, and seat of the worlds largest economic power), might have to pay an entirely different group of yokel farmers for our cash crops, and that might actually have some effect on the...hum, good point, screw you deep south/midwest, we're starting our own country and we're importing our damned vegtables from mexico!--205.188.117.12 04:52, 24 February 2006 (UTC)[reply]

Rhetorical questions are not appropriate here. --Halcatalyst 04:44, 24 February 2006 (UTC)[reply]

  • One of the reasons is that United States political system inproportionally favors the less populated states. So theoretically, farmers in these states have more political power than say the average new yorker. --Vsion (talk) 08:41, 24 February 2006 (UTC)[reply]
Nostalgia for the time when they were the backbone of the American economy & made up the majority of the population. A general preference for the country life (represented by the farmer) than for city life (yes most people live in the city but lots would prefer to live in the country) seems the most lkely to me. AllanHainey 11:31, 24 February 2006 (UTC)[reply]

China[edit]

Are Jiang, Deng and Hu loved by their people?

From my quick poll of 5+ people, mostly young adults from PRC,
  • Jiang: not much, regarded as a transitional figure
  • Deng: generally "respected" for opening up China economically,
  • Hu: not sure, judgment reserved. Note that Hu spent most of his career in rural regions, including Tibet; his ideology is largely unknown.
Of course, it is a very noisy sampling and result would vary with different demographic groups. Ask more people to get a clearer picture. --Vsion (talk) 07:58, 24 February 2006 (UTC)[reply]

Yahoo as moral agent?[edit]

How can yahoo justify helping the red chinese government oppress their people. Is possible that for this the U.S. can bring criminal sanctions against yahoos officers?

  • No. They have violated no US criminal laws. --Halcatalyst 04:47, 24 February 2006 (UTC)[reply]

Aryan National socialist party[edit]

Why is that the Aryam National socialist party (a Iranian group) who are fairly liberal by nazi standards, want to be identified with the nazis, all it does is hurt their credibility?

Is it possible that they don't want to be identified with the Nazi misuse of the term, but rather the true meaning of the term Aryan ? StuRat 04:57, 24 February 2006 (UTC)[reply]
  • Then how did I know you were thinking of the number 23?--205.188.117.12 05:20, 24 February 2006 (UTC)[reply]
  • <bzzzz> sorry, I was the one thinking of the number 23. Thanks for playing! Grutness...wha? 09:31, 24 February 2006 (UTC)[reply]
  • Can I just say that I'm going to try to use the phrase "fairly liberal by Nazi standards" at least once over the next month. DJ Clayworth 18:32, 24 February 2006 (UTC)[reply]
  • is there a prize for being the first person to spot where you use it? :) It's a good phrase, though. Grutness...wha? 06:37, 25 February 2006 (UTC)[reply]
  • i know that during world war I or II the shah of Iran changed the name of the country from Persia to Iran in order to pander to Hitler (Iran basically means Aryan) this was probably for political reasons and the fear of individualist political systems. Iran is deemed to be the origin of the Aryan race. uhh did i even answer your question? no - but i said some stuff. give me a cookie. Amirman 23:44, 2 March 2006 (UTC)[reply]

Chinese leaders[edit]

Why are all current chinese leaders engineers? Why is it none of them served their country in the military?

  • Interesting question. You can find information at the article on Chinese politics, which has links to the current Chinese leaders. --Halcatalyst 04:59, 24 February 2006 (UTC)[reply]
Apparently, most of these leaders started their careers in state-owned enterprises, which are those massive industrial organizations or bureaus in China. The most outstanding engineers will get promote to become bureau chiefs, which can be regarded as at the lowest ring in the hierarchy of power in the Chinese Communist Party, and from there they start to climb the political ladder to become the country's leaders. Professional administrators or executives are less common in China, hence most of these bureau chiefs are engineers or scientists by professions. Also most of the brightest students in China enroll in Tsinghua University, which is primarily an engineering university, unlike in other countries where most go to law or medicine schools.
In addition, China's history consists of a long series of imperial dynasties with numerous infighting and chaos before finally becoming a Republic in 1911. Traditionally, Chinese did not have a high regards of the military and were even suspicious of it. Therefore the Party today still exercises a very strong control over the military. Although the People's Liberation Army is now highly respected, a national leader with a military background would be highly unpopular in the present days, both within the Party and among the people. --Vsion (talk) 07:34, 24 February 2006 (UTC)[reply]
That's refreshing. Most leaders of countries are either military (very roughly in dictatorships) or former social sciences students. It's nice to know there's a country where the people in power actually know things. Any more such countries? I might emigrate (but rather not to China). :) DirkvdM 10:23, 25 February 2006 (UTC)[reply]
The current chancellor of Germany is a physicist (she even has a PhD, to boot!). — QuantumEleven | (talk) 08:43, 27 February 2006 (UTC)[reply]
  • A PhD is pretty much the entry level requirement for physicists. --Halcatalyst 23:33, 28 February 2006 (UTC)[reply]

Questions[edit]

Are all questions not subjective?

Here is one that isn't: "How many protons does a hydrogen atom have ?" Answer: one. StuRat 05:03, 24 February 2006 (UTC)[reply]
Heck, even simple relativistic adjustments make that a subjective statment--205.188.117.12 05:14, 24 February 2006 (UTC)[reply]
Please explain. StuRat 06:51, 24 February 2006 (UTC)[reply]
  • Not subjective, means the question is objective. So they're really asking "are all questions objective?" If properly phrased, questions are objective, but the answers can be subjective to boot. "Who is the best musician in the world?" would be a question that yields subjective answers. - Mgm|(talk) 08:52, 24 February 2006 (UTC)[reply]


"One" may or may not be a subjective answer to the question. But the question is about questions, not about answers. Is the question "How many protons does a hydrogen atom have?" a subjective question? The very fact that you disgree on the answer to what seems to a scientist like a simple question with only one answer proves that it is a subjective question. It's also a subjective matter deciding what the original question was really asking. The placement of "not" is interesting. The question could be viewed as neutrally asking "Are there any questions that are not subjective? (and if so, name one)". Or maybe it's more like "I assert that all questions are subjective. Don't you agree?". The way the reader perceives the question would influence the way they answer it. Any ambiguity guarantees the subjectivity of the question. Laywers, sociologists, psychiatrists, therapists and statisticians would all say that the way a question is framed has a profound bearing on the answers obtained. Maybe there is a good example of a truly non-subjective question, that is, a question the meaning of which can be interpreted in one and only one way (I didn't say could be answered in only one way), but I can't think of one. JackofOz 09:15, 24 February 2006 (UTC)[reply]

If the standard is that one person disagreeing with what everyone else believes makes it false, then I suppose every question must be considered subjective. For example, somebody will say "yes" when asked if the world is flat, and even though they are an idiot, we still have a disagreement, and hence a subjective question. StuRat 08:26, 27 February 2006 (UTC)[reply]

From space, few would dispute the Earth is round. To a person on the ground, for most practical purposes the Earth is flat, and saying so doesn't make them an idiot. But this is not even about scientific truths or mathematical certainties, it's about questions, language, communication and meaning. I think there is always more than one way to understand a question, and they all have their own validity. It depends on one's frame of reference.

  • "Do you have any apples in that basket?" - No.
  • "Do you have some apples in that basket?" - No.
  • "Do you have apples in that basket?" - Yes. JackofOz 20:28, 27 February 2006 (UTC)[reply]

lawyers[edit]

If a new lawyer wanted to handle refugee claim in the sprigfield Mass area, where might he apply for work?

  • true, but it would probably make sense to apply for one in Springfield. Or Sprigfield, as the case may be. (S)he might be able to get a job with Lionel Hutz. Grutness...wha? 09:43, 24 February 2006 (UTC)[reply]

The lawyer might contact the National Lawyers Guild for leads. JamesMLane t c 13:13, 26 February 2006 (UTC)[reply]

Thursday evenings[edit]

Are Thursday evenings a particularly good time to avoid homework? --Halcatalyst 05:03, 24 February 2006 (UTC)[reply]

  • Yes, except technically it's friday morning right now--205.188.117.12 05:11, 24 February 2006 (UTC)[reply]
    • That would entirely depend on the time zone you're in. Besides, he could be asking for next week's planning. -Mgm|(talk) 08:54, 24 February 2006 (UTC)[reply]
    • Is asking and answering one's own questions anonymously particularly fun? --Halcatalyst 05:15, 24 February 2006 (UTC)[reply]

The family guy and American dad[edit]

Stan Smith = George Jetson Both college educated introverted, company men Peter Grithin = Fred Flinston less educated extroverted men who put work second.

Any comment?

Wait... wait... wait... wait... Hold on a second... Are you saying that Hollywood rips off old ideas that ripped off older ideas that ripped off older ideas... and they tell us it is new!? Oh my god. I feel so gullible. At least I can rest safely knowing that the Simpsons is completely original without ever stealing a joke or parodying an older show. --Kainaw (talk) 14:36, 24 February 2006 (UTC)[reply]
But recursion is always kinda funny, especially when it gets out of hand. --17:58, 24 February 2006 (UTC)

Homework[edit]

I am a second semester third year law student, I do keep up with my homework, though I keep irregular hours. Though I thank the fellow wikipedian for his concern.

  • Are you planning to work with refugees in Springfield? --Halcatalyst 05:16, 24 February 2006 (UTC)[reply]

I have not descided, however I was exposed to the subject mater via a international law class I recently took.

Masturbation[edit]

In the Catholic church is this still a mortal sin? Or has it been down graded?

Abortion law[edit]

why are democrates so vehemently for abortion?

  • Why are you so vehemently opposed to sticking to one question at a time?--205.188.117.12 05:22, 24 February 2006 (UTC)[reply]

If US Democrats were "vehemently for abortion", as you suggest, then they would propose laws REQUIRING ABORTIONS. They only want to keep legal a woman's right to choose an abortion, this is not the same thing at all. StuRat 10:11, 24 February 2006 (UTC)[reply]

For future reference (please see the top of the page), it's always helpful if you're a little more specfic as to whom you're referring - there are other "democrates" in the world besides the US Democratic Party.
Also, I think it would be a gross overgeneralisation to say that "all democrats are pro-choice", there are plenty of people in the Democratic party who oppose abortion, just like there are Republicans who don't oppose it. You may be interested in reading about abortion, pro-life and pro-choice. — QuantumEleven | (talk) 13:18, 24 February 2006 (UTC)[reply]
Capitalisation would have helped here; Democrats. By the way, I thought he was asking about some Greek philosopher, but it turns out there is no Democrates. :) DirkvdM 10:28, 25 February 2006 (UTC)[reply]
Democritus ? StuRat 05:40, 1 March 2006 (UTC)[reply]
shhh, I know, it was just a lame joke about the spelling... DirkvdM 07:37, 2 March 2006 (UTC) [reply]

Hatred[edit]

Why does everybody seem to hate me?

  • Must be one of these out tonight, turned all us normal editors into these, you'd grumpy too if you were trying to type with furry paws--205.188.117.12 05:30, 24 February 2006 (UTC)[reply]
Paranoia? Or more specifically a Persecution complex? GeeJo (t) (c)  16:28, 24 February 2006 (UTC)[reply]

Judiaism,[edit]

Do Jews believe in a heaven and a hell?

  • There's no need to be rude. - Akamad 03:39, 25 February 2006 (UTC)[reply]
Below are two questions where I deleted responses from 205.188.117.12, which is a shared AOL account. It was odd how all of these sort of responses occured on the same day. I drew the line on the ones that were also unhelpful. I still would've left them in if they were from a registered user. I guess you can call me biased. -LambaJan 05:58, 1 March 2006 (UTC)[reply]

Sex[edit]

Why do people enjoy sex so much? While it seems fun, after a couple times it gets trite.

  • Better than sitting here asking annoying rhetorical questions on a thursday night?--205.188.117.12 05:31, 24 February 2006 (UTC)[reply]
If you think it's trite, you're not doing it right. JackofOz 06:12, 24 February 2006 (UTC)[reply]
It's a biological thing. As you get older, you will appreciate it a lot more. --Shantavira 13:07, 24 February 2006 (UTC)[reply]
I don't know about that, I think that when you're young you appreciate the chance to get some as well. As far as it being trite, maybe you're just not with someone who makes it enjoyable for you. Just a thought. Dismas|(talk) 14:46, 24 February 2006 (UTC)[reply]
You only fully appreciate sex when you share it with another human. That would be the immediate sollution to your problem - although possibly not the final. Celcius (Talk) Wiki be With us! 15:22, 24 February 2006 (UTC)[reply]
No one's mentioned evolution yet? The animals that didn't enjoy having sex died out, for the obvious reason. —Keenan Pepper 04:34, 25 February 2006 (UTC)[reply]

UAE[edit]

Is this nation a democracy, does it respect fundimental liberties?

  • Try reading United Arab Emirates and Politics of the United Arab Emirates. - Mgm|(talk) 08:59, 24 February 2006 (UTC)[reply]
    • No, it's not a democracy and there are a number of outstanding criticisms of its human rights record. Relative to the region, however, it's moderate, modern, and has achieved a number of notable successes in regards to human development (the education of women, for example). Labour rights remain nascent, but there is a growing movement to adopt greater safeguards for workers. It's also radically decentralized, so it's hard to speak for the entire place. If the current sturm und drang over Dubai in the U.S. is what prompted this, note that Dubai is essentially a city-state. You can drink there, but take a step over to Sharjah and you'll find it's dry.
    • One final note: the recent quote from a Senator in New York that the UAE "has a long history of supporting terrorism" is idiotic, insulting, and untrue. Marskell 10:47, 24 February 2006 (UTC)[reply]
You seem to mix up two things. Democracy simply means the population has a say in politics. There are various gradations in that, but a democracy may very well ignore fundamental liberties (whatever those are - or are the people to decide that too?). DirkvdM 10:32, 25 February 2006 (UTC)[reply]
Uh, what are the two things mixed up? "Democracy simply means the population has a say in politics" is rather reductionist. I'd say democracy means some form of franchise and none exists in the UAE. Even taking it in the more simple sense, people do not, basically, have a say in politics here. Marskell 09:49, 28 February 2006 (UTC)[reply]

Japan's immigration policy[edit]

I got the gist from reading a forum somewhere that Japan hardly takes in any immigrants. What is Japan's immigration policy? Can someone show me where to read more info about it or perhaps post it here directly? --Shultz III 05:35, 24 February 2006 (UTC)[reply]

Japan is a extremely homogenius country, they have strict immigration laws. --(Unsigned)
So then, where can I read more details about these laws? --Shultz III 06:38, 24 February 2006 (UTC)[reply]
Check out Japan; I'm sure there's a demographics section that can explain the issue. Mike H. That's hot 10:41, 24 February 2006 (UTC)[reply]
I heard it is complicated, mininum 5 years of living in Japan to qualify for permanent residency first and does not allow dual citzenship. I can't help you much, but you may be interested in the story of Arudou Debito, a naturalized Japanese. --Vsion (talk) 10:58, 24 February 2006 (UTC)[reply]

Julius and Ethel Rosenberg[edit]

Guilty or innocent?

I vote for guilty. StuRat 10:05, 24 February 2006 (UTC)[reply]

They were guilty, and it's not a question of voting. The court made its decision, that's the definition of guilty. (Just as John Wilkes Booth, Lee Harvey Oswald, James Earl Ray, Sirhan Sirhan, Albert de Salvo, O J Simpson, Michael Jackson and stacks of others are innocent). But whether the Rosenbergs actually did what they were alleged to have done, that's another question. JackofOz 10:18, 24 February 2006 (UTC)[reply]
My definitions is different, I only consider them guilty if they actually did it, regardless of the court verdict. I call the court verdict being "found guilty" to distinguish it from actually "being guilty". With any luck, there will be at least a rough correlation between the two. StuRat 11:04, 24 February 2006 (UTC)[reply]
It seems pretty clear that Julius was guilty of some form of espionage (even if not exactly that for which he was charged), while Ethel had only an accessory role at best. Ethel's life could have been rather easily spared if she had turned on Julius, or if Julius had taken the fall and pleaded guilty to spare her. So basically Ethel died just to keep up the pretence of Julius's supposed mythical absolute innocence. AnonMoos 13:55, 24 February 2006 (UTC)[reply]
Both of them could have been spared if they confessed or cooperated. It's somewhat of a mystery why they didn't, and why they instead chose to go to their graves for that, leaving two kids as orphans.--Fastfission 17:29, 24 February 2006 (UTC)[reply]
Our page on them goes over some of the main controversies in a fairly systematic fashion. Most of the controversy does not revolve, at this point, around whether or not Julius committed espionage, but on subsidiary issues, such as whether they were given a fair trial, whether the punishment was fair, whether the evidence was good, etc. --Fastfission 17:29, 24 February 2006 (UTC)[reply]
As an aside, primarily to Jack of Oz, I'd say you were not using the terms correctly. "Guilty" and "innocent", both connotationally and denotationally, describe whether or not someone has actually committed an infraction (though they are often used in reference to decisions made by a court). I'd humbly suggest the words "convicted" and "acquitted" as substitutes for your purposes. Preserving this distinction allows one to say (hypothetically) that O. J. Simpson was acquitted of a crime he is guilty of, and that the Rosenbergs were convicted of a crime they were innocent of. Having made this pedantic and trivial point, I should probably toddle off to Reference desk/Language where such things are more appropriate. :-) Jwrosenzweig 05:22, 27 February 2006 (UTC)[reply]
I think that's a very important distinction. I can't stand it when they say somebody is "guilty" or "innocent" when they are really only referring to the court verdict. I find "not guilty be reason of insanity" to be particularly annoying, the verdict should be "guilty AND insane", now there are two reasons to lock them up. StuRat 08:36, 27 February 2006 (UTC)[reply]
Also, in the US at least, nobody is ever found "innocent." The two verdicts that can come out of a trial are guilty and not guilty. (Discounting the cases where no verdict is returned). --LarryMac 20:41, 27 February 2006 (UTC)[reply]
I don't think it's trivial or pedantic at all, Jwrosenzweig. You make a good point. I have a great respect for the rule of law. This includes the possibility of a convicted person appealing an unsound or unfair court decision. In general, though, we rely on the courts to get it right, otherwise we may as well abandon the justice system altogether. Uninvolved third parties have no say as to whether a convicted person was really "innocent" or an acquitted person was really "guilty". That is simply a question of personal judgement or personal opinion that is irrelevant to the person's circumstances. Which is perhaps why OJ Simpson is a free man. Popular opinion has it that he really killed his wife, but the court disagreed. The court, and not popular opinion, is where such decisions are made. On the other side of the coin, the court convicted the Rosenbergs and sent them to the electric chair. Apart from whether one agrees or not with capital punishment, the court has spoken. Just as with OJ, others are free to form an opinion as to whether the outcome was fair and just, but such opinions have no bearing on their case. JackofOz 06:56, 1 March 2006 (UTC)[reply]
StuRat, your last comment is interesting. I hope you were having one of your usual jokes, because if not, I wonder if you'd feel the same way if one of your loved ones was facing justice. "Guilty" includes the concept of intentionality. If I kill you without intending to do so, a just trial would acquit me of murder, but might well convict me of lesser crimes such as manslaughter, negligent driving etc (I know terminology varies, but I hope you understand the distinction I'm making). A particular case of this is where I kill you but I was considered "insane" (however one defines that) at the time. A charge of murder in these circumstances should not succeed because it fails the intentionality test. There is a reasonable doubt about my motives and intentions, and if properly directed a jury should acquit. That is the whole basis of the "innocent until proven guilty" principle. However, letting a person whose insane actions have already killed somebody go free would in many cases be irresponsible, and some form of treatment where their movements are restricted would be called for, not as a punishment for them but to protect the public. I assume that's what you meant by "lock them up". (Now, I must work out how to kill StuRat but make it appear like an accident ....hmmm :-) JackofOz 06:56, 1 March 2006 (UTC)[reply]
I think people need to be locked up if they are dangerous, the reason really doesn't matter. If they kill for profit, because of a nuero-receptor imbalance, they ate too many Twinkies, or they are possessed by a demon. It doesn't matter to me, they are all dangerous and should all be locked up. What terminology is used doesn't much matter. StuRat 01:25, 2 March 2006 (UTC)[reply]

Pepsi Cola?[edit]

<Insert random question here>--205.188.117.12 05:39, 24 February 2006 (UTC)[reply]

Mommie Dearest liked it. Mike H. That's hot 06:26, 24 February 2006 (UTC)[reply]
Pepsi Cola? 42. Hmm.... I guess that's not it either. I guess I'll stick with "How many roads must a man walk down?" for now. --Kainaw (talk) 14:27, 24 February 2006 (UTC)[reply]

My mother[edit]

Why does dear mother have such contempt for me?

-Removed rude comment from shared AOL account -LambaJan 07:29, 24 February 2006 (UTC)[reply]
Do you expect us to answer this honestly, or sarcastically? GeeJo (t) (c)  13:03, 24 February 2006 (UTC)[reply]
Assuming your mother is an average female one could speculate upon literally thousands of reasons. Please expand your question to narrow the field of the contempt in question. Celcius (Talk) Wiki be With us! 15:12, 24 February 2006 (UTC)[reply]
I doubt she does. A boy's best friend is his mother. --George 17:38, 24 February 2006 (UTC)[reply]
Indeed. GeeJo (t) (c)  18:03, 24 February 2006 (UTC)[reply]
Thks guys for your references to Hitchcockles and Sophocles, but let's not add to contempt, real or imaginary.
Either a mother loves and admires her children or she is ill ... children can be deluded too. You think that dear mother behaves badly to you : try to talk about it, here if you want, but we are only children of the knowledge universe and maybe you want a nearer ear or a phone number. --DLL 21:13, 25 February 2006 (UTC)[reply]

Animal human hybred?[edit]

Is this scientificlly possibe? Ape man hybreds, is this possible?

probably not. inter-generic hybrids are highly unlikely, especially between complex vertebrates; the only one of which i'm aware is a elephant.se/Motty_the_elephant_crossbreed.php?open=Living%20Elephant%20Species single instance of a hybrid between asian and african elephants. furthermore, humans have 46 chromosomes while the great apes (we'll leave gibbons out of it for simplicity's sake) have 48, which would further complicate matters. not that this has stopped people from trying . . . and that's really the only way to find out.
no successes have been recorded to date. —Charles P._(Mirv) 06:39, 24 February 2006 (UTC)[reply]
See also: Chuman and Ilya Ivanovich Ivanov for more background, and Oliver the chimpanzee for a mistaken case. GeeJo (t) (c)  13:02, 24 February 2006 (UTC)[reply]
Since humans are animals, I don't see a problem here. DirkvdM 10:34, 25 February 2006 (UTC)[reply]
If the questioner is referring to President George W. Bush's call to outlaw such manimals, he was referring to something real but doing so in a way that only brought titters. Single human genes are transposed into other animals and plants. This is done to produce laboratory animals that will exhibit a phenotype that will allow them to be accurate models for drug therapy. The idea is, for example, getting mice with Parkinson's disease so that you can study therapies. (Currently, transgenic mouse models don't get human genes.) Some limited work like this has been done, but no one is trying to create a hybrid. People are just trying to get a few human traits, usually non-functional, so that they can examine them. Geogre 11:02, 27 February 2006 (UTC)[reply]

Sopranos[edit]

Is tpny soprano based on a real life figure?

-Removed rude comment from shared AOL account -LambaJan 07:29, 24 February 2006 (UTC)[reply]
Afraid not, he's a completely fictitious character, though doubtless there are some coincidental similarities between him and various Mafia capos. GeeJo (t) (c)  16:26, 24 February 2006 (UTC)[reply]

You might also get a case of life imitating art where real criminals emulate the behavior of Tony Soprano. Then again, criminals who squeal to the cops may be transformed into sopranos, as a result. StuRat 20:47, 25 February 2006 (UTC)[reply]

Aaaactually, the plot events on the show have been taken from actual "Mafia" events. Tony Soprano is a fictional thing, but there have been numerous journalistic "keys" to the show. Slate magazine (slate.msn.com) had organized crime reporters comment on Season 3, and they made the analogs clear. I also have some personal experience that leads me to believe that some things that alledgedly happened to alledgedly connected persons alledgedly in New York alledgedly showed up in The Sopranos in a transfigured way, alledgedly. Tony? No. Stuff that happens on it? Probably. Geogre 21:47, 27 February 2006 (UTC)[reply]
I remember reading some time ago of some gangsters who got caught because the FBI had a wire in their building & recorded them talking about the Sopranos & saying that the producers/writers had copied things that'd happened to them & people they knew. AllanHainey 15:28, 28 February 2006 (UTC)[reply]

SPAM!!!!1!![edit]

Whose idea was it to name repetitive, unwanted, email, solicitations after a type of processed sandwich meat?--205.188.117.12 06:30, 24 February 2006 (UTC)[reply]

It's almost too funny to be true. -LambaJan 07:23, 24 February 2006 (UTC)[reply]
Note also, 205.188.117.12, that you may have infringed copyright by typing it in capitals in the header (I kid you not (much)). Grutness...wha? 09:51, 24 February 2006 (UTC)[reply]

TRO[edit]

My son (a california resident) was served by his ex-fiance with a temporary restraining order. He plans on attending the hearing and would like to know what to expect. Will this TRO show up in a background check by prospective employers? Thanks for any assistance. T Smith.

A TRO is often entered without notice to the party restrained. Its purpose is to preserve the status quo until a hearing can be held. I don't know whether it would show up or not, but it shouldn't be a big deal. Your son could always explain that it was entered based only on the adversary's say-so, and that once he had a chance to present his side, it wasn't continued. (That assumes he prevails at the hearing. He should get a lawyer if possible.) JamesMLane t c 13:42, 26 February 2006 (UTC)[reply]

Violin virtuoso[edit]

I'm looking for the name of a male violin virtuoso. I'm not sure of the nationality--I believe Polish, but I could be wrong. He made several recordings in the 20s/30s, and died from complications of brain surgery undertaken to cure schizophrenia. I believe that he was in his late 20s (possibly 27) when he died, but other than that, I haven't been able to find him. I've looked on the Violin Virtuoso list, but no luck there. Anyone any idea of the name? Quite possibly it's easy to find, but I'm just blanking. Jude(talk,contribs) 13:14, 24 February 2006 (UTC)[reply]

Josef Hassid? Markyour words 14:09, 24 February 2006 (UTC)[reply]
That's him. Thanks very much! Jude(talk,contribs) 02:15, 25 February 2006 (UTC)[reply]

what are the products band by islamic countries[edit]

kindly give me the list of the produts in denmark--195.245.108.42 15:36, 24 February 2006 (UTC)Abdrrahim Abeeb Olamide[reply]

See Economy of Denmark GeeJo (t) (c)  16:16, 24 February 2006 (UTC)[reply]
Darn. I saw the header and thought this was going to be a question about "The Products" band. What's their latest video on MTV? When is the next album coming out? Is it true that Britney Spears sings backup on it? Just a boring economic question though. --Kainaw (talk) 16:20, 24 February 2006 (UTC)[reply]
Yeah, I'd heard that The Factors were getting back together. GeeJo (t) (c)  17:10, 24 February 2006 (UTC)[reply]
Whatever. Meanwhile, the killings over the cartoons continue. Economic boycott is much more rational. --Halcatalyst 17:47, 24 February 2006 (UTC)[reply]
Uh, there is no "the list" of Danish products. They make thousands of consumer goods, just like just about any other European nation. --BluePlatypus 22:40, 24 February 2006 (UTC)[reply]
I believe the calls were to boycott all Danish products. - Akamad 03:34, 25 February 2006 (UTC)[reply]
Or to buy more Danish products. Dalembert 12:17, 25 February 2006 (UTC)[reply]
As the only products I can think of which are famously associated with Denmark are their bacon and their lager, this may not be a very effective boycott... Shimgray | talk | 13:48, 25 February 2006 (UTC)[reply]
I was wondering about Novo Nordisk insulin, which is the dominant brand in Europe and much of Asia. A new excuse for Muslim kids who don't feel like taking their insulin. alteripse 00:58, 26 February 2006 (UTC)[reply]
Don't forget Lego. Proto||type 16:51, 27 February 2006 (UTC)[reply]

In the Kingdom of the blind the one eyed man is King[edit]

Sir / Madam, Could anyone out there help me find the answer to the origins of the above phrase. I've tried a search on this site to no avail. I believe it has something to do with some form of religious persecution against a group of french citizens from a few hundred years ago. Many poor people had their eyes put out save for one who became their leader to take them home. I think. Can anyone point me to a historical reference to this event please. Thankyou. (194.70.45.170 15:40, 24 February 2006 (UTC))[reply]

The quotation is usually attributed to Desiderius Erasmus, (see Wikiquote for some more choice quotes from him.) GeeJo (t) (c)  15:52, 24 February 2006 (UTC)[reply]
Hmm, apparantly he published it in Adagia, which was a collection of classical quotations, so while it's possible Erasmus was the first to commit the adage to paper, it's very probable he wasn't the first to voice it. Indeed, checking Wikiquote yields a very similar Arabic adage — "The one-eyed person is a beauty in the country of the blind". It is, however, very unlikely that we'll uncover the very first instance of the quotation, so Erasmus seems as good a person as any to pin it on. GeeJo (t) (c)  16:03, 24 February 2006 (UTC)[reply]
I like Tibor Fischer's version: "In the land of the blind, the man who knows how to use the white stick is king." --BluePlatypus 20:06, 24 February 2006 (UTC)[reply]

It's older than Erasmus. According to Stevenson's Book of Quotations it dates to Michael Apostolius's Proverbs ("Caecorum in patria luscus rex imperat omnis"). The first English use of the phrase (again, as in Stevenson's) was "probably in the translation in 1540 by John Palsgrave of the Comedye of Acolastus bu Fullenius. Erasmus's is given as one of many "frequent minor variations in use thereafter". Grutness...wha? 06:45, 25 February 2006 (UTC)[reply]

Proverbs are notoriously difficult to pin down. I once spent weeks trying to find out who first said that "We are dwarves standing on the shoulders of giants." The short answer (i.e. in English) was William Temple, but the long answer (who said it "first") has no end. It went to Charlemagne and beyond and beyond and beyond. Proverbs are like that. Aeolus Donatus was already complaining, "Damn those who said our sayings before us!" Geogre 19:19, 26 February 2006 (UTC)[reply]

Opera Attire[edit]

Tonight i am attending the opening night of an opera ,i will have to wear a jacket , but i dont know which one would be more appropriate , my dress is knee length with a split up one side, it is black and has a low cut back. i have either a calf length mink jacket , a calf length camel colour dress coat or would a sweater be better? I would greatly appriciate a prompt response , as tonight is the opera and i will have to leave at 4:00 to get to dinner on time. Thanks so much!!

  • You're asking about a matter of taste. Like beauty, it is in the eye of the beholder. Miss Manners could give you perfect advice; I would say, avoid the sweater at all cost. Either the coat or the jacket should be fine. --Halcatalyst 17:44, 24 February 2006 (UTC)[reply]
i can't advise you on the details of fashion, but there are practical concerns to consider: what's the weather like? (very cold, wear the mink; moderately cold, the other coat; not really cold, the sweater—if it's suitably dressy—or none of the above.) is there a place to check your coat at the opera house? (mink coats tend to be heavy, bulky, and too warm to wear indoors.) what about its climate control: is it typically hot (+75° F), cold (-66 ° F), or just right? (you might want to wear the sweater in addition to the coat.) how's your tolerance for temperature variations? (or you might not.) hope that helps. —Charles P._(Mirv) 17:48, 24 February 2006 (UTC)[reply]

Legalize Sports gambling[edit]

Should we legalize Sports Betting? By doing so we would be taking money from Organized Crime, as well as taxing the profits so that the States could benefit or do the moral issues outweigh the benefits. And are there anyother benifits or problems with it.

Who's "we"? Which country are you talking about? GeeJo (t) (c)  17:59, 24 February 2006 (UTC)[reply]

Speaking as an inhabitant of a country where it's fully legal, I would say yes. DJ Clayworth 18:25, 24 February 2006 (UTC)[reply]

It relies on subjective opinion - there is no true or false. Personally, I would say you should. Celcius (Talk) Wiki be With us! 19:06, 24 February 2006 (UTC)[reply]

The only problem that I can think of is addiction. If no-one gambles to excess, everyone wins. Unless they lose. And I live in a country where legalised gambling is um, legal.
Slumgum 20:13, 24 February 2006 (UTC)[reply]
Sorry GeeJo I was talking about the United States, in particular New Jersey. There is a current Law proposal to allow Sports Betting in Atlantic City. I was just wondering some of the details/Pros and Cons.
Put it this way, in Britain its legal and there are no calls to ban it as far as I know. Jameswilson 00:31, 25 February 2006 (UTC)[reply]
I think most people prone to addictive behavior in gambling will find other ways. Like internet poker or simply illegal gambling. Tax it and use some of the money to help those who need it - use the remainder for arts and culture. That's the Danish model anyway Celcius (Talk) Wiki be With us! 07:07, 25 February 2006 (UTC)[reply]
While I disagree with this argument (because I feel that everything should be legal and unregulated to speed up "death by stupidity"), some would argue that you are taking something currently limited because it is illegal and turning into something popular because it is legal. The taxes raised are earmarked for good things (like education or the arts), but there is also an increase in poverty and crime to support the habit. So, you have to find more money for welfare and police. In reality, the money raised by the legalization of the illegal activity ends up paying for the trouble the activity causes. --Kainaw (talk) 01:03, 26 February 2006 (UTC)[reply]

Reza Shah Kabir, Pahlavi[edit]

Dera Wikipedia

I would like to know where is really the dead corps of the belated Reza Shah Pahlavi The Great burried as he had been removed from the original grave.

According to our article on Mohammad Reza Pahlavi, he was buried in the ar-Rifai Mosque in Cairo. No mention is made of his body having been removed. David Sneek 20:42, 24 February 2006 (UTC) P.S.: Sorry, you meant Reza Pahlavi, of course.[reply]

Fishhook on Baseball cap[edit]

Can someone please tell me the meaning of wearing a fishhook on the brim of a baseball cap?

Next time you need a fishhook and don't have one, you'll wish you put one on your cap. Actually, this is a style popularized in outdoors magazines. I've primarily seen it in Bass Pro Shops ads. --Kainaw (talk) 20:35, 24 February 2006 (UTC)[reply]

Just to be different, why not glue a baseball onto your fishing hat ? StuRat 20:27, 25 February 2006 (UTC)[reply]

That could come in handy next time you need to catch a catcher. -LambaJan 04:24, 26 February 2006 (UTC)[reply]

Establishment Clause and faith based organizations?[edit]

Has any appellate court addressed the issue of Establishment Clause law and President Bush's executive order relating to faith based organizations?--68.238.200.118 20:19, 24 February 2006 (UTC)Janice 2/24/06 at 3:18 EST[reply]

Just to explain the question to non-Americans: The Establishment Clause prohibits the US legislature from passing any laws regarding the establishment of a state religion. The executive order in question was to end the existing ban on government support for religious charitable organizations. The question, then, is whether government support of religious organizations violates the US Constitution. StuRat 20:35, 25 February 2006 (UTC)[reply]
To explain it in a little more detail... In the U.S., the public school system is inferior to most private schools. It is not just a little inferior - it is nearly impossible to get an education in many schools. Rich people do not care. They pay to send their kids to private school. Poor parents pay education taxes, but have to send their kids to terrible public schools. So, some feel that they should get a voucher from their education taxes and use it to send their kids to private schools. Education vouchers exist in some areas already. The catch is that some private schools are managed by a religion (such as a private Catholic school). So, should vouchers, which represent government money, be allowed to pay for a private religious school? President Bush is one who believes that it is the parent choosing to spend their voucher on the private religious school, so it is not considered state support of the school. Others feel that no penny of government money should ever make it into the hands of a religious organization. Most people fall somewhere between those two views. --Kainaw (talk) 00:35, 26 February 2006 (UTC)[reply]
That is an issue in the US, but not the one asked about. States control the funding for education, and hence the use of vouchers, not the Federal Government, so they would not be affected by an Executive Order. At least that's my opinion, but then again, I went to a public school, so I must not know anything. StuRat 08:43, 27 February 2006 (UTC)[reply]
Correct. I got sidetracked in another conversation. Currently, the voucher debate is working through the court system to the Supreme Court (which is why it is a Federal issue). The executive order was about the permission for a military chaplain to include "in Jesus' name" in prayers for non-Christain service members. --Kainaw (talk) 16:23, 27 February 2006 (UTC)[reply]
School vouchers have already been ruled on in Zelman v. Simmons-Harris; the court said they were OK as long as they passed a five-part test. Chuck 21:19, 27 February 2006 (UTC)[reply]
  • To respond to the question... I don't think so. Any such ruling, one way or the other, would surely have been reported prominently and then discussed in high decibels in the US media. Nobody would have not been aware of it. But nobody here has heard of it. Therefore, it didn't happen. Q.E.D. --Halcatalyst 23:33, 26 February 2006 (UTC)[reply]
    • To expand slightly on the above, the judiciary in the United States (and elsewhere, I presume) is a purely reactive branch of government. That is, the court cannot merely address an issue it wishes to. Rather, the court must wait for a case to be filed that challenges the law in question--it can't act immediately if it thinks a law is unconstitutional. All this is to say that, while no court may yet have ruled, this does not indicate one way or the other whether or not the courts will uphold the order. It would take a decision explicitly upholding the new executive order to make its constitutionality more certain. Jwrosenzweig 05:12, 27 February 2006 (UTC)[reply]
I think some countries have a Constitutional Council to vet new laws. Jameswilson 23:32, 27 February 2006 (UTC)[reply]

Name of Belgium[edit]

Why is the country called "Belgium"? Most countries, as well as the historical name "Gallia Belgica" are feminine. Why is the name neutered? Common Man 20:24, 24 February 2006 (UTC)[reply]

It is my opinion that the Germans, who refer to it as "Belgien", were the source of the English "Belgium". The Dutch do not end it with an "n" or "m" themselves. It is just Belgie. --Kainaw (talk) 20:33, 24 February 2006 (UTC)[reply]
België, to be exact. Or Belgique. Let's not forget the Walloons. David Sneek 20:37, 24 February 2006 (UTC)[reply]
No-one was sure whether to call it Belgi-ë, Belgi-que or Belgi-en, so people ended up calling it Belgi-um?
Slumgum 20:39, 24 February 2006 (UTC)[reply]
Interesting idea, Kainaw. But it begs more questions: (1) Why would English take it from German? (2) Even in those cases where English did take it from German, it never added the ending "-um": Thuringia, Westphalia, Saxony, and possibly Bavaria. Why in this case? Common Man 20:47, 24 February 2006 (UTC)[reply]
Actually, all of these came to English through Latin. --Chl 02:01, 1 March 2006 (UTC)[reply]
English stole many words from German (as well as French, Spanish, Italian...) If Belgian diplomats were introduced to the English diplomats through German diplomats, then they would have first heard the German country name. --Kainaw (talk) 21:23, 24 February 2006 (UTC)[reply]
How old is the name "Belgium", anyway? The historical name is certainly old, but is the English name "Belgium" really older than the Belgian country? (19th century, that is) --BluePlatypus 22:23, 24 February 2006 (UTC)[reply]

The ultimate origin is from the tribe of Belgae who lived in the area in Roman times. The Roman name for the region was Gallia Belgica (i.e. "Belgic Gaul"). Belgium looks like a Latin neuter adjective form, but I'm not sure which Latin neuter noun it would be understood as implicitly agreeing with (maybe Regnum). AnonMoos 23:11, 24 February 2006 (UTC)[reply]

Ah, that's likely to be it: the English name comes from Regnum Belgium, the Belgian kingdom. Until the 18th century there was no political entity called Belgium, although something like it has existed since the 16th century (see History of Belgium and United States of Belgium). The choice of name is a clear reference to Gallia Belgica, and that seems to suggest an easy way for the Latin name to enter into English without intermediary. — Gareth Hughes 23:43, 24 February 2006 (UTC)[reply]
This is possible, but I don't know how likely it is. As you point out, Belgium first was "united states", and not a kingdom until 1831. By contrast, most other countries were either a "regnum" or an "imperium", but none ends in "-um". Moreover, if it is the adjective, we would need to explain why the "c" got dropped. Common Man 09:35, 27 February 2006 (UTC)[reply]
Maybe it was a mistake. It looks like the Latin equivalent of the existing Walloon word Belgique, which is a proper noun but is adjectival in form. The Latin adjective agreeing with the neuter word Regnum is Belgium. JackofOz 02:12, 1 March 2006 (UTC)[reply]

It seems that "Belgium" was the form used by Caesar www.perseus.tufts.edu/cgi-bin/vor?lang=la&formentry=0&lookup=Belgium, and "Belgica" was used only by later Latin authors. --Chl 02:01, 1 March 2006 (UTC)[reply]

A related, though not strictly relevant, point of iterest is that the Belgae were spread throughout a large part of north-wstern Europe - including not only what is now Belgium, but also southwestern England and parts of Ireland (where they are probably the origin of the legends of my own presumed ancestors, the Fir Bolg). Grutness...wha? 09:28, 2 March 2006 (UTC)[reply]

The gingham Fabric[edit]

Hello. I am conducting research on the history of the Gingham fabric. I am at my wits end, as every website I have visited contains the same information as Wikipedia. The report is on the origin of Gingham, how it was used where it was first made, and finally how it spread to the rest of the world. Thank you, by the way. It would be really nice if you could give me some links to websites, or books. I do not want to cheat by just copying info from you. I know this subject is hard to find, so thank you very much. End. Sincerely, Andre -------------------- IP 66.27.168.34 23:02, 24 February 2006

Wikipedia says Malay/Dutch but other sites www.google.co.uk/search?q=gingham+originally+bengal&hl=en&rls=GGLD%2CGGLD%3A2005-03%2CGGLD%3Aen say Bengal. But this Indian site www.ourindia.com/def1.htm#gingham here says the word has an Italian origin. Jameswilson 00:54, 25 February 2006 (UTC)[reply]

Hey, thanks for the help with gingham fabric, appreciate it. My partner totally bailed on me, he was supposed to do calico cotton. The stuff wikipedia has on calico is great, but is there anything else anybody can find on the history of calico? It woyuld be very much appreciated. Andre05:13, 26 February 2006 (UTC)05:13, 26 February 2006 (UTC)05:13, 26 February 2006 (UTC)~~

February 25[edit]

Why would Kim Il-sung have blue hair???[edit]

In the Korean and Japanese Wikipedias, Kim Il-sung is shown to have blue hair.

I can understand that Junichiro Koizumi dyed his hair blond to make himself more popular to the country's youth (at least something along those lines), but Kim Il-sung would be the last ruler you'd expect to have dyed hair.

I guess Junichiro dyed his hair to somewhat emulate rock stars who have hair similar to his, but though that was relevant in Japan, how is Kim Il-sung dyeing his hair relevant in North Korea?

ja.wikipedia.org/wiki/%E9%87%91%E6%97%A5%E6%88%90 Kim Il-sung article on the Japanese Wikipedia See how dyed it is?? Why would a dictator want to dye his hair that color? --Shultz III 00:43, 25 February 2006 (UTC)[reply]

I think it's more likely that someone has rather overdone the saturation when processing the photograph (see how glowing his skin is). Overdone, saturation can pretty much manufacture colour where little existed in the original scene. -- Finlay McWalter | Talk 00:49, 25 February 2006 (UTC)[reply]
Is that Kim Il-sung or Marge Simpson?
Slumgum 00:53, 25 February 2006 (UTC)[reply]
Doesn't look dyed to me, more likely it's just due to the lighting conditions, the film used and any post-processing. --BluePlatypus 02:05, 25 February 2006 (UTC)[reply]

It doesn't look blue on my computer. Maybe the color on your monitor needs adjusting. StuRat 20:22, 25 February 2006 (UTC)[reply]

Hair match the blazer, both look plain blue. See also Shiva. --DLL 20:53, 25 February 2006 (UTC)[reply]
It is perhaps interesting, though not directly relevant except to challenge expectations: blue is the traditional hair dye colour of respectable, elderly, middle class ladies in the UK. See Blue rinse brigade. Though in my younger days when I bleached my hair and dyed it blue I was never mistaken for a respectable, elderly, lady. Anyway, it may be that if his hair is died there are sound cultural reasons. Notinasnaid 17:23, 26 February 2006 (UTC)[reply]

How Many?[edit]

Honestly, guys. How many licks does it take to get to the center of a tootsie roll pop? I must know! ... It's homework...

See Tootsie Pop. It's like a koan; it's not meant to have a definite answer. —Keenan Pepper 04:12, 25 February 2006 (UTC)[reply]
The answer is (g x p x s)/m where m is the mass of the hard shell, g is the glossal surface area contacting the pop, p is the erosion power per lick and s is the solvent strength of the saliva. Just insert your own personal values for the variables and it's no longer a secret. alteripse 12:06, 25 February 2006 (UTC)[reply]
Wouldn't the number of licks be inversely proportional to g, p, and s? —Keenan Pepper 17:28, 25 February 2006 (UTC)[reply]
Oh, yeah. Sorry. Did it make you lick smaller? alteripse 04:34, 26 February 2006 (UTC)[reply]
Sadly, this has been tested in real experiments. An experiment with a licking machine at Purdue found that an average of 364 licks was required. A separate test using 20 humans found an average of 252 licks was required. A Univ of Michigan licking machine required an average of 411 licks. A Swathmore School experiment using humans found an average of 144 licks was required. www.tootsie.com/memoriesLicksMachine.html --Kainaw (talk) 20:34, 25 February 2006 (UTC)[reply]
Too many jokes. Must not... type them... - Taxman Talk 04:28, 26 February 2006 (UTC)[reply]
You mean like, "Swarthmore got there sooner because everyone knows they suck?" Yeah, I'm glad that I, for one, didn't type that. (The last Tootsie Pop commercial I remember seeing in the late 1960's had "Mr. Owl" answer the question by licking it once, twice, biting to the center, and saying, "Three" was the answer.) Geogre 19:15, 26 February 2006 (UTC)[reply]
I always knew you were ahead of your time, Geogre. ;-) Now we have proof, as Tootsie Pop indicates the first Tootsie Pop commercial (and the one to which you refer) first aired in 1970. :-) Jwrosenzweig 04:54, 27 February 2006 (UTC)[reply]
Well, 1970 is the late 1960's, you know. :-) </Year-2000-is-20th-century-debate> Oddly, I recall that commercial and no other from them. They aired it quite a bit, and it was at first more amusing and then more annoying than other commercials. Geogre 10:57, 27 February 2006 (UTC)[reply]
The commercial from then I remember best, and maybe the only one ever where I particularly noticed the advertiser (Alka-Seltzer), is www.allesoverballen.com/engels/SPICY.html "Mama Mia, That's a Spicy Meatball". --Halcatalyst 05:14, 28 February 2006 (UTC)[reply]
I remember that one is circa 1974-5, when the other memorable commercial was "I Can't Believe I Ate the Whole Thing," which was also Alka Seltzer and is now being remade in the US. Geogre 13:40, 28 February 2006 (UTC)[reply]

statute of limitations[edit]

what is the time limit for prosecution of forgery in california? thanks,popeye

P.S. Do you want clean new dollars, half-price ? Olive. --DLL
  • Three years after the discovery of the crime. Or maybe six. (CA Penal Code 473 sets the penalty for forgery as imprisonment in the state prison, and 801 sets limitation as three years, but I might be interpreting something long. Luckily, IANAL.) --jpgordon∇∆∇∆ 18:41, 26 February 2006 (UTC)[reply]

Settlement Amount for M. Lewinsky[edit]

I have heard from a not-so-informed friend that Monica Lewinsky obtained $43,000,000 from the civil lawsuit against William J. Clinton? Was my friend inventing facts or is this correct? Can someone explain the pecuniary relief and if she and her lawyers obtained any money, what would the legal basis for it have been? He did not harass or rape her; it was consensual.

--66.81.192.18 05:02, 25 February 2006 (UTC)[reply]

I very much doubt that sum is accurate. According to his autobiography, he earned less than his wife prior to becoming President, and remains the PotUS with the lowest net worth in modern history . I'd guess that the $875,000 per year salary he gets nowadays is more than enough, but a sum that large would require him to file for bankruptcy. He paid out $850 000 to Paula Jones over the sexual harassment lawsuit, along with a $90,000 fine for contempt of court, but that nowhere near approaches the 43million figure. GeeJo (t) (c)  14:01, 25 February 2006 (UTC)[reply]
I didn't even realize Monica had sued him. Do you have lnks to any news articles on this? Thanks. --Nelson Ricardo 18:52, 25 February 2006 (UTC)[reply]

What did she sue for ? The dry cleaning bill for her dress ? StuRat 20:17, 25 February 2006 (UTC)[reply]

  • No actual expense there. Hmmmm.... I wonder if it's turned up on eBay yet? --Halcatalyst 22:16, 25 February 2006 (UTC)[reply]
  • I understand that Clinton was impeached for obstruction of justice and perjury when he said that he has no relationship with M. Lewinsky. (1) What was the basis for questioning him in the first place since adultery is not a crime? Other than the funny parts, what was the legal basis for even questioning Clinton about this matter? (2) Also, did Lewinsky file a civil suit or did she not? --66.81.192.109 23:42, 25 February 2006 (UTC)[reply]
Adultery is not a crime, but sexual harassment can be (the questioning was in relation to the Paula Jones incident). If you look at our article on the Impeachment of Bill Clinton, it goes into the details. --Fastfission 04:06, 26 February 2006 (UTC)[reply]
  • I remember my friend saying that this was a trap for Clinton; that Lewinsky purposely kept that semen-stained dress for the future lawsuit against him. But now you guys are making me doubt whether such a lawsuit even existed? --66.81.192.109 23:49, 25 February 2006 (UTC)[reply]
You are probably thinking of the Paula Jones lawsuit. She was paid an out-of-course settlement of $850,000. --Fastfission 04:06, 26 February 2006 (UTC)[reply]
  • One more thing, it is possible for Clinton to have earned less than his wife prior to his Presidency, but I am sure he had a lot assets. Assets>income. --66.81.192.109 23:53, 25 February 2006 (UTC)[reply]
    • There's no doubt in my mind that Bill Clinton earned less than his wife before he became president; I don't know what his exact salary was, but as of 1999 the Arkansas governor's salary was $68,448 www.stateline.org/live/ViewPage.action?siteNodeId=136&languageId=1&contentId=14239; a partner in a major law firm such as Hillary Clinton would have earned significantly more than that even in 1992. Furthermore, I don't know how Bill Clinton could have amassed significant assets before his presidency except from what he and Hillary owned jointly; he had spent the majority of his time since graduating from law school as the governor of Arkansas, plus two years as state attorney general for which his salary would have been even lower. Anyway, I have no recollection of Monica Lewinsky suing Bill Clinton. --Metropolitan90 00:56, 26 February 2006 (UTC)[reply]
  • If I recall correctly, Bill Clinton become rich only after he completed his Presidency, mainly from his book sales and giving speeches. He was probably in debt before then because of all the legal fees. He wouldn't have a problem getting a loan though. --Vsion 16:12, 27 February 2006 (UTC)[reply]
  • A Smart Loan. A man walks into a bank in downtown Manhattan and asks for the loan officer. He tells the loan officer that he is going to Italy on business for two weeks and needs to borrow $5,000. The bank officer tells him that the bank will need some form of security for the loan, so the man hands over the keys to a new Ferrari. The car is parked on the street in front of the bank.
  • The loan officer agrees to accept the car as collateral for the loan. The bank's president and its officers all enjoy a good laugh at the man for using a $250,000 Ferrari as collateral against a $5,000 loan. An employee of the bank then drives the Ferrari into the bank's underground garage and parks it there. Two weeks later, the man returns, repays the $5,000 and the interest, which comes to $15.41.
  • The loan officer says, "Sir, we are very happy to have had your business, and this transaction has worked out very nicely, but we are a little puzzled. While you were away, we checked you out and found that you are a multimillionaire. What puzzles us is, why would you bother to borrow $5,000?"
  • The man replies: "Where else in Manhattan can I park my car for two weeks for only $15.41 and expect it to be there when I return?" --Halcatalyst 05:09, 28 February 2006 (UTC)[reply]

indian writing in english drama[edit]

why is Asif Curimbhoy called 'a dramatist of public event' ?

You may struggle to get an answer. Google barely knows the guy www.google.co.uk/search?hl=en&client=firefox-a&rls=org.mozilla%3Aen-GB%3Aofficial&q=Asif+Currimbhoy&btnG=Search&meta=, and you're not very specific about who's calling him "a dramatist of public event". GeeJo (t) (c)  14:04, 25 February 2006 (UTC)[reply]

Age of Majority[edit]

Please can you tell me when the age of 21 was set for the age of majority on england. Thanks

The age of majority in England is 18, not 21 — it's similar for most Commonwealth countries. GeeJo (t) (c)  12:18, 25 February 2006 (UTC)[reply]
True, but it was 21 before that, and I expect that that was the original question. I expect it was a traditional age, and that there was no date where it came into being. Sam Korn (smoddy) 13:54, 25 February 2006 (UTC)[reply]
It is a complex question. Majority allows you to do plenty of things, e.g. : voting, sometimes driving a car, getting married without parent's consent, inheriting without a legal guardian, and so on. Democratic elections are very young ; cars have at most one century ; the required ages may have been different through the ages - in different countries also (I do not specifically refer to England). --DLL 20:41, 25 February 2006 (UTC)[reply]
The age of majority in England has been 18 or 21 for different matters for a while. It isn't consistent. Nor is it the same for boys and girls in the past. I.e. there's no way to answer the question without knowing which time period is being asked about. Geogre 19:12, 26 February 2006 (UTC)[reply]

Foreigners fighting in the Spanish Civil War[edit]

I know that there were many, many non-Spaniards (including George Orwell) fighting for the Republicans during the Spanish Civil War, but were there many who came to Spain to fight with Francisco Franco's Nationalists? If so, were there special battalions for them, or were they placed into regular combat units? —DO'Neil 11:48, 25 February 2006 (UTC)[reply]

In a limited number of cases, yes, foreign volunteers did travel to Spain to fight for the Nationalists - the Englishman John Amery, for example, who later joined the SS. However, they were overshadowed by the "volunteers" sent by Italy and Germany; military units, under the pretence of being private volunteers, deployed there. See, for example, the Condor Legion. Shimgray | talk | 13:46, 25 February 2006 (UTC)[reply]
And by the Americans: see Abraham Lincoln Brigade, part of the International Brigades. Rmhermen 03:47, 26 February 2006 (UTC)[reply]
No, like he said, to fight with the nationalists, not against them. Superm401 - Talk 10:29, 26 February 2006 (UTC)[reply]

World War II[edit]

no question

Try the search bar for World War II. GeeJo (t) (c)  13:47, 25 February 2006 (UTC)[reply]

cost of education[edit]

What is the average cost of education for, as an example, K-12+college, in US or in other countries? How much has this cost changed over the last decade? Thanks. Browni 13:50, 25 February 2006 (UTC)[reply]

  • You'll need to define the question more clearly. Cost in USD or some other currency, or the cost taking into account purchasing power parity, given that salaries vary widely between different countries. Do you mean cost to the student or cost to the government? A good number of countries have free higher education. Should living costs be included? What about the large number of third-world countries who have virtually no higher education? What qualifies as a college education? 3 years? 4 years? The number of students in higher studies varies, and so does the amount of time they spend in higher studies. --BluePlatypus 18:31, 25 February 2006 (UTC)[reply]
  • Just as an example, suppose I have a 6-year old kid, how much should I expect to spend to cater for his education till he finish university (in US, Europe, China, India, Nigeria, etc.)? Is there a comparison between countries? Since we have the GDP figures, are there any analysis on how much of this GDP is spend on education (for any country) ? --Browni 23:22, 25 February 2006 (UTC)[reply]
As BluePlatypus said, you should distinguish between cost to you or to the government. You seem interrested in the former but also seem to ask about the latter. In the Netherlands, when I studied (about 20 years ago) the cost to the student was about 500 € per year and I once heard the actual cost (to the government) was more like 10.000 €. I don't know what would be included in that, but certainly not housing because that is completely separate (although I believe one university has a campus). DirkvdM 08:54, 26 February 2006 (UTC)[reply]
In the UK, the annual income for a university per undergraduate is on the order of £6,000 (it varies by subject), with an optional additional levy now coming into play - it's likely the real cost, at least for science students, is closer to £10,000 per year. The student pays about £1200 of that, with the additional levy meaning that up to £3000 can be borne by them. Shimgray | talk | 19:23, 26 February 2006 (UTC)[reply]
Here in America we consider affordable merit based education to be the work of communism, and hold it in a contempt usually reserved for science and math, and other things jesus doesn't like--64.12.116.72 00:30, 27 February 2006 (UTC)[reply]

bulbul tarang (Indian Banjo)[edit]

can someone wrire an article on this acient musical instrumant, www.google.co.il/search?hl=iw&q=bulbul+tarang&btnG=%D7%97%D7%99%D7%A4%D7%95%D7%A9+%D7%91%D7%92%D7%95%D7%92%D7%9C&meta= click here for more. thank you, 82.81.128.112 18:34, 25 February 2006 (UTC)[reply]

The magic of Wikipedia is that you can write the article. Just click on the link you made in the title and start typing. When you are done, click "save page" and the article will be there for everyone to read. --Kainaw (talk) 18:49, 25 February 2006 (UTC)[reply]
Actually you will need to be logged in to start an article. But don't worry, I've started it for you: Bulbul tarang. So now you can edit and improve it. --Commander Keane 19:26, 25 February 2006 (UTC)[reply]
i dont have enough money to expand the article, can't someone else do is instead?82.81.128.112 21:10, 25 February 2006 (UTC)[reply]
Money!? What does money have to do with a free encycolpedia? That makes absolutely no sense. --Kainaw (talk) 21:52, 25 February 2006 (UTC)[reply]
Internet time can cost money. Think about it.
Slumgum 22:05, 25 February 2006 (UTC)[reply]
Most public libraries provide free internet access to members. Next time you feel like spending a big chunk of time online, consider checking out a library. I think many even have AIM. Shaggorama 05:32, 26 February 2006 (UTC)[reply]
Another option for reducing internet connect time is to write the article in a separate Word document, then cut and paste it into the Wikipedia article. That last action would take a very short space of time. JackofOz 10:47, 26 February 2006 (UTC)[reply]
If this person is in rural India, as I suspect they might be, there might not be another option than some form of pay-per-time-used computer establishment, where the cost could very well be applied to offline use. -LambaJan 17:45, 26 February 2006 (UTC)[reply]

February 26[edit]

Song Name[edit]

I am trying to find out the name of a song, it is played often on television during a parody of a 1950's kitchen appliance commercial or along those lines, it's very upbeat and has kind of a "doot doot" sound, and it may have been used in The Sims. I seem to recall it being used in old PSA's as well as parodies thereof. Can anybody help me?

72.240.71.205 05:12, 26 February 2006 (UTC)[reply]

Have you tried www.musipedia.org/ this site? Or try themefinder.org/ here or tunespotting.com/ here. --Shantavira 10:02, 26 February 2006 (UTC)[reply]
I'll take one wild guess: "Anything You Can Do (I Can Do Better)", from the 1940s musical Annie Get Your Gun, was in a retro-style kitchen commercial a few years ago and is often parodied. It kind of goes "doot doot". ×Meegs 11:08, 26 February 2006 (UTC)[reply]
How about the theme from I Dream of Jeannie ? That goes "doot doot, doot doot duh do doot". I hope my advanced musical notation wasn't too much for you. StuRat 06:39, 27 February 2006 (UTC)[reply]

UK companies and trade with India[edit]

What are some companies in the UK that trade or invest in India?

The British East India Company. -LambaJan 17:42, 26 February 2006 (UTC)[reply]
That's a little out of date. Plenty of modern British companies invest or trade with India. Banks are probably a good place to start. www.indiainbusiness.nic.in/invest-india/forbank.htm this link might be helpful. DJ Clayworth 18:31, 27 February 2006 (UTC)[reply]

attracting birds to house[edit]

Do anybody has suggestions on how to attract birds to my garden?

1. Eliminate pets, especially cats. 2. Feed the birds. 3. Add nesting boxes. Notinasnaid 17:17, 26 February 2006 (UTC)[reply]
Also, plants, particularly herbs and flowers. They attract insects that birds like to eat. -LambaJan 17:39, 26 February 2006 (UTC)[reply]
Don't expect quick results. After you establish a feeding station, it will take up to 3 weeks for the birds to find it, especially in the spring time, when natural sources of food are plentiful. Do not leave the feeding station in an overly exposed area. You want to see the birds, but they don't really like being out without cover, so put the station near a natural cover (a tree or a set of bushes). Shrubbery also allows the birds to hide when they suspect predators, so these are good. Finally, people often overlook the fact that birds need water as much as food, and, if you are in a dry-ish area, a nice bird bath can attract birds as well as a feeding station.
On the feeding station, be aware that seed gets old, and it gets moldy. Do not dump a pile of seed out at first. Know what kinds of birds are near you and feed appropriately. Put out black oil sunflower seeds for best general results, suet for tree-clinging birds, and thistle seed for finches. In each case, though, put out only a bit until birds start attending. Once they are attending, keep it filled. If you have trouble with squirrels and other pests, consider thistle seed and nailing a suet block to a tree trunk in a cage. Geogre 19:09, 26 February 2006 (UTC)[reply]
We have an article on birdfeeding, though it's not particularly helpful. Perhaps Geogre might like to do some work on it. --Shantavira 13:00, 27 February 2006 (UTC)[reply]
I gave it a shot, but the article sure suffers from wiki-itis (a thousand paragraphs saying the same thing, each tacked on), and it's misnamed. I don't know what style sheet says that bird feeding is a single word. Oh, well. I did try to smoothe it a little and add some how-to information. Geogre 21:42, 27 February 2006 (UTC)[reply]

Counter-reformation[edit]

I was doing some research on the reformation when the idea of a 'counter-reformation'? popped up, i was wondering what this was?

February 27[edit]

governmental question[edit]

Who is the head of local (CA) government?

Arnold Schwarzenegger is Governor of California. --Halcatalyst 03:55, 27 February 2006 (UTC)[reply]

Or did you perhaps mean Canada? Dismas|(talk) 06:02, 27 February 2006 (UTC)[reply]
Stephen Harper is Prime Minister of Canada. Or did you perhaps mean Central America? Or perhaps Carlisle? Grutness...wha? 06:33, 27 February 2006 (UTC)[reply]
Maybe he meant Cagliari, which would be Renato Soru. Or, it could be Catamarca Province, which would be Eduardo Brizuela del Moral. (Abbreviations are fun.) --Kainaw (talk) 01:38, 28 February 2006 (UTC)[reply]
François Bozizé is the President and Elie Doté is the Prime Minister of the Central African Republic. The previous answers are obvious examples of inherent bias. User:Zoe|(talk) 03:13, 28 February 2006 (UTC)[reply]
The moral: Caca questions produce caca answers. JackofOz 11:08, 28 February 2006 (UTC)[reply]

World War I battle[edit]

Are their any good websites or organizations that have a good detailed history and photos of the battle of lemberg in what is now Russia.

There were three Battles of Lemberg in WWI: we have a decent article on the third one. Markyour words 02:13, 27 February 2006 (UTC)[reply]

Pre-industrial apprenticeship system[edit]

In the pre-industrial apprenticeship system, how frequently was an apprentice's master his own father, as opposed to someone else with whom a contract was signed? Were there particular crafts in which this was more common?

  • You can learn a lot from our Apprenticeship article and its links and references. My guess would be that no contract was required if the son was learning his father's trade. --Halcatalyst 14:43, 27 February 2006 (UTC)[reply]
  • Exactly: A son was not an apprentice. An apprentice was contracted. A son was duty-bound. Children would be farmed out as apprentices when the parent either had no trade to teach or had no room for another worker. It was not something parents did happily nor children accepted joyfully. Geogre 15:03, 27 February 2006 (UTC)[reply]

Well, OK. But since an apprenticeship contract was ordinarily overseen by the guild, did the guild oversee a father-son relationship in any way? Or, in other words, to the extent that the years of apprenticeship led to someone who had a real skill--a skill that was "certified" by the guild--what was the actual procedure for certification in cases where there was not a contract? Did the father simply present his son to the guild at some point, as someone who he said had completed training in the craft?

There were cases of people becoming apprentices to relatives. I believe Benjamin Franklin was apprenticed to his older brother (at his father's insistence), for example. StuRat 10:46, 28 February 2006 (UTC)[reply]
That would have been unusual and one of those, "Son, I'll lend you money, but you need to sign a loan agreement" things. Done, but done to make a point. Guilds.... In the UK, the guilds weren't very involved by, say, 1700. They existed, and there were very small scale guilds that exerted some control, but the lack of regulation of apprenticeships is one of the most common working class complaints. You run across many complaints of masters abusing apprentices and many complaints of apprentices running amock. The apprentice was in a very strange position -- somewhere being a member of the master's family and being the master's slave. Youngsters always hoped for the former, and generally that's what they got, but the master acted in loco parentis for many legal matters (in Jacobean drama, for example, you see that the master was responsible for managing the apprentice's marriage and burying a dead apprentice, etc.). The guilds in the UK and US were largely out of the picture by then. Geogre 13:45, 28 February 2006 (UTC)[reply]

Connection with someone who is dying[edit]

Wikipedia:Clueless newbies is a veritable fount of fun questions... such as this one...

Is it true that some people have a connection with people and they can feel what they are going through and can see them dying and seeing visions of love ones .

CHEERS Ćð

QuantumEleven | (talk) 09:56, 27 February 2006 (UTC)[reply]

Assuming the questioner is referring to some kind of connection beyond normal empathy, their question is about telepathy. The short answer is "no, it doesn't exist". --Robert Merkel 21:40, 27 February 2006 (UTC)[reply]

Marriage License[edit]

I hear you don't actually need a marriage license to get married (in the United States). But is that just refering to the fact that you don't need legal recognition in order for your religion to recognize your marriage? Or can you actually get the legal benefits/recognition of marriage (income tax stuff, social security, etc) without the license? Cyllya 10:54, 27 February 2006 (UTC)[reply]

In the US, each state regulates marriage. You do need a marriage license. The ceremony can be performed by a civil or religious authority; the state doesn't care much about who does this or what form the ceremony takes. After the parties and their witnesses sign state forms, that's it, as far as the state is concerned. The legal benefits then follow. If you need proof that you're married, you go to the state, usually the county recorder. Religious recognition is up to the religious bodies themselves. They may or may not see a civil marriage as valid.
Currently there is religious and political controversy about the meaning of marriage, including gay marriage. --Halcatalyst 14:37, 27 February 2006 (UTC)[reply]
You can be married in some states without a license--see Common-law marriage. But the requirements can be rather vague, and if you're trying to get the legal benefits of marriage, you're much better off getting a license. If you try to claim a common-law marriage, you're subject to the whims of the local bureaucracy and/or courts, who may or may not decide that you're actually married. Chuck 21:30, 27 February 2006 (UTC)[reply]
It also costs more to get a divorce with common law marriage in South Carolina, Kansas, Missouri, and Iowa. I haven't checked all 50 states, but I assume it is the same most everywhere. The reason it costs more is because there is no marriage to dissolve, so it requires more work on the lawyer's part. At least, that is the reason I was given when I called around to ask, primarily because I didn't believe it would cost more. --Kainaw (talk) 01:31, 28 February 2006 (UTC)[reply]
So what's happening: they agree to marry (non-public); they make it public somehow (how?); then they want to divorce? And the problem is, they can't get unmarried because there's not good evidence that they're married? I can sort of understand that from a legal point of view, though it's certainly unintuitive. --Halcatalyst 03:44, 28 February 2006 (UTC)[reply]
I assume it isn't the divorce that is important in common-law marriage dissolution - it is the "I want half" issue. Suppose you have a roommate for five or six years and you decide to move out and take all your stuff. That roommate can try to argue that you are a common-law spouse and try to take half your stuff. So, I guess there is more work than a regular divorce. --Kainaw (talk) 14:13, 28 February 2006 (UTC)[reply]
I think that what commonlaw marriage is about is that after a couple has been living like they've been married for a while, the state decides it's time for them to participate in the tax structure like they're married. From their financial point of view, in most U.S. states anyway, it would make more sense for them to marry to begin with. As for the divorce, I agree with what Kainaw said. -LambaJan 05:40, 1 March 2006 (UTC)[reply]

See Banns of marriage as an alternative to a marriage license in some jurisdictions. Samw 23:42, 4 March 2006 (UTC)[reply]

Industrial Relations[edit]

Generally, i have found the Wikipedia page emmensely helpful in my studies, however, i struggled to find information on the following topic:

In looking at Industrial Relations in Britain since 1945, how has the public expenditure spent on these policies changed over this time period. Obviously, the Thatcher effect changed matters, but did other governments make any difference?

Many thanks guys, hope you can help!

You can also try UK miners' strike (1984-1985), Trade unions in the United Kingdom, etc. AllanHainey 15:34, 28 February 2006 (UTC)[reply]

J.B. Bury[edit]

I have the book, "The Invasion of Europe by the Barbarians" by J.B. Bury, and i know that it is pretty old, i think it was written around 1930 or so, so i was wondering if it is still considered pretty accurate in the field of archaeology and history. ~katie

Pretty accurate, yes, as far as it goes. It doesn't reflect more recent archaeological discoveries and research, and there have been new interpretations of the available facts since it was written, but it's still reasonably useful. I wouldn't rely on it exclusively, though. —Charles P._(Mirv) 03:49, 28 February 2006 (UTC)[reply]

February 28[edit]

2 Dollar Bill[edit]

To Whom It May Concern,

I need some major help.

On the back of the 2 dollar bill there are 42 men who signed the declaration of Independence. What are their names?

In the original John Trumball portrait there were 47 men. What five were left off the 2 dollar bill?

There were originally 56 signers of the Declaration of Independence. Two were left off of the 2 dollar bill due to their objections about the constitution. Who were they?

Thank you,

Charly

First, the painting is not the signing of the Declaration of Independence. It is the presentation of the Declaration of Independence by the drafters. As for the 5 left off, they are the ones on the edges. Compare the painting to the bill. None were left off the bill for objections to anything. They were left off because it looked better to leave off the extra guys at the edge.
Note: This is an encyclopedia. You would have received an answer much quicker if you checked U.S. two-dollar bill before asking a question. --Kainaw (talk) 01:28, 28 February 2006 (UTC)[reply]


I did look there first. This is a trivia question that my political science professor asked. I am still looking for the answer. Sorry if I annoyed you, by you thinking that I did not look it up first, however, I did and would still like to know the answer. In addition, their opposition to the constitution was due to the fact that it originally did not include the Bill of Rights.

Thanks again, Charly

It is not annoying. Question like "Pepsi Cola?" are annoying. The problem with your question is that it is based on a false premise. The painting is not the signing of the Declaration of Independence. It is of the presentation of the Declaration of Independence. The people in the painting are not the signers of the Declaration of Independence. While most of the men in the paintind did sign it, some signers are not in the painting and some men in the painting did not sign it. You are left with two choices: Give your teacher the answer he wants even though it is wrong -or- research the painting and explain (with references) why his questions are not valid. --Kainaw (talk) 01:52, 2 March 2006 (UTC)[reply]
www.americanrevolution.org/decsm.html This site will help get you started. Wikipedia has articles on all the men. You can prove to your teacher which ones signed the Declaration of Independence and which ones didn't. --Kainaw (talk) 01:58, 2 March 2006 (UTC)[reply]
OTOH, they're not very common these days. I suppose you could go to a bank, but they might be scarce even there. And, just possibly, the inquirer might be outside the US. --Halcatalyst 03:34, 28 February 2006 (UTC)[reply]
They're not that uncommon. I have two or three in my wallet right now. Dismas|(talk) 11:33, 28 February 2006 (UTC)[reply]
I'm amazed -- haven't seen one in many years. --Halcatalyst 00:03, 1 March 2006 (UTC)[reply]
Comparing them, it looks to me like there are 48 guys in the picture and 6 guys are left off, two on the right, four on the left. --BluePlatypus 04:16, 28 February 2006 (UTC)[reply]
Obviously, another example of the right wing conspiracy to cut out the lefties! Geogre 13:47, 28 February 2006 (UTC)[reply]
Naw, it's the middle finally getting a backbone. --Halcatalyst 00:05, 1 March 2006 (UTC)[reply]
They are 42 on the 2 doll bill because 42 is a happy number. --DLL 15:50, 4 March 2006 (UTC)[reply]
  • That's right: 42 is the inverse of 24 (four omitted on the right and two on the left). Which proves that the Mint is run by a shadowy group of numerologists who aim to take over the world by buying out all the companies and governments using $2 bills. --Halcatalyst 15:58, 4 March 2006 (UTC)[reply]

British Petroleum and trade with India[edit]

Hello, I have been trying to find information on companies in India that the UK company 'British Petroleum' trades with. I read the BP article on Wikipedia and while it was helpful, I could not find links to such companies. Any Indian business journals, newspapers or articles would be very helpful as well. Thank you---DrtyHOBO

  • 72.14.207.104/search?q=cache:CbTNeVkT1IsJ:www.ipan.com/CLIENTS/British%2520Petroleum/british-petroleum.htm+British+Petroleum+trade+india&hl=en&gl=au&ct=clnk&cd=2 This Google cache link has some mention of BP's operations in India. Castrol India is owned by BP, and is in fact their largest operation in that country. You can download their Annual Reports www.castrol.com/castrol/castrolhomepage.do?categoryId=3230&source=castrolglobalcountrylisting here. Your question is a bit vague as to what kind of "trade" you are interested in finding out about: as a fuel resource provider, BP would deal with hundreds, if not thousands, of Indian companies, so you may have to narrow down what sort of companies you want in your list. --Canley 02:24, 28 February 2006 (UTC)[reply]

I'm just having trouble locating the names of any companies in India that BP trades petroleum or petrochemical products to. I just want to find out a few names of companies in India so I can research their trade agreements and relationships, but not subsidiary companies like castrol. Anything will help-thanks

Apostolic Succession from Old Catholic or Independent Catholic Church[edit]

I have several questions relating to Apostolic succession from the Old Catholic Church or Independent Catholic Churches, particularly with reference to the United States (although the answers may be true worldwide, I don't know):

  • (1) When someone is consecrated as a Bishop why are they sometimes later reconsecrated one or more times?
  • (2) Suppose Adam has been consecrated as a Bishop by Carl; why then does it sometimes happen that Adam will later consecrate Carl as a Bishop?
  • (3) Why do people consecrated as Bishops often start new churches, rather than continuing the church of the person(s) who consecrated them?
  • (4) How significant is it to be consecrated as a Bishop? Are there a lot of Catholic Bishops in the United States who are independent of Rome?

As an example of (1) see www.united-catholic-church.org/Who%20We%20Are/apostoli.htm where Bowman is consecrated twice in 1996, and again in 1998. For (2), see www.tboyle.net/Catholicism/The_Costa_lines-_P-T/The_Reeves_lineage.html where Reeves is consecrated by Strong, and then Reeves later consecrates Strong. As an example of (3), Bowman begins the United Catholic Church despite one of his consecrators being with American Catholic Church and another from the Catholic Church of The Americas. I don't think this refers to the physical church; people Bowman has consecrated (many, anyway) identify with his UCC, apparently. I may not have phrased all this well, but that's because I really don't have much of a grasp on it. Thanks! Schizombie 01:55, 28 February 2006 (UTC)[reply]

One would guess that the church is not sure of the apostolic lineage of some of their bishops, that they feel pretty sure, but not absolutely sure. Each bishop can trace a different route of consecration, so it would be possible that they assess some lines as stronger than others. Then again, there are reconsecrations at anniversaries and with changes of office, and these differ by church. (E.g. a dean of a cathedral may be reconsecrated, even if he is a bishop, and not just invested.) Not much of an answer, but since these churches have been denounced, they may have disruptions in their episcopal lines. Geogre 03:15, 28 February 2006 (UTC)[reply]
I have no idea about any of the specific individuals or groups you mentioned, but one thing which is fairly obvious is that a number of the micro-splinters and fragments of factions of so-called "Old Catholics" have only a somewhat tenuous connection with traditional mainstream Christianity, and that some in fact are basically just new age groups with very remote claims to "Utrecht" connections. AnonMoos 03:21, 28 February 2006 (UTC)[reply]
Thanks for these responses. They're of some help, though I still don't understand it entirely. Schizombie 10:59, 1 March 2006 (UTC)[reply]
I don't know that there's much to understand, other than that this is behavior many or most people would consider strange. --Halcatalyst 04:59, 4 March 2006 (UTC)[reply]
That does seem to be the case, though there would seem to be a sizeable number of people who engage in it anyway, though how many I don't know. Schizombie 07:09, 4 March 2006 (UTC)[reply]
  • The recent (Roman) Catholic schismatics? Speaking ignorantly, I would say their numbers related to the number of separate "churches" is arithmetic rather than geometric. But I just thought of this article: Sedevacantism. Hope it helps. --Halcatalyst 15:51, 4 March 2006 (UTC)[reply]

Idea of "Purity" being the color white[edit]

What social/environmental/psychological factors are involved in the idea of "purity" being represented by a solid color, namely white?

Is this a homework question by any chance? - Akamad 03:15, 28 February 2006 (UTC)[reply]
Also, isn't it sort of obvious? Have a spaghetti dinner wearing a white shirt and then with a red shirt. You'll be able to spot the impurity right off with the former. Although white is the presence of all colors, it is that presence in exact saturation and balance; anything imbalances it and shows as a stain. Geogre 03:16, 28 February 2006 (UTC)[reply]
I think it's obvious only because were acculturated into thinking that way. Brides nowadays generally wear white to symbolise their purity (= virginity), but in some European cultures a bride wore black or some dark colour. White is considered the colour of death in some places, and also a white flag denotes surrender. Colours mean whatever convention says they mean. JackofOz 04:13, 28 February 2006 (UTC)[reply]
If you think it's all relative, you eat the yellow snow. Myself, I will stay with the pure white snow. StuRat 10:40, 28 February 2006 (UTC)[reply]
Ooh no, that's too cold on my fillings. I prefer pizza. JackofOz 11:02, 28 February 2006 (UTC)[reply]
It's obvious because white shows stains. It's not obvious because of linguistic determinism. Against a bright background, all dark things show. In the natural world, "stains" are darker than the artificial brightness of colors. The brightest is white. It's not obvious that white means virginity, only "unstained." The idea that sexual intercourse is a stain is cultural. The observation that something on white shows up better than something on burnt umber isn't. Geogre 13:49, 28 February 2006 (UTC)[reply]
The pure aspect from an artistic standpoint isn't just that stains show on white. It is that with paints you can mix colors to get any color you want, except white. White is pure. Black is inundated with every color. With light, it is the exact opposite, but when the concept of pure white was formed, nobody had color lights (even white ones).
The debate about the pureness of white is just as cultural as the pureness of white. For some, it is an attempt to prove that people with dark skin colors are automatically assumed to be guilty of some crime because they aren't pure white. This assumes that a jury looks at someone and judges them based purely on how easy it would be to mix paints and get their skin color. It is very stupid, but humans are great at rationalizing stupid concepts and getting excited about them. How about a million man march to make painters stop using white canvases? --Kainaw (talk) 21:22, 28 February 2006 (UTC)[reply]

New question (wiki disappearing)[edit]

Is there a risk that Wikipedia could disappear? LG

Conceivably, the Wikimedia Foundation could go defunct. However, becasue the content is under the GNU Free Documentation License, the encyclopedia can always be restarted elsewhere and should be permanent. Superm401 - Talk 03:59, 28 February 2006 (UTC)[reply]
In the long run of things, nothing lasts forever. - Akamad 04:37, 28 February 2006 (UTC)[reply]
  • But, if the foudation goes defunct, where is the Wikipedia stored? on their servers? can they be relocated, or at least the information? LG
Yes, the Wikipedia's primary storage is on Wikimedia foundation servers in a colo in Florida. However, dumps of the entire database are downloadable, and as well as backups made by the administrators indeed a number of organizations do so on a regular basis. You can see the instructions on how to do so at Wikipedia:Database download. It is highly unlikely that all of these independently-kept copies would be destroyed short of a major global catastrophe. --Robert Merkel 04:59, 28 February 2006 (UTC)[reply]
It's worth noting that - due to the gfdl licence - a considerable amount of wikipedia is available from other sites as well. So even if the worst happened and Wikipedia disappeared entirely, a significant poportion of it would still be online (unless the web went down too, of course). Grutness...wha? 07:07, 28 February 2006 (UTC)[reply]
But even then there would still be loads of people with the encyclopedia on a few dvd's and if somehow the Internet would fail (although it's specifically designed never to do that) and continue to fail, sneaker net would take over. Unless some electromagnetic disaster (whatever) causes all computers to cease functioning. In that case we'd be screwed (although the collapse of Wikipedia would probably not be the biggest problem we're facing then). Has anyone thought of printing Wikipedia? How big a book would that be? To make this a bit more general, are we relying too much on the continued functioning of our information technology? DirkvdM 09:48, 28 February 2006 (UTC)[reply]
The English Wikipedia is 8GB of text for the latest version of each article. Assuming about 3KB per full size standard page (depends on spacing, font, etc), that would be almost 3 million pages, or over 5,000 volumes 500 pages each. In other words, you'd have to be choosy and filter it down somehow. - Taxman Talk 19:28, 28 February 2006 (UTC)[reply]
Quite separately there is the risk that Wikipedia could peak, then decline eventually beyond the point of recovery. This could happen if (to oversimplify) the ratio of bad guys to good guys gets too high. It may be that as Wikipedia gets close to being finished the good guys will drift away, leaving it to destruction by the bad guys. Notinasnaid 18:38, 28 February 2006 (UTC)[reply]
Then we'd just employ different methods using some implementation of stable versions. That could be where editing is by invite only once merit and/or qualifications are shown on the wide open Wiki. There's already plenty of discussions on implementing the stable versions part at least. - Taxman Talk 19:28, 28 February 2006 (UTC)[reply]
There is the chance that when we run out of cheap oil we won't have enough energy to run all the computers - and most other things. AllanHainey 12:21, 1 March 2006 (UTC)[reply]
Or, alternatively, run out of oil to make the plastics to make the computers in the first place! Hopefully, by that point, we will either have found a way to make plastics from something else than oil, or invented an even better material. — QuantumEleven | (talk) 08:45, 2 March 2006 (UTC)[reply]
Or, that the sun might emit a pulse so powerful that all the electronic gear on one side of the planet would be completely fried, an event that would bring our civilation to a stunning halt. I've read that this is to be expected once in 300 years. If it does, WP won't matter. --Halcatalyst 04:57, 4 March 2006 (UTC)[reply]
After the Internet got rebuilt, Wikipedia could be reinstalled from data dumps on CDs and DVDs. Superm401 - Talk 03:31, 6 March 2006 (UTC)[reply]

Claus Sluter birth date[edit]

Could you help me find where and when 14th century sculptor, Claus Sluter, was born?

Thank you

--69.180.135.226 04:02, 28 February 2006 (UTC)[reply]

Unfortunately, in the words of Myers & Copplestone (editors)'s massive tome The History of Art - "Little is known about Sluter's origins." It is known he was active in the early 1380s and died circa 1405-1406, and that he was Dutch, having probably done much of his early work in Brussels before coming to Dijon, but that's about all. Grutness...wha? 07:13, 28 February 2006 (UTC)[reply]

Ugly Laws[edit]

Hello,

I'm looking for examples or information on "ugly laws" such as the one below

"No person who is diseased, maimed, mutilated or in any way deformed so as to be an unsightly or disgusting object or improper person to be allowed in or on the public ways or other public places in this city, shall therein or thereon expose himself to public view, under a penalty of not less than one dollar or more than fifty cents for each offense. [Chicago, Ill., _Mun. Code_, Sections 36-34 (1966) (repealed 1974)]"

Not exactly what you requested, but interesting enough, according to www.msnbc.msn.com/id/9221959 "China’s Nanjing are banning taxi drivers who are bald, wear their hair too long, have mustaches or wear too much makeup" --Vsion 22:57, 28 February 2006 (UTC)[reply]
Sounds like the way hippies were treated in the west in the 60's and backpackers around the world in the 70's and 80's. But that was never law as far as I know. Is this? DirkvdM 07:30, 1 March 2006 (UTC)[reply]

Anti- Communism[edit]

What was the effect of the Anti-Communist movement in America on the countires Internal politics, generally between 1947-1956? The significance of Joe McCarthy?

  • Look through your text book and class notes. If you missed too many classes and don't have the text book, look at Red Scare or Cold War. Geogre 13:52, 28 February 2006 (UTC)[reply]

The history of jackson, new jersey, named after and where do the cottrell family com in to this?[edit]

I am writing to find out when jackson, new jersey was named adn after who it was named.also i would like to know which john cottrell was granted land after the civil war, the land was in both ocean and monmouth county. my husband is a decendent of the cottrell family and i am trying to get information on this issue. also could you find for me any other cottrell's of jackson, newjersey or surrounding areas?

If you don't get an answer that suits you here, you might want to try contacting someone from the township. Their web page is linked to at the bottom of the article for Jackson. Dismas|(talk) 14:16, 28 February 2006 (UTC)[reply]

Who composed the Benny Hill song?[edit]

I need to find out who wrote the infamous Benny Hill song, the instrumental one that sounds all zany. I do not know if it was created for the show, or for some other reason, but the song is heavily associated with Benny Hill.

Boots Randolph. The song is called Yakety Sax. This information can be found in the Benny Hill article. --LarryMac 15:14, 28 February 2006 (UTC)[reply]

Robotron? Anybody?[edit]

When I was growing up, I seem to remember a television show about transforming robots battling for the safety of earth. If I'm not mistaken, this well-loved show from my childhood was called Robotron. Can anybody help resurrect a lost memory?

Try Transformers (toyline) or Transformers series. AllanHainey 15:40, 28 February 2006 (UTC)[reply]
Robotron was a cartoon in the late 80's. I think it was done by the same people who did Voltron. There isn't a Wikipedia article on it, so I have to by 25-year old memory here. What I remember is that two channels picked up the same Japanese cartoon. It was about some robots that protected the Earth. The plot was always the same: Bad guys come. Commercial. The good guys get called in to stop them. Commercial. The good guys get into a minor battle. Commercial. The bad guys look like they will win. Commercial. The good guys for Robotron/Voltron mega-robot and kill the bad guys with a single whack from the big mega-sword. Commercial. The really weird thing is that the exact same video was on both channels, but the dialog was dubbed differently to the point that the little plot in the show was changed. Robotron: "The Grigloc warriors are here to kill all the cute bunny rabbits. We must stop them!" Voltron: "The Marknur overlords are coming to mine away all our platinum. We just stop them!" --Kainaw (talk) 21:33, 28 February 2006 (UTC)[reply]
Are you thinking of Robotech? --Robert Merkel 22:13, 28 February 2006 (UTC)[reply]

WHY? (Activist Judges)[edit]

why is it that most of the more activist judges seem compelty unconcerned with states rights?--Ask a question11 16:18, 28 February 2006 (UTC)[reply]

This is pushing way into the realm of a loaded question, but here's my take on it: restricting states' rights is currently viewed as a form of activism. It's worth noting, however, that our article on states' rights fairly points out that recent references "have generally been opportunistic" on the part of both pro and con viewpoints. Consequently, nearly all action regarding states' rights can be viewed as activistic, but views both favorable and opposed are easy to find. — Lomn Talk 18:01, 28 February 2006 (UTC)[reply]
Are you talking about judges in general or in a specific country? DirkvdM 07:32, 1 March 2006 (UTC)[reply]
When in doubt, assume Merka. That policy hasn't failed us yet... *grin* — QuantumEleven | (talk) 08:12, 1 March 2006 (UTC)[reply]

Winston Churchill[edit]

A simple question as to whether Sir Winston Churchill's name actually contained another "h" making it "Churchhill?" I have seen both and am researching his genealogy but that difference makes a difference. Thank you!

Our article says his legal name was Winston Leonard Spencer-Churchill. David Sneek 21:21, 28 February 2006 (UTC)[reply]
When researching genealogy you must be aware that family names often vary in spelling. It is entirely possible that some people with that name have spelled it Churchhill (or Church Hill or any other variation you can imagine), while others spelled it Churchill, whether they were related or not. This sort of variation has become much less common in the last couple of centuries (six signatures of William Shakespeare survive and no two of them are spelled the same way!), but it died out gradually and you can't just assume it hasn't happened. --Anonymous, 22:27 UTC, February 28, 2006.
Also note that he is a Spencer - they just picked up the added Churchill in the mid-1800's (his father's generation.) Rmhermen 23:25, 28 February 2006 (UTC)[reply]
He's not descended from John Churchill, 1st Duke of Marlborough? Marlborough was an enormously controversial person in my own area of special knowledge, and I had always thought that the marriage into the Spencers was matrilineal and not patrilineal. Geogre 00:47, 1 March 2006 (UTC)[reply]
He was descended from Spencers and Churchills. John Churchill was the 1st Duke of Marlborough. His title passed to his daughter Henrietta Churchill, 2nd Duchess of Marlborough. It then passed to her nephew Charles Spencer, 4th Earl of Sunderland, who became 3rd Duke of Marlborough. His son and grandson, both named George Spencer, were 4th and 5th Dukes of Marlborough. The 5th Duke (Winston's great-great-great-grandfather) changed his name legally from Spencer to Spencer-Churchill in 1817, and this is the legal surname that all future Dukes of Marlborough have borne. Had the 5th Duke not done that, Winston would have been Sir Winston Spencer. What Winston's father Lord Randolph Spencer-Churchill did was to drop the Spencer part of the name in public life. Although the legal surname remained Spencer-Churchill, Randolph started to refer to the family as "Churchill", and Winston followed suit. JackofOz 01:43, 1 March 2006 (UTC)[reply]

REGIONAL INTERGRATION[edit]

What are five different forms of regional integration? What are some examples?

When is your homework due? Markyour words 23:28, 28 February 2006 (UTC)[reply]
Subnational entity will give you more than enough ideas. --Halcatalyst 00:10, 1 March 2006 (UTC)[reply]

John F. Kennedy speech at 1956 Democratic National Convention[edit]

I asked this before but I'll ask it again. Where can I find an ONLINE transcript of John F. Kennedy's speech at the 1956 Democratic National Convention. I tried Google and got nowhere. --Blue387 23:09, 28 February 2006 (UTC)[reply]

It depends on what you look for when you try Google. "Democratic national convention 1956" kennedy speech garnered 49 hits, including the www.cs.umb.edu/~serl/jfk/index93.htm John F. Kennedy Library index to its document holdings. That might help. --Halcatalyst 14:59, 1 March 2006 (UTC)[reply]

Why was suffrage limited to property?[edit]

One thing I didn't understand about the framers of the U.S. Constitution was that suffrage was initially limited to white, property owning males. What was the motivation for this? --Blue387 23:22, 28 February 2006 (UTC)[reply]

  • I don't see where the Constitution says anything of the sort; until the Fifteenth Amendment, I don't think the Constitution controlled suffrage in any way, leaving it instead to the states. Am I having a lapse in my historical memory? --jpgordon∇∆∇∆ 00:31, 1 March 2006 (UTC)[reply]
You are correct. Voting is a state issue. It is not part of the Constitution. Each state had a constitution and, to my knowledge, all but two of them required voters to be white property-owning males in 1789. Also, to my knowledge, the state constitutions were written by the "founding fathers" - the original Continental Congress. --Kainaw (talk)
Anyone who claims to know what the "founding fathers" were thinking is, by rule, deluded. With that in mind, consider who this left out: The British, French, and Spanish people who did not own land in the U.S., but were doing trade there could not vote. This was the basis of a government "by the people". Of course, women were left out because nobody considered them smart enough to vote at that time (even though there had been many women in politics by then). In all reality, it wasn't as much an intelligence issue as a family one. An average woman had 10-15 kids. When could she have time for education or political banter? As for the "white" clarification - even Lincoln considered non-whites a lesser race and he wasn't even born yet. So, you can make your own guess at the motivation. Then, you can back up your ideas by saying that the founding fathers agree with you! --Kainaw (talk) 00:35, 1 March 2006 (UTC)[reply]
Right, it was a matter of education. Only rich white men got an education in the western world. Which makes some sense. In a democracy the voters need to know what they vote about, so if large groups of people aren't educated it makes little sense to let them vote. Which is why even fullfledged democracies have a voting age. But this meant that these people voted for their own interrests. Workers may not have been educated, but they weren't stupid, so they fought for equal rights. Which they got. It turned out the prerequisite of education among the voters wasn't all that important as long as they voted for educated people. But that is getting to be less the case it seems, so maybe either voters need to get a better education or only educated people should be allowed in politics. Not sure, just a thought. DirkvdM 07:45, 1 March 2006 (UTC)[reply]

Logic[edit]

Is it true that Logic can only hope to be consistent 'within' itself? or has anyone come up with an argument that says logic has the possibility to justify itself beyond itself?. I know this is a question that seems silly at first glance...but here is an exapmple... I know it's ficticious but let's assume it's the case. let's say I make a computer program that can 'square the cirlce', confuse up with down, blah blah blah...all those 'illogical' things and maybe even resemble quantum mechanics...let's say I put an AI inside my program...then the logic of that AI will be illogical to us...but it may seem completely logical to the AI...so now picture the case that our logic is 'fake'...I know that to think about this is completely useless, but I'm just curious if OUR logic can justify itself beyond itself somehow...I doubt it...but I bet logicians know better.--Cosmic girl 23:39, 28 February 2006 (UTC)[reply]

Logic is a framework that is true to itself - all the pieces in the framework hang together in order for the laws of logic to work - this is occasionally called an organon, though technically that relates specifically to the teachings of Aristotle. But whether there is any firm foundation to the structure is a question which has haunted phoilosophers for centuries - this is the field of metaphysics. It is possible to build logical structures that are "false" according to our understanding - they can be consistent without being valid. In fact, many of the exercises used in intermediate level logic texts show this: "If all fish are Italian, and the banana is a fish, then the banana must be Italian." There is no guarantee that any particular organon is correct o=r has any firm foundation. At various points in the history of philosophy, people have come up with what they believe is a firm grounding of a logical structure in incontrovertible facts - the most famous being Descartes's "I think, therefore I am", but ebven this has the logical fallacy that there's no way of knowing you think unless you assume you are to start with (so it's really the recursive "If I am and I am thinking, then I am"). But I digress... no logic has yet been proved to represent true reality - or even that a true reality exists. But our logic works very well and seems to be hard-wired into us as a way to cope with the world, and shows no apparent major contradictions with it. As such, it is the best tool we have or are likely to have in understanding the universe. Our view of things may not be perfect, but it's a pretty sensible and consistent view. Grutness...wha? 00:16, 1 March 2006 (UTC)[reply]
If nothing else, logic must be only consistent within itself, as all systems require comparison to external systems for verification. Logic cannot be compared to any system outside of logic. This can be a cheering or depressing insight. It can make logic look bigger or smaller. It depends upon what you want to do with this insight. You can argue that logic is no more than a syntax of the subjective state in good operation. You can argue that logic may be the single system prior to the chaos of existence. <shrug> At the very least, we could wish that presidents of the United States appealed to it more often. Geogre 00:51, 1 March 2006 (UTC)[reply]
I agree that the rules of logic can only be asked to be consistant within their own framework, but they were developed by people who lived on this earth and were consistantly immersed in the physical laws that govern its existance. The rules of logic are each expressions that can be found to be logical within the rules of this existance. Our computer languages are built from this logic also. If you wanted to create a universe inside a computer where an AI creature thinks it's perfectly reasonable for a circle to be a square you would need to flesh out that universe. 'What logical rule would allow a circle to be the same as a square?' 'What else would that rule govern?' Perhaps you could simply start with a set of rules and have the AI tell you what it sees. That's a bit of a mind b(l)ender. -LambaJan 05:21, 1 March 2006 (UTC)[reply]
Of course then you couldn't communicate with the AI because you couldn't trust that it's words for it's perceptions line up with your definitions of those words. It could say 'circle' when you know you put a square there. Or 'circle' and you did put a circle there, but is it really seeing what you would call a circle? sorry, I couldn't help it. -LambaJan 07:33, 1 March 2006 (UTC)[reply]


I would make the AI much less smarter than me..like in less dimensions...so that I could have a superior understanding...(I'd really never do it, I'm not that mean...just as a mind experiment...I'd never make the AI a sentient being...).--Cosmic girl 20:48, 1 March 2006 (UTC)[reply]

Logic and mathematics are formalisations of the way we think (our apriori knowledge). If something does not fit our logic we cannot think nor speak of it. Let me be mean and throw Wittgenstein at you: "Wovon man nichts sagen kann, darüber muss mann schweigen." Then again, if we designed that computer program and run it in a computer we built then it falls within the realm of our logic because our computers are designed according to the rules of our logic (quite literally - they're physical manifestations of our rules of logic). Then again, we may design the genotype of the AI, but that does not mean we can understand its phenotype. We have limited brains and lifespans. The same rules we use, implemented in an almost infinitely larger entity could come up with conclusions we could never have reached (but would we accept those conclusions from a 'machine'?). But then this is still based on our logic and I get back to Wittgenstein. Still not much of an answer, sorry. DirkvdM 08:23, 1 March 2006 (UTC)[reply]

I don't know german :( .--Cosmic girl 20:49, 1 March 2006 (UTC)[reply]

Wittgenstein is not mean! He's weird, but he's not mean. :-) (I have yet a Different interpretation of logic, myself. I see it, as a formalized study as opposed to the mere syntax of the mind, as arising particularly and peculiarly in a specific set of historical moments. When Aristotle's Prior Analytics came along, it was in the context of a mature democracy. Socrates, Plato, and Aristotle were reacting to a world where an impressive speech got power, and therefore they needed a system by which good from bad speech could be distinguished (Rhetoric) and by which good from bad proposals could be discerned (logic). As the medieval historical need and frisson (dang! sorry!) was different, it changed the nature of logic as a study. As the 20th century's doubt and angst changed its comfort with knowledge and certainty, we get the questions about logic that were dormant and inherent from the start.) (I take this view because I hold out no hopes for transcendence through the mind and hold my metaphysics to be mystical.) Geogre 11:08, 1 March 2006 (UTC)[reply]

Even mysticism has a logic to it ...and it might be wrong...there's no way a mystic can assure what he's seeing is 'true' it's only true to himself...I guess...--Cosmic girl 02:24, 2 March 2006 (UTC)[reply]

Re logic and reality: Albert Einstein knew a thing or two about logic and he said: "Reality is merely an illusion, albeit a very persistent one". JackofOz 12:53, 1 March 2006 (UTC)[reply]

I knew that quote...but it says reality is an illusion not logic is an illusion.--Cosmic girl 20:51, 1 March 2006 (UTC)[reply]

I think the point is that logic cannot be used to prove that something objectively exists in reality. As Grutness said above, "No logic has yet been proved to represent true reality - or even that a true reality exists." JackofOz 21:52, 1 March 2006 (UTC)[reply]

Awesome! I asked the question with exactly that same idea in mind...I just wanted to listen to the experts.lol.--Cosmic girl 02:25, 2 March 2006 (UTC)[reply]

What I mean by this question is that I think logic is trapped within itself...I mean, it can be consistent, but it can never have any claim as to what is 'true reality' or if there is any at all...but like the first wikipedian that answered this question said...it's the best thing we have. what I was asking was if there was any attempt to 'prove' logic beyond itself that was valid...I guess there's none.--Cosmic girl 20:54, 1 March 2006 (UTC)[reply]

Gödel's incompleteness theorems[edit]

Gödel's incompleteness theorems support this thought. The basic idea is that a mathematical (i.e., logical) system can't be both complete and consistent. --Halcatalyst 21:32, 1 March 2006 (UTC)[reply]

Take a look at the question I asked about that at the math section...I got missunderstood I guess... I mean,it seems they didn't see this conection that I saw and you just pointed out. but I have a doubt about that...what does it mean to be or complete or consistent but not both? I mean, let's say a logical system is consistent...then it's not complete, why is it not complete? and also...if it is 'complete' ...how can it be since it 'lacks' consistency? and so, is this saying that this logical system can only be complete if it is 'irrational' (inconsistent)? wow! why? I need to understant Gödel's theorems!.--Cosmic girl 02:29, 2 March 2006 (UTC)[reply]

My understanding is that the theorems that are actually specific enough to prove something can't prove that that thing is true. So, a logical proof for equation 'x' can proove x within its own internal logic, but it cannot say that 'x' is true by any measure outside of the proof itself. Does that sound right to the rest of you? -LambaJan 19:18, 2 March 2006 (UTC)[reply]

I still don't get it ...is there any easy explanation of the Gödel theorems?.--Cosmic girl 23:35, 3 March 2006 (UTC)[reply]

Well, no. You need to know a lot of math to even know what they mean. We laypeople can skim off a little of the top layer of meaning and perhaps believe we understand, but it ain't necessarily so. Gödel's theory is pretty safe from this sort of misuse, but you certainly can't say the same about Heisenberg's Uncertainty principle, for example. All sorts of people think this is a general explanation for everything. Turn it around, and you have physicists striving for GUT (Grand unification theory.) So the one is going to borrow a principle and apply it to everything, and the other is going to discover a principle and say it is everything. Ohmiohmy. --Halcatalyst 04:49, 4 March 2006 (UTC)[reply]

"About which one cannot say anything, over it man must be silent" - thus Googl translates Wittgenstein . Would AI logic give results that we could not understand and still be plain logic ? I think that it depends only of the choices made in relation to data. Our logic talks about data from our common experience of the current world. We know that when we begin logic plays with italian bananas, it is a play : AI does not know that first time.

We need to feed AI with collected data, namespaces, referential common sense ... to avoid it giving back crap as seen in our sense.

Now, the measure of truth we can impart to our data is the measure of truth of the results of logic. The truth of the very process of logic itself is given by us and it can't prove itself. You can see that AI is built only with that logic of ours, with transistor gates saying "and, or, not" ...

Logic itself is based upon assumptions or axioms such as "non contradiction" and "causality" : it's built in babies'brains, our brains do not change easily. The risk would be to built AI logic with only such axioms. If we try a wider logic game, including contradictory axioms such "A is B and also non-B", as we began to observe with quantic things, then the big risk is that results will grow quickly hard to evaluate, even if the rules are strongly enforced. --DLL 15:42, 4 March 2006 (UTC)[reply]

I see, but I was proposing a case in which we where smart enough and had the thechnology to do that...and even if it's not possible, we can't prove it isn't for 'someone' or 'something'... and I didn't mean for the AI to only come up with 'crap', lol...the AI would come up with 'crap' for us in this world but with crap which is consistent with it's internal logic (illogic) ( a logic we created)...so the AI would think that WE are the illogical ones if it came to think about us... I put this example to imply that we are the AI...(it's hypothetical...of course...I'm not that crazy yet).So...what I mean is that our logic is completely consistent for us, it's usefull and what not, but we can't ever prove it's TRUE not only for itself but for 'beyond' itself also...I mean, it can't ever prove it's ABSOLUTELY true...nor can any sort of logic (that's my guess)but then again I may be trapped in a meaningless wordplay here...I think I need a prophesional logician for this question. :S what did Russell have to say about this?.--Cosmic girl 17:21, 4 March 2006 (UTC)[reply]

This is beginning to sound strikingly similar to Meta-God, where I eventually drew the line and said something along the lines of 'maybe, but this is the reality we have. So... does it matter?' -LambaJan 22:32, 4 March 2006 (UTC)[reply]

True...I just wanna know what logicians have to say regarding this problem. --Cosmic girl 02:06, 5 March 2006 (UTC)[reply]

Perhaps I'm wrong, but I'm under the impression that they don't say anything about it. I think this is a much bigger picture than they tend to look at. -LambaJan 20:48, 5 March 2006 (UTC)[reply]

I think you are right, I haven't found any thoughts on this...I thought the Gödel theorems where about this but I don't know anymore... thank you all for your responses and patience, lol. XD --Cosmic girl 17:02, 6 March 2006 (UTC)[reply]